360
Acromegaly Stem Statement Please examine hands, face, look and proceed. Patient has headaches, increased sweatiness Approach 1. Hands a. Palm downwards – large, doughy, spade shaped, OA, double pinch test b. Palm upwards – sweatiness, CTS, wasting of thenar eminence, numbness 2. Elbows – ulnar nerve thickening 3. Proximal myopathy 4. Face – Transfrontal scar, prominent supraorbital ridges, greasy skin, broad nose, hirsute, thickened lips, macroglossia, teeth indentation marks on the side of the tongue, prognathism, splaying of teeth, malocclusion of teeth 5. Neck – Goitre 6. Lower limb – bowed legs, OA, pitting edema from CCF/CCB, heelpad thickened 7. Request for patient to remove shirt to inspect the trunk and axillae a. Skin tags b. Coarse body hair c. Acanthosis nigricans d. Gynaecomastia, galactorrhoea e. Kyphosis 8. Request a. Visual fields – bitemporal hemianopia, fundoscopy for angiod streaks b. CVS – cardiomegaly c. Abdomen – organomegaly, testicular atrophy, PR bleed for Ca colon d. BP - Hypertension e. Urine dipstick – glycosuria Presentation Sir, this patient has acromegaly as evidenced by presence of coarse facial features with prominent supraorbital ridges, broad nose and thick lips; a/w macroglossia with teeth indentation marks on the side of the tongue. There is also presence of splaying of the teeth with malocclusion and prognathism. I did not notice any scars on the forehead to suggest previous Transfrontal surgery. There is also no goitre There is presence of a large, spade like doughy hands with no sweating detected. There is no wasting of the thenar eminence and Tinel’s sign was negative. There are also no thickened ulna nerves at the elbows and no proximal myopathy. No features of OA of the hands and no bowing of the tibia. No pedal edema but presence of thickened heelpads. I would like to complete the examination by 1. Asking the patient to remove his shirt to look for Acanthosis nigricans, coarse body hair, skin tags, kyphosis and gynaecomastia/galactorrhea 2. Visual fields for bitemporal hemianopia 3. CVS – cardiomegaly 4. Abdomen exam for organomegaly

Mrcp short case telling skills

Embed Size (px)

Citation preview

Page 1: Mrcp short case telling skills

Acromegaly

Stem Statement Please examine hands, face, look and proceed. Patient has headaches, increased sweatiness Approach

1. Hands a. Palm downwards – large, doughy, spade shaped, OA, double pinch test b. Palm upwards – sweatiness, CTS, wasting of thenar eminence, numbness

2. Elbows – ulnar nerve thickening 3. Proximal myopathy 4. Face – Transfrontal scar, prominent supraorbital ridges, greasy skin, broad nose, hirsute,

thickened lips, macroglossia, teeth indentation marks on the side of the tongue, prognathism, splaying of teeth, malocclusion of teeth

5. Neck – Goitre 6. Lower limb – bowed legs, OA, pitting edema from CCF/CCB, heelpad thickened 7. Request for patient to remove shirt to inspect the trunk and axillae

a. Skin tags b. Coarse body hair c. Acanthosis nigricans d. Gynaecomastia, galactorrhoea e. Kyphosis

8. Request a. Visual fields – bitemporal hemianopia, fundoscopy for angiod streaks b. CVS – cardiomegaly c. Abdomen – organomegaly, testicular atrophy, PR bleed for Ca colon d. BP - Hypertension e. Urine dipstick – glycosuria

Presentation Sir, this patient has acromegaly as evidenced by presence of coarse facial features with prominent supraorbital ridges, broad nose and thick lips; a/w macroglossia with teeth indentation marks on the side of the tongue. There is also presence of splaying of the teeth with malocclusion and prognathism. I did not notice any scars on the forehead to suggest previous Transfrontal surgery. There is also no goitre There is presence of a large, spade like doughy hands with no sweating detected. There is no wasting of the thenar eminence and Tinel’s sign was negative. There are also no thickened ulna nerves at the elbows and no proximal myopathy. No features of OA of the hands and no bowing of the tibia. No pedal edema but presence of thickened heelpads. I would like to complete the examination by 1. Asking the patient to remove his shirt to look for Acanthosis nigricans, coarse body hair, skin tags, kyphosis and gynaecomastia/galactorrhea 2. Visual fields for bitemporal hemianopia 3. CVS – cardiomegaly 4. Abdomen exam for organomegaly

Page 2: Mrcp short case telling skills

5. BP 6. Screen for DM 7. Ask for symptoms of headache, increase sweatiness and recent increase in shoe or glove size. Questions

1. What is acromegaly? a. Due to excess GH activity as a result of a pituitary macroadenoma occurring post

puberty 2. What are the indicators of activity?

a. Skin tags b. Increased sweatiness, headache c. Increased size of goitre/visual field loss/size of hands/Shoe size d. Hypertension e. Glycosuria

3. What are angiod streaks? a. Degeneration and fibrosis of Bruch’s membrane

4. List causes of macroglossia. a. Acromegaly b. Hypothyroidism c. Amyloidosis d. Down syndrome e. Haematological malignancy

4.What is the pathology of acromegaly? Pituitary macrodenoma 5. What are the complications?

a. Metabolic and endocrine i. Diabetes mellitus in 20% of patients

ii. Hypertriglyceridemia in 40% of patients b. Cardiovascular

i. Hypertension ii. Cardiomyopathy and CCF

c. Respiratory i. Acute dyspnea and stridor (upper airway narrowing)

ii. Obstructive sleep apnea d. Abdomen

i. Colonic polyps and malignancies (ie, colon cancer) ii. Organomegaly, testicular atrophy

e. Neuromuscular i. Proximal myopathy

ii. Nerve root compression – CTS, radiculopathy iii. Spinal stenosis

f. Calcium and bone metabolism i. Hypercalciuria

ii. Hyperphosphatemia iii. Urolithiasis

Page 3: Mrcp short case telling skills

6. How do you investigate? a. Confirm the diagnosis by OGTT to look for non supressibility of GH (2ng/ml), can

also screen for DM b. Other useful blood Ix

i. IGF-1 – as a baseline and monitoring disease activity and treatment ii. Prolactin levels as 20% are associated with hyperprolactinaemia

1. low in hypopit 2. High because 1. Co-secretion 2. compression of pit stalk with

interference of dopaminergic suppression of prolactin production iii. Pituitary function (SST, TFT, FSH/LH/Testos/Oestradiol) iv. Calcium levels – MEN type 1 syndrome

c. Imaging (after diagnosis is confirmed) i. MRI of the pituitary fossa – macroadenoma

ii. X-rays 1. Skull – Enlarged sella turcica, enlarged frontal, ethmoid and

mastoid sinuses, thickened calvarium, enlarged mandible 2. CXR – cardiomegaly 3. Hand and feet X-rays – terminal phalangeal tufting and thickened

heel pad (>23mm thick on a lateral X-ray) d. Others

i. Formal perimetry ii. Obtain old photos

iii. ECG - LVH 7. How would you manage?

a. The definitive therapy is surgical which can be via transphenoidal or the transfrontal approach

b. Radiation therapy if pt is not a suitable candidate c. Medical therapy

i. Bromocriptine – dopamine agonist (PO) ii. Octreotide or long acting type (SC, daily vs monthly)

iii. GH receptor antagonist – pegvisomant which is a recombinant DNA analogue (SC daily)

8. What are the conditions with excess GH besides acromegaly? a. MEN type 1 (PPP) b. McCune Albright syndrome – Polyostotic fibrous dysplasia, sexual precocity and

café-au-lait spots c. Carney Complex – multicentric tumors in multiple organs, pigmented skin lesions

and pigmented nodular hyperplasia (aut dominant)

Ankylosing Spondylitis

Approach Stem Statement

1. Look at this patient (no apparent endocrine, rheumatology or neurological abnormalities) 2. Examine patient’s gait 3. Low back pain, chest, back, neck, look at the ceiling

Page 4: Mrcp short case telling skills

Proceed 1. Introduce (thank you for letting me exam you, my name is… how do I address you) 2. Ask if there is pain 3. Stand the patient, walk him, turn around and return to original position 4. Touch toes with his fingers 5. Look left and look right 6. Touch your chest with your chin

Presentation Sir, this patient has Ankylosing Spondylitis as evidenced by a stooped, question-mark posture with loss of lumbar lordosis and a fixed kyphosis with extension of the cervical spine in an attempt to maintain a horizontal visual gaze. There is also a protuberant abdomen. Spinal movements are restricted as evidenced by the finger-to-toe test, with limited flexion and lateral movements of the cervical spine. I would like to complete the examination by doing the “heels, hips and occiput” test and measure the occiput-to-wall distance, as well as a modified schoeber’s test and chest expansion (5cm). I would like to look for

shoulder and knee joint involvement

extra-articular involvement and complications

Differential diagnoses o Skin – to rule out Psoriasis which is a possible differential diagnosis o Abdominal examination to look for signs of inflammatory bowel disease o Reiter’s syndrome

Questions 1. Tell me about AS.

a. Seronegative spondyloarthropathies, chronic inflammatory arthritis affecting the SI joints with fusion of the spinal vertebrae, associated with HLA b27

b. 3rd to 4th decades, males 3X c. Symptoms: back pain worse in the morning and with rest and improves with

activity d. Signs : limited lateral flexion of the lumbar spine is the first sign of spinal

involvement followed by loss of lumbar lordosis e. Investigation and management

2. What are the associated conditions? a. Anterior uveitis, iritis b. Atlanto-axial subluxation or dislocation c. Apical fibrosis d. Aortic regurgitation e. AV nodal block, arrythmias f. Amyloidosis g. Achilles tendonitis, plantar fasciitis

Page 5: Mrcp short case telling skills

3. What is the heels-hips-occiput test? a. Ask the patient to place his heels, hips and occiput against a wall all at once b. Inability of the occiput to touch the wall c. Can measure the wall-occiput distance

4. What is Schoeber’s test? a. Draw a line joining the “dimple of Venus” b. 5cm mark below c. 10cm mark above d. Forward flexion e. <5 cm implies limited spinal mobility

5. Why is there a protuberant abdomen? a. This occurs as a result of restricted chest expansion from a fixed spine b. Hence resulting in a predominantly diaphragmatic breathing c. With resultant protuberant abdomen

6. What are the other types of conditions that can present with sacroilitis? a. Psoriasis b. Reiter’s (reactive arthritis)

i. Can be urogenital (Chlamydia) or gastrointestinal (Shigella, campylobacter, salmonella)

ii. Triad of urethritis, arthritis and conjunctivitis iii. Cs have circinate balanitis (small shallow painless sores) and keratoderma

blenorrhagica (small hard papules on palms and soles) c. Enteropathic arthritis (these are the seronegative spondyloarthropahy which are

associated with HLA B27) 7. How do you diagnose AS?

a. Rome or New York Criteria b. Based on:

i. Radiological features of sacroilitis ii. Symptoms of back pain (lumbar spine or dorsolumbar junction)

iii. Physical signs of limited spinal mobility in all 3 planes and chest expansion <2.5cm

8. How would you investigate? a. Imaging – AP views and the SI joints and AP/lat of spinal vertebrae

i. Early – erosions and sclerosis of the SI joints ii. Later – Syndesmophytes in the margins of lumbosacral vertebrae

iii. Advanced – “bamboo” spine b. Blood test (not so important)

i. Elevated ESR ii. HLA B 27 (95% AS is positive but small percentage of B27 positive develop

AS) 9. How would you manage?

a. Education and counseling i. Chronic disease

ii. Genetic counseling (HLA positive, siblings has 30%)

Page 6: Mrcp short case telling skills

1. 50% chance of transferring genes to children 2. 1/3 of children who has HLA B27 will have AS 3. Overall risk is 1/6

b. Non-pharmological i. Lifelong regular exercises

ii. Involving the PT and OT c. Pharmological

i. NSAIDs ii. MTX, sulphasalazine

iii. Anti TNF and anti CD-20 d. Surgical therapy

What are the indications for starting immunomodulators?

TNF blocking agents are recommended for the treatment of active AS after having failed treatment for the patient’s predominant clinical manifestation

Aortic Regurgitation

Examination Proceed as per cardiovascular examination On detecting AR, to examine eyes for Argyll-Robertson pupil and auscultate the femorals for pistols shots (Traube sign) and Duroziez sign. Presentation Sir, this patient has aortic regurgitation that is severe. My findings are: Presence of a high-pitched early diastolic murmur heard best at the left lower sternal edge and is loudest at end expiration with the patient sitting forwards. It is a grade 4/6 murmur and is associated with a diastolic thrill. It is severe as the murmur is of a long duration associated a soft second heart sound with a S3 present. There is also an Austin Flint murmur with presence of a mid diastolic murmur heard at the apex not associated with an opening snap. The apex beat is displaced at the 6th IC anterior axillary line and is thrusting in nature. This is associated with evidence of CHF with bibasal crepitations, raised JVP at 4 cm with a prominent V wave as well as bilateral peal edema. Peripheral examination showed no evidence of IE. The pulse is bounding and collapsing in nature at a rate of 90 bpm in SR. There is no RR or RF delay to suggest coarctation of the aorta. In addition, quinke’s sign was negative. There was no conjunctival pallor but Corrigan’s sign and brachial dance were present. Muller’s sign, Duroziez and Traube’s signs were not detected. In terms of etiology, there is no evidence of symmetrical deforming polyarthropathy to suggest RA and patient does not have a Marfanoid habitus or a high arched palate. There is no Argyll-Robertson pupil to suggest lewitic disease.

Page 7: Mrcp short case telling skills

To complete my examination, I would like to take patient’s BP looking for wide pulse pressure and severe hypertension. I would also want to look at the patient’s temperature chart. In summary, this patient has got AR that is severe with complication of CCF. Possible causes for this patient’s AR are Rh heart disease, infective endocarditis or congenital bicuspid valve. Questions What are the signs of severity of AR? S3 Austin Flint murmur (functional mdm at the apex due to regurgitant jet striking the anterior

leaflet of the MV, therefore obstructing flow from the LA into the LV) Soft S2 Duration of the decrescendo murmur and loudness of murmur (cf with AS) Apex beat displaced and thrusting CCF Wide pulse pressure Hill’s sign What are the characteristic signs of AR? Collapsing pulse Brachial dance Quinke’s sign (visible capillary pulsation in the nail bed) Corrigan’s sign (Visible Carotid pulsation in the neck) De Musset’s sign (head nodding in time with the heart beat Muller’s sign (pulsation of the uvula) Traube’s sign(pistol shots) and Duroziez sign(to and fro murmur on sl compression of the

femoral artery) What are the causes of a collapsing pulse? AR PDA An aortopulmonary window A ruptured aneurysm of the aortic sinus Active Paget’s High fever Severe Anaemia Pregnancy What would you expect to find on taking this patient’s blood pressure? Wide pulse pressure Severe hypertension (with functional AR) UL and LL discrepancy with systolic in LL>UL = Hill’s sign How do you differentiate an Austin Flint murmur from mitral stenosis? Opening snap Loud S1 Tapping apex beat, which is not displaced

Page 8: Mrcp short case telling skills

What are the causes of AR? Valvular

Rh, IE and congenital biscupid valve (associated with CoA) Aortic root dilatation

Syphilis, RA, AS, Marfan, severe hypertension Acute causes

IE, trauma, Aortic dissection, rupture of sinus of valsalva How would you investigate? ECG – LVH with diastolic overload pattern – deep but narrow Q, isoelectric ST, and tall T

waves in left praecordial leads CXR – valvular calcification, cardiomegaly, pulmonary congestion, widened aorta 2D echo Confirm Dx Assess cause Severity Complications

How would you manage this patient? Education Medical Antibiotic prophylaxis Treat underlying cause Treat complications such as CCF, IE Vasodilators – ACE and CCB

Surgical Indications Symptomatic – CCF, angina and severe AR LV ESD >55mm Aortic root >55mm Reduction of EF >5% on exercise

Types of surgery What is the prognosis? -> 4% develop symptoms, CCF or both annually

Aortic Stenosis

Presentation Sir, this patient has Aortic stenosis that is severe in nature. My findings include: Presence of an ejection systolic murmur heard best at the aortic area and radiates to the carotids. It is a grade 4/6 systolic murmur a/w with a systolic thrill. It is severe as there is an early ejection click a/w a long systolic murmur with delayed peaking of the murmur. I could not detect an S4 and the second heart sound is soft. There was also no paradoxical splitting of the second heart sound.

Page 9: Mrcp short case telling skills

The apex beat is heaving in nature and is displaced, located at the 6th IC space at the just lateral to the mid-clavicular line. This is associated with signs of congestive cardiac failure as evidenced by presence of bibasal crepitations, raised JVP at 3 cm with prominent V wave and bilateral pedal edema but she does not require supplemental oxygen. Peripheral examination does not reveal any stigmata of IE. The pulse is regular at 84bpm and is anacrotic/pulsus parvus et tardus in nature. There are no features suggestive of haemolytic anaemia with no conjunctival pallor and patient is not jaundice. I would like to complete my examination by taking the patient’s blood pressure to look for a narrow pulse pressure as well as his temperature chart. I would also like to enquire on patient’s symptoms of angina, syncope and dyspnea as these are important prognostic markers. In summary, this patient has got aortic stenosis that is severe in nature with complication of congestive cardiac failure. There is no evidence of infective endocarditis or haemolytic anaemia. The most likely causes include Rh heart disease, calcified biscupid aortic valve or degenerative calcified aortic valves. Questions What are the differential diagnoses for an ejection systolic murmur?

AS

PS

HOCM

MVP/MR

Coarctation How do you differentiate between them?

AS and PS – expiration and inspiration

AS and HOCM – Valsalva, squatting

AS and MVP – location and clicks

AS and Coarctation – differential pulse What are the types of pulses associated with aortic stenosis?

Pulsus parvus et tardus – means low volume pulse with delayed upstroke due to a reduction in systolic pressure and a gradual decline in diastolic pressure

Anacrotic pulse – small volume pulse with a notch on the upstroke What does a normal pulse volume in AS mean?

The travsvalvular gradient is <50 mmHg What does a palpable systolic thrill implies?

It means that the transvalvular gradient is > 40mmHg What does the second heart sound indicate about the aortic stenosis?

Soft second heart sound means poorly mobile and stenotic valve

Reversed splitting means mechanical or electrical prolongation of ventricular systole; S2 is normally created by the closure of the aortic valve followed by the pulmonary valve, if

Page 10: Mrcp short case telling skills

the closure of the aortic valve is delayed enough, it may close after the pulmonary, creating an abnormal paradoxical splitting of S2.

Single second heart sound implies fibrosis and fusion of the leaflets

Normal second heart sound implies insignificant stenosis What is Gallavardin phenomenon?

Systolic murmur may radiate towards the apex, which may be confused with a MR murmur

How can haemolytic anaemia result from aortic stenosis? --MAHA from severely calcified aortic valve What are the causes of aortic stenosis?

Rheumatic heart disease (<60)

Degenerative calcification (>75)

Calcified biscupid (60-75, males) What are the severity markers?

Early ejection click

Long Systolic murmur

Late peaking of the murmur

4th heart sound

Paradoxical splitting of S2

Heaving apex beat which is displaced

Systolic thrill

Pulsus parvus et tardus

Narrow pulse pressure

Symptoms (ASD)Px Angina5 years S yncope 3 years Dyspnea (Most impt)2 years How do you differentiate AS from aortic sclerosis?

No severity signs as above ESM which is localised to aortic area with a normal S2 in elderly person

How do patients present?

Asymptomatic and incidental finding

Angina o Increase oxygen requirement for hypertrophied LV with hypoperfusion of the

subendocardial myocardium

Syncope o Cardiac arrythmias o Peripheral vasodilatation eg post exercise without concomitant increase in CO o Transient elctromechanical dissociation

Dyspnea o Implies LV dysfunction and heart failure

How would you investigate?

ECG – LVH with strain, 1st degree heart block, LBBB

Page 11: Mrcp short case telling skills

CXR – Calcified aortic valve, cardiomegaly, pulmonary congestion

2D echo o Dx o Severity

LVH, EF

Severity Area Transvalvular gradient

Mild >1.5 <25

Moderate 1-1.5 25-50

Severe <1 50-80

Critical <0.7 >80

Complications eg IE How would you manage?

Education Medical

o Antibiotic prophylaxis o Rx complications such as arrythmias and CCF (caution with antihypt to avoid

reducing preload) o Statins may have a role in reducing calcification of the aortic valve o Surgical treatment

Indications Symptomatic and severe Asymptomatic but has

Area<0.6

LV systolic dysfunction

Hypotension on exercise

VT

LVH>15mm Moderate AS but going for Sx for CABG, MVR or aortic root surgery

o Options Valve replacement (Sx of choice) Valvuloplasty (for moribund patients)

What are your thoughts on a young person with AS murmur but a normal aortic valve?

Supravalvular stenosis o Can be isolated or associated with Williams syndrome o It is an inherited disorder, autosomal dominant, Ch 7 o Features of elfin facies, hypertension and mental retardation with other cardiac

lesions such as PS

Subvalvular stenosis What abdominal condition is associated with AS?

Angiodysplasia of the colon (PR bleed)

Page 12: Mrcp short case telling skills

What is pulse pressure?

Difference between systolic and diastolic pressure

Normal – 40mmHg

Wide - >60 mmHg

Narrow - <25mmHg

Note: There is no official definition but studies usually measures the pulse pressure as a continuum

Ascites

(Think of CLD, Budd-Chiari, renal failure or heart failure, hypothyroidism, also malignancy, TB) Presentation Sir, this patient has gross ascites. There is presence of abdominal distension with an everted umbilicus. There is a positive fluid thrill as well as shifting dullness. This is not associated with any abdominal tenderness and patient is able to lie flat for the examination. There is also abdominal scar marks suggesting abdominal tap has been done. I am unable to palpate the liver and it has a span of 12 cm in the right mid-clavicular line. The spleen is not palpable or percussible. The kidneys are not ballotable. There are no other masses palpable in the abdomen. There are no stigmata of chronic liver disease such as leukochynia, clubbing, palmar erythema, spider naevi, gynaecomastia or loss of axillary hair. There is also no hepatic fetor or a hepatic flap. Patient is not jaundice and there is no conjunctival pallor. There is associated pedal edema up to the knee level with sacral edema but no periorbital edema. There are no signs of renal failure such as a sallow appearance or uremic fetor. Patient also does not have any features to suggest hypothyroidism such as a cream and peaches complexion, macroglossia, hoarseness of voice or bradycardia. He is not cachexic looking and there are no palpable cervical LNs. He is not toxic looking. I would like to complete my examination by

CVS looking at the JVP with the patient seated 45 degrees to look for raised JVP with steep x and y descent, early S3 suggestive of constrictive pericarditis

Urine dipstick for proteinuria

Temperature chart for fever (TB)

Rectal examination for a rectal mass In summary, this patient has got gross ascites that is not associated with any intra-abdominal organomegaly or masses of which no apparent cause is found clinically. The possible differential diagnoses include cirrhosis of the liver, Budd-chiari syndrome, nephrotic syndrome or protein-losing enteropathy, congestive cardiac failure or intra-abdominal malignancy or TB. Questions What are the causes of abdominal distention?

Fat, fluid, flatus, faeces, fetus and organ enlargement What is ascites?-Pathologically accumulation of fluid in the peritoneal cavity How much fluid must be present before there is flank dullness?-1.5 L of ascitic fluid How would you approach a patient with ascites clinically?

Page 13: Mrcp short case telling skills

Abdominal examination o Liver

Look for jaundice, spleen and stigmata of CLD – cirrhosis of liver Liver palpable and smooth – think of Budd-chiari Liver palpable and hard and nodular – think of malignancy

o Kidneys--Look for evidence of kidney failure and anasarca o Look for congestive cardiac failure or constrictive pericarditis o Look for features of hypothyroidism o If all above absent, think of

TB peritonitis Intra-abdominal malignancy

Carcinomatosis peritonei Secondaries-Liver Colon Ovaries Pancreas What are the causes of ascites? A-Serum ascites albumin gradient >1.1g/dl = portal hypertension (97%accuracy)- Cirrhosis of the liver Budd-Chiari CCF Constrictive pericarditis Malabsorption Meig’s syndrome Hypothyroidism B-Serum ascites albumin gradient< 1.1g/dl Intra-abdominal malignancy TB Nephrotic syndrome Protein losing enteropathy What is the pathophysiology of ascites in cirrhosis of the liver?

The chief factor is splanchnic vasodilatation

Cirrhosis leads to increased resistance to portal flow

Leading to portal hypertension

Portal hypertension results in local production of vasodilators, with splanchnic arterial vasodilatation

(1) Arterial underfilling Early stage – minimal effect on effective arterial volume as can be compensated by increase in plasma volume and cardiac output

o Later stage - splanchnic vasodilation so marked that effectve arterial pressure falls and results in activation of vasoconstrictors and atrial natriuretic factors

Sodium and fluid retention and expansion of plasma volume contributing to ascites Impaired free water execretion leading to dilutional hyponatraemia Renal vasoconstriction with hepatorenal syndrome

(2) Increase in splanchnic capillary pressure with lymph formation exceeding return therefore ascites

How would you investigate to determine the cause of the ascites? (Liver, renal, heart, thyroid, TB)

Ascitic tap o Cell count, albumin, and total protein concentration if cirrhosis and dxSee attached o Others

Infection – c/s and g/s AFB.. Malignancy - cytology o <0.1% of Cx such as hemperitoneum or bowel perforation o 1% of abdominal wall hematoma

Page 14: Mrcp short case telling skills

o 2FB cephalad and medial to the ASIS in the left lower quandrant

Imaging o USS/CT

Liver – cirrhosis, budd-chiari Renal

o Echo and ECG o CXR (TB, Pl effusion)

Bloods o LFT, Renal, TFT, FBC

How would you manage a patient with ascites secondary to cirrhosis of the liver?

Treat the underlying cause

Avoid alcohol or medications that are toxic to liver

Management of ascites o General measures

Salt restriction <2 g/day Fluid restriction <1l/day (for ascites, edema with Na <130)

o Specific measures Diuretics (Spironolactone, frusemide initially)

Aim to 0.5kg/day if no peripheral edema

Aim 1kg/day if presence of peripheral edema also

Increase diuretics with spironolactone up to 400mg/d or frusemide 160mg/d

Paracentesis-If >5L then requires albumin administration (8g per L of fluid removed)

TIPSS (Transjugular Intrahepatic portosystemic shunt)

High rate of shunt stenosis; up to 75% at 1 year o Liver transplant

5 year survival rate for cirrhosis with ascites is 30-40% vs 70-80% for post liver transplant

MELD score (Model for End Stage liver disease which has bilirubin, creatinine and INR)

Consider for those with refractory ascites, SBP or HRS

Manage other complications of cirrhosis How do treat and prevent spontaneous bacterial peritonitis?

Defined as >250 polymorphs per ml of ascitic fluid

Commonly E coli, Klebsiella and pneumococci

Translocation of bacteria from intestinal lumen to LNs then bacteremia

Rule out secondary peritonitis o Loculated infection or perforated viscus o Fluid

>1000 polymorphs.LDH > upper limit of serum Low glucose.High protein >1 g/L CEA > 5ng/ml.ALP >240u/L

Page 15: Mrcp short case telling skills

Treatment o 3rd generation cephalosporin . IV albumin to prevent HRS Prevention o Indications

After 1 episode of SBP as recurrence as high as 70%/year In patients with acute variceal bleed Ascitic fluid protein concentration<1g/dl (controversial)

o Prophylaxis with ciprofloxacin or norfloxacin What does development of ascites in a patient with cirrhosis of the liver means?

Decompensation

Occurs in 50% of patients within 10 years of diagnosing compensated cirrhosis

Poor Px o only 50% survive beyond 2 years o poor quality of life o increased risk of infection and renal failure

ASD

Presentation Sir, this patient has atrial septal defect as evidenced by presence of a wide and fixed splitting of the second heart sound. There is presence of an ejection systolic murmur over the pulmonary area which is louder on inspiration, implying presence of a pulmonary systolic murmur. This is a grade 3/6 murmur and there is no associated systolic thrill. There is no associated mid-diastolic flow murmur to suggest relative tricuspid stenosis or Lutembacher’s syndrome (Acquired MS and ASD). There was also no associated PSM to suggest an ostium primum defect (TR, MR, VSD). The apex beat is not displaced and is located in the 5th IC space just medial to the mid-clavicular line. There is no complication of Eisenmenger’s syndrome; there is no evidence of pulmonary hypertension; is not clubbed and no central cyanosis. There is also no evidence of congestive cardiac failure. There are no stigmata of infective endocarditis. Patient is in atrial fibrillation with an irregularly irregular pulse and is rate controlled at a rate of 84 bpm; there are also no bruises to suggest over-anticoagulation. There is no evidence of any thumb defects to suggest Holt-Oram syndrome. The patient also does not features of Down’s syndrome.

Page 16: Mrcp short case telling skills

I would like to complete my examination by examining patient’s chest for pneumonia as patients are prone to recurrent chest infections as well as a neurological examination to look for evidence of stroke due to paradoxical embolus. In summary, this patient has got an ASD with complications of AF. There are no complications of pulmonary hypertension, heart failure or Eisenmenger’s syndrome. There is also no infective endocarditis. This patient has ASD is most likely due to an ostium secundum atrial septal defect which is a congenital heart condition. Questions What are the types of ASDs?

o Ostium secundum type o 90% o common congenital heart condition o Most remain asymptomatic o If small <2 cm, normal life expectancy with no symptoms o Larger defects may present in the second or third decades with dyspnea or

fatigue o defect in the fossa ovalis with no involvement of the AV valves

o Ostium primum type o 10% o Failure of fusion of the septum primum with the endocardial cushions o AV valves affected – MR, TR and VSD

o Sinus venosus type o Defect in the septum just below the entrance of the SVC (inverted P waves in the

inferior leads) How do patients present?

o Secundum o Asymptomtic o Symptomatic

Fatigue, dyspnea Right heart failure AF Recurrent pulmonary infections Paradoxical emboli

o Primum o In addition to the above

Failure to thrive, poor development IE Syncope (heart block)

What are the complications of ASD? o Pulmonary hypertension, heart failure, Eisenmenger’s o AF, IE (primum defects) o Recurrent chest infection, paradoxical emboli

Page 17: Mrcp short case telling skills

What are the various types of murmurs that can be associated with ASD and what do they mean?

o Pulmonary ejection systolic murmur and mid-diastolic murmur at the triscuspid area implies increased flow of blood through the pulmonary and triscupid valve respectively due to left to right shunting of blood via the ASD

o MS murmur means acquired Rh heart disease affecting the mitral valve in Lutembacher’s syndrome

o MR, TR or VSD murmur implies that ASD is of the ostium primum type What is the mechanism of a split second heart sound?

o A split S2 is caused physiologically during inspiration because the increase in venous return overloads the right ventricle and delays the closure of the pulmonary valve

Why is there wide and fixed splitting of the second heart sound in ASD? o With an atrial septal defect, the right ventricle can be thought of as continuously

overloaded because of the left to right shunt, producing a widely split S2, with the pulmonary valve closing much later cf to the aortic valve

o It is fixed because the atria are linked via the defect, inspiration produces no net pressure change between them, and has no effect on the splitting of S2

How do you differentiate between a flow mumur through the pulmonary valve vs a PS murmur? o PS murmur is a/w P2 that is soft, delayed and varies with respiration

What are the conditions that can cause a wide splitting of the second heart sound? o Increase RV volume – ASD, VSD, PR o Increase RV pressure – PS o RV conduction delay –RBBB o Increase LV emptying – MR, VSD

What is Eisenmenger’s syndrome? o It implies a reversal of a left to right shunt as a result of the development of pulmonary

hypertension o This occurs in conditions such as ASD, VSD or PDA o Patients are markedly clubbed and deeply cyanosed o The defect must not be repaired once this complication occur due to high mortality risk

What is Lutembacher’s syndrome? o Acquired Rh MS o ASD

What is Fallot’s trilogy? o ASD o RVH o PS

What is Holt-Oram syndrome? o ASD secundum type o Hypoplastic thumb with accessory phalanx o Autosomal dominant

Page 18: Mrcp short case telling skills

How would you investigate? o ECG

o Secundum – Partial RBBB, RAD o Primum – LBBB, LAD, low atrial rhythm o Sinus venosus – inverted P in inferior leads o Pulmonary hypertension – p pulmonale, RVH

o CXR o Cardiomegaly o Pulmonary hypertension

Prominent pulmonary trunk Enlarged RA and RV

o Shunt vascularity/pulmonary plethora (well visualised pulmonary arteries in the periphery of the lung

o Small aortic knob o Echocardiogram

o Diagnosis - demonstrate the defect o Cardiac catheterisation

o Determine the severity and direction of shunt How would you manage?

o Counsel o Medical

o No antibiotic prophylaxis required, repaired or unrepaired o Treatment of complications such as heart failure and AF o May consider anticoagulation if there is evidence of bidirectional shunting to

prevent strokes from paradoxical emboli o Surgery

o Early childhood – closure is recommended at 5-10 years of age to prevent complications

o Small ASDs can be left alone(5 mm or less) o Large ASDs or pulmonary to systemic flow ratio>1.5 o Closed surgically or transcatheter button or clam-shell devices o Closure prevents pulmonary hypertension and RHF but does not alter incidence of AF

How would you counsel a patient with ASD who intends to get pregnant? o Pregnancy is well tolerated in patients with small and hemodynamically insignificant ASD o For large defects with pulmonary hypertension, Eisenmenger’s syndrome, avoid

pregnancy as there is increase morbidity and mortality both to fetus and mother o Routine closure before pregnancy as complications of progressive pulmonary vascular

disease may develop Asking for Postmortem (Academic)

Introduction

Introduce yourself

Clarify patient’s relatives identity and relation

Esatblish that he is the main spokesperson

Page 19: Mrcp short case telling skills

Offer Condolences

Pause – allow relative to express emotions Task

Summarise current admission clinical condition leading to death

Explain that Drs are surprised about suddeness of death

Other possible explanations for sudden death, Drs unsure

Pause

Absorb news

May challenge you on why these alternative explanations

One way to determine is postmortem (6)

Does relative understand what a postmortem is

Did the deceased express any objections previously

Benefits of a postmortem

Est cause of death, family to come to terms with death

Infor for familial disease

Infor to help drs treat other pts

For research purposes

Details of postmortem

Full postmortem vs limited postmortem

When, where, by whom and how long

Within 2 –3 working days

Pathologist

3 hrs with lab Ix up to several weeks

Body released on the day of examination unless in late pm

Delay if relative requests for parts/organs to be reuinited

Details if relative requests

Detailed internal examination

May not be 100% conclusive

Pathologist, hospital mortuary

External exam

Internal exam via an incision anteriorly

Internal organs

Incision at the base of scalp

Reassure that body not disfigured

If suspicious findings,

Retention of tissue and fluids for Ix to est cause

Retention of organ if agreeable (for research or education)

Sign consent form if agreeable

Will inform in writing if organ retained

Infor on length of retention

If at any point relative wants back, will dispose in the proper way or return for cremation/burial

Page 20: Mrcp short case telling skills

Fixed and examine and returned to body

Results of findings can be obtained from consultant in charge

Relative’s decision

Yes – will still give the death cert for funeral arrangements

No – respect her wishes and explain death certificate Dx Informing about Postmotem (Coroner’s case or Procurator Fiscal)

Same as above

Except

Coroner is an independent officer

Drs have discussed the case with Coroner

Coroner will ask the pathologist to carry out a postmortem

Findings will be sent to the Coroner who will then issue the death certificate

Representative of the Coroner will inform relatives when to proceed with funeral arrangements

Bilateral Enlarged Kidneys

Presentation Sir, this patient has bilateral enlarged kidneys. There are bilateral masses in the flanks which are bimanually palpable and ballotable with a nodular surface. Of note, I am able to get above both masses. Percussion note was resonant over both kidneys and they move inferiorly with respiration. They are not tender in nature and there was no renal bruit. There is no associated hepatomegaly and the liver span is 12 cm at the right mid-clavicualar line. The spleen is not enlarged. There is no ascites detected clinically and the bladder is not palpable or percussible. The patient does not have a sallow appearance and not cachexic looking. There are no pruritic scratch marks or bruising. There is also no leukonychia or Terry’s nails. There is no conjunctival pallor to suggest anaemia and no features of polycythemia such as a plethoric facies or conjunctival suffusion. Patient is not in fluid overload as there is no pedal oedema, he is able to lie flat and is not oxygen dependent. There is no Kussmaul’s breathing pattern and also no flapping tremor or uremic fetor. He does not have any acromegalic features, no DM dermopathy and no adenoma sebaceum to suggest tuberous sclerosis. There is no evidence of renal replacement therapy such as AVF, TK cathether or a transplanted kidney. I would like to complete my examination by

checking the patient’s temperature chart for fever,

blood pressure for hypertension

fundoscopy for hypertensive changes

urine disptick for hematuria, proteinura and pyuria

Cardiovascular examination for signs of MVP or AR

Neurological examination for a third nerve palsy secondary to berry aneurysm or any evidence of a stroke

Page 21: Mrcp short case telling skills

FHx of aneurysm or SAH (5% risk overall but 20% if positive FHx) In summary, this middle age gentleman has got bilateral enlarged kidneys with no complications of chronic renal failure detected clinically. There is also no evidence that the patient is undergoing renal replacement therapy. The most underlying etiology is Adult Polycystic kidney disease. Questions What are the causes of bilateral enlarged kidneys?

APCK

Commoner o Acromegaly (hepatosplenomegaly) o Early diabetic nephropathy o Bilateral hydronephrosis

Rare o Tuberous sclerosis o Amyloidosis o Von-Hippel Lindau disease

Autosomal dominant Multiple angiomata in the retina, CNS Cysts in liver, kidneys pancreas RCC, phaeochromocytoma

What are the conditions that can result in bilateral renal cysts? o Polycystic kidneys-Dominant and recessive

Simple cyst Von Hippel Lindau Tuberous sclerosis What are the complications of APCK? (Renal and Extra-renal Cx)

Fever o UTI, pyelonephritis, pyocyst

Hypertension (75%) o Activation of RAA from intra-renal ischaemia from architectural distortion o Malignant hypertension

Renal artery stenosis from compression Renin producing cyst

Pain o Chronic pain o Acute pain

UTI Nephrolithiasis Cyst rupture Haemorrhage into cyst Upper tract obstruction

Massively enlarged cyst

Page 22: Mrcp short case telling skills

Clot

Stone

Anaemia o CRF o Persistent gross hematuria

Polycythaemia o Increased erythropoietin production

Malnutrition o CRF o Bilateral renal enlargement with early satiety

Acute renal failure o Malignant hypertension o UTI o Nephrolithiasis (Uric acid)

Chronic renal failure

Renal cell carcinoma (rare)

Extra-renal o Abdominal – cysts in liver, spleen, pancreas, ovaries; colonic diverticular disease o Cardiac – MVP(25%), AR, TR o Intracranial aneurysm (III nerve palsy), SAH (3%)

What are the complications of CRF?

Fluid

Electrolytes – Hyperkalaemia

Acid-base – Metabolic acidosis

Uremia and its complications

Hypertension

Anaemia (NCNC)

Secondary and tertiary hyperparathyroidism

Renal bone disease Why are patients with CRF sallow?

Impaired execretion of urinary pigments combined with anaemia What are the types of signs in the nails that you can detect in patients with CRF?

Hypoalbuminaemia o Leukonychia o Muehrcke’s nails (paired white transverse line near the distal end of nails)

Renal failure o Terry’s nails (distal brown arc 1mm or >) o Mee’s line (single white line; also in arsenic poisoning) o Beau’s line (non-pigmented indented band = catabolic state)

What are the causes of anaemia in patients with CRF?

Erythropoeitin deficiency

Anaemia of chronic disease

Fe deficiency anaemia – blood loss, nutrition

Page 23: Mrcp short case telling skills

Folate deficiency – nutrition What is Adults Polycystic Kidney disease?

Multisystemic, progressive disease, 1 in 400 to 1 in 1000 people

Characterised by cysts formation and enlargement in the kidneys and other organs

Autosomal dominant with almost 100% penetrance

Focal cystic dilatation of the renal tubules

2 predominant type o 85% - APCKD 1 on Ch 16 o 15% - APCKD 2 on Ch 4 o 3rd type of which loci is not fully known

Presents clinically in the 3rd or 4th decades with o Hematuria, hypertension, recurrent UTI, pain and uremia o Stroke

By age 60 years, 50% will require RRT

Poorer Px – males, PCK 1 and early onset of clinical features

Mortality o ESRF (1/3) o Stroke and other hypertensive Cx (1/3/) o Others

How do you investigate?

Blood Tests o FBC o Biochemical o CRF – Ca, PO4, iPTH, Uric acid, urinalysis

USS (useful >20 years old); Ravine’s criteria o At risk patients, 20-30yrs: 2 cysts in 1 kidney or 1 cyst in each kidney o At risk patients, 30-60 yrs: 2 cysts in each kidney o At risk patients, >60 yrs: 4 cysts in each kidney

Other imaging(CT and MRI)

MRA for patients with high risk of an aneurysm, Ba enema and Echocardiogram

Genetic testing o For young people with no cysts on USS who are potential organ donors

How would you manage?

Education and counselling, regular follow up, screening of first degree relatives

Avoidance of medications that can precipitate renal impairment such as NSAIDs or tetracycline antibiotics

Medical treatment o Hypertension with ACE inhibitors or ATII RA o UTI, cysts infection – usually GN bacteria therefore use Bactrim or fluroquinolones with

good renal tissue penetration o Pain treatment o Renal failure – medical treatment and RRT for those with ESRF o Antibiotic prophylaxis

Page 24: Mrcp short case telling skills

Surgical treatment o Pyocyst – drainage o Cystectomy o Nephrectomy o Alcohol sclerosant o RRT o Aneurysm clipping, MVP with MR

Brainstem Death & Organ Donation

Introduction (Introduce, clarify, comfortable, aim, what she understands) Discussion

(medical/legal/ethical, listen/summarise/advice, social/social, ICE)

Pick up from what relative understand about patient’s condition so far

Explain that o “No signs of improvement” o “Not regaining consciousness” inspite of sedation being turned off o “Reason is a large bleed in the brain” o “Irreversible brain damage”, “will never wake up” o “Reliant on the machine to breathe” o 2 Consultants not involved in the management for assessment

Tests Pronounced dead

o Patient has passed away o “Inappropriate for the team to continue ventilating a dead person” o Next step is to stop the machine and what would happen o Clarify that turning the machine off will not cause his death as he is already dead

Organ donation o Dead but “no damage to other organs” o “As doctors, it is our responsibility to discuss with you about organ donation for such

situations” o Clarify

Any objection from the deceased or patient has a signed donor card or in the donor registry

Any family objections Any religious objection

o If yes, not for organ donation o If no objections

Explain indications and benefits for other people Procedure

Transplant coordinator and team

They will speak to the family

Ensure no cancer or hep B/C

If agreeable, we will continue ventilation till organs are retrieved

Father will experience no pain as he is brainstem dead

Page 25: Mrcp short case telling skills

If need time to think,

“ I understand that you and your family is going through a difficult time”

“ I do not expect you to make a decision right now

“ I will be around the whole of today and we can meet up later” Closing

Summarise (skip)

Advice and assure (offer condolences)

Course of action (continue ventilation till we meet later; preparation of death procedures- time of deatn would be when the test was done this am)

Support o Provide a room for you and your family o A nurse, counsellor

Next appointment Discussion Breaking Bad News – Chronic Ilness

Introduction (3) o Introduce myself o Confirm her identity o Ask if she has brought a family member and if she wants him to be around

Opening statement (3) o State the purpose of the visit o How have you been since the procedure/last visit o What have the previous doctor explained to you/what she understands

Issues (4) o Note the tasks stated

Breaking bad news (BBN) Address patient’s concerns

o Consider all topics in ethics BBN, end of life, negligence, counselling, pregnancy, driving, religion, consent

and others eg angry pt o BBN

Warning shot Tell her the report/news Allow pauses; anticipate anger, denial and expecting the worst What she understands about report Explain simply eg “lymphoma is a type of cancer that is treatable” Give a Silver lining for the bad news (treatable, well established treatment,

notb al pts with bad Px) Allow pauses and reaction

Page 26: Mrcp short case telling skills

More detailed explanation if patient is ready o Address her concerns

ICE o Explain plans

Refer her immediately to specialist, “I am not the expert”, best available therapy

Ix plans Mx plans and treatment options Assure her – integrated center

o Her social support Family, trusted friend Work, Others eg Church

Closing (8) o Summarise management plan o Assure patient o Arrange another appointment to meet up in a few days’ time to answer queries o Bring a family member or friend next appointment o Offer contact no o Support group o Contact GP if she wants as she maybe comfortable with him

Arrange transport for her Conditions

Multiple sclerosis

Diabete Mellitus

Rheumatoid arthritis

Motor neuron disease Breaking Bad News – Terminal Illness eg Cancer

Introduction o Introduce myself o Confirm her identity o Ask if she has brought a family member and if she wants him to be around

Opening statement o State the purpose of the visit o How have you been since the procedure/last visit o What have the previous doctor explained to you o What she understands about the disease

Issues o Note the tasks stated

Breaking bad news (BBN) Address patient’s concerns

o Consider all topics in ethics BBN, end of life, negligence, counselling, pregnancy, driving, religion, consent

and others eg angry pt

Page 27: Mrcp short case telling skills

o BBN Warning shot Tell her the report/news Allow pauses What she understands about report Explain simply eg “lymphoma is a type of cancer that is treatable” Give a Silver lining for the bad news (treatable, well established treatment) Allow pauses and reaction More detailed explanation if patient is ready

o Address her concerns ICE

o Explain plans Refer her immediately to specialist, “I am not the expert” Ix plans Mx plans and treatment options Assure her – integrated center

o Her social support Family, trusted friend Work, Others eg Church

Closing o Summarise management plan o Assure patient o Arrange another appointment to meet up in a few days’ time to answer queries o Bring a family member or friend next appointment o Offer contact no as well as support group o Contact GP if she wants as she maybe comfortable with him o Arrange transport for her

Bronchiectasis

Presentation Sir, this patient has got bronchiectasis affecting both lower lobes as evidenced by late, coarse inspiratory crepitations heard best posteriorly in the lower one third bilaterally. Patient has a productive cough with large volume of purulent sputum with hemoptysis associated with clubbing. Chest excursion was reduced bilaterally with a normal percussion note and vocal resonance. Trachea is central and the apex beat is not displaced. There are no signs to suggest presence of COPD. (There is concomitant COPD with a reduced chest excursion bilaterally, hyperinflation of the chest associated with hyperresonance on percussion with loss of liver and cardiac dullness. There is presence of ronchi and a prolonged expiratory phase. Vocal resonance is normal. Trachea is central and apex beat is not displaced.) There is complication of pulmonary hypertension with a loud and palpable component of the second heart sound associated with a left parasternal heave. There is also cor pulmonale with a

Page 28: Mrcp short case telling skills

raised JVP of 3 cm with prominent a wave associated with bilateral pedal oedema. Clinically there are no signs of polycythemia such as plethoric facies or conjunctival suffusion. He is not in respiratory distress (with a RR of 14 bpm without use of accessory muscles of respiration). There are no signs of respiratory failure (he does not require any msupplemental oxygen and there is no central cyanosis; there is also no flapping tremor of the hands and no bounding pulse). There is also no nicotine staining of the fingers, patient is not cachexic looking and no enlarged Cx LNs. With regards to aetiology, there is no dextrocardia or a nasal voice to suggest possible Kartagener’s syndrome. In addition, there is no symmetrical deforming polyarthropathy to suggest RA or any cutaneous signs of SLE. There is no kyphoscoliosis. With regards to treatment, patient has a steroid metered-dose inhaler, salbutamol and ipratropium metered-dose inhalers by the bed side. I would like to complete the examination by looking at the temperature chart for fever as well as an abdominal examination to look for splenomegaly from amyloidosis which can result from bronchiectasis. A neurological examination is useful to screen for deficit as patients are prone to brain abscesses. In summary, this patient has bronchiectasis affecting both lower lobes with complications of pulmonary hypertension and cor pulmonale. There is no concomitant COPD and no polycythemia. He is clinically not in respiratory failure. The possible causes for this patient’s bronchiectasis are post infective causes such as post viral, bacterial, TB or ABPA, connective tissue disease such as RA or SLE, congenital conditions such as cystic fibrosis, Kartagener’s syndrome or hypogammaglobulinemia. Questions What are your differential diagnoses for a patient that is clubbed and has crepitations?

o Bronchiectasis o Pulmonary fibrosis o Mitotic lung lesion o Abscess

What is bronchiectasis? o Definition: permanent dilatation of the bronchi o Pathology: Retained secretions and chronic inflammation o Clinical course: Chronic, progressive with recurrent infective exacerbations o Clinical: Symptoms - productive purulent cough, dyspnea and hemoptysis and Signs:

coarse late inspiratory crepitations with a 3 layered purulent sputum What are the causes of bronchiectasis?

o Focal o Luminal blockage – FB, broncholith o Arising from the wall – mitotic lesion of the lung o Extrinsic – enlarged LNs esp middle lobe from TB/fungi; displacement of airways

post lobar resection o Diffuse

o Post infectious conditions Bacteria – Pseudomonas, Hemophilus, Pertussis TB

Page 29: Mrcp short case telling skills

Aspergillus (for upper lobe or proximal bronchiectasis) as in allergic bronchopulmonary aspergillosis from type III immune complex reactions.

Virus – adenovirus, measles, influenza o Congenital conditions

Cystic fibrosis Alpha 1 Antitrypsin deficiency Kartagener’s syndrome of immotile ciliary syndrome Hypogammaglobulinemia

NB: Immunodeficiency form secondary causes such as cancer, chemotherapy or immune modulation post transplant

o Rheumatic conditions RA (1-3% of patients) SLE Sjogren’s

o Others Yellow nail syndrome (yellow nails, bronchiectasis, pl effusion and

lymphedema) Young’s syndrome(secondary ciliary dyskinesia from mercury intoxication) Inflammatory bowel disease (UC or Crohn) Congenital kyphoscoliosis Idiopathic (50%)

What is bronchiectasis sicca? o “dry” bronchiectasis o Presents with recurrent hemoptysis and dry cough o Affects the upper lobes therefore good drainage o Usually from past history of granulomatous infection eg TB

What is Kartagener’s syndrome? o It is a type of immotile ciliary syndrome o Comprising of

o dextrocardia, situs inversus o bronchiectasis, sinusitis, frontal sinus dysplasia, otitis media o infertility

o Resulting in poor ciliary function with retained secretions and recurrent infections and thus bronchiectasis

What is cystic fibrosis? o Most commonly due to mutations to CFTR (CF transmembrane conductance regulator)

with F508 o Recurrent respiratory infections with pancreatic exocrine deficiency and short stature o Upper lobe involvement o Staph aureus, Ps aeuroginosa o Elevated sweat Na and Cl concentrations

What are the differences in bronchiectasis vs COPD? o They may both occur concomitantly

COPD Bronchiectasis

Page 30: Mrcp short case telling skills

Cause Cigarette Infection, genetic

Infection Secondary Primary

Organism S. pneumoniae, Haem Haem, Pseudomonas

Symptoms Dyspnea, chronic cough Dyspnea, hemoptysis, productive

Sputum Mucoid clear 3 layered, purulent

CXR Hyperlucency, hyperinflated

Airway thickening, dilated

What are the complications of bronchiectasis? Pneumonia, collapse, pleural effusion, lung abscess, pneumothorax, hemoptysis Brain abscess Sinusitis Amyloidosis How would you investigate? The diagnostic investigation of choice is a HRCT but simple Ix such as CXR and LFT are also useful:

o CXR – Diagnosis, extent and complications o 90% abnormal o Diagnosis

specific

dilated and thickened airways

Ring shadows (seen on end)

Tram lines Non-specific

Linear or plate-like atelectasis

Scattered irregular opacities

Focal pneumonitis o Extent and distribution o Complications

Pneumonia, abscesses, pleural effusion o Lung function test

o Obstructive pattern with FEV1/FVC <70% o Severity of obstruction based on FEV1 o Reversibility with beta agonist

40% of patients have >15% improvement o High-resolution computer tomography scan of the thorax

o Non-contrast study with 1 mm cuts every 1 cm with acquisition time of one second during full inspiration (requires patient cooperation); 90% sensitivity

o Diagnostic -Dilatation of airway lumen >1.5X cf to a nearby vessel

- Signet ring sign (dilated bronchus with its pulmonary artery) - Lack of tapering of an airway toward the periphery with presence

Page 31: Mrcp short case telling skills

of bronchi within 1 cm from the pleura - Reid’s classifications

Cylindrical or tubular

Varicose

Saccular or cystic -Useful also in elucidation cause of focal bronchiectasis

o Assess distribution Usually lower lobes If upper lobes – suspect Cystic fibrosis or ABPA If proximal bronchiectis, ABPA If ML or lingula – M. avium complex

o Complications How would you manage?

o Non-Pharmacological o Education and counselling o Stop smoking, vaccinations (yearly influenza and 3-yearly pneumococcal) o Chest percussion and postural drainage (no evidence actually)

o Rx underlying cause o Pharmacological

o Rx acute exacerbations o O’Donnell’s 4/9 symptoms of exacerbations

Increased dyspnea Increase cough Increase sputum production Increased wheezing Fever Lethargy, malaise Changes in chest sounds Reduced pulmonary function Radiographic changes consistent with a new pulmonary process

o Antibiotics targeting Haem, Ps and Strep and Moraxella Fluoroquinolones Others

MAC – Rifampiciin, ethambutol and Azithro till c/s negative for 1 year

ABPA – augmentation of corticosteroids and use of itraconazole 200mg bd for 4 weeks then 200mg om for 4 more weeks

o Bronchodilator therapy such as beta agonists and anticholinergics with inhaled corticosteroids

o Improve lung function (FEV1) and reduce sputum volume o No effect on mortality

Page 32: Mrcp short case telling skills

o Aerosolised recombinant human DNAse for cystic fibrosis (not for other causes of bronchiectasis)

o Surgery o Focal---Removal of obstructing tumour or FB o Diffuse

o Segments that are most damaged and contributing to recurrent acute exacerbations

o Segments involved with uncontrolled haemorrhage o Removal of segments suspected of harbouring drug resistant organism such

as MDR MTB or MAC o Lung transplant

How do you manage complication of hemoptysis? o Quantify

o If >600mls /day = massive o Lie on the affected side o Protect airway o Bronchoscope or CT to determine site of bleed o Interventional radiology or surgical removal

Brown-Sequard Syndrome e

Presentation Sir, this patient has a Brow- Sequard syndrome with a right hemisection of the cord at level T10. This is because Monoparesis of the right lower limb Reduced vibration and proprioception on the same side Loss of pain sensation on the contralateral limb below the level of T10 Loss of pain sensation on the right T10 dermatome There are no complications of DVT Pressure sores Charcot’s Incontinence Functionally, he has difficulty with ambulation as I notice a wheel chair at his bed side. The causes for a hemisection of the spinal cord at the level of T10 on the right are Trauma PID Spondylosis Tumor Abscess Multiple sclerosis Therefore, I would like to complete the examination Examining the back Per rectal examination for lax anal tone Multiple sclerosis – INO, optic atrophy with RAPD

Page 33: Mrcp short case telling skills

Central and Branch Retinal Vein Occlusion

Presentation Central Retinal Vein Occlusion Sir, this patient has got central retinal vein occlusion affecting the right eye as evidenced by grossly tortuous and engorged retinal veins, especially near the disc. There are also numerous haemorrhages seen scattered all over the retina of varying shape and sizes with presence of cotton wool spots (blood and thunder appearance). There is also papilloedema. There is no evidence of rubeosis irdis. The left eye is unaffected but has presence of hypertensive retinopathy grade 2 with silver wiring and irregular retinal arterioles associated with arteriovenous nipping. (Look for causes for in the other eye eg hypertensive or diabetic changes) I would like to complete my examination by

VA and Visual fields

Checking her BP

Urine dipstick for glycosuria

Urine for Bence Jones protein for multiple myeloma

Look for LNs and hepatosplenomegaly, bruising and purpura for macroglobulinaemia

Ask about h/o of glaucoma or use of OCPs (in females of reproductive age group) Branch Retinal Vein Occlusion Sir, this patient has a branch retinal vein occlusion affecting the superior/inferior temporal/nasal vein in his right eye as evidenced by a fan-shaped distribution of retinal flame-shaped haemorrhages with cotton wool spots which radiate from the arteriovenous crossings. These changes have affected the macula of the right eye. There is presence of hypertensive changes of grade 2 with silver wiring and irregular retinal arterioles with arteriovenous nipping. There is no exudates noted and the disc margins are sharp. I would like to complete my examination by

Checking his VA (normal unless affecting the macula) and VF (nomal or presence of a quandrantonopic fieled loss)

Blood pressure

Urine dipstick for glycosuria Questions What are the vessels that are involved in retinal vein occlusion?

CRVO – central retinal vein (occlusion is behind the cribiform plate)

BRVO – in front on the cribiform plate o Most commonly the superior temporal retinal vein o Followed by inferior temporal retinal veins o The nasal retinal veins

How do patients present?

CRVO

Diminished visual acuity – to counting fingers (incomplete loss)

Page 34: Mrcp short case telling skills

Reduced visual fields

Floaters

Acute secondary glaucoma 3 months later from rubeosis irdis from an ischaemic retina

Although retinal changes may improve in weeks, VA fails to be restored

BRVO

VA normal unless macula affected

Can have sudden visual loss if there is vitreous haemorrhage

Normal VF or monocular quadrantic field defect

Floaters How do you differentiate between CRVO and papilloedema?

CRVO has reduced VA whereas papilloedema affected only in late stages

CRVO unilateral involvement in contrast to papilloedema What are the usual sites of occlusions in BRVO

AV crossings

Along the main veins (DM)

Edges of the optic disc, resulting in hemispheric involvement

Peripherally as in sickle cell disease What are the causes of retinal vein occlusion?

CRVO o Hypertension o Diabetes mellitus o Glaucoma o Hyperviscosity syndrome

Waldenstrom’s macroglobulinaemia Multiple myeloma OCPs

BRVO o Diabetes mellitus o Hypertension o Need to differentiate with retinal vasculitis

How would you investigate? CRVO

BP

Fasting blood glucose

Screen for glaucoma

Screen for multiple myeloma (BJ proteins) and macroglobulinaemia

Enquire regarding OCPs and stop with alternate use of contraceptive methods BRVO

o BP o Fasting blood glucose, urine dipstick o ESR, CRP and ANA and ANCA for vasculitis

How would you manage this patient?

Page 35: Mrcp short case telling skills

CRVO o Identify causes and risk factors and correct or treat them o Fluoroscein angiography to identify areas of retinal ischaemia o Panretinal photocoagulation to prevent rubeotic glaucoma

BRVO o Identify and treat causes and risk factors o Fluoroscein angiography to identify areas of retinal ischaemia o Laser photocoagulation may be necessary to ablate damaged vessels if there is

macular edema or risk of vitreous haemorrhages from abnormal new vessels What is the prognosis?

CRVO o Mild to severe o Mild cases, asymptomatic o In severe cases, markedly reduced VA to counting fingers with complications of

neovascularisation of the retina, rubeosis iridis(1 month) and robeotic glaucoma (3 months)

BRVO o Mostly asymptomatic o Macula affected with VA loss but can recover o Occassionally leads to gradual visual loss

Central Retinal Artery Occlusion

Presentation Sir, this patient has a right sided CRAO on fundoscopy as evidenced by

RAPD

Pale retina

Foveola cherry red spot

Attenuated retinal vessels

Intra-arterial emboli (10-20%) I did not notice any laser scar marks to suggest laser panretinal photocoagulation and there are no complications of rubeosis iridis Complete the examination

Visual acuity

Pulse (AF), DM dermopathy, xanthelasma, BP

Palpate the temporal area for tenderness in GCA Questions What is the epidemiology of CRAO?

1 in 10 000

Male 2X

Unilateral in 99% How do patients present?

Sudden, painless visual loss (counting fingers to light perception)

10% has a history of amaurosis fugax – TIA affecting the retinal artery and lasting for minutes and classically described as a loss of vision in a curtain descending fashion

Page 36: Mrcp short case telling skills

Significant cardiovascular risk factors What are the causes?

Emboli (Hollenhorst plague = cholesterol embolus within the arteriole)

Arteritis (GCA, SLE, PAN) Why is there a “Cherry red spot”?

Ischaemia of the retina at the posterior pole renders the retina pale, white and milky; thus the choroid is seen through the fovea as a cherry red spot

How would you manage?

No effective treatment o Within 100mins, severe retinal cell injury o Total irreversible damage occurs 4 hrs after onset

Within 48 hrs, may attempt o Digital massage for at least 15 mins to dislodge any emboli o Administration of carbogen therapy (95% O2 and 5% CO2) o Medications to lower intraocular pressure (IV acetazolamide) o Anterior chamber paracentesis

Laser panretinal photocoagulation to reduce risk of progression to neovascular glaucoma

Manage cardiovascular risk factors o DM, hypt, lipids, smoking o Echo and USS carotids o Antiplatelets

What are the differential diagnoses for sudden painless loss of vision?

CRAO

CRVO

Retinal detachment

Submacular haemorrhage form age-related macular degeneration

Vitreous haemorrhage commonly from DM retinopathy

Cerebellar

Examination Stem statement

Giddiness, falls, unsteadiness

Face, Speech, ULs and LLs Unilateral

Upper limbs

Screen for pronator drift, ensure patient can see your finger!

Cerebellar signs

Dysmetria with intention tremor

Dysdiadochokinesia

Dyschronometria

Power for ataxic hemiparesisSensory

Temperature/Pain loss in syringomyelia and LMS

Tone for cogwheel and leadpipe rigidity

Page 37: Mrcp short case telling skills

Skin for neurofibromatosis Pulse for AF

Face

Gaze evoked nystagmus (in the direction of gaze), INO, RAPD

Speech (Count 1 to 20; British Constitution; West Register Street)

Cerebellar speech – jerky, explosive and loud; irregular syllables

CNs

CPA

LMS

III nerve palsy in Benedikt’s syndrome Xanthelesma

Lower limbs

Dysmetria and intention tremor for toe to finger test

Dyssynergia for heel-shin test

Dysdiadochokinesia for foot tapping test

DM dermopathy

Sit up with hands folded and tests for pendular jerks

Gait

Broad based gait with veering towards the side of the lesion

Request to test visual fields for hemianopia Bilateral

Upper Limbs

Cs of cerebellar (dysmetria, dysdiadochokinesia and dyschronometria)

Sensory – loss of temperature/pain for syringomyelia

Parkinsonism

NF features

Alcoholic features – dupytren’s contracture, stigmata of chronic liver disease

Face

CNs

Bilateral CPA tumor

Multiple sclerosis

Eyes

Gaze evoked nystagmus

KF rings

INO, RAPD

Mouth

Gingivial hypertrophy

Macroglossia

Telengiectasia

Parotidomegaly

Goitre

Speech

Page 38: Mrcp short case telling skills

Cerebellar speech

Hoarseness of voice

Lower limbs

Cerebellar signs

Clawing of toes (Friederich’s ataxia)

Sit – truncal ataxia and pendular jerks

Gait – cerebellar gait Presentation Unilateral Sir, this patient has a right sided unilateral cerebellar lesion as evidenced by presence of a right dysmetria, dysdiadochokinesia and dyschronometria of the right upper limb. The right lower limb also demonstrates presence of right dyssynergia on heel shin test, with right dysmetria and intention tremor on toe-finger test and dysdiadochokinesia. This is associated with a gazed evoked nystagmus on rightward gaze with a broad based gait with veering towards the right. I did not detect any cerebellar speech or any truncal ataxia. There are no associated cranial neuropathies. In particular there was no evidence of any cerebello-pontine angle lesion with CN V, VI, VII and VIII intact. (There are also no signs of neurofibromatosis such as neurofibromas or café-au-lait spots.) There is also no evidence of lateral medullary syndrome or III nerve palsy to suggest Benedikt’s syndrome. There is also no pronator drift on the right to suggest a right ataxic hemiparesis. Patient is in sinus rhythm and not in atrial fibrillation with no xanthelesma or diabetic dermopathy. There are also no bruises to suggest overanticoagulation. There are no signs of Parkinsonism to suggest presence of multiple system atrophy. There are also no associated features of multiple sclerosis such as RAPD or INO. I would like to complete the examination by looking at 1. The patient’s temperature chart for fever (abscess in posterior fossa) 2. Visual fields for a left sided hemianopia, which can occur with a right posterior circulation stroke 3. I would also like to do a fundoscopy for papilloedema for a SOL in the right cerebello-pontine lesion as well as for optic atrophy from demyelinating disease. In summary, this patient has got an isolated right cerebellar lesion. The differential diagnoses include cerebral vascular infarction or haemorrhage or a space-occupying lesions such as a mitotic lesion or an abscess. Bilateral Sir, this patient has bilateral cerebellar lesions as evidence of dysmetria with intention tremor bilaterally associated with dysdiadochokinesia. Similar findings were also present on

Page 39: Mrcp short case telling skills

examination of the lower limbs. There is also presence of multi-directional gaze evoked nystagmus associated with a cerebellar speech, truncal ataxia and a broad based gait. There is no evidence of bilateral CPA lesion with no CN V, VI, VII and VIII abnormalities. Patient is in sinus rhythm and not in AF with no xanthelesma or diabetic dermopathy. There is no evidence of KF rings to suggest presence of Wilson’s disease. There is also no RAPD or INO to suggest multiple sclerosis. There is also no gingival hypertrophy to suggest chronic phenytoin use. Patient has no goitre or features of hypothyroidism such as a cream and peaches complexion, no hoarseness of voice or macroglossia. There are also no features of chronic ethanol ingestion such as Parotidomegaly, dupytren’s contracture or stigmata of chronic liver disease. There is no associated Parkinsonism signs to suggest multiple system atrophy such as presence of cog-wheeling or leadpipe rigidity. There are also no neurofibromas present to suggest presence of NF type 2. Patient is also not cachexic looking and there is no clubbing to suggest underlying malignancy. I did not detect any telengiectasia to suggest presence of Ataxia telengiectasia and there is pes cavus to suggest Friederich’s ataxia. (Think of Wilson’s, MS, Phenytoin, Hypothyroid, Alcohol, Parkinsonism, NF, paraneoplastic, telangiectasia and FA) I would like to complete the examination by 1. Looking at the temperature chart for fever 2. Performing a neurological examination of the lower limb to look for spastic paraparesis 3. I would also like to do a fundoscopy for Optic atrophy, which may suggest demyelinating disease. In summary, this patient has bilateral cerebellar syndrome. Possible causes include drugs such as phenytoin, metabolic conditions such as hypothyroidism, chronic ethanol ingestion, paraneoplastic conditions and infection such as enteroviruses and bilateral cerebellar strokes. Questions What are the differential diagnoses for a unilateral cerebellar syndrome?

Isolated

Cerebrovascular accident – infarction or haemorrhage

SOL in posterior fossa – abscess or mitotic (primary vs secondary)

Associated

CN

CPA and/or neurofibromatosis

Lateral medullary syndrome

Jugular foramen (Arnold-Chiari or Dandy-Walker)

Benedikt’s syndrome

Ataxic hemiparesis (lacunar stroke)

Parkinsonism in Multiple system atrophy

Page 40: Mrcp short case telling skills

Demyelinating such as Multiple sclerosis What are causes of bilateral cerebellar syndrome?

Acquired

Infection

Viral – HIV, Enteroviruses

Spirocheatal – Lymes and Tabes dorsalis

Others – Toxoplasmosis and CJD

Metabolic

Wilson’s disease

Hypothyroidism

Drugs

Phenytoin and Carbamazepine

Lithium

Alcohol

Causes bilateral cerebellar signs

Causes isolated lower limb cerebellar signs

Affects the anterior vermis

Due to thiamine deficiency Multiple system atrophy Neurofibromatosis type 2 with bilateral CPA tumor Bilateral Strokes Paraneoplastic – Ca lung or ovary

Hereditary

Ataxia telangiectasia

Autosomal recessive

Childhood with death by 20s or 30s

Ataxia, choreathetosis and telengiectasia on the face, eras and conjunctiva and skin crease

Low IgA with recurrent chest infections and lymphoreticular malignancy

Friederich’s ataxia

Scoliosis, pes cavus

Spastic paraparesis, dorsal column loss, absent ankle jerks What are the signs of a midline lesion (cerebellar vermis) and what are the causes?

Signs : truncal ataxia, abnormal heel-toe walk test, cerebellar speech

Causes : Midline tumor, paraneoplastic What are the causes of cerebellar signs with spastic paraparesis?

Friederich’s ataxia

Spinocerebellar ataxia

Arnold-Chiari Malformation

Lesion at the craniospinal junction eg meningioma

Syringomyelia

Page 41: Mrcp short case telling skills

Multiple sclerosis

Syphilitic meningomyelitis how are cerebellar signs located?

Limb ataxia = cerebellar lobes

Gait ataxia = anterior vermis

Truncal ataxia = posterior vermis What are the differences between cerebellar and sensory ataxia?

How would you investigate?

Imaging – MRI brain

Blood tests according to the causes How would you manage?

Depends on underlying cause

Charcot’s Joint

Examination

Inspection

Skin

DM dermopathy, callus, ulcer, amputations, loss of concavity of the foot arch, loss of leg hair, shiny skin;

Leprosy skin changes, i.e. hypopigmented macules

Muscle - wasting

Bone - Joint involvement and stage of Charcot’s

Notice any nearby foot orthoses

Palpate

Joint for tenderness, osteophytes and crepitations, passive ROM for hypermobility

Feel for thickened nerves

Feel pulses

Sensory testing – pinprick, temperature; vibration and proprioception

Motor testing

Cerebellar Sensory ataxia

Site Cerebellar Posterior column, nerves

Tone Reduced Normal

Reflexes Normal or pendular Reduced

Sensory Normal Reduced

Sphincter disturbance

Nil Affected when posterior column involved; overflow incontinence

Page 42: Mrcp short case telling skills

Presentation Sir, this patient has a right ankle Charcot’s joint. The right ankle is enlarged and deformed with crepitus and hypermobility of the joint. It is not warm or tender. There is loss of sensation to pinprick in a stocking distribution and there is also loss of vibration and proprioception up to the ankles. In terms of function, I noticed the presence of an ankle-foot orthoses by the patient’s side. There is no muscle wasting to suggest disuse atrophy. The most likely aetiology is diabetes mellitus as evidenced by presence of DM dermopathy, with callus formation over the pressure points of the feet and loss of concavity of the foot arch. There is also loss of skin hair on the lower limbs and the skin has a shiny appearance. There is no hypoaesthetic, hypopigmented macules or palpable thickened nerves to suggest leprosy. To complete the examination

Stand the patient for Rhomberg’s sign, walk for functional assessment

Check the back for meningocele

Check the upper limbs for dissociated sensory loss

Examine the eyes for Argyll-Robertson pupils

Urine dipstick for glycosuria

Check for signs for chronic ethanol ingestion In summary, this patient has a right ankle Charcot’s joint secondary to diabetes mellitus and

able to ambulate with an ankle foot orthosis. Questions What is Charcot’s foot? It is a chronic, progressive, degenerative, neuropathic arthropathy resulting from a disturbance from the sensory innervation of the affected joint. What are the causes of a Charcot’s foot?

Diabetes mellitus (toes and ankles)

Leprosy

Alcoholic neuropathy

Tabes dorsalis (hips and knees)

Myelomeningocele

Syringomyelia (upper limbs eg shoulder)

Others – HSMN, congenital insensitivity to pain What are the stages of Charcot’s foot?

Atrophic form – usually forefoot with osteolysis of the distal metatarsal; X-ray shows MT resembling a pencil point

Hypertrophic form (mid or rear foot and ankle); Eichenholtz classification system: Stage 0 – Clinical stage with signs and symptoms but no joint deformity yet Stage 1 – Acute (Developmental or fragmentation stage)

-Periarticular fracture with joint dislocation with unstable deformed foot -Tender, red and swollen, mimicking infection (but afebrile, normal TW and good DM control and no break in skin) or gout

Stage 2 – Subacute (Coalescence stage)

Resorption of bone debris

Page 43: Mrcp short case telling skills

Stage 3 – Chronic (Reparative stage)

Restabilization with fusion of the involved fragments

Enlarged and deformed, non tender What is the pathogenesis of Charcot’s foot? 2 theories:

Neurotraumatic – loss of pain sensation and proprioception combined with repetitive and mechanical trauma to the foot

Neurovascular – Autonomically stimulated vascular reflex that causes hyperemia and periarticular osteopenia with contributory trauma

How would you investigate?

Establish the diagnosis

In acute stage

X-ray – to rule out OM; MRI or Indium scanning for infection

FBC for a normal TW; ESR or CRP normal in Charcot’s joint

Stage the Charcot’s joint from radiographs

Demonstrate loss of protective sensation of the foot

Semmes-Weinsten (10-g or 5.07 gauge) monofilament

Applied with just enough pressure to bend the monofilament

Positive if 4 out of 10 sites affected

Establish the cause--FPG for DM, VDRL How would you manage?

Education and counselling

Early Dx to prevent deformity

PT/OT – immobilization with total contact cast initially, paying particular attention to proper foot care and footwear; with ankle foot orthoses and custom made footwear

Medications--Symptomatic in the acute stages, bisphosphonates

Surgical – in chronic stage eg foot stabilisation procedures, exostosectomy

Chorea

(Beware the Parkinsonism with dyskinesia!) Approach

1. Introduce, sit the patient 2. Lift up hands

a. Involuntary athetoid movements/choreiform movements b. Choreic posture c. Dish spooning d. Pronator drift e. Milk-maid’s grip f. Look for wasting of the muscles and joint deformities g. Look for erythema marginatum and subcutaneous nodules h. Check for Parkinsonism i. SLE signs

3. Check for long tract signs especially if hemiballismus or one sided 4. Eyes

Page 44: Mrcp short case telling skills

a. KF rings b. Conjuctival suffusion c. Dysthyroid eye disease, nystagmus d. Plethoric facies e. Darting tongue f. Goiter

5. Walk the patient – effeminate gait, Parkinsonian gait Presentation Sir, this patient has chorea/athetosis affecting her left hand. I say this because of presence of brief, abrupt, irregular, quasi-purposeful movements of the left hand with writhing and twisting movements (athetosis). There is choreic posturing of the left hand with a flexed wrist and an extended mcpj; with dish spooning and milk maid grip, associated with darting tongue and an effeminate gait. There were no features of Parkinsonism to suggest that dyskinesia is secondary to L-dopa therapy. There was no evidence of erythema marginatum or subcutaneous nodules which can occur in rheumatic heart disease. There is also no cutaneous rash to suggest SLE. There is also no pronator drift. There are also no KF rings or nystagmus to suggest Wilson’s disease. There are no signs of polycythemia rubra vera as I did not notice any plethoric facies, conjunctival suffusion or pruritic scratch marks. There are also no goiter or thyroid eye signs. I would like to complete the examination by performing a cardiovascular examination to look for evidence of rheumatic heart disease, a mini-mental state examination for dementia as this occurs in Huntington’s chorea, as well as take a drug history of neuroleptics and L-dopa and a past history of encephalitis. Questions What are the different types of movement disorders that you know about?

Tremors

Resting tremor of Parkinsonism

Intention tremor of Cerebellar

Postural tremor of outstretched hands

Anxiety

Thyrotoxicosis

Alcohol

Drug induced – salbutamol, terbutaline, theophylline, Li

Drug withdrawal – BZD, opiates

Familial

Chorea (globus pallidus)

Athetosis

Hemiballismus (subthalamic nucleus)

Infarct

Others – abscess, tumor, MS, AVM

Search for CVS risk factors

Page 45: Mrcp short case telling skills

Rx – haloperidol, treat CV risk factors and Sx eg contralateral thalomotomy or pallidotomy

Orofacial dyskinesia

Secondary to antipsychotics usually, in pts with SZ

One of the 4 EPSE

Acute dystonia (oculogyric)

Parkinsonism

Akathisia (restless legs syndrome)

Tardive dyskinesia (or orofacial dyskinesia) What are the causes of choreathetosis?

CVA/tumors affecting the globus pallidus (Benedikt’s syndrome - III)

Metabolic – Wilson’s disease

Endocrine – Hyperthyroidism, post-hyperglycemia

CTDs – SLE

Polycythemia

Rheumatic heart disease – Sydenham’s chorea o Most recover within one month

Huntington’s Chorea

Drugs – neuroleptics, L-dopa, phenytoin, OCPs

Post encephalitis

CO poisoning What is Huntington’s disease?

Young adult, chorea and dementia

AD, Chr 4, CAG trinucleotide repeats

Chronic Liver Disease

Prensentation Sir, this patient has decompensated chronic liver disease with portal hypertension, splenomegaly and ascites. My findings include: Presence of an enlarged spleen that is palpable 3cm from the left costal margin. It is non-tender, firm in consistency, smooth surface, regular edge, notch border with no splenic rub. I am unable to get above this mass. The liver is not enlarged with a span of 12 cm in the right mid-clavicular line. The kidneys are not ballotable. There is presence of ascites with shifting dullness and this is not associated with tenderness.

Page 46: Mrcp short case telling skills

He is deeply jaundice and bruising noted on the ULs and LLs with presence of stigmata of CLD including leukonychia, clubbing, palmar erythema, spider naevi and gynaecomastia with loss of axillary hair. There is also presence of bilateral edema. Complications: He is cooperative with the examination with no flapping tremor to suggest hepatic encephalopathy. There are no enlarged Cx LNs and patient is not cachexic looking. There is also no conjunctival pallor noted. Aetiology: I did not find any parotidomegaly, dupytren’s contracture, tattoos, surgical scars or thrombosed veins. Treatment: I did not notice any abdominal tap marks but patient has sinus bradycardia, indicating use of beta-blockers. I would like to complete my examination by looking at the patient’s temperature chart for fever and a rectal examination for hard impacted stools or malena. In summary, this patient has decompensated chronic liver disease with portal hypertension, splenomegaly and ascites. There is presence of bruising, leukonychia, jaundice with no evidence of hepatic encephalopathy. A. The most likely etiology for this gentleman is 1. Chronic ethanol ingestion due to presence of parotidomegaly but no dupytren’s contracture;

presence of hepatomegaly 2. Chronic hepatitis B infection as patient has tattoos. 3. Chronic hepatitis C infection as I note an abdominal surgical scar with possibility of

transfusion in the past B. In the local context, the most likely underlying etiology is chronic ethanol ingestion, chronic hepatitis B and C. C. The most likely aetiology is chronic ethanol ingestion as I notice that this patient has presence of parotidomegaly. In view that there is also a hard irregular liver that is palpable, it raises the possibility of an underlying mitotic lesion of the liver. D. The most likely aetiology is 1. Primary biliary cirrhosis as she is a middle-aged lady with evidence of CLD with pruritus, xanthelasma and generalised pigmentation. 2. Hemochromatosis as he is a middle-aged gentleman with slate-grey appearance with presence of diabetic dermopathy. I would like to complete the examination by examining the CVS for CMP, urine dipstick for glycosuria and for small testes secondary to pituitary dysfunction.

Page 47: Mrcp short case telling skills

3. Wilson’s disease as the patient has a short stature associated with Kayser Fleisher rings of the eyes and tremor and chorea of the affecting the left upper limb. 4. Haemolytic anaemia (Thalassemia major/intermedius, Hereditary spherocytosis) as the patient has a short stature associated with hyperpigmentation and thalassemic facies with frontal bossing, flat nasal bridge and maxillary hyperplasia. I would like to complete the examination by examining the CVS for CMP, urine dipstick for glycosuria and for small testes secondary to pituitary dysfunction. Questions What is cirrhosis of the liver? Defined pathologically Diffuse liver abnormality Fibrosis and abnormal regenerating nodules What are the causes of liver cirrhosis? Chronic ethanol ingestion Viral hepatitis – B and C In UK, the risk for hep C is blood transfusion before Sept 1991 or blood products before

1986 Cardiac failure Others Autoimmune chronic active hepatitis (female) Primary biliary cirrhosis (female) Primary sclerosing cholangitis Haemochromatosis (male) Hemolytic disease Wilson’s disease Alpha 1 AT deficiency Galactosemia Type 4 glycogen storage disease Budd-Chiari (in malignancy- PRV or intraabdominal, AI, OCPs, IBD and PNH) Drugs – MTX (Look for RA or Psoriasis; Bx before starting MTX and bx every 1.5g

accumulated dose), amiodarone, isoniazid, methyldopa (MAMI) Cryptogenic

What are the complications of cirrhosis? (5) Portal hypertension

Ascites – Tense ascites, SBP Splenomegaly – thrombocytopenia Varices

Hepatorenal syndrome---Dx Cr Clr <40 Absence of other causes for renal impairment Absence of Cr improvement, proteinuria (0.5g/d), hematuria (<50/hpf) and urinary Na <10

Page 48: Mrcp short case telling skills

Type 1 = rapidly deterioration in renal fn ie doubling of serum Cr in < 2wks to >221 umol/l

Type 2 = stable or slowly progressive that does not mean criteria for type 1 Hepatic encephalopathy

Stages 1 – depression, euphoria, sleep disturbance, slurred speech; may have asterixis, normal EEG 2 – lethargy, moderate confusion; asterixis present; abnormal EEG 3 – marked confusion, arousable; asterixis present; abnormal EEG 4 – coma; abnormal EEG

Coagulopathy – low platelets and reduced clotting factors HCC How do you stage cirrhosis of the liver? Child-Pugh staging

o Consists of 5 parameters with score ranging from 5 to 15 o Prognosticate o 5 parameters ( 2 clinical and 3 Ix)

o Bilirubin (<34, 34-50, >50 umol/l) o INR (<1.7, 1.7-2.3, >2.3) o Albumin (>35, 28-35, <28) o Ascites (mild, moderate, severe) o Encephalopathy (absent, I and II, III and IV)

o A – 5-6 pts (1 year 100%, 2 year 85%) o B – 7-9 (1 year 80%, 2 year 60%) o C – 10-15 (1 year 45%, 2 year 35%)

How would you investigate? (Note the STEM STATEMENT) (5) Confirming the dxAbdominal USS or CT

Establishing the aetiology Hep markers, CAGE questionnaire, liver Bx in selected cases

Prognosticate LFT – Albumin, bilirubin.. INR

Complications Endoscopy of the upper GIT Mitotic change – USS and AFP Evaluation of renal function – urea, electrolytes and Cr Evaluation of ascitic fluid

Cell count Ascites albumin (SAAG) Gram stain and C/S Others – AFB smear and c/s, cytology

Evaluation for liver transplant o 5 year survival rate for cirrhosis with ascites is 30-40% vs 70-80% for post liver transplant o MELD score (Model for End Stage liver disease which has bilirubin, creatinine and INR) o Consider for those with refractory ascites, SBP or HRS

Page 49: Mrcp short case telling skills

When should an abdominal paracentesis be done for a patient with cirrhosis and ascites? Newly diagnosed to r/o SBP Symptomatic – fever, abdominal pain, encephalopathy, GI bleed How would you manage? (4) Education and counselling---Stop drinking alcohol, regular follow up Manage the underlying disease

o Hepatitis B General measures (stop alcohol, hep A vaccination) Lifelong surveillance for HCC with USS and AFP Antiviral for

Immune clearance phase( HBeAg +, ALT raised)

Reactivation phase ( HBe Ag -, ALT raised, HBV DNA raised) IFN alpha (SE : influenza-like; neutropenia and thrombocytopenia;

neuropsychiatric and unmasking AI disease) Lamivudine (well tolerated but YMDD mutant)

o Hepatitis C At risk are IVDAs and transfusion pre 1989 (Singapore) or pretransfusion

Sept 1991 or blood pdts before 1986 (UK) General measures Surveillance (HCC and screen for HIV) Indications

HCV RNA levels (>50 IU/ml)

Raised ALT

Bx showing fibrosis and inflammation Treatment

Peg interferon

Ribavirin o Alcoholic liver disease

>21u/wk in males and >14u/wk in females 100% of normal liver develops fatty liver 35% develop alcoholic hepatitis 20% develop cirrhosis 40% of alcoholic hepatitis develop cirrhosis Maddrey’s discrimination function

PT x Bil x 4.6

>32 = severe

Treat with corticosteroids or total enteral nutrition (20-30 kcal/kg/day)

o Others (see notes below) Manage the complications

o Hepatic encepholpathy Treat precipitants (see below)

Page 50: Mrcp short case telling skills

Prevent

Low protein diet

Lactulose o Hepatorenal syndrome

Treatment with

Noradrenaline infusion, telipressin or midodrine with octreotide plus

Albumin infusion (1g/kg on D1 then 20-40g/day)

For 5-15 days Prevention (in patient with cirrhosis and ascites)

IV albumin NB that hemodialysis does not help in this condition

o Ascites (see ascites) o Upper GI bleed

Secure VS Urgent endoscopy Operative Prevention

Propanolol to reduce HR by 25% or to 55-60 bpm

Variceal banding o HCC

Definitive treatment o Liver transplant o MARS (Molecular adsorbent Recirculating system)dialysis as an interim measure

before liver transplant What are the factors precipitating decompensation? Infection – SBP, pneumonia, UTI GI bleed Constipation Diuretics and electrolyte imbalance Diarrhea and vomiting Sedatives Surgery What are the nail changes of hypoalbuminaemia? Leukonychia, ie nail bed opacify indicating an albumin level <30g/dL; affecting the thumb

and index nails bilaterally initailly Muehrcke’s lines – transverse white lines What are the causes of palmar erythema? CLD RA, thyrotoxicosis and polycythaemia Pregnancy, normal finding What are the causes of anaemia in cirrhotic patients? Anaemia of chronic disease

Page 51: Mrcp short case telling skills

Fe deficiency from GI bleed Hemolysis from hypersplenism Folate and B12 from poor nutrition How many spider naevi should be present to be considered as significant? More than 5 When examining a patient with signs of chronic liver disease, think of:

Primary biliary cirrhosis

Clinical Female middle age CLD with pruritus, xanthelesma, generalised pigmentation, hepatosplenomegaly Stages Asymmptomtic with normal LFTs (positive Abs) Asymptomatic with abnormal LFTs Symptomatic – lethary and pruritus Decompensated

Commonly associated with sicca syndrome, arthralgia, Raynauds, Sclerodactyly and Thyroid disease

Ix Raised ALP, Anti-Mitochondrial Ab – M2 Ab, IgM Lipids Other tests for CLD Histology – Granulomatous cholangitis

Mx Symptomatic Urosdeoxycholic acid Cholestyramine Fat soluble vitamins Immunosuppression – Cyclosporin, steroids, AZA, MTX, tacrolimus, colchicines

Liver transplant

Hemochromatosis

Clinical Male Slate-grey appearance, hepatomegaly Affects Liver – cirrhosis and cancer Pancrease – DM Heart failure (CMP) Pituitary dysfunction Pseudogout

Therefore requests Urine dipstick, CVS examination and testicular examination

Autosomal recessive, HLA-A3, Ch 6 – HFE gene, increased Fe absorption with tissue deposition,

Page 52: Mrcp short case telling skills

Ix Raised ferrritin, transferrin saturation and liver Bx Mx

Non-pharmological Avoid alcohol Avoid shellfish as they are susceptible to Vibrio vulnificus

Venesection Dy/Dx of generalised pigmentation Liver – hemochromatosis in males and PBC in females Addison’s Uremia Chronic debilitating conditions eg malignancy Chronic haemolytic anaemia

Wilson’s disease

Clinical Short stature Eyes KF rings - greenish yellow to golden brown pigmentation of the limbus of the cornea

due to deposition of Cu in Descemet’s membrane at 12 and 6 o’clock position. Also occurs in PBC and cryptogenic cirrhosis

Sunflower cataract Extrapyrimidal Tremor and chorea Presents as difficulty writing and speaking in school

Pseudogout Penicillamine complications Myasthenic – ptosis Lupus – malar rash, small hand arthritis

Urinalysis for glycosuria from proximal RTA Autosomal recessive, Ch 13, increased Cu absorption and tissue deposition Ix Low serum ceruloplasmin, increased 24H urinary Cu Liver Bx – increased Cu deposition Mx Penicillamine

Ulcerative Colitis

Clinical o Skin – erythema nodosum, pyoderma gangrenosum o Joint arthropathy – LL arthritis, AS, sacroilitis o Aphthous ulcers o Ocular – iritis, uveitis and episcleritis

Page 53: Mrcp short case telling skills

o CLD – Cirrhosis, chronic active hepatitis, fatty liver PSC, Cholangiocarcinoma, metastatic colorectal cancer, amyloid

Cranial Nerves

Conforming

Superior Orbital syndrome (see VI nerve palsy)

Cavernous sinus syndrome (see VI nerve palsy)

Cerebellar Pontine Angle syndrome o Involvement

V1-3 (tinnitus and deafness earliest symptom then vertigo; loss of corneal reflex is the earliest sign)

VI VII VIII IX Cerebellar

o Causes Tumor

Primary o “Acoustic neuromas” – schwannomas of the vestibular o Meningiomas, haemangioblastomsa, medulloblastomas o Choleastoma

Secondaries o NPC ( loss of corneal reflex and V2 early) o Lymphoma

Aneurysm For bilateral lesions

Bilateral “acoustic neuromas” in NF type 2 o Examination

Examine CNs ULs for cerebellar signs Proceed to check neck for LNs Look for NF features (café au lait spots, neurofibroma, freckling and Lisch

nodules) o Presentation

Sir, this patient has righ/left CPA lesion as evidenced by There is no enlarged Cx LNs to suggests secondaries There is also no evidence of NF Possible etiologies includes

o Questions What is the CPA?

Shallow trangular fossa lying between the cerebellum, lateral pons and the petrous temporal bone

Histology?

Page 54: Mrcp short case telling skills

Schawannoma Ix?

Imaging – CT/MRI/Angio

Audiography

ENT to exclude NPC Mx

Microsurgical resection

Stereotaxic radiosurgery (Cx rate same as surgery)

Lateral medullary syndrome

o 5 vessels involved (wedge shaped infarction of the lateral aspect of the medulla and the inferior surface of the cerebellum) PICA Vertebral artery (most common artery that is involved) Lateral medullary artery (superior, middle, inferior)

o Areas affected Descending sympathetic fibres

Ipsilateral Horner’s syndrome

Ptosis, meiosis and anhidrosis Spinothalamic tract

Contralateral hemi-sensory loss of pain and temperature Descending tract and nucleus of V

Ipsilateral loss of pain and temperature of the face Nucleus ambiguus(X) and IX

Hoarsenss of voice, dysphagia, hiccups Vestibular nuclei

Nystagmus, vertigo, nausea Cerebellar (restiform body of the inferior cerebellar peduncle)

Ipsilateral ataxia and gait ataxia o Examination

CN examination Go to ULs for loss of pain and temperature and cerebellar Check for AF and DM dermopathy Visual Fields for homonymous hemianopia (posterior circulation)

o Sir this patient has right/left LMS as evidenced by State the findings Mention NG Aetiology – infarction affecting the vertebral artery or the PICA, LMA Did not find any xanthelesma or DM dermopathy, or AF Request for BP and asking patient on symptoms of dysphagia

Medial medullary syndrome

o Triad of XII, medial lemniscus and pyrimidal tract o Ipsilateral wasted tongue, contralateral loss of vibration and propioception and

contralateral hemiparesis respectively

Page 55: Mrcp short case telling skills

o Either vertebral artery or lower basilar

Bulbar palsy

o Bilateral involvement of LMN IX, X, XI and XII o Examination

Proceed with CN Do Jaw jerk Requests to examine speech, and gag reflex Requests to examine ULs for fasciculations and dissociated sensory loss

o Presentation Patient has bulbar palsy as evidenced by weakness of the soft palate, wasted

tongue with fasciculations a/w a nasal voice and a normal or absent jaw jerk o Causes (MGS, NNNP)

MND GBS Syringomyelia Poliomyelitis, NPC, neurosyphilis and neurosarcoid

Pseudobulbar palsy

o Bilateral UMN lesions of the IX, X and XII, V and VII (III/IV and VI are spared) o Examination

Proceed with CN Do jaw jerk Request for speech, gag reflex and enquire emotional lability Request for AF, DM dermopathy and xanthelesma Requests for ULs to look for UMNs

o Presentation Patient has PBP as evidenced by presence of sluggish palatal movement,

small, stiff and spastic tongue a/w brisk jaw jerk with “Donald duck” speech (slow, thick and indistinct)

No AF, DM or xanthelasma No evidence of mix UMN and LMN signs to suggest MND No RAPD or INO to suggest multiple sclerosis Possible causes (BMM)

Bilateral stroke

MND

Multiple sclerosis

Syringobulbia

o See syringomyelia o Extension of syrinx to involve the brainstem o V(descending tract of V), VII, IX, X, XI, XII and Horner’s syndrome o Usually unilateral

Jugular foramen syndrome

o Involvement of the IX, X, XI (XII maybe affected due to proximity) o Unilateral

Page 56: Mrcp short case telling skills

o Examination CN exams Proceed to check for enlarged Cx LNs And request to assess speech for husky voice and bovine cough

o Presentation Sir, this patient has right/left JFS as evidenced by Notice that this patient is on NG No enlarged Cx LNs Possible etiologies includes

o Questions Causes

Ca of the pharynx (commonest cause), tumor, neurofibroma

Basal meningitis

Paget’s disease, trauma

Thrombosis of the jugular vein IX, X and XI leaves the skull via jugular foramen (between the lateral part of

occiput and the petrous part of the temporal bone) XII leaves via the anterior condylar foramen Isolated XI implies injury to XI in the neck eg stab wounds

Non-Conforming

Myasthenia Gravis (see Myasthenia Gravis)

Miller Fisher Syndrome o Variant of Guillain Barre syndrome o Characterise by triad of ophthalmoplegia, ataxia and areflexia o Cs by anti G1Qb antibodies o Rare o Good prognosis with recovery beginning within 1 month of onset and complete

recovery within 6 months o Some maybe left with residual weakness and 3% will have relapses

Guillain-Barre syndrome

Mononeuritis multiplex

Migraine (paralytic)

Paget’s

Base of skull (trauma)

Basal meningitis

Brainstem strokes or multiple sclerosis

Collapse

Presentation Sir, this patient has a right upper lobe collapse as evidenced by reduced chest excursion of the right hemithorax associated with dullness on percussion and reduced vesicular breath sounds and reduced vocal resonance affecting the upper one third of the right hemithorax. This is associated with tracheal deviation to the right. There was no displacement of the apex beat.

Page 57: Mrcp short case telling skills

Patient’s respiratory rate is 14 bpm and is not in respiratory distress or failure. With regards to aetiology: There are signs to suggest underlying malignancy. Patient is cachexic looking and is clubbed. I did not detect any tenderness in the wrists to suggest HPOA. There were no enlarged palpable Cx LNs. There is also no conjunctival pallor or jaundice noted. I did not detect any associated pleural effusion or raised hemidiaphragm. There were no signs of SVCO, patient’s does not have a right Horner’s syndrome and there is no wasting of the intrinsic muscles of the right hand (hoarseness of voice for left sided lesion). In considering other etiologies: There are also no ronchi on auscultation to suggest asthma or allergic bronchopulmonary aspergillosis. There are no Mantoux testing detected on the upper limbs but endobronchial TB is a possible differential diagnosis. I did not find any signs of treatment such as radiotherapy hyperpigmentation or side effects of chemotherapy such as alopecia, phlebitic veins or oral ulcers. I would like to complete my examination by looking at the patient’s temperature chart as well as examining his sputum. In summary, this patient has a right upper lobe collapsed and is clinically comfortable. In view that patient has: 1. A history of weight loss and is cachexic looking, the possible diagnoses include endobronchial

mitotic lesion or an endobronchial tuberculous infection. 2. Fever/cough/hemoptysis/Cx lymphadenopathy, the possible diagnoses includes an

endobronchial tuberculous infection, endobronchial mitotic lesion or a collapse consolidation from a pneumonia.

3. Complications of Right Horner’s syndrome, signs of SVCO, clubbed with HPOA, the most likely cause is an endobronchial mitotic lesion affecting the right upper lobe.

Questions What are the causes of a lung collapse?

Intraluminal – Mucus plugging from asthma or ABPA, FB

Endobronchial tumor, TB

Extrinsic compression – enlarged LNs for mitotic lesion (pri or sec), lymphomas or TB What is Brock’s syndrome?-Collapse of the right middle lobe from enlarged LNs How would you investigate?

Simple investigations o CXR o ABG, FBC and biochemical profile o Sputum AFB smear and c/s and cytology

Diagnostic - Bronchoscopy and Bx

Page 58: Mrcp short case telling skills

Staging – CT thorax and abdomen with adrenal cuts, bone scan

Physiological staging: Lung function test o FEV1 >1.5 o Transfer factor>50%

How would you treat the patient?

Depending on the underlying cause

For mitotic lesion o Multidisciplinary approach o Education and counselling, support groups and stop smoking o Symptomatic treatment o For non-small cell

Assessment for surgical resectability

Staging (up to stage IIIA); ie once T4, N3 or M1 not a candidate

Physiological staging Chemotherapy

Neoadjuvant

Adjuvant Radiotherapy

Adjuvant

Palliative Palliative

Radiotherapy o Pain, bone mets o Dyspnea from bronchial obstruction, dysphagia o SVCO, pancoast syndrome

Chemotherapy o For small cell

Chemotherapy How does patient with bronchogenic carcinoma present?

Primary tumor--Cough, dyspnea, hemoptysis, pneumonia

Mediastinal spread o SVCO, Horner’s, pleural effusion, phrenic nerve palsy, hoarseness of voice, T1

wasting, pericardial effusion o Metastasis--Liver, bone, brain, skin, adrenal glands

Paraneoplastic symptoms o Systemic effects--LOA, LOW, fatigue

What are the paraneoplastic syndromes?

Endocrine o PTH-related peptide (hypercalcemia) – SCC o SIADH – Small cell (usually asymptomatic) o ACTH – Cushing’s (usually hypokalemic metabolic alkalosis) o Gynaecomastia

Neurological

Page 59: Mrcp short case telling skills

o Subacute cerebellar degeneration o Peripheral neuropathies o Lambert-Eaton syndrome

Cardiovascular--Non-thrombotic endocarditis

Renal-----Nephrotic syndrome, GN (membranous)

Skin o Migratory venous thrombopleblitis (Trosseau’s sign) o Acanthosis nigricans o Dermatomyositis o Zoster

MSK o Clubbing, HPOA

Haematological o DIC o Anaemia

What is SVCO?

Tumour with obstruction of the SVC

Plethoric facies

Facial and UL oedema

Conjunctival suffusion

Undersurface of the tongue with multiple venous angiomata

Fixed engorgement of the neck veins

Stridor

Upper chest telangiectasia

Radiation marks (NB think of polycythemia which also have plethoric facies)

Causes o Lung carcinoma, especially small cell o Lymphoma o Others – mediastinal goitre

Communiaction Skills and Ethics

5 mins 1. Read carefully 2. Identify medical issues, legal issues and ethical issues 3. Understand your task

12 mins

1. Introduce yourself (name and position) 2. Clarify patient’s relatives identity (if appropriate) 3. Position comfortably 4. Explain purpose of interview 5. Find out what he or she understands 6. Set to accomplish tasks and prioritise

Page 60: Mrcp short case telling skills

Medical issues

Ethical Issues

Legal Issues 7. While setting out to accomplish (6)

Empathy, apologise

Body Language

Listen, pauses

Layman terms

Summarise and feedback from patient

Offer support (MSW, support groups, glass of water, tissue paper) 8. Social History and support 9. ICE, feelings

Ideas

Concerns

Expectations

Feelings

Social support 2 mins

1. Summarise 2. Course of action and plan 3. Offer another appointment with the consultant and any significant other 4. Offer to give him or her your hospital pager

1 min 1. What do you think are patient’s problems? How did it go? Summarise.

a. Address medical, legal and ethical issues b. Identified patient’s or relative’s ideas, concerns, expectations and feelings c. Put patient’s or relative’s ICE and feelings at ease

2. How would you have done the interview differently? 3. What are the ethical and legal issues?

Communication and Ethics

Breaking Bad News o Chronic Disease o Terminal illness o Death o Postmortem

End of life issues o DNR o AMD o Withdrawal of treatment o Organ transplant and brainstem death o Persistent vegetative state o Euthanasia

Negligence

Page 61: Mrcp short case telling skills

o Misdiagnosis o Delay in diagnosis o Wrong medications

Counselling o Genetics o Non-compliance o Chronic disease o HIV testing o Others

Smoking cessation Insulin therapy Unrelated live organ donation HRT

Pregnancy and disease o DVT o Valve replacement o Pregnancy and pharmacy

Fitness to drive o Epilepsy and Neurological o CVS o Visual disorders

Consent o Consent for clinical trial o Consent for teaching o Consent for procedures o Consent from the mentally incapacitated/Capacity to make decision o Consent for emergency operation

Religion o Religious views on medical treatments o Religious views organ donation o Blood transfusions and Jehovah’s witness

Others o Colleague with hepatitis B o Needlestick injury o Informed refusal of therapy o Fitness to fly o Fitness for anaesthesia o Violent and abusive patient

Consent Consent for Procedure/treatment

Explanation of condition leading to the procedure, update condition

Page 62: Mrcp short case telling skills

Explain indications

Explain contraindications

Explain procedure details (pre, during and post)

Explain side effects and monitoring after that

Explain consequences of not doing the procedure

Explore concerns and expectations

Always check back with the patient if he/she understands

Offer options and option not to treat

Give time to consider Consent from a Mentally Incapacitated Patient/ Emergency

Same as above but note:

Verify patient’s relative status, is she the official proxy (In Scotland)

It is the responsibility of doctor to decide mental capacity o If limited, have to explain to patient still in simple terms

Best interest principle o Options considered and individualized o Patient’s opinion previously, advance statement

Autonomy overrides beneficence o Patient’s views from a third party, eg close relative o Necessity principle

Procedure to ensure improvement or prevent deterioration, safe patient life

Note that even in emergency, if patient has refused treatment previously which is life saving, doctors should not proceed

If discrepancy regarding patient’s capacity, engage senior psychiatrist opinion or “appointed medical practitioners” (in Scotland)

Advance directives o Patient has a right to refuse treatment o Patient can request for treatment but doctors are not obliged to follow o Only exception to requesting for treatment is that doctors should proceed with

CPR if patient insist even though doctors do not feel it is appropriate

Above applies for temporary incapacitation as in patient involved in RTA with ICH

If there is a conflict between relatives and doctor, plan is to o Speak to my consultant o Say that we acknowledge relatives and patient’s wishes o Ask her to speak to her other relatives o Make another appointment with consultant and significant others

Assessment of capacity – ReC test (Comprehend and retain treatment information, believe it, weight it up and arrive at choice)

Consent from Children (<16 years)

Page 63: Mrcp short case telling skills

In Scotland, if child is deemed to understand and make his/her own decision, the child can make decision, ie “Gillick competent”; this applies only to consenting and not to refusal of treatment

If there is discrepancy between child and doctors opinion in “best interest” principle, seek advice from the Courts

Parental consent can be taken if child cannot make consent o Just one parent’s consent is sufficient o Both parents must be married at the time child was conceived and born o If not married, only mother has parental rights

An adult is 18 years and older, but wrt consent, 16 and 17 year olds are considered as adult

For a pregnant female, she makes decision for both herself and the child even though decision may endanger the life of the unborn child

Consent for Clinical Trial

Update condition

Explain aim and purpose of trial and that he is a suitable candidate

Providing an option for him

Assure that it has been approved by Ethics committee

Explain trial o Method of treatment o Drug vs placebo o Blinding o Advantages o Side effects o Monitoring

Reassure o No change in standard medication and care of treatment whether or not he ants or

do not want to participate; treatment of complication o Others have participated in the trial o Autonomy – can opt out any time o Beneficience – trail may stop prematurely if there is an obvious danger/advantage o Confidentiality – assure this but sponsors and ethic committee and GP o Financial aspects of extra Ix and Mx

Give time to decide o Information and phone no o Obtain consent in writing

Consolidation

Presentation Sir, this patient has a right upper lobe consolidation as evidenced by reduced chest excursion of the right hemithorax associated with a dull percussion note, bronchial breath sounds and

Page 64: Mrcp short case telling skills

crepitations and increased vocal resonance. These signs were best heard in the upper one third anteriorly in the right hemithorax. The trachea is central and apex beat is not displaced. There are no signs to suggest that the patient is in respiratory distress or in failure. With regards to aetiology, an underlying malignancy is considered as there is associated complication of SVCO (hoarseness of voice if left sided), with plethoric facies, ruddy complexion a/w oedema of the face and upper limbs associated with suffusion of the eyes, fixed engorgement of the neck veins, dilatation of the superficial veins of the neck, and venous angiomata detected on the undersurface of his tongue. There is no Horner’s syndrome, wasting of the intrinsic muscle of the hands and no soft heart sounds to suggest pericardial effusion. There was also no associated pleural effusion or a raised right hemidiaphragm. He is also clubbed with HPOA and has nicotine staining of his fingers. He is cachexic looking with enlarged palpable cervical LNs. There is also thrombophiblitis of the forearms which may suggest Trosseau’s sign. (if there are no signs of cancer, proceed to mention TB/pneumonia ie mantoux testing, toxic looking, productive cough with purulent sputum; DVT ie swelling and tender calves) There is presence of radiation therapy marks on the right chest wall as well as side effects of chemotherapy such as alopecia and oral ulcers. In summary, patient has a right upper lobe consolidation complicated by SVCO. He is not in respiratory distress. The underlying cause is most likely a mitotic lesion of the lung. Question What are the causes of a consolidation?

Infection o Pneumonia o Abscess o TB o Aspergilloma, cryptococcoma, hydatid cyst

Neoplastic or mass o Carcinoma o Lymphoma

Pulmonary infarction How would you investigate?

Simple o CXR o ABGs, blood tests FBC and biochemical profile, blood c/s o Sputum

Directed tests o Infection o Cancer – Bronchoscopy and Bx, CT staging, bone scan, physio staging o Infarction

What are the causes of an unresolved pneumonia?

FB

Tumor

Page 65: Mrcp short case telling skills

Abscess

Inappropriate antibiotics, resistant organism

Bronchopulmonary sequestration o Rare, congenital o Non-functioning lung tissue with anomalous arterial supply with no connection to

the bronchopulmonary tree What are the extrapulmonary manifestations of Mycoplasma?

CNS – meningitis, encephalitis

CVS – percarditis, myocarditis

Hepatitis, GN

DIC, AIHA

EM, SJS

Arthralgia, arthritis What are the complications of pneumonia?

Local o Abscess o Empyema o Respiratory failure

Sepsis o Septic shock o ARDS o MOF o DIC

what are the pulmonary eosinophilic disorders?

Defined as radiographic infiltrates with hypereosinophilia (>1.5 x109L)

Includes o Churg-Strauss – Asthma with vasculitis and hypereosinophilia o Tropical pulmonary eosinophilia – high anti-filarial Ab o Chronic pulmonary eosinophilia – cough, progressive SOB, weight loss with

“photographic negative” pulmonary oedema ie diffuse peripheral pulmonary infiltrates

How do you manage pneumonia? BTS 2004 Depending on the type of pneumonia

o CAP o HCAP o HAP o note that HCAP is Rx the same way as CAP in BTS guidelines

Risk Stratify - CAP o CURB-65 score – 6 point score from 0 to 5

Confusion

Urea >7 mmol/L

Page 66: Mrcp short case telling skills

RR >30 bpm

BP <90/60

Age 65 and above o 0 to 1 – low risk of death and home Rx o 2 – increased risk of death and short hospitalization or supervised home

treatment o 3 and above – high risk of death and urgent hospitalization

Ix – FBC, CRP, Bld c/s (for those with severe indicators or co-morbidities), sputum g/s and c/s, Lg and Pneumococcal Urine Ag only for severe pneumonia Mx

o Use of SpO2 monitoring advocated in GP setting o NIV not for use in severe pneumonia unless in ICU setting o Antibiotics

CAP- Penicillins or macrolides; fluoroquinolones if intolerant or selected inpatient treatment with PO moxifloxacin preferred over levofloxacin

o Discharge planning

Should not be discharged within 24Hrs if have >1 of

T >37.8

HR>100

RR>24

SpO2<90%

BP sys <90

Inability to maintain oral intake

Abnormal mental status How do you mange pulmonary infarction? BTS 2003

All patients with suspected PE should have clinical probability assessed Hence evaluate patients for

o SOB or tachypnea, chest pain or hemoptysis o Absence of an alternative explanation (a) o Presence of a major risk factor (b)

Therefore if o (a) and (b) present = high probability o Only (a) or (b) present = intermediate probability o None present = low probability

D-dimer o To use only if low or intermediate probability o Highly specific = if negative, no need to do imaging o Not to order if high probability; image immediately

Imaging o Done within 1 hr for massive and 24 hrs for non massive o Imaging options

CTPA Isotope lung scan

Page 67: Mrcp short case telling skills

Facilities available

CXR normal

No cardiopulmonary disorder

Standardised reporting criteria

Non-diagnostic results followed by other imaging USS DVT

Can be used if presence of clinical DVT and if positive, is sufficient to confirm VTE

If negative, cannot be used to confirm absence of VTE Management Massive (BP compromised or cardiopulmonary collapse)

Thrombolysis with alteplase (No lowering BP effect cf with streptokinase)

Thrombus fragmentation and IVC filters if expertise present and available

Non-massive PE

NO thrombolysis

Use of heparin before imaging in high or intermediate clinical probability

Use LMWH as first choice

Use unfractionated heparin if o First dose bolus o Massive PE o Acute reversal desired

If VTE confirmed o Commenced oral anticoagulation o INR 2-3 o Duration is 4-6 wks for temporary risk factors. 3 months for

first idiopathic episode and 6 months for other clinical situations

Special populations o Pregnancy

- LMWH during pregnancy - UFH approaching delivery -UFH stopped or reduced 4-6hrs prior delivery -Oral anticoagulation

commenced after delivery and continued for 6 weeks or 3 months after PE whichever is longer

o Cancer patients-- -Oral anticoagulation as per above

-Duration unknown -Higher risk of thrombosis and bleeding If recurrent thrombosis

Page 68: Mrcp short case telling skills

INR 3-3.5 or LMWH + anticoagulation or IVC

COPD

Presentation Sir, this patient has severe COPD that is complicated by pulmonary hypertension, cor pulmonale and polycythemia. He is tachypneic at rest and requires use of intranasal oxygen supplementation. Patient has got hyperinflated chest with reduced chest expansion bilaterally at 2cm. The percussion note is resonant with loss of liver and cardiac dullness. There is prolonged expiratory phase with expiratory ronchi. Vocal resonance is normal. Trachea is central and apex beat is not displaced. There is complication of pulmonary hypertension as evidenced by loud and palpable P2 associated with a left parasternal heave. There is also cor pulmonale with raised JVP of 4cm with giant V waves associated with bilateral pedal oedema. There are also features of polycythemia with plethoric facies and conjunctival suffusion. The patient is in respiratory distress. He is tachypneic at rest with a RR of 20 bpm and uses his accessory muscles of respiration at rest. He is also in respiratory failure with presence of central cyanosis and is oxygen dependent. However, he does not have a flapping tremor or a bounding pulse to suggest CO2 retention clinically. In terms of aetiology, the presence of nicotine staining of his fingers implies significant history of smoking. He is not clubbed. The Cx LNs are not enlarged and he is not cachexic looking. There is presence of steroid MDI as well as bronchodilators by his side. There is no evidence of a hoarse voice or oral thrush or other features of chronic systemic steroid usage. I would like to complete the examination by testing patient’s forced expiratory time, checking his temperature and examining his sputum. In summary, this patient has got severe COPD with complications of pulmonary hypertension, cor pulmonale and polycythemia. He is in respiratory failure and respiratory distress. The most likely aetiology is smoking. How do you dx COPD?

Clinical (>35 years, smoking, wheeze, SOB, cough with sputum, winter bronchitis)

Airflow obstruction – FEV1/FVC<70 and FEV1<80 o Mild – 50-80% o Mod – 30-50% o Severe - <30%

Exclude differential diagnoses

Page 69: Mrcp short case telling skills

o Asthma (>400mls to dilators or PO pred 30mg OM 2 weeks or variation in PEFR >20%)

o Cancer o Bronchiectasis o ILD

How do you grade the severity of dyspnea?

MRC scale o 1 – SOB on strenuous exercise o 2 – on hurrying or up hill o 3 – walks slower than contemporaries and stops for breaths o 4 – stops for breath after walking 100m o 5 – SOB on ADLs

How would you investigate admitted with an acute exacerbation?

FBC (anaemia, polycythemia), biochemical, theophylline levels, ABG

Blood C/S if febrile

CXR

Spirometry

ECG, 2D echo How would you manage?

Non-pharmacological o Stop smoking o Regular follow up (if >500 mls decline over 5 years implies accelerated decline) o Pneumococcal and influenza vaccination o Pulmonary rehabilitation for MRC 3 or above(PT/OT) o MSW, Nurse o Assessment of inhaler technique (should not clean space more than once a month

due to increased static) Pharmacological

o Bronchodilators Beta agonist and anticholinergics Short acting and long acting Improves symptoms and exercise capacity

o Theophylline o Steroids

Reduce exacerbations and decline in health status Used if

FEV1<50%

2 or more exacerbations a year requiring antibiotics or steroids Prophylaxis against osteoporosis (once >65)

o Mucolytics (not anti-tussive) o Management of exacerbation

Bronchodilator - Nebs or via spacer Systemic steroids

Page 70: Mrcp short case telling skills

IV aminophylline Antibiotics such as macrolide (increase volume or purulence) Oxygen therapy

Intranasal if hypoxic

NIPPV if hypercapneic and pH 7.25-2.35

Intubation

Manage complications o Hypoxemia – assessment for need of LTOT

PaO2 <55 mmHg PaO2 <60 mmHg and presence of

Polycythemia

Pulmonary hypertension

Cor Pulmonale

Nocturnal hypoxemia For at least 15 hrs/day if not 20 hrs

o Cor pulmonale Diuretics

o Polycythemia >55% consider venesection

Surgical o Bullectomy

Single large bulla FEV1<50%

o LVRS Upper lobe bullous involvement FEV1>20% TLCO >20% PO2 <45

o Transplant When would you order a AAT levels?

COPD with o Young o Family H/O o No smoking history

Not recommended for AAT replacement if present; treat the COPD How would you advice on air travel?

FiO2 at 15% - hypoxemia

Pressurised at 8000 ft – pneumothorax

Assessment o H/O and PE and H/O of problems encountered during previous flight

Page 71: Mrcp short case telling skills

o Spirometry o SpO2 <95% on RA

50m walk test Hypoxic challenge test

Bring inhalers in the hand luggage

Inform the airline

See Link to Air Travel Advice

Counselling ---Genetics

Introduction o What she understands about the condition o Why she wants the test – must not be coerced o ICE

Clarify about the Index case – diagnosis and accuracy o Draw a family tree (parents, siblings, cousins, aunts and uncles) o Is she married o Is there a confidante (should not be someone who is also at risk)

Explain the condition (check what she knows and what she wants to know) o Nature – symptoms and signs o Treatment o Prognosis o AR, AD, X-linked

Explain procedure of test o Pre-test counselling, referral to regional genetic center o Test itself: Blood test, genetic make-up o False positive and false negative o Post test follow up, assignment of counsellor

Implications o Medical, worry o Social – job, marriage, children o Financial - insurance, social security o Careful as the test may reveal status of relative who does not want to know o Careful as the test may reveal other information such as parent may not be the

biological parent o May need DNA from a relative who may not want to do or know the test

Plan o Advice to make another appt in 1 month’s time to think about it o Options – not taking the test, deposit for research of future use o There is time – no need to make the decision now o Referral to regional genetic center if she wants to proceed with the test

-Multidisciplinary team consisting of a neurologist, geneticist, MSW, psychiatrist

Counselling - HIV Testing

Scenarios 1. Patient wants a test

Page 72: Mrcp short case telling skills

2. Patient referred to clinic for a possible AIDs defining illness 3. Screening – Pregnancy, blood donation, insurance

Introduction

The standard steps applies

For scenario 1, find out why he wants a test

For scenario 2, find out about his condition eg SOB and explain the results of Ix which led to possible diagnosis of HIV; then do an ICE on him

Task – 5 stages (explore patient’s ICE after each stage)

Explain nature of HIV infection o Transmission o Difference HIV and AIDs o How to reduce transmission

Explore his risk activities, last date of such a risk and perception of need of test o Sex o Drugs o Blood products o Occupational risk, tattooing, travel o Previous HIV test

Benefits and difficulties o Advantages

Patient – allow appropriate medical care, prophylactic care, future decisions, reducing anxiety about not knowing

Loved ones or others – subsequent sexual partners, spouse, vertical transmission

o Difficulties Anxiety Impact on family, partners and work Insurance Explore how he would take it if test is positive

Details of test o Test procedure – blood test, detection of antibodies to HIV, sensitive test and if

negative no retesting; if positive, will do a confirmatory test o Results ready within 24 hours o Confidentiality o Negative results – no Ab detected implying that no HIV or window period (usually 3

months but maybe longer; therefore repeat 3-6 month’s later if recent exposure) o Positive results – very rarely it is a false positive o Indeterminate – follow up blood test for direct virus testing and repeat Ab testing;

could be seroconversion period o Who he wants to inform him of test result (eg GP), and who he wants to be present o Who to share result with – regular sexual partner, GP o Post test counselling, support group, lealets, internet, phone no

Informed decision and consent form voluntarily

Page 73: Mrcp short case telling skills

Closing

Summarise

Advice and that he need not decide right away

Advice on current problem and treatment plan

Avoid transmission

Ask if he wants to screen for other STDs, Hep B, C and Syphilis

Special situation – pregnant mother o TOP (24 weeks in UK) o Prophylactic treatment with AZT to reduce vertical transmission o Avoidance of breast feeding

Counseling--Non-Compliance

Introduce and ask how he has been

Update him that his condition is not improving based on objective findings

Offer reason such as medications not taken properly, have you been having difficulty with the medications

Patient may confess and justify o Side effects o Inconveneience o Inability to hold the medications o Forgetfulness o Unsure the indications o Don’t know how to take the medications or the right dose, complicated o Not working and he feels fine o Cannot swallow, bad taste o Too many tablets

Patient may deny and say that everything is fine o Explain that you are concerned that he is not improving o Explain the test shows no improvement o Ask how much tablets he has left at home, red discoloration of urine with rifampicin

and that I am aware that he has not collected his prescription o Then explore above reasons

Acknowledge difficulty with long term medication and side effects, understand patient’ difficulties

Explain condition and importance of treatment

Plan o To solve the problems he had earlier attributed to, explain side effects o Eg DOT, 3X a week rather than daily, less tablets with combinational o Involve GP o Alternative employment because infectious risk

Counseling – Other Topics Smoking

Smoking history

Page 74: Mrcp short case telling skills

o how long, how many packets, type and brand o Increasing/decreasing o Pleasure, after work, when stress o Tried quitting, what happened then o Withdrawal symptoms o Illness – cough, dyspnea, CVS o FHx of smoking and illness

Social Hx o Married, children o Who he stays with and lives with o What are their opinion

Benefits of quitting o CVS, resp o Breathe easier, taste sharper o Financial o Good for children and loved ones o Many have been successful

Fears and offer solution o Withdrawal (restless, irritable and lack of sleep)

nicotine replacement (gum, patches, inhalers) and bupropion These double the chance of successfully quitting NRT start on quit date (2 weeks supply initially); have locaolised reation

and may have mild sleep disturbance Bupropion start 1 week before quit date (3 weeks supply initially); 0.1% of

seizures and 0.1% of severe hypersensitivity reaction; 3% has rash/urticaria/pruritus and c/o dry mouth and insomnia

Not for <18, breastfeeding or pregnant o Weight gain – up to 5 to 10 pounds in the first few months is expected but can

exercise and have healthy snacks

Any intention to quit, will to quit o No – can meet up again; phone no, leaflets o If yes, can refer to a NHS Stop Smoking Service

If not keen, then offer advice on quit date and nicotine replacemen/bupropion; anticipate likely failure triggers

If keen, then make the referral! Organ Transplant

The new Human Tissue Act in UK comes into effect on 1/9/06 o If someone who wants to donate, ie on the NHS Donor register or Donor Card

carrier or expresses wishes to donate organs upon death, relatives cannot object o Unrelated Live Organ Donation

Allowed from the 1/9/06 “paired” or “pooled” or “altruistic”

Page 75: Mrcp short case telling skills

Paired = 2 couples in which they are not compatible within but compatible with the other couple

Pooled = >2 couples

Alder Hey Scandal whereby organs of the dead were taken and stored without consent of the deceased or relatives

In UK, it is an “Opt In” system

Hormone Replacement Therapy

Informed decision

Types o Creams and gels (estrogen) which should not be used >3-6 months because

systemic absorption of estrogen o Tablets, patches, implants, nasal spray (estrogen), Mirena(IUD with

progesterone), vaginal ring with estrogen only and for hysterectomised patients and “bleed-free HRT”

o All women with uterus must have progesterone

Uses o For menopausal symptoms which normally lasts for 6 months to 2 years which

includes hot flushes, vaginal atrophy, frequency, urgency and dyspareuina, night sweats and irritability

o For premature menopause up to age 50 o Osteoporosis o Reduce risk of colorectal cancer

Risk of o CHD o CVA o PE/VTE o Breast Cancer, ovarian cancer, uterine cancer o Dementia

Side effects o Breast tender o Vaginal discharge o Nausea, weight gain, irregular bleeding, headaches o Progesterone can cause premenstraul tension, depression, ankle swelling and

jaundice o Increase size of gallstones

Alternatives o Homeopathic alternatives and use of phyto-estrogens

Compensation

For asbestosis, silicosis, coal worker’s pneumoconiosis, byssinosis and mesothelioma

Get form B1; MO from Social Security visit; compensation backdated 3 months except for mesothelioma; have up to 1 month to dispute degree of disability

Page 76: Mrcp short case telling skills

Cushing’s Syndrome Examination

Exposure – remove shirt Seated

Face

Round or moon facies

Plethora

Telangiectasia

Cataracts, anaemia

Oral thrush, buccal pigmentation

Hirsutism

Acne Neck

Supraclavicular fat pads

Dorsal hump or buffalo hump (interscapular fat pad) Upper limbs

Hands

clubbing, nicotine staining, hyperpigmentation

RA, SLE

Bruising

Papery thin skin (use 2 index fingers in a circular fashion)

Peripheral wasting of Uls

Proximal myopathy

Acanthosis nigricans Abdomen

Truncal obesity

Purple striae Lower Limb

Edema

Bruising Stand the patient up

Look for kyphoscoliosis

Palpate/percuss for tenderness (osteoporosis and vertebral collapse)

Ask the patient to squat and then stand up Request

BP

Urine dipstick

Examine the lungs for asthma and pulmonary fibrosis

Visual field assessment although majority of pituitary adenoma are microadenoma

Panhypopituirism

MEN type 1

Page 77: Mrcp short case telling skills

Presentation Sir, this patient has got Cushing’s syndrome. There is presence of moon-facies with facial plethora and telengiectasia. There is presence of hirsutism, acne, oral thrush and cataracts or conjunctival pallor. This is associated with supraclavicular as well as interscapular fat pad deposition. There is truncal obesity associated with purple striae. There is bruising of the skin and the skin is papery thin skin with proximal myopathy and lower limb edema. There is no evidence of acanthosis nigricans. There is kyphoscoliosis with tenderness of the spine. There was no clinical evidence of RA such as symmetrical deforming polyarthropathy or SLE. I would like to complete the examination by

Respiratory examination for evidence of asthma or pulmonary fibrosis

BP

DM

Ask history of exogenous steroid intake

Virilisation – deepening of voice, breast atrophy, clitoromegaly Questions What are the causes of Cushing’s syndrome? Rule of 90:10

90% exogenous and 10% endogenousof the 10% endogenous

90% ACTH dependent and 10% ACTH-independent (adrenal adenoma and carcinoma)

of the 90% ACTH dependent

90% are Pituitary(Cushing’s disease) and 10% are ectopic ACTH (bronchial carcinoid, small cell lung ca, pancreatic carcinoma, non-teratomatous ovarian tumor) of the Pituitary adenoma

90% are microadenoma

10% are macroadenoma ACTH independent

Adrenal adenoma

Adrenal carcinoma

Micro/macronodular adrenal hyperplasia

Part of Carney complex (pigmented skin lesions with endocrine and mesenchytmal tumors)

McCune Albright syndrome What are the causes of PseudoCushing’s? DOA

Depressions, drugs

OCPs, obesity

Alcoholism, acute illness What are purple striae? Purple striae are due to the weakening and disruption of the collagen fibres of the dermis leading to exposure of the underlying vascular tissue. They can be found on the abdomen, the upper arms and on the medial aspects of the thighs.

Page 78: Mrcp short case telling skills

What are the signs suggesting ectopic ACTH secretion?Absence of Cushingoid habitus, prominent edema and hypertension and marked muscle weakness. What are the features that suggest adrenal carcinoma?Virilisation in the female, gynaecomastia in a male and a palpable abdominal mass. What is the significance of hyperpigmentation in a Cushingoid patient? It implies that Cushing’s syndrome is due to ACTH excess due to presence of MSH like activity of the ACTH molecule. What is Nelson’s syndrome?

Nelson syndrome occurred formerly as a result of bilateral adrenalectomy for Cushing’s disease

Resulting in absent negative feedback of cortisol on the pituitary adenoma, with expansion of the pituitary adenoma with headache, bitemporal hemianopia and panhypopit eventually

Occurs in 20% of such patients in the past

Hyperpigmentation occurs due to melanocyte stimulating component of the precursor molecule of ACTH.

How would you investigate this patient?

Screen with

24H urinary cortisol or

overnight dexamethasone suppression test

1mg between 11pm to 12 midnight

Serum Cortisol at 8 am the following morning (>2mcg/dl)

Confirm diagnosis with a low dose dexamethasone suppression test

Determine the cause with

Plasma ACTH

High dose dexamethasone suppression test

Imaging studies (MRI pituitary or CT adrenals)

Others

CXR – if ectopic ACTH suspected

AXR – for adrenal calcification

CRH test (distinguish ectopic CRH vs Cushing’s disease)

Inferior petrosal sinus sampling (distinguish primary and ectopic source of ACTH when above tests are inconclusive)

How would you manage this patient? Treatment is directed at the primary cause of the syndrome:

Exdogenous steroids

Withdrawal if possible

If not possible

Monitor for complications and treat them

BP

DM

PUD

Osteoporosis

Endogenous

Page 79: Mrcp short case telling skills

ACTH dependent

Cushing’s disease

Transphenoidal hypophysectomy

Transfrontal hypophysectomy

Pituitary irradiation

Bilateral adrenalectomy with pituitary irradiation

Ectopic ACTH

Locate the source and treat appropriately

If unable to, adrenalectomy or medical therapy ACTH independent

Adenoma/carcinoma – unilateral adrenalectomy

Hyperplasia – bilateral adrenalectomy

Medical therapy only used if surgical not possible, eg metastatic adrenal carcinoma or ectopic ACTH; can use mitotane, ketoconazole, metyrapone, aminogluthithemde, trilostane and etomidate.

Dermatology Overview

Note: Number in brackets refer to the page numbers of “250 Cases In clinical Medicine” by R.R Baliga, 3rd Edition.

Generalised Maculopapular rash (389) Erythema multiforme (423) Steven Johnson Syndrome (423)

Dy/Dx – Staphylococcal Scalded Syndorme Purpura (see notes)

Henoch Scholein Purpura (399) Systemic Lupus Erythematous (417)

Bullous eruption (396) Dermatitis herpetiformis (456) (dy/dx scabies) Herpes zoster Herpes labialis

Urticaria (496) Vasculitis Urticaria pigmentosa (500)

Eczema (475, 484) Kaposi (460) Mycosis fungoides (498) Psoriasis (see notes)Dy/dx (mycosis fungoides and Bazex syndrome) Lichen Planus (see notes) Purple hue – dermatomyositis, sarcoid Erythroderma (see notes) Sezary syndrome (mycosis fungoides) Multiple lump NF Dercum’s disease

Multiple exostosis

Page 80: Mrcp short case telling skills

Gardner’s syndrome

Face

Alopecia (472) Scarring (infective/Discoid lupus, Lichen planus) Non-scarring (alopecia areata, totalis, universalis/telogen effluvium/male balding)

Acne (470) Rosacea (454) SLE (417) Dermatomyositis (see notes) Mitral stenosis Seborrheic dermatitis (493) Telengiectasia (403)

Sturge Weber (468) Peutz Jeghers (463)

Hereditary Haemorrhagic Telengiectasia (403) Shovlin Criteria (epistaxis, telangiectasia, visceral and AD) Dy/dx (CLD, AI, rosacea, lupus pernio,radiation)

Lupus pernio (435) Nose disfigurement Lupus pernio Lupus vulgaris Leprosy Rhinophyma

Xanthelesma (See Limbs) Hirsutism (428) Radiation marks (444)

Limbs

Ulcers of Lower Limbs (477,478)

Pyoderma gangrenosum (466)

Loose skin Pseudoxanthoma elasticum (451) (also has grouped papules) Ehlers Danlos (508)

Hyperextensible Joints Ehlers Danlos (508) Marfan (see rheumatology) MEN type 2B Klinefelters

Ichthyosis (401)--Inherited or malignancy(Breast, haematological)

Raynaud’s phenomenon (415)

Phleblitis migrans (421)

Erythema ab igne and livedo reticulais (426)

Bad nails Psoriasis

Page 81: Mrcp short case telling skills

Lichen planus

Alopecia areata (472)

Fungal (482)

Lipoatrophy (433)

Xanthelasma (439)

Tendon xanthomaa (446)

Eruptive xanthoma (448) (dy/dx- pseudoxanthoma elasticum, molluscum contagiosum 494)

Palmar xanthoma (450)

Tuberous xanthomas Shin lesions

Necrobiosis lipodica diabeticorum (442)

retibial myxedema (514)

Erythema nodosum (480)

DM dermopathy

Erythema ab igne

Livedoreticularis Melanoma (490)

Hypopigmentation Vitiligo (412) Post inflammatory

Pityriasis alba

Tinea versicolor

Dermatomyositis

Examination “Examine this patient’s skin/face/hands” “Look and proceed” “This patient has dysphagia, please examine her” (Similar to sun-exposed rash)

Face

Helitrope rash

Neck and shoulder – shawl sign

Weakness of neck flexion

Conjunctival pallor (associated with myeloproliferative or GI malignancies)

SLE or SScl for overlap syndrome Hands

Grottron’s sign

Vasculitis, capillary loops at the base of fingernails

Raynaud’s phenomenon

Calcinosis (usually in children)

SLE or SScl or RA for overlap syndrome Upper limbs

Elbows for rashes

Page 82: Mrcp short case telling skills

Tenderness of muscles

Test power, demonstrating proximal weakness

Loss of reflexes

Show no loss of sensory Knees for rash Request to screen for underlying mitotic lesions such as breast, respiratory and abdominal examination and screen for interstitial fibrosis.

PresentationSir, this patient has got dermatomyositis. There is the presence of heliotrope rash, which is a purplish-blue rash, around the eyelids and periorbital area and on the dorsum of the hands. This erythematous rash is also present on the neck and the shoulders, ie in a “shawl” distribution as well as on the sun-exposed areas. There is also involvement of the extensor surfaces of the elbow and knees. There is also periorbital edema. Examination of the hands reveals also presence of Grottron’s papules, which are flat-topped, violaceous papules over the dorsum of the knuckles and interphalangeal joints. The erythematous rash spares the phalanges. There is presence of nailfold vasculitis and telengiectasias. The cuticles are irregular, thickened and distorted. There is hyperkeratosis of the palms which resembles a mechanic’s hands. I did not notice any Raynaud’s phenemenon. There is also no calcinosis. There is tenderness of the muscles with proximal weakness. There is no sensory loss. There is also weakness of neck flexion. I did not detect any clinical features of Systemic sclerosis or systemic lupus erytthromatosis or rheumatoid arthritis to suggest an overlap syndrome. I would like to complete the examination by screening for any associated underlying mitotic lesion. There are also no features of chronic steroid use. Questions What is dermatomyositis? It is an idiopathic inflammatory myopathy with characteristic cutaneous findings. How do you diagnose DM/PM? 4 out of 5 criteria:

Progressive, proximal, symmetrical muscle weakness

Raised CK

Cs EMG findings

Cs findings on muscle Bx

Compatible dermatological findings What are the types of dermatomyositis?

Dermatomyositis

Polymyositis

Amyopathic dermatomyositis (no muscle involvement, just skin features) How do you classify? 5 Groups, Group 1 to 5 respectively

Idiopathic polymyositis

Page 83: Mrcp short case telling skills

Idiopathic dermatomyositis

A/w neoplasia

Childhood a/w vasculitis

A/w collagen vascular disease How would you investigate this patient?

Creatinine kinase levels – raised and reflects disease activity

ANA levels, anti-Mi-2, anti-Jo1

EMG – myopathic changes which are spontaneous fibrillations, salvos of repetitive potentials and short duration of polyphasic potentials of low amplitudes

Muscle biopsy – necrosis and phagocytosis of muscle fibres, with interstitial and perivascular infiltration of inflammatory cells.

Ba swallow – for atonic dilated esophagus (if stem statement states patient has dysphagia)

Other Ix to rule out malignancy (breast, lungs and GIT, ovaries) and mixed CT disease What is your differential diagnosis for myositis with raised CK levels?

Statin, chloroquine and colchicine What are some disorders associated with myositis?

Drugs

Infectious – Lyme’s disease, CMV

Eosinophilic myositis Outline your management.

Educate and counselling

Treat underlying malignancy

General meausures

Skin – sun avoidance and sunscreens

Muscle – bedrest, PT and OT, ST and bed elevation if dysphagia

Medical treatment

Steroid treatment (prednisolone 1mg/kg/day)

IVIG, methotrexate or azathioprine

Calcium channel blockers eg diltiazem for calcinosis What is the prognosis? Depends on

Presence of underlying malignancy

Severity of myopathy

Presence of cardiopulmonary involvement

Dextrocardia

Examination After the routine examination Request to examine the abdomen for a liver on the left side of the abdomen for situs inversus Presentation Sir this patient has dextrocardia as evidenced by:

o Right apex beat o Heart sounds that are better heard on the right than on the left

Page 84: Mrcp short case telling skills

The heart sounds are normal and there are no murmurs detected. Apex is not displaced located at the right 5th IC just medial to the midclavicular line and has a normal characteristic. She is in SR with a rate of 84bpm On examination of his lungs posteriorly, there was no evidence of coarse late inspiratory crepitations to suggest bronchiectasis and patient does not have a nasal voice to suggest sinusitis. (Katargener’s syndrome) There is no evidence of Turner’s syndrome. (mention this only if female!) I would like to complete my examination examining the abdomen for a left sided liver for situs inversus. In summary this patient has got dextrocardia and is well clinically and is of congenital etiology. Questions What is the significance of situs inversus in patients with dextrocardia?

o It usually implies that there is no significant cardiac malformation What conditions is dextrocardia associated with?

o Kartagener’s syndrome – a type of immotile ciliary syndrome o Triad of

Bronchiectasis Sinusitis, otitis media and dysplasia of the frontal sinuses Infertility

o Turner’s syndrome o Asplenia – PBF may show Heinz bodies and Howell-Juoly bodies

What is situs inversus? o Right sided apex and right descending aorta o Left lung having 3 lobes and right lung with 2 lobes o Left sided liver and right sided stomach o Right descending colon

What is dextroversion? o Right sided apex and left sided descending aorta o Left sided stomach

What is levoversion? o Left sided apex and right sided descending aorta o Right sided stomach

Diabetic Retinopathy

Presentations (Can still use the older classification) Sir, this patient has: (a)Type and location

1. Background diabetic retinopathy affecting the inferior/superior temporal/nasal quadrants of the retina (usually seen in the posterior pole, ie area between the superior and inferior temporal quadrants) as evidenced by

a. Microaneurysms b. Dot, blot or flame shaped haemorrhages c. Hard exudates

Page 85: Mrcp short case telling skills

2. Preproliferative diabetic retinopathy affecting the inferior/superior temporal/nasal quadrants of the retina

a. Cotton-wool spots b. Venous dilatations, beading, looping or segmentation

3. Proliferative retinopathy diabetic retinopathy a. Neovascularisation

i. At the disc ii. Affecting the inferior/superior nasal/temporal quadrants of the retina

4. Diabetic maculopathy a. Circinate formatiuon of hard exudates at or near the macula b. Macular edema (cannot be seen by direct ophthalmoscopy)

(b)Treatment for (3) and (4) 1. Focal photocoagulation scars 2. Panretinal or scatter photocoagulation scars 3. Macular photocoagulation scars

(c)Complications (for proliferative) 1. Vitreous haemorrhages 2. Fibrosis with traction retinal detachment 3. Optic atrophy (for all)

(d)Associations (mention if present) 1. Xanthelesma 2. Cataracts 3. Hypertensive changes 4. Robeosis irdis

Questions What are microaneurysms?

They are well-defined red dots seen in the superficial retinal layers which represents outpouching of the retinal capillaries; earliest sign of diabetic retinopathy

Can also be seen in o Hypertensive retinopathy o Collagen vascular disease o Severe anaemia o Dysproteinaemia

What are flame-shaped haemorrhages?

Superficial bleed shaped by nerve fibres into a fan shape which points towards the disc What are dot and blot haemorrhages?

Formed as a result of rupture of microaneurysms with bleeding into the deep layer of the retina

How do you differentiate between dot haemorrhages and microaneurysms?

This is difficult and differences includes:

Microaneurysms are well defined and last for months to years whereas dot haemorrhages tend to be have an irregular outline and disappears within a few days

Page 86: Mrcp short case telling skills

Fluoroscein angiography of which microaneurysms are hyperfluoroscent whereas dot haemorrhages are hypofluoroscent

What are hard exudates?

These are minute, yellow, well defined deposits of lipo-protein and lipid-laden macrophages

What are cotton-wool spots?

Build up of axoplasmic material due to interrupted flow caused by ischaemia from capillary occlusion in the retinal nerve fibre layer

What are IRMAs?

It stands for intraretinal microvascular abnormalities. They are remodelled capillary beds without proliferative changes and are collateral vessels that do not leak on fluoroscein angiography. Usually found on the borders of non-perfused retina

What is neovascularisation?

Formation of abnormal new vessels on the retinal surface and at the optic disc as a result of ischaemia

These are fragile and tend to bleed into the vitreous leading to vitreous haemorrhages and fibrous tissue formation with resultant traction retinal detachment

What is clinically significant macular edema?

Thickening of the retina at or within 500 microns of the centre of the macula

Areas of thickening 1 disc area or larger, any part of which is within 1 disc diameter of the centre of the macula

Hard exudates at or within 500 microns of the center of the macula, if associated with thickening of the adjacent retina

What is the pathogenesis of diabetic retinopathy?

Earliest stages are characterised by increased vascular permeability, leading to fluid accumulation in the retina (seen by leakage of fluoroscein dye into the vitreous humor)

Later there is vascular closure causing retinal ischaemia leading to neovascularisation of the retina

These new vessels are prone to complications of vitreous haemorrhages, fibro-proliferative changes, retinal detachment and neovascular glaucoma

How can diabetes mellitus affect the eye?

Eyelids – xanthelasma (association)

Extraocular – mononeuritis multiplex, diabetic third (spares the pupils and associated with headache; resolves within 3 months) or sixth nerve palsies

Anterior chamber – neovascular/rubeotic glaucoma

Iris – rubeosis irdis

Pupils – Argyll Robertson pupil, RAPD

Lens – cataracts(higher incidence and occurs at a younger age), refractor errors (occurs due to fluctuation in the blood sugar level especially when starting treatment; it is a benign condition)

Vitreous body - haemorrhages

Retina – DM eye changes, hypertensive, CRAO, lipaemia retinalis

Optic nerve – optic atrophy, ischaemic papillitis

Page 87: Mrcp short case telling skills

Orbit – mucormycosis How can patients present clinically?

NPDR – asymptomatic

PDR o Asymptomatic o Reduced VA or blindness as a result of complications

CSME o Asymptomatic o Reduced in VA o Paracentral scotoma o Decrease in central vision

What conditions can cause blindness in diabetic eye disease?

Macular edema

Retinal detachment

Vitreous haemorrhages

3.6% of type 1 and 1.6% of type 2 were legally blind according to the WESDR When should we screen for diabetic eye disease?

Type 1 DM – within 3-5 years of Dx of DM

Type 2 DM – at diagnosis

Pregnancy with pre-existing DM – prior conception and first trimester How should we screen for diabetic eye disease?

Fundal photography

Indirect ophthalmoscopy with slit-lamp biomicroscopy

Direct ophthalmoscopy through dilated pupils How frequent should patient’s be followed up?

No retinoapthy – annually

NPDR o Mild and no retinal edema – 6 to 12 monthly o Presence of retinal edema – 4 to 6 monthly o Macula affected or severe – 1-4 monthly

PDR – 1 to 4 monthly How soon must you refer a patient with diabetic eye disease to the ophthalomologist?

All patients with DM retinopathy needs a referral to an ophthalmologist

Immediately (1 day) o Sudden loss of vision o Retinal detachment

Urgently (within 1 week) o Neovasculariastion o Pre-retinal or vitreous haemorrhages o Rubeosis irdis

Soon (within 4 weeks) o Pre-proliferative changes

Page 88: Mrcp short case telling skills

o Macular diabetic changes o Unexplained drop in VA

What are the risk factors for diabetic eye disease?

Poor glucose control o WESDR (Wisconsin Epidemiologic study of DR) o DCCT (Diabetes Control and Complications trial)

Hypertension o UKPDS (UK prospective diabetic study) o Aim < 130/80 mmHg

Hyperlidaemia

Renal disease – aggressive treatment of renal disease may slow progression of DM retinopathy and prevent neovascular glaucoma

Cigarette smoking

Duration of diabetes (non-modifiable) o After 20yrs, nearly all patients with type 1 and 60% of type 2 will have DR

Pregnancy (non-modifiable) How do you manage patient’s with diabetic eye disease?

management of the diabetic eye includes: o mPrompt referral to the ophthalomologist (see above) o Macular edema – focal or grid macular laser o NPDR – none or consider scatter laser if severe o PDR – scatter laser o Vitreous surgery

Non-resorbing vitreous opacities Traction retinal detachment threatening or involving the macula Progressive fibro-proliferative diabetic retinopathy Combined rhegmatogenous and traction retinal detachment

Pay attention to o Glycaemic control o Blood pressure o Quit smoking o Screen for other DM complications especially DM renal disease o Control hyperlidaemia

Engage patient o Education o Importance of regular follow up

How would you manage a patient who requires laser therapy but has cataracts?

If fundal visibility permits, laser treatment administered prior to cataract surgery

If not, cataract surgery followed by prompt laser treatment how effective is laser photocoagulation?

For visual salvage in maculopathy o Effective in 50-60% of cases

For reduction of already formed abnormal new vessels on the retina

Page 89: Mrcp short case telling skills

o It can abolish new vessels in up to 80% of patients with PDR and follow up showed that disease had stabilised or cured

o In pan-retinal photocoagulation, it reduces the ischaemic and hypoxic retina, reducing angiogenic factors and neovascularisation

o It can alsobe used to treat microaneurysms

There may be loss of peripheral vision

Dystrophia Myotonica

Approach to Congenital Myopathies 1. Duchenne’s, Becker’s 2. Myotonia

Dystrophia myotonica (fascioscapular dystrophies can mimic appearance)

Congenital myotonia

Hereditary paramyotonia 3. Fascioscapulahumeral dystrophies, limb-girdle dystrophies, distal myopathies Examination Examine patient’s face or hands (Can be short case of locomotor or in CNS station) Examine the hands

Demonstrate difficulty opening hands after shaking

Repeatedly open and close the hands

Percussion myotonia of the thenar eminence

(proceed with hand examination with function assessment if locomotor station)

Demonstrate weakness in the forearms (especially) and hands

No sensory loss

Loss of reflexes

Check the pulse (dysrhythmias, small volume pulse) Examine the face

Myopathic facies

Expressionless

Triangular facies

Wasting of the temporalis, masseter (palpate these muscles when patient clenches teeth)

Frontal balding

Bilateral ptosis

Close his eyes and open

Tongue for percussion myotonia

Gum hypertrophy from phenytoin toxicity

Swan-neck appearance with wasting of the SCM (test for weakness of SCM), weakness of flexion of the neck

Nodular thyroid enlargement Request

Face

Cataracts - posterior subcapsular and stellate

Assess Speech – slurring due to myotonia of the tongue and pharyngeal muscle

Page 90: Mrcp short case telling skills

Chest examination

Gynecomastia

Cardiovascular examination – dilated cardiomyopathy (split S1, mitral murmur, low BP and pulse volume)

Testicular atrophy

Urine dipstick for diabetes mellitus

Lower limbs – bilateral footdrop Presentation Sir, this patient has got dystrophia myotonica as evidenced by

A myopathic facies that is triangular in appearance with an expressionless look. There is wasting of the facial muscles involving the temporalis and masseter muscles associated with frontal balding and bilateral ptosis. He had difficulty opening his eyes after firm closure. There was myotonia affecting the tongue.

There is also a swan-neck appearance with wasting of the sternocleidomastoid muscles with weakness of flexion of the neck. On shaking his hand, there was a delay in releasing his grip. In addition, after making a fist, he was unable to quickly open it especially after doing this repetitively. There was also presence of percussion myotonia of the thenar eminence. There is presence of proximal myopathy and wasting with involvement of the forearms and hands. There are also reduced reflexes with no sensory loss detected. Function is relatively preserved.

With regards to complications

His pulse is regular at 80 bpm with a small volume pulse suggesting dil CMP

There was no gum hypertrophy to suggest chronic phenytoin use.

There is nodular thyroid enlargement. I would like to complete my examination by

Face

Cataracts - posterior subcapsular and stellate

Assess Speech – slurring due to myotonia of the tongue and pharyngeal muscle Chest examination

Gynecomastia

Cardiovascular examination – dilated cardiomyopathy (split S1, mitral murmur, low BP and pulse volume)

Testicular atrophy

Urine dipstick for diabetes mellitus

Lower limbs – foot drop with high steppage gait (tibial nerves are affected early) Questions What are the types of muscular dystrophies you know of? 1. Duchenne’s

Sex linked

Pseudohypertrophy of the calves or deltoids

Gower’s sign, proximal weakness

Cardiomyopathy

Page 91: Mrcp short case telling skills

Becker’s

Sex linked

Later onset and less severe form of Duchenne’s 2. Limb-girdle

Autosomal recessive

Shoulder and pelvic girdle affected

Third decade

Sparing of the face and heart Fascioscapulohumeral

Autosomal dominant

Bilateral, symmetrical weakness of the facial and SCM with bilateral ptosis

Weakness of the shoulder muscles and later the pelvic girdle muscles 3. Dystrophia myotonica Congenital myotonia Hereditary paramyotonia 4. Distal myopathies eg Welander’s myopathy What is myotonia? Continued contraction of the muscles after voluntary contraction ceases, followed by impaired relaxation. What is dystrophia myotonica?

Characteristic clinical appearance with myotonia and weakness with no sensory loss

Autosomal dominant with a trinucleotide (AGC) repeat disorder on chromosome 19

Anticipation – phenotypic expression worsens with each successive generation

Onset in the 3rd or 4th decade

Males>females

In addition to the characteristic facies and musculoskeletal involvement

Intellectual and personality disorder

Cataracts – posterior subcapsular cataracts which are stellate type

CVM – dilated cardiomyopathy and conduction defects

Resp – recurrent infection from weakness of the bronchiolar musculature, hypoventilation and post-anaesthetic respiratory failure

Abdomen – dysmotility and dysphagia

Testicular atrophy and gynecomastia

Diabetes mellitus

Nodular thyroid enlargement How would you investigate?

Confirm Diagnosis

EMG – dive bomber pattern ie waxing and waning of the potentials

Muscle biopsy shows no inflammatory changes with type 1 fibre atrophy which is characteristic but not diagnostic

DNA analysis

Muscle enzymes are normal Screen for Complications

Page 92: Mrcp short case telling skills

FPG – screen for diabetes mellitus

ECG – heart blocks, small P, prolonged PR, notched QRS and prolonged QTc

CXR – enlarged heart

Slit-lamp examination for cataracts How would you manage?

Education, genetic counselling

PT/OT – eg foot orthosis for foot drop

Medications – phenytoin for myotonia, other anti-myotonic medications such as quinine and procainamide should be avoided due to aggravation of cardiac conduction defects; however it is the weakness that causes disability and not myotonia

Pacemaker for 3rd degree heart block or symptomatic such as syncope How would you counsel the patient’s family?

Vertical Autosomal dominant, children 1 in 2 Anticipation DNA analysis is available for some families for prenatal diagnosis

Horizontal Screen with clinical examination Slit-lamp examination EMG

What are the other types of myotonia disorders? Myotonia congenita (Oppenheim’s disease)

Autosomal dominant or recessive Presence of myotonia without other features of dystrophia myotonica Present at infancy with difficulty feeding with subsequent improvement No weakness and reflexes are preserved Herculean appearance Channelopathies

Hereditary paramyotonia Autosomal dominant Cold-induced myotonia

what are your differential diagnoses for dystrophia myotonica? Facies appearance – Facioscapulohumeral dystrophy

Autosomal dominant, onset at age 10-40, Chr4, normal lifespan Face – ptosis, difficulty closing eyes, facial weakness and speech impaired Normal IQ Neck – wasted SCM and weakness Shoulder – winging of scapula, weakness of pectoralis, trapezius, biceps and triceps and hypertrophy of deltoids Occasionally affecting the anterior tibialis Normal CK

Proximal weakness – FSH, limb-girdle, prox myopathy causes, MG limb Girdle dystrophy (see prox myopathy)

Page 93: Mrcp short case telling skills

Distal weakness – Welander’s distal myopathy, nerve problem Myotonia – Congenital myotonia, hereditary paramyotonia

Flaccid Paraparesis

Examination

Complete the LL examination

Commonly

HMSN

Polio

(infantile hemiplegia)

Spina Bifida

Cauda Equina Syndrome

GBS/CIDP

MND (see spastic paraparesis)

Diabetic amyotrophy (See proximal myopathy)

Concentrate on

Ataxia – Miller Fisher Variant, Tick Paralysis

Sensory

No sensory abnormalities

Myopathies

Neuromuscular

Nerves – certain conditions eg GBS, multifocal motor neuropathy

Anterior Horn Cell

Glove and stocking

Peripheral neuropathy

HMSN, paraneoplastic

Mild and patchy = GBS

Sensory level (Acute)

Cord compression

Cord infarction

Transverse myelitis

L5 and S1 sensory loss in spina bifida Typical features of HMSN

Pes cavus, clawing of toes, contractures of Achille’s tendon, inverted champagne bottles (wasting of distally and stops abruptly at the lower one third of thighs; also similar distal wasting distally in the ULs)

LMN – reduced tones and no clonus, reduced reflexes and downgoing plantar response, weakness, bilateral footdrop

Sensory – no sensory or mild glove and stocking

Gait – high steppage gait of foot drop

Marked deformity with minimal disability

Others

Feel for thickened nerves (lateral popliteal nerve)

Page 94: Mrcp short case telling skills

Examine the hands for small muscle wasting and clawing

Examine spine for scoliosis

Feel for thickened Greater Auricular nerves

Wheelchair, calipers Examine

Back

Kyphoscoliosis

Spina bifida – scars, tuft of hair, dimples, sinus or naevus

Per rectal examination

Saddle anaesthesia and cauda equina syndrome

Incontinence – fecal and urinary

Upper limbs

CNs- fatiguibility, GBS (bilateral VII)

Functional aids Presentation

Obvious disease

HMSN

Sir, this patient has got HMSN/CMT as evidenced by

Bilateral pes cavus with clawing of toes and distal wasting of the lower limbs with a inverted champagne bottle appearance; there is hypotonia with reduced reflexes and downgoing plantar responses a/w weakness of the lower limbs of power 4/5 with bilateral foot drop; there is no associated sensory disturbance; she has a high steppage gait form bilateral foot drop and is able to walk independently inspite of the marked feet deformity; I also noticed presence of wasting and clawing of the upper limbs; there is no palpable thickened lateral popliteal nerve.

I would like to complete my examination by examining the spine back for scoliosis and palpate for other sites of thickened nerves

Mention walking aids or wheelchair Polio

Sir this patient has monoparesis of the right LL most likely due to polio

A shortened right lower limb associated with wasting. It is hypotonic with reduced reflexes and downgoing plantar response and is flaccid with a power of 3/5. There is no sensory weakness.

There is no UMNs or shortened wasted right UL to suggest infantile hemiplegia

Examination of the back did not reveal any cutaneous signs of spina bifida.

Mention any walking aids/wheelchair Not so obvious

Sir, this patient has got flaccid paraparesis as evidenced by

Presence of hypotonia with reduced reflexes a/w with downgoing plantar responses bilaterally; I did not detect any fasciculations. There is weakness of the LLs with a power of 3/5. There is no associated cerebellar signs in the LLs and no sensory loss to pin prick, propioception and vibration.

Complete my examination

Page 95: Mrcp short case telling skills

Back

Per rectal

ULs for ataxia, flaccid paresis

CNs for cranial neuropathies Questions What are the causes of flaccid paraparesis?

Acute myopathies

Inflammatory myopathy (polymyositis, dermatomyositis)

Rhabdomyolysis (extreme exertion, drugs, viral myositis, crush injury etc.)

Acute alcoholic necrotizing myopathy

Periodic paralyses (hypokalemic, hyperkalemic)

Metabolic derangements (hypophosphatemia, hypokalemia, hypermagnesemia)

Thyroid or steroid myopathy

Neuromuscular

Myasthenia gravis

Botulism

Tick paralysis

Other biotoxins (tetradotoxin, ciguatoxin)

Organophosphate toxicity (can also cause neuropathy)

Lambert-Eaton Myasthenic Syndrome (LEMS)

Nerve

Diphtheria

Porphyria

Drugs & Toxins (arsenic, thallium, lead, gold, chemotherapy – cisplatin / vincristine)

Vasculitis (incl. Lupus, polyarteritis)

Paraneoplastic and Paraproteinemias

Multifocal motor neuropathy

Nerve roots

Guillian Barre Syndrome

Lyme disease

Sarcoidosis

HIV

other viruses (CMV, VZV, West Nile)

Cauda equina syndrome (lumbar disc, tumour, etc.)

Plexus lesions (brachial plexitis, lumbosacral plexopathy)

Anterior Horn Cell (motor neuron diseases):

Amyotrophic lateral sclerosis (ALS) – with UMN findings

Poliomyelitis

Kennedy’s disease (spinobulbar atrophy / androgen receptor gene)

other spinomuscular atrophies (inherited)

Anterior spinal artery syndrome (with grey matter infarction)

Spinal Cord (corticospinal tract diseases):

Page 96: Mrcp short case telling skills

Inflammatory (Transverse myelitis)

Subacute combined degeneration (B12 deficiency)

Spinal cord infarction

other myelopathies (spondylosis, epidural abscess or hematoma

Brain

Pontine lesions (eg. Central pontine myelinolysis, basis pontis infarct or bleed)

Multifocal lesions (multiple metastases, dissemination encephalomyelitis [ADEM], multiple infarcts or hemorrhages – eg. DIC, TTP, bacterial endocarditis)

What is Charcot Marie Tooth disease? Hereditary sensory motor neuropathy Consisting of 7 types of which types 1,2 and 3 are the most common types

Type 1 – A demyelinating neuropathy, aut dominant, absent tendon reflexes, enlarged nerves; Chr 17

Type 2 – An axonal neuropathy, aut dominant (mild and present later), normal deep tendon reflexes, nerves not enlarged; Chr 1

Type 3 – rare, hypertrophic neuropathy of infancy, thickened nerves, aut recessive (Dejerine Sottas disease)

Physical findings

Above plus

Others – optic atrophy, retinitis pigmentosa and spastic paraparesis Ix

Rule out other causes of neuropathies

EMG/NCT

Biopsy

Genetic testing Mx

Eductaion and counselling and family screening

PT/OT, AFOS

Medical Rx – pain relief, avoid obesity

Surgical treatment Px

Normal life expectancy

Disease usually arrest in middle life

Disability varies Dy/Dx of hereditary disease

Hereditary amyloidosis

Refsum’s disease – accumulation of phytanic acid

Fabry’s disease – deficiency of alpha galactosidase What is poliomyelitis?

Enterovirus, picorna virus, with IP of 5-35 days, oro-fecal route or contaminated water, 3 serotypes

Page 97: Mrcp short case telling skills

Replicate in the nasopharynx and GIT and then to lymphoid tissue and then hematological spread with predilection to the anterior horn cells of the spinal cord or brainstem with flaccid paralysis in spinal or bulbar distribution

4 forms

Inapparent infection

Abortive – nauseas, vomiting and abdominal pain

Nonparalytic – above plus meningeal irritation

Paralytic – paralysis and wasting; bulbar or spinal distribution

Occasionally, can get postpolimyelitis syndrome which results in weakness or fatigue in the initially involved muscle groups 20-40 years later

Ix

Viral c/s from stool, throat and CSF

Antibodies Mx

Educationa and counselling

Non-medical

PT/OT

Care of limbs Medical

Rx complications

Pain

Respiratory failure

Clear bowels Prevention

Inactivated polio vaccine – Salk vaccine which is administered parenterally

Oral live vaccine – can result in poliomyelitis in immunodeficient individuals Dy/Dx

Spina bifida

Infantile hemiplegia – hypoplasia of the entire side of the left side with UMN sign on the affected side

What is Spina Bifida?

Incomplete closure of the bony vertebral canal with similar anomaly of the spinal cord

Usually in lumbosacral region, can also involve the cervical region and is associated with hydrocephalus

Look for

Scars, tuft of hair, dimples, sinus, naevus, lipoma

Asymmetric LMN signs of LLs

L5 and S1 dermatomal sensory loss

Bladder involvement

X-ray: sacral dysgenesis, laminar fusion of the vertebral body. Scoliosis

Page 98: Mrcp short case telling skills

Multifactorial aetiologies, with folic deficiency and use of Na Valproate, siblings with spina bifida has higher risk

Prevented with use of folic acid early in pregnancy

Can be tested with amniotic serum AFP, serum AFP or USS What is cauda equina syndrome?

The cauda equina refers to the nerve roots that are caudal to the termination of the spinal cord; any lesion below the 10th Thoracic vertebrae

Low back pain, unilateral or bilateral sciatica, saddle anaesthesia, bladder and bowel disturbances and variable motor and sensory LL abnormalities

Causes – trauma, PID, spondylosis, abscess, tumor (ependymoma and NF)

Anatomy

Spinal cord starts from the foramen magnum to the level of L1 vertebrae

Add 1 to Cx vertebrae

Add 2 to Tx vertebrae 1-6

Add 3 for Tx vertebrae 7-9

T10 and T11 vertebrae = lumbar segments

T12 and L1 = sacral and coccygeal

Conus medullaris = T9 to L1 vertebrae

Conus Medullaris Cauda equina

Presentation

Acute Chronic

Reflexes Knees preserved; ankle absent

Both knees and ankles absent

Motor Spastic para; symmetrical

Flaccid para; asymmetrical

Sensory More LBP, less radicular

Less LBP, more radicular

Sensory Perianal Saddle

Impotence

Frequent Less frequently

Sphincter Occurs early Occurs late

What is Guillain Barre Syndrome?

Auto immune, antecedent Campylobacter infection

Bimodal – young adults or the elderly

Motor, sensory and autonomic dysfunction

Progressive ascending muscle weakness, variable patchy sensory loss, hyporeflexia and autonomic disturbances such as tachycardia and labile BP

Post GI or resp infection, 2-4 weeks of onset of symptoms which may progress over hrs to days and recovery over months; complicated by respiratory failure

Subtypes

Acute inflammatory demyelinating neuropathy

Page 99: Mrcp short case telling skills

Acute motor axonal neuropathy

Acute motor-sensory axonal neuropathy

Miller Fisher Syndrome (Ataxia, areflexia and ophthalmoplegia; antiGQ1b Ab)

Acute panautonomic neuropathy Ix

CSF shows albuminocytologic dissociation (<10 mononc cells and high prot)

AntiGQ1B Ab in MFS and anti GM1 implies poorer Px

NCT – demyelination

FVC (15-20ml/kg < or NIF 25cmH2O<) = may require ventilation Mx

Emergency

ABCs, and pacing maybe required

IVIG

Plasma exchange

Steroids

PT/OT

Prevention Cx – DVT prophylaxis Px

Most 85% will have full recovery by 6-12 months

Complications from respiratory failure and cardiac dysrythmias and labile BP

Footdrop

Approach Bilateral

LMN -Peripheral neuropathy (see peripheral neuropathy) UMN- Cord lesion

Unilateral Once dorsiflexion impaired

Check eversion (Common peroneal nerve = dorsiflex and eversion)

Check inversion and plantarflex = posterior tibial nerve

If foot drop and inversion and eversion is lost with normal plantarflexion, then L5 nerve root

If all gone = posterior tibial+common peroneal, sciatic nerve or plexus/roots Knee flexion intact

Go to sensory

Peripheral neuropathy

Common peroneal nerve palsy (sensory loss over dorsum of the foot)

Determine if common peroneal nerve or

Deep branch only or

The superficial branch only If knee flexion weak, test hip abduction and internal rotation and intact

Go to sensory

Sciatic nerve

Page 100: Mrcp short case telling skills

If hip abduction and internal rotation is weak

Go to sensory

Nil = anterior horn cell

L4 and L5 dermatome = plexus or root

Once site is located, go for the cause

Note walking aids Questions

Common peroneal nerve palsy (L4 and L5)

Anatomy

the sciatic nerve divides at the popliteal fossa into the tibial and common peroneal nerves

The posterior tibial nerves effects plantar flexion and inversion of the foot

The common peroneal nerves winds round the neck of the fibula, covered by s/c tissue and skin only and prone to extrinsic compression

It then divides into the

Superficial branch: foot everters and sensation to lateral calves and dorsum of the foot

Deep branch : toe dorsiflexors and dorsiflexion of the ankle and sensation to the first interdigital web space

Therefore wasting of the peroneous and anterior tibialis muscles; weakness of dorsiflexion of the foot and eversion; foot drop and high steppage gait and loss of sensory over the lateral aspect of the calf and dorsum of the foot

Causes of mononeuropathy (3 Sx and 3 Medical causes)

Trauma

Surgical

Compression at the neck of the fibula (habitual leg crossing, cast, brace)

Infection – Leprosy

Inflammatory – CIDP

Ischaemic - Vasculitis

Part of mononeuritis multiplex (Endo, AI, infection, infiltrative and cancer) Ix = NCT and EMG Mx

PT/OT – 90 degrees splint at night

Sx – for severed nerve or excision of ganglion

Sciatic nerve (L4 L5 S1 S2)

Weakness of the knee flexion also

Knee jerk is intact but ankle jerks affected and plantar response absent (for common peroneal nerve, all reflexes intact)

L5 nerve root

Weakness of hip abduction and internal rotation as well as loss of foot inversion (cf with common peroneal nerve)

Gait Assessment

Page 101: Mrcp short case telling skills

“Examine this patient’s gait”

Procedure o Stand and Rhomberg’s o Walk and turn and return o Heel to toe o Heel walk o Tip toe o Squat

Look for the obvious o Ankylosing spondylitis o Chorea o Hemiplegic gait o Antalgic gait

Small paces o With stooped posture and paucity of arm swing

Parkinsonian o With upright gait, normal armswing

March a petit pas Diffuse cerebrovascular disease

Feet separation o Broad

Cerebellar-

Unilateral or bilateral + high stepping = Sensory ataxia

Peripheral neuropathy or dorsal column loss o Crossing over

Scissoring gait Spastic – cerebral palsy, MS, cord compression

High stepping with normal feet separation o Unilateral or bilateral footdrop

Pelvis rotating o Waddling gait

Proximal myopathy or congenital dislocation of the hips

Apraxic gait – disjointed o Frontal lobe – CVA, SOL, hydrocephalus

Bizarre in consistent = functional gait, Huntington’s chorea

Gaze Palsies INO

Examination (example right INO) o Cs

Abduction of the left eye with nystagmus a/w failure of adduction of the right eye on leftward gaze

Page 102: Mrcp short case telling skills

The right eye is able to independently adduction Saccadic eye movement – horizontal saccade is abnormal with the right eye

lagging behind the left eye o Lesion is in the

Pons – convergence is intact Midbrain – convergence is lost

o Proceed with other CNs examination Multiple sclerosis (RAPD) Myasthenia gravis

o Limbs Multiple sclerosis – cerebellar signs CVA – DM dermopathy, xanthelasma, AF

o Request for fundoscopy (optic atrophy)

Presentation o Sir this patient has a right INO as evidenced by Cs o The lesion is in the midbrain (anterior INO) or pons (posterior INO) o Evidence for MG o Evidence of MS o Evidence for CVA

Questions o What causes a right INO?

Lesion in the right medial longitudinal fasciculus that affects connects the ipsilateral third nerve innervation to the right medial rectus to the left gaze center (parapontine reticular formation ie PPRF)

o What are the causes of INO? Multiple sclerosis Brainstem infarction Pontine glioma Infections

Lyme’s disease

Syphilis

Viral Drug intoxication (phenothiazines, TCAs, phenytoin, CMZ) Trauma

o How would you Investigate? As above etiologies (MG, MRI, FPG, lipids, lyme titre, VDRL, drug)

o How would you manage Mx of Multiple sclerosis Mx of infarction and risk factors

Typically resolves with time WEBINO (Walled-eye Bilateral INO)

Bilateral INO with exotropia and failure of convergence

Lesions in the pons and midbrain

Page 103: Mrcp short case telling skills

Due to multiple sclerosis, vascular, gliomas and Wernicke’s Fisher’s one-and-a-half syndrome

INO and ipsilateral gaze palsy

Due to lesion in the MLF and adjacent gaze center Conjugate upward vertical gaze palsy

Midbrain lesion

MS, vascular, tumor Conjugate downward vertical gaze palsy

Midbrain or

Foramen magnum o Arnold-Chiari, Dandy Walker o Acquired lesions (tumor, vascular, demyelination, abscess)

Supranuclear gaze palsy

Progressive suprnuclear gaze palsies (see Parkinson’s disease) o Loss of saccadic(frontal lobe) and pursuit movements (Occipital lobe) o Loss of downward gaze, then upward gaze then horizontal gaze o Can be overcome by Doll’s reflex

Parinaud’s syndrome o Loss of vertical gaze, nystagmus on convergence, PseudoArgyll-Robertson pupils o Causes – Ms, vascular, pinealoma

Goitre Examination

Look and proceed, Look at the eyes/face (Grave’s ophthalmopathy) Examine hands (thyroid signs) Examine lower limbs (pretibial myxoedema) Examine her neck (start from neck) Assess her thyroid status (start from peripheries)

General inspection – thin, fidgety and may have choreoathetoid movements

ULs o Both ULs up with dorsum facing upwards

Tremors Acropachy (thyroid clubbing) Onycholysis (Plummer’s nails – especially ring finger) Skin for vitiligo

o Both ULs with palm facing upwards Sweaty palms Palmar erythema

o Proximal weakness o Pemberton’s sign o Measure pulse for ST or AF o Reflexes

Eyes o Look

Page 104: Mrcp short case telling skills

Chemosis, keratitis, prominent caruncle and tarsorrhaphy Lid erythema and periorbital edema Exomphthalmos and lid retraction (Dalrymple’s sign)

o Move Lid lag (von Graefe’s sign) Ophthalmoplegia

Order of muscles affected “I’M So Lazy” o Inferior, medial, superior and lateral recti

Neck o Goitre – swallow water o Look for scar (think of hypothyroid and hypoparathyroid) and distended neck

veins o Walk to the patient’s back

Observe for proptosis Palpate the goitre (soft, smooth vs nodular, large, tender) Palpate for Cx LNs, carotid pulsations

o Listen for bruit o Palpate for tracheal deviation and SCM weakness on MNGs o Percussion of sternum

LL o Pretibial myxedema

Complete examination o Reflexes for hyperreflexia o Cardiovascular examination

Wide pulse pressure (if clinically hyperthyroid) and systolic hypertension ESM,CCF Gynaecomastia

o If there is a scar, request to perform Trousseau’s sign and Chvostek’s sign for hypoparathyroidism, assessment for hoarseness of voice

o Abdominal examination may reveal hepatosplenomegaly in Grave’s disease Presentation Grave’s disease Sir, this patient has got Grave’s disease and is clinically hyperthyroid complicated by Grave’s ophthalmopathy. There is presence of a diffusely enlarged, smooth and firm goitre which is associated with a bruit and is non-tender. There are no palpable LNs and tracheal is central with no dullness to percussion of the sternum. Pemberton’s sign is negative. There is evidence of hyperthyroidism. Patient is thin looking and is anxious and fidgety with presence of fine tremors of the outstretched hands, sweaty palms, with palmar erythema and a resting sinus tachycardia. I did not notice any thyroid acropachy or onycholysis. There is also no evidence of proximal upper limb weakness. Examination of the eyes reveals presence of lid retraction with a staring appearance. There is no chemosis, keratitis or evidence of tarsorraphy. There is evidence of exomphthalmos and proptosis. There is no ophthalmoplegia.

Page 105: Mrcp short case telling skills

There is no evidence of pretibial myxedema. Multinodular Goitre Sir, this patient has MNG and is hyperthyroid complicated by atrial fibrillation. There is presence of an enlarged goitre with multiple nodules bilaterally with a dominant nodule in the right lobe of the thyroid gland. This is non tender. There is no associated Cx LN and the carotid artery is palpable. There are no signs of compression such as stridor, negative Pemberton’s sign with no dullness to percussion of the sternum. There are signs of hyperthyroidism. The patient is in atrial fibrillation; did not notice any easy brusibility or obvious hemiplegia Questions What is Grave’s disease?

Autoimmune disease

TSI binds to and stimulates the TSH receptor on the thyroid cell membrane

Resulting in excessive synthesis and secretion of thyroid hormone

2% in women and 0,2% in men; 2nd to 4th decades What are the clinical signs specific to Grave’s disease?

Grave’s ophthalmopathy

Pretibial myxedema

Thyroid acropachy

Diffuse goitre

Lymphoid hyperplasia What is Grave’s Ophthalmopathy?

Characterised by

edema and inflammation of the extraocular muscles

increase in orbital connective tissue and fat

edema is due to hydrophilic action of the glycosaminoglycans secreted by fibroblast

inflammation is due to infiltration by lymphocytes and macrophages

Worst in

Smokers, elderly males

Post radio-iodine treatment

Severe hyperthyroidism

Can occur pre, during or post diagnosis of hyperthyroidism How do you assess activity of the eye disease?

Retrobulbar pain

Pain on eye movement

Eyelid erythema

Conjunctival injection

Chemosis

Swelling of the caruncle

Eyelid edema Points system together with degree of proptosis (Hertel’s ophthalmometer), reduced VA and eye movements What is pretibial myxedema?

Page 106: Mrcp short case telling skills

Specific feature of Grave’s disease

Types o Lymphedema type

Symmetrical, well defined, waxy and shiny peau d’orange appearance Red but not inflamed, swollen but not edematous

o Nodular type o Plague type

Occurs on the shins, anterior lateral aspects

Can also occur as localised dermopathy at sites of trauma

Characterise by edema, accumulation of glycosaminoglycans and lymphocytic infiltrates

Usually after treatment of hyperthyroidism, especially after radioactive iodine What are the signs of hyperthyroidism?

Resting tachycardia (important)

Sweaty palms

Tremors

Hyperreflexia

Thyroid bruit What are the causes of hyperthyroidism?

Primary

Grave’s disease

Toxic MNG (Plummer’s disease)

Toxic adenoma

De Quervain’s thyroiditis

Post partum thyroiditis (Characteristics: Reduced radionuclide uptake, low T3/T4 ratio and raised Thyroglobulin level)

Secondary

Pituitary

Struma ovarii, hydatidiform mole or choriocarcinoma (ectopic TSH)

Exogenous

Overtreatment (eg in thyroid cancer)

Factitious

Drug induced – Lithium, amiodarone (type 1 i.e. iodine induced and type 2 i.e. inflammatory thyroiditis)

What are the differential diagnoses of swellings in the neck?

Midline o Thyroid gland which rises on swallowing o Thryroglossal cysts which also rises on swallowing but also moves on sticking out

the tongue o Submental LNs

Lateral o LNs o Salivary Glands o Skin – sebaceous cysts or lipoma

Page 107: Mrcp short case telling skills

o Cystic hygroma o Pharyngeal pouch

How would you grade the goitre? WHO grading:

Grade 0 : not palpable or visible

Grade 1A: palpable goitre

Grade 1B : palpable and visible only on neck extension

Grade 2: Visible goitre at primary position

Grade 3: Obvious goitre from a distance What is Pemberton’s sign?

Elicited by asking the patient to lift his arms above her head

Development of plethora, cyanosis, inspiratory stridor and respiratory distress and distension of neck veins

Test for thoracic inlet obstruction due to a retrosternal mass How do you differentiate between thyroid acropachy and HPOA?

Radiographically o Thyroid acropachy new bone formation has a soap bubbles appearance on the

bone surface with coarse spicules o HPOA new bone formation in a linear distribution

What are the associated clinical conditions with Grave’s disease?

Diabetes mellitus

Vitiligo

Pernicious anaemia

Addison’s disease

Myasthenia gravis

Alopecia areata How would you investigate this patient with Grave’s disease?

Confirm the diagnosis o Thyroid stimulating hormone levels (aka thyrotropin levels) o Free thyroxine levels; KIV serum free tri-iodothyronine o Autoantibodies such as TSH receptor Ab(TRAb), thyroid peroxidase antibodies

and thyroglobulin antibodies o Occasionally, to differentiate between Graves and autoimmune thyroiditis,

radionuclide scan which shows diffuse uptake in Graves and no/low uptake in autoimmune thyroiditis

Ophthalmopathy o CT or MRI orbits to rule out retrobulbar tumor or AVM especially in unilateral

exomphthalmos How would you investigate this patient with MNG?

Activity assessment – fT4 and TSH

Imaging – CT neck to look for obstruction

Radionuclide in a predominantly “hot nodule”

Page 108: Mrcp short case telling skills

What is T3 thyrotoxicosis?

Hyperthyroid symptoms and sign

normal fT4 (thyroxine) level

elevated T3 (triiodothyronine). What is “sick euthyroid”?

Occurs in patient’s with severe illness or physical trauma

Alterations of peripheral transport and metabolism of thyroid hormones

Low fT4 and T3 and inappropriately low TSH How would you manage this patient? (1) Grave’s disease:

Medical therapy o Symptomatic treatment with propranolol o Carbimazole, methimazole and propylthiouracil o All inhibit thyroid peroxidase and hence thyroid hormone synthesis o PTU also inhibit conversion of fT4 to fT3, useful in crisis o CMZ and MTZ useful as fewer tablets and once daily dosing o Treated for 12-18 months and 30-40% will remain euthyroid o If it recurs, likelihood of remission on medications is low o Minor adverse effects (5%)

fever, rash, urticaria and arthralgia o Major adverse effects (0.5%) (CMZ and MTZ are dose related and PTU is not)

agranulocytosis

Advised to stop the drug if develop fever, sorethroat or mouth ulcers

Severe hepatotoxicity Vasculitis Lupus-like syndrome

Radio-iodine (131-I at dose 5 to 15 mCi) o 90% will become euthyroid within 2 months. o Contraindicated in pregnant and breast-feeding mothers, children and adolescent o Side effects

Almost all will become hypothyroid Neck pain Worsened thyrotoxicosis for several days post treatment

Prevented with CMZ/MTZ pre-treatment for 1-2 months and stopped 3-5 days before treatment; try not to use PTU as this decrease efficacy of I -131 treatment

Observed if mild or treat with beta blockers

Should not give antithyroid medications unless severe or expected to be severe due to poor control at the time of I-131 administration

Worsening of ophthalmopathy

especially in smokers and severe hyperthyroidism

Page 109: Mrcp short case telling skills

Administration of glucocorticoids can prevent worsening

Thyroidectomy o Indications includes “Cs”: Cancer (dominant nodule), cosmesis, compression o Effective in 90% o Not a/w worsening of Grave’s ophthalmopathy o Side effects : recurrent laryngeal nerve, hypoparathyroidism (1-2%) o Medications given prior to surgery and Lugol’s iodine given 7-10 days prior to

surgery (2) MNG

Render euthyroid with thionamide

As spontaneous remission does not occur, ablative therapy required

No obstruction – Radio-iodine

Obstruction – Surgical (3) Toxic Adenoma

Render euthyroid with thionamide

Radio-iodine – hypothyroidism side effect is less compared to Grave’s disease as the toxic adenoma suppresses the other thyroidal tissue

Surgical – Lobectomy (4) Subacute thyroiditis

Should not Rx with thionamides

Rx with propranolol, aspirin, NSAIDs and glucocorticoids How would you counsel a young woman with thyrotoxicosis who wishes to be pregnant?

Ideally, pregnancy should be avoided until hyperthyroidism is adequately treated because the rate of fetal loss is high

If it occurs or recurs during pregnancy, then o Treat with PTU

Lowest dose possible such that fT4 is at the upper range of normal Combination therapy contraindicated because PTU passes the placenta

but thyroxine doesn’t, resulting in fetal hypothyroidism PTU better because of better binding to proteins and therefore less

transplacental transfer theoretically; also CMZ a/w rare side effects of aplasia cutis congenita, esophageal and choanal atresias

o Can also be safely treated with surgery in the second trimester with almost no risk of death in experienced hands

o In the 3rd trimester, TSI levels declines and remission of hyperthyroidism occurs; stopping medications is possible then

o 1-5% of fetuses may be hyperthyroid resulting in IUGR and tachycardia o up to 750mg/d PTU or 20mg CMZ can be safely used in lactating mothers

How do you treat Graves ophthalmopathy?

General measures o Maintenance of euthyroidism o Stop smoking o Sleep with head raised

Page 110: Mrcp short case telling skills

o Use of artificial tears o Diuretics o Tinted glasses

Specific measures (for severe disease) o Glucocorticoids – 40-80mg OM then taper over 3 months o Radiotherapy o Surgical decompression

Stable disease o Surgery for lid retraction, exomphthalmos or diplopia

Gouty Hands

Examination of the Hands – Sequence

Tophi, joint deformity

Feel joints for active arthritis

Palmar erythema, dupytren’s contracture (alcohol), finger pulps for tophi

Test function – pincer and grip, coarse and fine

Look at the extensor surface and elbows (olecranon bursae)

Sallow appearance, dialysis (Renal failure)

Pinna or helix of the ear

Pleithoric, parotidmegaly, bleeding and hypertrophic gums

Look at the feet for joints, deformity, active arthritis, diabetic dermopathy

Feel the achilles tendon and infrapetallar region

Request o Walk patient if feet are involved o BP o Urine dipstick for glycosuria, hematuria (stones)

Presentation Sir, this patient has chronic tophaceous gout affecting his hands and his feet. On examination of the hands, there is asymmetrical swelling affecting the small joints of the hands with tophi formation which has resulted in severe deformity of the hands and feet. I also noticed that these tophi are exuding chalky material. On palpation, there is no tenderness and joints are not warm to suggest active arthritis. There is wasting of the intrinsic muscles of the hands. There is also presence of tophi on the extensor aspects of the forearms, the left olecranon bursae, the right helix/pinna of the ear as well as the small joints of the feet. I looked for but did not detect any tophi on the achilles tendon or the infrapetallar region. In terms of function, he is able to perform pincer and handgrip movement and his hand function is relatively preserved; able to perform door knob turning and cap a pen, as well transfer coins and unbutton his shirt. I noticed that the patient is not obese looking, no DM dermopathy or xanthelasma as these are a/w gout. There is also no evidence of chronic ethanol ingestion such as palmar erythema, dupytren’s contracture and parotidomegaly. There is no sallow appearance to suggest chronic renal failure. I also did not detect any conjunctival pallor or suffusion, hypertrophic or bleeding

Page 111: Mrcp short case telling skills

gums and patient is not pleithoric which may suggest presence of lymphoproliferative disease or polycythaemia. There are no psoriatic skin lesions I would like to complete the examination by walking to patient to assess function as I noticed that his feet is affected by gouty arthritis, take his blood pressure as well as a urinalysis to look for glycosuria as well as hematuria for UA stones and proteinuria for UA nephropathy. A detailed drug history, dietary history and alcohol consumption. Questions What is gout? Gout is a disorder of purine metabolism, resulting in hyperuricaemia either from overproduction(75%) or undersecretion of uric acid, resulting in deposition of urate crystals in the joints or bursae. Patients typically present with acute monoarthritis of the first MTPJ, with pain swelling and exquisite tenderness which peaks within hours and lasts for days. It affects the joints of the lower limbs initially in the majority of patients which includes the MTPJ, ankles and knees. It can also subsequently affects the joints of the upper limb. What are the stages of gout? Acute gouty arthritis Intercritical period Chronic tophaceous gout What does tophi indicate? Severe, recurrent and chronic gout. Where are the commonly areas to look for gouty tophi? Hands, extensor aspect of the forearms, olecranon bursae Helix if the ears Toes, Achilles tendons, infrapetaller regions What are the clinical manifestations of gout? Asymptomatic hyperuricaemia Acute arthritis Chronic, recurrent arthritis Tophaceous gout Uric acid nephrolithiasis Uric acid nephropathy What are the triggering factors of gout? Alcohol ingestion Foods – sweetbreads, liver, kidneys and sardines Drugs – Thiazide diuretics, aspirin, cyclosporine, pyrazinamide and ethambutol Dehydration and fasting Surgery, Trauma What are the causes of gout? Primary – associated with obesity, diabetes mellitus, hypertension and high TGs Secondary Drugs Chronic ethanol ingestion Chronic renal failure

Page 112: Mrcp short case telling skills

Polycythaemia, lymphoproliferative, myeloproliferative Psoriasis How would you investigate? Definitive investigation would be aspiration of the involved joint, looking for intracellular deposition of needle-shaped crystals that is negatively birefringent under polarised light, within leukocytes. They react with nitric acid and NH4OH to give a purple color (Murexide test) Blood Ix – Uric acid levels which may be normal during an acute attack X-ray of the joints may show erosive arthropathy from tophi with overhanging edges associated with punctuate to diffuse calcification. How would you manage? Education and counselling, including dietary advice and avoidance of alcohol PT/OT if tophaceous gout for preservation of function manage associated hypertension and diabetes mellitus Medications – acute attack and prophylaxis Surgery – rarely for cosmetic reasons, arthroplasty How would you treat an acute attack? NSAIDS – Indomethacin (50mg tds) Colchicine 2 ways:0.5mg hrly till GI side effects or max of 5 mg, or 0.5mg tds Intrarticular steroids (triamcinolone 20mg) Systemic steroids (Prednisolone 30mg OM and tails over 7-10 days) How would you prophylax against gouty attacks? Prophylactic agents used are iniated under colchicine cover which includes: Xanthine oxidase inhibitor Allopurinol New agents – Uricase, febuxostat Uricosuric acid agentsprobenecid or sulfinpyrazone losartan fenofibrate What are the indications for allopurinol? Recurrent gouty attacks > 3 times a year Chronic tophaceous gout Uric acid nephropathy Persistently high uric acid level Conditions that may predispose an individual to gouty attacks, prior to chemotherapy or radiotherapy which may induce tumor lysis What are the side effects of allopurinol? Side effects occur in 3-5% Rash, diarrhea, drug fever Leucopenia, thrombocytopenia Allopurinol hypersensitivity syndrome Erythematous rash, fever, hepatitis, hypereosinophilia and renal failure What are the other crystal arthropathy that you know about?

Page 113: Mrcp short case telling skills

Pseudogout – Acute arthritis resulting from deposition of calcium pyrophosphate dihydrate crystals in the joints which are rhomboid shaped positively birefringent crystals under polarised light. Calcium hydroxyapatite crystals deposition in the large joints such as knees and shoulders, affecting the elderly. What are the differential diagnoses? Septic arthritis Overlying cellulitis Trauma What is your differential diagnosis for chronic tophaceous gout? Florid tendon xanthomata Yellow and not chalky Adherent to tendon and not joint Does not involve the bursae, ie no olecranon or pinna lesions No active arthritis

Hemiparesis/Hemiplegia

Examination “This man has a left hemiparesis, please examine him”

For this, examine the UL and LL

Locate o Brainstem

Weber’s syndrome (III and contralateral hemiplegia) Millard-Gubler (VI and VII and contralateral hemiplegia; usually a/w

contralateral loss of proprioception and light touch as the medial lemniscal damage)

o Subcortical – lacunar; a/w UMN VII Pure motor (50%) Pure sensory (5%) Mixed motor and sensory (35%) Ataxic hemiparesis (10%) Dysarthria clumsy hand syndrome (rare)

o Cortical signs Do abbreviated version Gaze preference, sensory and visual neglect, hemianopia and dysphasia if

dominant lobe involved

Causes or risk factors o Pulse, Carotid bruit, murmur o Dyslipidaemia stigmata (xanthelasma, xanthomas, thickened TA) o DM dermopathy o Tar stains o Bruising, telangiectasia

Function and complications o Upper limb

Page 114: Mrcp short case telling skills

o Gait o Pressure sores, NG, urinary catheter

Request o BP o Urine dipstick o Fundoscopy for papilledema (to rule out SOL which is a possible differential)

Presentation Sir, this patient has got a left hemiparesis as evidenced by

State the UL and LL findings

State the level of the lesion and justify as above

Mentioned the causes as above

Mention the functional status and complications Questions What are your differential diagnoses?

Vascular o Ischaemic (80%)

Intracranial thrombosis Extracranial embolism – heart, extracranial arteries, paradoxical Lacunar strokes – small vessel disease from DM or hypt as a result of

lipohyalinosis Dissection

o Haemorrhagic (Intracerebral, SDH, SAH)

Space occupying lesion

Infective – abscess, meningoencephalitis

Seizures

Toxic-metabolic – Hypoglycaemia, HypoNa What are the 4 neuroanatomic stroke syndromes?

Anterior cerebral artery - affect frontal lobe function, producing altered mental status, impaired judgment, contralateral lower extremity weakness and hypoesthesia, and gait apraxia.

Middle cerebral artery (MCA) - contralateral hemiparesis, contralateral hypoesthesia, ipsilateral hemianopsia (blindness in one half of the visual field), and gaze preference toward the side of the lesion. Agnosia is common, and receptive or expressive aphasia may result if the lesion occurs in the dominant hemisphere. Since the MCA supplies the upper extremity motor strip, weakness of the arm and face is usually worse than that of the lower limb.

Posterior cerebral artery occlusions affect vision and thought, producing homonymous hemianopsia, cortical blindness, visual agnosia, altered mental status, and impaired memory.

Vertebrobasilar artery occlusions are notoriously difficult to detect because they cause a wide variety of cranial nerve, cerebellar, and brainstem deficits. These include vertigo, nystagmus, diplopia, visual field deficits, dysphagia, dysarthria, facial hypoesthesia, syncope, and ataxia. Loss of pain and temperature sensation occurs on the ipsilateral face and

Page 115: Mrcp short case telling skills

contralateral body. In contrast, anterior strokes produce findings on one side of the body only.

How would you investigate?

Confirm the diagnosis o Imaging – CT or MRI if posterior stroke (Diffusion-weighted imaging)

For diagnosis and type

For complications eg hydrocephalus

Blood Ix o FBC – polycythemia o Coagulation profile o Biochemical – HypoNa

ECG – AF, MI (60% a/w with AF or MI)

CXR – Enlarged mediastinum suspicious of a dissection

2D echo (cardioembolic course)

Carotid ultrasound scan (significant stenosis - >70%) and transcranial doppler

Young patient – young stroke work up (10) o ANA, dsDNA, ESR o Protein C, S o Anti Thrombin III o Factor V leiden or APC resistance o Anticardiolopin IgM/IgG o Homocystine o VDRL

What are the limitations of CT brain?

Unable to visualise the posterior fossa structures such as the brainstem and the cerebellum

Maybe normal up to 6 hours of onset o After 6 hours – hypodense area o Early signs on CT (5) – loss of grey-white differentiation, insular ribbon sign,

sulcal asymmetry, hyperdense MCA sign and obscuration of the LN

MRI – Diffusion weighted imaging which has a high sensitivity – looked for hyperintense signal

How would you manage?

Multidisciplinary approach

Education and counselling

PT/OT and ST – speech and swallowing, caregiver training, prevention of bed sores

Medications o Antiplatelets (Aspirin, persantin, Clopidogrel, Ticlid) o Anticoagulation

Correct risk factors o Hypertension o Hyperlipidaemia

Page 116: Mrcp short case telling skills

o Diabetes mellitus

Surgical o Intracranial bleeds o Hydrocephalus

How would you manage the patient acutely?

Airway, breathing and circulation

Control BP if bleed otherwise allow high BP in ischaemic stroke up to 220/120

Treat fever

Control of blood sugar

Determine if bleed or ischemic

If ischaemic stroke, assess for possibility of reperfusion therapy ie National Institute of Health Stroke Scale (NIHSS) using alteplase ie recombinant tissue plasminogen activator; within 3 hours and important to note inclusion and exclusion criteria.

Treat complications o Seizures o Raised intracranial pressure (Hyperventilation, elevate the head, mannitol)

Hepatomegaly

Presentation Sir, this patient has an enlarged liver without any signs of liver cirrhosis. (The mass in the RUQ is a liver mass as I am unable to get above the mass and am able to trace the edge of the liver past the midline of the abdomen.) It is massively/moderately enlarged with a

Size – cm from the right costal margin with a span of cm

Edge – regular or irregular

Surface – smooth or nodular

Consistency – soft, firm, hard

Tender

Pulsatile

Bruit The spleen is not palpable or percussible. The kidneys are not enlarged. There is no associated ascites. Examination of the peripheries:

Jaundice, bruises, stigmata of CLD and edema of LL

Hepatic flap

Causes o Cachexia, Cx LNs, conjunctival pallor o Dupytren’s contracture, parotidomegaly o Toxic looking, rashes or injected throat or enlarged tonsils o CCF – presence of raised JVP

Page 117: Mrcp short case telling skills

I would like to complete the examination by checking patient’s temperature chart for fever (if infective cause is a differential) and a rectal examination for masses (if secondaries are a differential). In summary, this patient has an enlarged liver that is (state the important Cs). Hence my differential diagnosis includes: Massive

HCC/Secondaries/myeloprolif

RVF

Alcoholic liver disease Mild-moderate

As above plus

Infection

Viruses – EBV, CMV, hepatitis A & B

Bacteria – Weil’s disease (leptospirosis), meliodosis, abscesses, TB, brucellosis, syphilitic gumma

Protozoal – hydatid cysts, amoebic abscess

Malignancy – lymphoproliferative, myeloproliferative, primary, secondary, adenoma from OCP

Infiltrative – sarcoid (erythema nodosum, lupus pernio), amyloid, fatty liver

Endocrine – acromegaly, hyperthyroid

Collagen Vascular disease

Chronic hemolytic anaemia( AI, thalassemia, HS)

Reidel’s lobe

Possibility of minimal CLD signs with just hepatomegaly

PBC

Hemochromatosis Tender

Liver abscess/infective (viral/bacterial/parasitic)

HCC/Secondaries

Right Heart Failure/Budd chiari Pulsatile

TR

HCC

AVM Hard/Irregular

Mitotic (primary/Secondary)

Page 118: Mrcp short case telling skills

Macronodular cirrhosis ie >3mm (post hepatitis B, C, Wilson’s and AAT) (Micronodular cirrhosis implies alcoholic liver cirrhosis)

Amyloidosis/Hydatid cyst/granulomatous disease/gummatous disease/APCKD Questions What are the causes? See above How would you investigate?

According to the most likely etiologies

Think of o Blood Ix – Dx and PX o Imaging o Liver Bx

How would you manage?

According to the most likely etiologies (Don’t forget that 40% of CLD has no peripheral stigmata of CLD – therefore think of Alcoholic liver cirrhosis, PBC and Hemochromatosis in the right setting)

Hepatosplenomegaly

Presentation Sir this patient has hepatosplenomegaly without evidence of cirrhosis of the liver. (I say this because enlarged masses in the right and left hypochondrial regions of which I am unable to get above theses masses and is not bimanually palpable or ballotable. Hence, these are unlikely to be due to kidney masses.) The liver is enlarged--Size, edge, surface, consistency, tender, bruit or pulsatile The spleen is enlarged --Size, edge, surface, consistency, tender Kidneys are not enlarged and no associated ascites Peripheral examination

CLD stigmata, jaundice, bruises

Hepatic encephalopathy

Causes o Pallor, cachexia, Cx LNs, PRV o Toxic, rashes, tonsils o Chronic ethanol ingestion o CCF o SBE, SLE, RA, Hemolytic anaemia

Page 119: Mrcp short case telling skills

I would like to complete the examination In summary, this patient has hepatosplenomegaly that is associated with. The differential diagnoses are: (Determine which is the predominantly enlarged organ eg massive liver with small spleen or massively spleen with small liver; determine if there is any Cs liver findings such as pulsatile liver; if both are mildy enlarged then combine the causes) Massive Splenomegaly (>8 cm)

CML

Myelofibrosis

PRV

Chronic malaria

Kala-azar (visceral leshmaniasis)

Others(Gaucher’s, rapidly progressive lymphoma) Moderately Enlarged (4 to 8 cm/ 2-4 FB)

Myeloproliferative

Lymphoproliferative

Haemotological – AI, ITP, Thalassemia and HS

Chronic malaria

Cirrhosis Mildly Enlarged(4cm</1-2FB)

Myeloproliferative, Lymphoproliferative

Infections

Viral – CMV,EBV

SBE, splenic abscesses, leptospirosis, Meliodosis, TB, Typhoid, Brucellosis(farmer)

Acute malaria

Infiltrative – Amylodosis, Sacoidosis

Endocrine – Acromegaly, thyrotoxicosis

Collagen vascular – SLE, Felty’s

Chronic haemolytic – Thalassemia, AI, HS, ITP Tender spleen

Infective causes

Acute myeloproliferative and lymphoproliferative Pallor(same as moderately enlarged spleen)

Myeloproliferative

Lymphoproliferative

Malaria

Page 120: Mrcp short case telling skills

Hemolytic anaemia(Thal and AIHA)

AI – Felty’s, SLE

Cirrhosis of the liver with portal hypertension Lymph Nodes

Lymphoproliferative(CLL/lymphoma)

Infective(IMS, Meliodosis, CMV, TB, HIV) Massive Liver

HCC/Secondaries/myeloprolif

RVF

Alcoholic liver disease Mild-moderate Liver

As above plus

Infection

Viruses – EBV, CMV, hepatitis A & B

Bacteria – Weil’s disease (leptospirosis), meliodosis, abscesses, TB, brucellosis, syphilitic gumma

Protozoal – hydatid cysts, amoebic abscess

Malignancy – lymphoproliferative, myeloproliferative, primary, secondary, adenoma from OCP

Infiltrative – sarcoid (erythema nodosum, lupus pernio), amyloid, fatty liver

Endocrine – acromegaly, hyperthyroid

Collagen Vascular disease

Chronic hemolytic anaemia( AI, thalassemia, HS)

Reidel’s lobe

Possibility of minimal CLD signs with just hepatomegaly

PBC

Hemochromatosis Tender Liver

Liver abscess/infective (viral/bacterial/parasitic)

HCC/Secondaries

Right Heart Failure/Budd chiari Pulsatile Liver

TR

HCC

AVM Hard/Irregular Liver

Page 121: Mrcp short case telling skills

Mitotic (primary/Secondary)

Macronodular cirrhosis (post hepatitis B/C, Wilson’s and AAT)

Amyloidosis/Hydatid cyst/granulomatous disease/gummatous disease/APCKD Questions What are the causes? How would you investigate? How would you manage? (See notes on individual causes in Splenomegaly and Hepatomegaly) History Taking Station Steps

1. 5 mins - Read carefully and prepare headings as in (2) 2. 12 mins – Examination itself

a. Introduce, shake hands b. Are you comfortable? Positioning! c. “I understand from the GP letter that…” d. Past history, drug allergy e. Present complaints

i. Address individual problems ii. Etiology and differential diagnoses

iii. Associations of condition iv. Complications of disease and treatment v. Conditions that may affect treatment

f. Women – Menstrual history, pregnancy g. Social history

i. Work – job nature, boss, colleagues ii. Family – husband, children

iii. Others – Church activities, exercise h. Family History

Page 122: Mrcp short case telling skills

i. Smoker, Alcoholic j. ICE – Ideas, Concerns and Expectations, Feelings

3. 2 mins – Wrap Up a. Any other significant things you would want to bring up b. Summarise c. Assure d. Management plan

i. Investigations ii. Symptomatic treatment

iii. Letter to employer iv. Arrange next consult with family and significant others v. Discuss the case with the consultant

4. 1 min – Gather your thoughts a. List patient’s problems

i. Medical ( Disease, associated conditions, side effects) ii. Social

iii. Concerns b. What is the diagnosis or differential diagnoses for the presenting complains? c. Summarise

5. 5 mins – Discussion of case with examiners a. Answer the above (4) b. Investigations c. Management

HOCM

Presentation Sir, this patient has got Hypertrophic obstructive cardiomyopathy. There is presence of a ESM heard best at the LLSE. It is a grade 3/6 murmur as it is not associated with any systolic thrill. In addition, there is presence of a MR mumur with a PSM heard bst at the ap3ex beat and radiates to the axilaa. It is a grade 4/6/ murm,ru as it is associated with a systolic thrill. The first heart sound is soft and there is no associated third or fourth heart sounds. The apex beat is not displaced located at the 5th IC space just medial to the midclavicular line. It has a double apical impulse (say this only if not if AF). There are no complications of congestive cardiac failure. However the JVP is raised at 3 cm with a prominent ‘a’ wave. Examination of the peripheries did not show any stigmata of infective endocarditis. He is is SR at a pulse rate of 84 bpm and has a characteristic bifid pulse (only if not in AF; if in AF, say shrap, rising and jerky pulse)

Page 123: Mrcp short case telling skills

I did not notice any clinical features to suggest Friederich’s ataxia. I would like to complete my examination by taking performing the valsalva manoeuvre or standing to accentuate the murmurs as well as take the patient’s blood pressure and look for fever from the temperature chart. A neurological examination would be useful to screen for any signs of stroke. In summary, this patient has got a HOCM with an ESM and MR murmur associated with a double apical impulse, bifid pulse and a raised JVP with prominent ‘a’wave. There are no complications of heart failure, AF or IE. This is a genetic condition. Questions What is HOCM?

o Hypertrophic Cardiomyopathy o Genetic cardiac disorder caused by missense mutation in the genes that encode proteins

of the cardiac sarcomere; autosomal dominant o Resulting in hypertrophy of the ventricular septum with LV outflow tract obstruction o 1 in 500, male:female 1:1 o Variable penetrance o Variable expression

o Asymptomatic (majority) o Symptomatic

o Angina, syncope, dyspnea, palpitations o Sudden death (Ventricular fibrillation) (overall annual mortality in 1%) o Complications of CCF, AF, IE and thromboembolic stroke

Why is there a ‘double apical impulse’?

o Presence of a LV heave with a prominent presystolic pulse caused by atrial contraction o A differential diagnosis is LV aneurysm

Why is there a prominent ‘a’ wave?

o Due to forceful atrial contraction against a non-compliant right ventricle What is Brockenbrough-Braunwauld-Morrow sign?

o Reduced pulse pressure in the post-extrasystolic beat o Occurs in HOCM and AS

What are the causes of HOCM?

o Familial o Friederich’s Ataxia o Idiopathic

How would you investigate?

o ECG

Page 124: Mrcp short case telling skills

o Normal in 25% o Tall QRS in precordial leads with ST-T changes, Q in inf and lateral leads o LAD o AF

o CXR o Normal o LA enlargement, LVH

o Echocardiogram o Diagnostic

Asymmetrical septal hypertrophy Systolic anterior motion of the anterior mitral valve leaflet Diastolic dysfunction

o Severity Septal thickness >18mm Outflow tract gradient > 40mmHg as rest

o Complications MR IE

o TMX for those with angina o Holter monitoring looking for arrythmias especially presence of VT

How would you predict poor outcome?

o Family history of sudden death o History of syncope, cardiac arrest o Poor BP response to exercise o Holter monitoring with ventricual arrythmias detected, esp spontaneous VT o Echo findings

How would you treat?

o Education and counselling with screening of first degree relatives o 50% chance of being affected o Screen with ECG and 2D echo

Annually for adolescent (12-18) And 5 yearly

o Treatment is directed at symptom relief and prevention of sudden cardiac death o Relief symptoms

Beta blockers If cannot tolerate, verapamil but caution I patients with sever

symptomatic obstruction because of increase death especially after first few doses

Beta blockers and disopyramide o Rx complications

Rx CCF Rx AF

Page 125: Mrcp short case telling skills

Rx and prevention of IE Prevention of sudden death

Amiodarone

Pacing (dual-chamber pacing) o Septal ablation with alcohol or surgery

o Surgical septal myomectomy (Gold standard)

Hypertensive Retinopathy

Presentation Sir, this patient has got grade 4 hypertensive retinopathy as evidenced by

Presence of narrowed, tortous and irregular retinal arteries with increased light reflex, demonstrating the silver/copper wiring

There is arterio-venous nipping

With evidence of flame shaped and blot haemorrhages and cotton wool spots and hard exudates, especially around the macula, forming a macular star

Associated with papilloedema I did not notice any concomitant features of diabetic retinopathy. However the patient does have xanthelesma. There are no signs of chronic renal failure such as sallow appearance. Patient does not have any cushingoid or acromegalic or polycythaemic features which are conditions associated with hypertension. I would like to complete my examination by taking the blood pressure of this patient as well as examine his

Cardiovascular system

RR/RF delay – coarctation of the aorta (state if young)

Evidenced of LVH

S4 if BP>180/110 (state if grade 3 or 4 changes noted)

Abdominal examination for renal bruit (renal artery stenosis) or ballotable kidneys (polycystic kidney disease) or palpable adrenal masses

Urine dipstick for proteinuria, casts, glycosuria

CNS for signs of previous CVA Questions How do you grade hypertensive retinopathy?

Keith Wagner Classification

Grade 1 – Arteriolar narrowing, tortousity, irregular calibre with copper/silver wiring

Grade 2 – Arteriovenous nipping

Grade 3 – flame-shaped and blot haemorrhages, cotton wool spots and hard exudates

Grade 4 – Papilloedema

Page 126: Mrcp short case telling skills

Clinical features and prognosis of grade 3 and 4 are the same Explain their physical appearance?

Young patients, retinal arterioles react to hypertension via constriction, hence arteriolar constriction or narrowing

In older patients, there is arteriosclerosis hence irregular calibre. Also the thickened walls shows a widening of the normal light reflex, giving the blood column a copper appearance (copper wiring) or silver appearance (silver wiring)

At arteriovenous crossing, the thickened arteriolar walls displace and constricts the veins, resulting in AV nipping

The arteriolar may be damaged by necrosis leading to flame shaped haemorrhages, cotton wool spots caused by microinfarcts, as well as retinal edema

Chronic retinal edema at the macula results in hard exudates radiating from the macular, ie macular star

Finally papilloedema results How do patients normally present?

Normal vision

Except when there is associated macular involvement What are the causes for retinal haemorrhages?

Diabetic retinopathy

Hypertensive retinopathy

CRVO

Severe anaemia, leukaemia What are the causes for cotton wool spots?

DM preproliferative retinopathy

Hypertensive retinopathy

Anaemias, leukaemias

HIV, infective endocarditis What are the causes for hard exudates?

Diabetic retinopathy

Hypertensive retinopathy What are the causes of hypertension?

90% are essential hypertension

10% are secondary Renal causes – Chronic renal disease, polycystic kidneys, renal artery stenosis Endocrine – Cushing’s, Acromegaly, Phaechromocytoma, Conn’s, hyperparathyroidism Others – Coaractation, OCP usage, pre-eclampsia, polycythaemia

Page 127: Mrcp short case telling skills

How would you investigate a patient with hypertension?

Urea and electrolytes

Urine for protein, glucose and cast

Fasting lipids and glucose

ECG, CXR If indicated clinically, ie

Young hypertensive <50

Requiring >2 antihypertensive

Sudden deterioration in control of BP

Features suggestive of secondary causes on clinical examination How would you manage?

Lifestyle- Exercise, eat healthily, stop smoking

Pharmocotherapy

Interstitial Lung Disease

Presentation Sir, this patient has interstitial lung disease affecting both lower lobes (upper lobes) as evidenced by fine velcro-like late inspiratory crepitations heard best posteriorly(anteriorly) in the lower one third bilaterally. This is associated with clubbing(50%) and a non-productive cough. Chest excursion was reduced bilaterally with a normal percussion note and vocal resonance. Trachea is central and apex beat is not displaced. There are no signs of pulmonary hypertension or cor pulmonale. There are also no features of polycythemia. Patient respiratory rate is 14 breaths per minute and there are no signs of respiratory distress. There are also no signs of respiratory failure. There is also no nicotine staining of the fingers and I note that the patient is cachexic looking with wasting of the temporalis muscles. In terms of aetiology, there is no symmetrical deforming polyarthropathy of the hands to suggest RA, or cutaneous signs to suggest presence of SLE, dermatomyositis or scleroderma as these conditions may be complicated by pulmonary fibrosis. With regards to treatment, patient is not Cushingoid and does not have papery thin skin or steroid purpura to suggest chronic steroid usage. On inspection there are no surgical scars to suggest open lung biopsy. I would like to complete the examination by asking for a detailed drug history as well as an occupational history. In summary, this patient has got pulmonary fibrosis affecting bilateral lower lobes. There are no complications of pulmonary hypertension, cor pulmonale and polycythemia. He is clinically not in respiratory failure and has no features of chronic steroid usage. The differential diagnoses include collagen vascular disease, drugs, occupational causes and idiopathic pulmonary fibrosis. Questions

Page 128: Mrcp short case telling skills

What are the differential diagnoses for clubbing and crepitations? Pulmonary fibrosis Bronchiectasis Lung abscess Mitotic lung conditions What are the characteristic auscultatory findings? Late, fine inspiratory crepitations Velcro-like Disappears or quietens with the patient leaning forwards What are the causes of fibrosis? Upper Lobes S – Silicosis, sarcoidosis C- coal worker pnemoconiosis H- histiocytosis A- Ankylosing spondylitis, ABPA R – radiation T – TB

Lower lobes R- RA A-Asbestosis S- Scleroderma I – Idiopathic pulmonary fibrosis O- others ie drugs Cytotoxics – MTX, Aza, bleomycin, bulsulphan, cyclo, chlorambucil CNS - Amitryptyline, phenytoin and carbamazepine CVS - Amiodarone, hydralazine, procainamide Antibiotics - Nitrofurantoin, isoniazid Antirheumatics – Gold, sulphasalazine

Both N – Neurofibromatosis, Tuberous sclerosis E – Extrinsic allergic alveolitis (acute symptoms within 6 hrs of inhaled allergens eg

farmer’s lungs) P – pulmonary haemorrhage syndromes A – alveolar proteinosis Primary Secondary – Inhaled organic dusts(Silica, Al), chronic infection, malignancy

Lymphangiomyomatosis How would you classify interstitial lung disease? (ATS/ERS 2001) Diffuse parenchymal lung disease(DPLD) of known cause Collagen Vascular disease RA, SLE, Dermatomyositis, Systemic sclerosis

Page 129: Mrcp short case telling skills

Occupational/Environmental Asbestosis, silicosis, extrinsic allergic alveolitis

Drug related Cytotoxic, CNS, CVS, Antibiotics and antirheumatic

Idiopathic IPF Other idiopathic interstitial pneumonias DIP AIP LIP NSIP Cryptogenic organising pneumonia Respiratory bronchiolitis

Granulomatous Sarcoidosis

Others - LAMs, histiocytosis How would you diagnose idiopathic pulmonary fibrosis? Clinical-radiological-pathological diagnosis Clinical

o Exclusion of other causes of ILD o >50 yrs, insidious onset of dyspnea, > 3months, non-productive cough o Typical physical findings

Radiological (see below) Pathological (see below) How would you investigate? The diagnostic Ix of choice is a HRCT of the thorax but simple IX such as CXR and LFT are useful: CXR Diagnostic bilateral basal reticulonodular shadows, peripheries, which advances upwards honeycombing in advanced cases (gps of closely set ring shadows) loss of lung volume

Extent and distribution Complications

Lung function Restrictive pattern (reduced TLC or VC with increased FEV1/FVC ratio) Severity of restriction based on TLC Reduced transfer factor (impaired gas exchange)

HRCT scan Dx – patchy reticular abnormalities, focal ground glass, architectural distortion, volume

loss, subpleural cyst, honeycombing (no consolidation or nodules) Extent and severity – basal, peripheral, subpleural Complications

Page 130: Mrcp short case telling skills

NB: Similar to that of collagen vascular disease and asbestosis Others Bronchoscopy – lavage Predominantly lymphocyte responds to steroids and better Px= not UIP Predominantly neutrophils and eosinophils means poor Px= UIP (if >20% of

eosinophils to consider eosinophilic lung disease) Lung biopsy IPF – Usual interstitial pneumonia

Bloods ABGs To rule out causes

How would you manage? Education and counselling Stop smoking Regular follow up and vaccinations

Treat underlying cause Pharmacological Trial of steroids If responding continue steroids If not responding, cyclophosphamide or azathioprine

Antifibrotic agents Eg penicillamine which has not been proven to be useful

Surgical Lung transplant (single lung transplantation)

Manage complications Cor pulmonale - diuresis for heart failure Polycythemia - venesection if Hct >55% Respiratory failure – Oxygen therapy Monitor for lung cancer

What are the good prognosticating factors? Young age Female Short duration Ground glass appearance on the CXR Minimal fibrosis on lung biopsy What is the clinical course of patients with IPF? Gradual onset Progressive Median survival from time of dx about 3 years What are the causes of death?

Page 131: Mrcp short case telling skills

Cor pulmonale Respiratory failure Pneumonia Lung carcinoma What is Hamman-Rich syndrome? Rapidly progressive and fatal variant of interstitial lung disease

Isolated Third Nerve Palsy

Examination

Complete the examination routine for eyes or CN as instructed

Proceed to look for intortion of the affected orbit by tilting the head towards the involved site or looking for intortion when asking patient to look down and medially of the affected eye; patient maybe tilting his head voluntary away from the side of the lesion (implies 4th nerve palsy)

Rule out

Thyroid, MG

Superior orbital syndrome and Cavenous sinus syndrome

Proceed with

Neck for LNs

Examine the upper limbs for Cerebellar, hemiplegia, EPSE and areflexia

Look dor DM dermopathy

Request

Corneal reflex (reduced or absent)

Visual fields (bitemporal hemianopia)

Fundoscopy for optic atrophy (MS), DM or hypertensive changes

Visual acuity

Blood pressure

Urine dipstick

Temperature chart

Headache or pain Presentation Sir, this patient has an isolated right third nerve palsy as evidenced by presence of

Divergent strabismus involving the right orbit which is in a “down and out” position

Complete ptosis/partial ptosis of the right eye

Dilated pupil which is not reactive to direct light and to accommodation There is no ptosis or superior rectus palsy of the left eye to suggest a III nerve nuclear lesion. There are no associated CN palsies to suggest superior orbital fissure syndrome or cavernous sinus syndrome. I did not find any associated 4th CN palsy with presence of intortion on asking the patient to adduct the right eye and look downwards. The 6th CN is also intact. There is also no paraesthesia of the ophthalmic division of the 5th CN. Gross VA is also intact.

Page 132: Mrcp short case telling skills

There are no signs of Graves ophthalmopathy (no conjunctival suffusion and proptosis or lid edema of the right eye) There is no evidence of fatiguiability to suggest myasthenia gravis. On examination of the neck, I did not find any enlarged cervical LNs. There is also no evidence of hemparesis, cerebellar signs, areflexia or tremors or chorea on examination of the upper limbs. I also did not notice any diabetic dermopathy. I would like to complete the examination by:

Corneal reflex (reduced or absent)

Visual fields (bitemporal hemianopia)

Fundoscopy for optic atrophy (MS), DM or hypertensive changes

Visual acuity

Blood pressure

Urine dipstick

Temperature chart

Headache or pain In summary, this patient has an isolated right third nerve palsy. The possible causes include… Questions What is the course and anatomy of the 3rd CN?

Nuclear portion – at the midbrain

Fascicular intraparenchymal portion – close to the red nucleus, emerges from cerebral peduncle

Fascicular subarachnoid portion – meninges, PCA aneurysm(between the PCA and internal carotid)

Fascicular cavernous sinus portion – sella turcica between the petroclinoid ligament below and interclinoid above

Fascicular orbital portion – superior orbital fissure Axons run ipsilateral except those to the (1)superior rectus which is innervated from the contralateral 3rd nucleus and (2) the levator palpebrae which has innervations from both nuclei. Hence, right sided 3rd nerve palsy can have contralateral ptosis which is often milder than the ipsilateral ptosis; also the ipsilateral superior rectus can still be affected due to involvement of the contralateral fascicular intraparenchymal midbrain portion of the left 3rd nerve. For pupillary reflex and accommodation, it is served by the Edinger-Westphal nucleus and all axons are ipsilateral. What are the causes of an isolated 3rd nerve palsy?

Brainstem

Infarct, haemorrhage, tumour, abscess, multiple sclerosis

For nuclear lesions

Will also have contralateral ptosis and elevation palsy

Page 133: Mrcp short case telling skills

May have bilateral 3rd nerve palsies (+/- INO)

For fascicular midbrain lesions

Weber’s (+ contralateral hemiplegia) – base of midbrain

Northnagel (+ contralateral cerebellar) – tectum of midbrain

Benedikt’s (+ contralateral hemiplegia, contralateral cerebellar and contralateral tremor, athetosis and chorea) – tegmentum of midbrain, red nucleus

Peripheral

Subarachnoid portion- PCA aneurysm, meningitis, infiltrative, others eg sarcoidosis

Cavernous sinus lesions- Tumour(pituitary adenoma, meningioma, cranipharyngioma), cavernous sinus thrombosis, inflammatory (Tolosa-Hunt syndrome which is a non-caseating granulomatous or non-granulomatous inflammation within cavernous sinus or superior orbital fissure that is treated with steroids) and ischaemia from microvascular disease affecting the vasa nervosa, mononeuritis multiplex

Orbital- tumor (meningioma, hemangioma), endocrine (thyroid) and inflammatory(orbital inflammatory pseudotumor ie Tolosa Hunt)

Mononeuritis multiplex, Miller Fischer and MG

Don’t forget migraines and myasthenia! (emergency – Coning, Giant cell Arteritis and aneurysm)

How would patient present?

Diplopia

Ptosis

Symptomatic glare from failure of constriction of pupil

Blurring of vision on attempt to focus of near objects due to loss of accomodation

Pain in certain etiologies

Diabetes mellitus

Tolosa-Hunt syndrome

PCA aneurysm

Migraine What are the causes of a dilated pupil?

III nerve palsy

Optic atrophy (direct light and accommodation absent with intact consensual reflex)

Holmes Adie Pupil (Myotonic pupil) o Unilateral o Slow reaction to bright light and incomplete constriction to convergence o Young women o Reduced or absent reflexes

Mydiatric eye drops

Sympathetic overactivity Why does a PCA aneurysm results in pupillary involvement whereas conditions such as DM or hypertension spares the pupil?

The pupillary fibres are situated superficially and prone to compression whereas ischaemic lesions tends to affect the core of the nerve thus sparing the pupillary fibres

Page 134: Mrcp short case telling skills

How would you investigate?

Imaging o CT, MRI o Angiogram

Blood test

Fasting blood glucose, ESR

TFT and edrophonium

LP How would you manage?

Medical 3rd nerve palsy Education – watchful waiting and avoid driving, heavy machinery and climbing high places Treat underlying conditions such as DM and hypertension Watchful waiting

Spontaneously recover within 8 weeks Symptomatic treatment

NSAIDs for pain

If complete ptosis, no need to treat diplopia

Use eye patch for severe diplopia and a prism Fresnel paste on for mild diplopia

Surgical 3rd nerve palsy - surgery

Isolated VI Nerve Palsy Examination (Examine patient’s eyes, CNs)

Complete the eye examination

Rule out MG, thyroid

Rule out Cavernous Sinus syndrome, Superior orbital fissure syndrome

Proceed to check cranial nerves

V, VII and VIII for CPA tumor

XI, X, XI, XII for any base of skull lesions

Examine upper limbs

Hemiplegia (long tract signs suggesting brainstem)

Cerebellar signs (CPA lesion, Miller Fisher usually truncal and gait)

Reflexes for areflexia (Miller Fisher)

Examine the neck for LNs and mastoid for tenderness (Gradenigo’s syndrome)

Request

Fundoscopy (papilloedema from raised ICP, optic atrophy for MS)

Field testing (bitemporal hemianopia)

Acuity (reduced in orbital lesions)

Corneal testing for reduced sensation form V1 involvement

BP

Urine dipstick

Temperature chart for fever(meningitis)

Ask for retrobulbar pain

Page 135: Mrcp short case telling skills

Presentation Sir, this patient has an isolated right sided VI nerve palsy as evidenced by

Convergent strabismus at primary gaze

Failure of abduction of the right eye

With diplopia where the image is side by side and furthest apart on rightward gaze, with disappearance of the outer image on covering the right eye. This suggests a right lateral rectus muscle weakness and hence a right VI nerve palsy.

There is no evidence of a III or IV or V1 palsy which may suggest cavernous sinus or superior orbital syndrome. There is also no involvement of the VII or VIII nerve palsy and no cerebellar signs to suggest A CPA lesion There are no associated CN lesions of IX to XII and no enlarged Cx LNs. There is no fatiguibilty to suggest MG and no thyroid eye signs. There are no enlarged LNs or tender mastoid. There is also no hemiparesis, no cerebellar signs and reflexes are present. I would like to complete my examination by:

Fundoscopy (papilloedema from raised ICP, optic atrophy for MS, DM or hypt changes)

Field testing (bitemporal hemianopia)

Acuity (reduced in orbital lesions)

Corneal testing for reduced sensation form V1 involvement

BP

Urine dipstick

Temperature chart for fever(meningitis)

Ask for retrobular pain In summary, this patient has an isolated right sided VI nerve palsy. The aetiology includes…. Questions What is the course of the VI nerve?

Nucleus of CN VI located in the pons, sending motor neurons supplying the lateral rectus muscle of the ipsilateral eye;

also internuclear neurons that project (via MLF) to medial rectus subdivision of contralateral oculomotor (CN III) nucleus (ie lesion of nucleus causes ipsilateral horizontal gaze palsy)

nerve fascicles exit the pons anteriorly and course through subarachnoid space where it runs a vertical (upward) course, along ventral surface of pons (confined by adjacent AICA)

pierces dura overlying clivus, entering into Dorello's canal, contacting tip of petrous pyramid (part of temporal bone; adjacent to mastoid air cells)

enters the cavernous sinus after passing through the petroclinoid (Gruber's) ligament

runs in body of sinus (rather than lateral wall where CN III, IV, V located) along foramen lacerum (near internal carotid artery)

enters orbit via superior orbital fissure and shortly thereafter pierces lateral rectus muscle

Page 136: Mrcp short case telling skills

What are the causes of a unilateral VI nerve palsy?

Brainstem (Pons)– infarct, haemorrhage, abscess, demyelinating (look for VI and VII palsies due to close proximity)

Aneurysm (ectatic basilar artery)

Meningitis

Infective – TB, Fungal, HIV, Syphilis, Lyme

Mitotic – leptomeningeal carcinaomatosis, secondaries(NPC), lymphoma, radiotherapy

Sarcoidosis

Trauma

CPA lesions

Petrous temporal bone (Gradenigo’s syndrome)

Cavernous sinus syndrome

Superior orbital syndrome

Miller Fisher syndrome

Mononeuritis multiplex

DM, hypertension

MG

Raised ICP NB: - most common aetiology of isolated abducens lesion is post-viral in children and ischemia / infarction (eg diabetic) in adults What are the causes of bilateral VI nerve palsy?

Leptomeningeal causes (see above)

Miller Fisher syndrome

Mononeuritis multiplex

MG

Raised ICP

Wernicke’s encephalopathy (Ophthalmoplegia, confusion and ataxia a/w Korsakoff’s psychosis from thiamine deficiency)

What are the syndromes associated with VI nerve palsy? Central

Raymond’s syndrome – ipsilateral VI with contralateral hemiparesis

Millard-Gubler syndrome – Ipsilateral VI and VII with contralateral hemiparesis Peripheral

Gradenigo syndrome – inflammation of the tip of the petrous bone from mastoiditis; VI and V(gasserian ganglion therefore ipsilateral pain) and VII

CPA

Cavernous sinus

Superior orbital Congenital

Mobius syndrome (VI with facial diplegia)

Duane’s syndrome (Congenital absence of VI nuclei with III nuclei innervating the lateral recti; orbit retraction on adduction and protrusion on abducting

Page 137: Mrcp short case telling skills

How do you evaluate diplopia?

Evaluation of diplopia follows 3 rules o The double vision is maximal in the direction of gaze in the affected muscle o The false image is the outer image o The false image arises form the affected eye

What is Cavernous sinus syndrome?

Lesion in the cavernous sinus leading to

Ophthalmoplegia, V1, Horner’s, proptosis, chemosis and pain

Causes include carotid aneurysm, carotid-cavernous fistula, tumour and thrombosis, Tolosa Hunt syndrome

What is superior orbital fissure syndrome?

Lesions occurring at the superior orbital fissure leading to

Ophthalmoplegia, V1, Horner’s, proptosis, chemosis, pain and Optic Nerve

Causes includes meningiomas, hemangiomas and thyroid eye disease, Tolosa Hunt syndrome What is Miller Fisher syndrome?

Triad of ophthalmoplegia, ataxia and areflexia

Cs by present of anti GQ1B antibodies

Variant of GBS What are the causes of mononeuritis multiplex?

Endocrine – DM, hypt

AI – Churg Strauss, Wegener’s, PAN, Sjogren’s, RA, SLE

Infective – Lyme, leprosy

Infiltrative – Amyloid, sarcoid How would you investigate?

Blood tests fasting glucose, ESR, ANA, VDRL

Imaging CT brain or MRI brain Vascular imaging if proptosis / chemosis looking for dural carotid-cavernous fistula or cavernous sinus thrombosis

Examine CSF (also if systemic illness, immunocompr, bilateral, other CN deficits)

8-30% will remain cryptogenic after work-up How would you manage?

Education on Px if ischaemic (see below) and Ix and avoid driving, climbing high places and operating heavy machinery

Treat underlying cause Example control DM and BP

Symptomatic treatment

Patch

Prism

If chronic, chemodenervation with botox or strabismus surgery What is the prognosis?

most either spontaneously improve or have underlying lesion found

Page 138: Mrcp short case telling skills

ischemic palsies almost always recover completely in 2-4 months while some recovery seen in half of traumatic cases (but take over one year)

if no recovery over 3-6 months then suspect underlying lesion such as tumour

few may have chronic isolated abducens palsy of unknown cause

follow regularly looking for emergency of new localizing signs and ensure adequate neuroimaging and ENT evaluation

Lichen Planus

Presentation Sir, this patient has Lichen planus as evidenced by Cs Grouped and confluent Flat-topped, polygonal, sharply defined papules Violaceous Wickham’s striae Koebnerisation Pruritic scratch marks Post-lesional hyperpigmented macules

Distribution Flexor areas of the ULs – wrists and forearms Sacral area, groin Palms and soles

Nails Dystrophy of nail plates Longitudinal ridging Longitudinal melanonychia Subungal hyperkeratosis Onycholysis Pterygium Complete loss of nail bed

Scalp Scarring/cicatrical alopecia

Mucosal White lacy-like lesions, asymptomatic Ulcers for erosive form

There are no surgical scars noted and patient is not jaundiced with no stigmata of chronic liver disease which may suggest Hepatitic C infection I would like to complete the examination Drug history Antihypertensive – B-blockers, thiazides, methyldopa Antimalarials – quinine Anti-diabetic (Tolbutamide) Phenothiazines, Gold

Page 139: Mrcp short case telling skills

Occupational history Color film developer

In summary this patient has lichen planus affecting his upper limbs, nails and oral mucosa and is troubled by pruiritus. Questions What are the types of Lichen Planus? Hypertrophic (plague-like lesions on the tibia; Afro-carribean) Erosive (mouth ulcers with risk of SCC) Bullous Follicular Guttate How would you Ix? Skin biopsy with IF Dx is triad of Typical skin lesions T-cell infiltration of the dermis in a band pattern IgG and C3 immunofluorescence at the basement membrane of the dermis

How would you manage? Education Px – skin lesions are not premalignant; oral ulcers can progress to SCC Most resolves within 6-18 months

Pharmological Steroids – topical, intralesional and systemic Cyclosporin – topical for mouth lesions, systemic MMF Retinoids PUVA

What is Pterygium of the nails? Cuticle invades the nail bed; Cs of Lichen planus What are the differential diagnoses for white lesions of the mouth?

Lichen Planus

Candidiasis

Secondary syphilis

Leukoplakia

Squamous papilloma What are the differential diagnoses of oral ulcers?

Erosive Lichen planus

Pemphigus vulgaris

SJS

Drug eruptions

Infective – HSV

Inflammatory bowel disease

Aphthous ulcers

Page 140: Mrcp short case telling skills

Behcet’s disease What are the differential diagnoses for violaceous lesions?

Lichen planus

Sarcoid (lupus pernio)

Dermatomyositis (eruptive xanthomas of the skin may mimic the polygonal lesions of LP)

Lobectomy/Pneumonectomy

1. Sir, this patient has left lobectomy as evidenced by a left sided thoracotomy scar associated with asymmetrical deformity of the chest. There is reduced chest excursion of the left and the percussion note is dull in the lower third of the left hemithorax with decreased breath sounds and vocal resonance in this area. The tracheal is not deviated and the apex beat is not displaced. 2. Sir, this patient has a left pneumonectomy as evidenced by a left thoracotomy scar associated with asymmetrical deformity of the chest. There is reduced chest wall excursion on the left with a dull percussion note and absent breath sounds and vocal resonance over the entire left hemithorax. Trachea is deviated towards the left with the apex beat displaced in the same direction. This is associated with hyperinflation of the right chest with hyper-resonant percussion note and loss of liver dullness. He is comfortable at rest with a RR of 12 bpm with no evidence of respiratory failure or distress. With regards to aetiology 1. I did not detect any coarse inspiratory crepitations to suggest bronchiectasis nor was there any ronchi or prolonged expiratory phase to suggest COPD or ABPA. 2. Patient is not clubbed and there is nicotine staining of the fingers. There is no palpable enlarged cervical lymph nodes and he is not cachexic looking to suggest mitotic lesion of tuberculosis. I would like to look at the patient’s temperature chart as well as his sputum. In summary, this patient has a left lobectomy/pneumonectomy and the possible causes include: 1. surgical resection for early stage mitotic lesion of the lung or a SPN of uncertain cause. 2. Resistant lung abscess 3. Mycetoma 4. Treatment modality for pulmonary tuberculosis in the past (1950s, pt should be late 60s) 5. lung volume reduction surgery in COPD (lobectomy) 6. Localised bronchiectasis or its complications 7. Trauma Questions What are the indications for a lobectomy/pnemonectomy? (7)

Causes of lobectomy and pneumonectomy o Mitotic lesion or SPN of uncertain nature o Abscess

Page 141: Mrcp short case telling skills

o Bronchiectasis – localised or complications o Mycetoma, ABPA (remaining lungs may have ronchi) o TB treatment o LVRS for COPD o Trauma

Lobectomy and splenectomy o TB o Sarcoidosis

What are the contraindications to a lobectomy/pneumonectomy in a lung cancer patient?

Resectability o T4 (mediastinal structures), N3 (contralateral mediastinal or hilar or ipsilateral

supraclavicular LNs or scalene) or M1 o Tumor within 2 cm of carina (potentially curable by radiotherapy) o Bilateral endobronchial tumor (potentially curable by radiotherapy)

Physiologic staging o Severe pulmonary hypertension o CO2 retention o FEV1< 1L o Transfer factor <40% o Concomitant disease that would shorten life expectancy o Recent MI in the past 3 months o Borderline function with cardiopulmonary exercise testing with a maximal oxygen

consumption <15ml/kg/min What are the indications for lung volume reduction surgery in COPD patients?

Emphysema

Predominantly upper lobes

No or mild pulmonary hypertension (PASP > 45 mmHg)

No concomitant disabling disease

FEV1>20%

DLCO >20% Lower Limbs Overview

Pes cavus

CMT

Spina Bifida

Poliomyelitis

Spinal cord tumours

Freiderich’s ataxia/spinocerebellar degeneration

Syringomyelia

Cerebral Palsy

Muscular dystrophies

Fasciculations (LMN type, MND)

Wasting

Bilateral

Page 142: Mrcp short case telling skills

Proximal weakness

Wasting distally (Pes Cavus, Peripheral neuropathy)

Spastic paraparesis (L&P 104)

Cerebellar (MS/FA/Syphilitic meningomyelitis/Craniospinal jn/SCA)

Sensory level (Lumbar/Thoracic/Cervical – ULs/Above – high Cx, CP)

Dorsal Column Loss (SACD/Taboparesis/MS/FA)

Mixed (Babinski + absent reflexes – see below)

Friederich’s ataxia

SACD

Tabo-paresis

MND

UMN + cauda equina or peripheral neuropathy(CVA+alcoholic/DM)

MND

Flaccid paraparesis

Wasted

GBS/CIDP/HMSN/Hansen’s

Poliomyelitis

Spina Bifida

No wasting

Peripheral – GBS, HMSN, paraneoplastic, paraproteinemia, amyloid

Cord compression

Others – Miller-Fisher, MG, Periodic paralysis, botulism/diphtheria/organophosphate/Hg/Pb, AIP(BP)

MND

Bilateral Footdrop Unilateral

Foot Drop

Bilateral (Peripheral neuropathy – motor predominant, flaccid, spastic)

Unilateral

Peripheral neuropathy, CPN

Sciatic nerve

Root or anterior horn cell

Look for complications- trophic ulcer, interventions – walking callipers

Unilateral – Peripheral neuropathy, lumbosacral plexus, polyradiculopathy, polio (LMN)

Brown-sequard (UMN)

Diabetic amyotrophy

Hemiparesis (UMN)

Sensory loss

Peripheral neuropathy

Mononeuropathy

Polyradiculopathy

Page 143: Mrcp short case telling skills

Lumbosacral Plexus

Dissociated sensory loss, spinal cord level

Others

Gait

Cerebellar

Unilateral

Cerebellar – Vascular, MS, SOL eg abscess or tumour

Combined – Lateral medullary syndrome, CPA tumour, ataxic hemiparesis

Bilateral

hypothyroidism, Wilson, Alcoholic cerebellar degeneration(spares the ULs), drugs phenytoin, paraneoplastic, Parkinson plus

large CVAs, SOL, MS

plus all causes of spastic and cerebellar

Midline – paraneoplastic, midline tumour

Spastic and Ataxic combined

Spinocerebellar degeneration

Friederich’s ataxia

Multiple sclerosis

Syphilitic meningomyelitis

Craniospinal junction abnormalities – Arnold-Chiari, meningioma Non conforming

Myasthenia Gravis Mononeuritis multiplex Motor neurone disease

Giddiness/Unsteadiness protocol

Giddiness o Cerebellar o Vestibular o Postural BP

Unsteady gait o Cerebellar o Parkinsonism o Sensory ataxia (Proprioception) o Others – hemiplegic gait, cervical myelopathy etc etc

Examination for unsteady gait o Start with Lower Limbs FIRST

As per LL protocol Concentrate on cerebellar, sensory ataxia and Parkinsonism Examine the gait!

o Proceed with Parkinsonism protocol if Parkinsonian gait o Proceed with cerebellar protocol if cerebellar signs

Lung Transplant Presentation 1

Page 144: Mrcp short case telling skills

Sir, this patient has a bilateral lung transplant as evidenced by presence of a median sternotomy scar associated with normal underlying respiratory findings. The patient has a wrist tag indicating that he is a lung transplant patient. Chest excursion is normal with a normal percussion note, vesicular breath sounds and vocal resonance. Trachea is central and apex beat is not displaced. Hence there are no signs to suggest underlying rejection of the transplanted lung. Patient’s respiration rate is 14 bpm and there are no sign of respiratory distress or failure. Peripheral examination was normal. There was no evidence of nicotine staining, clubbing or short stature. Patient is not Cushingoid with no steroid purpura or thin skin to suggest chronic steroid use. There is also no hypertrichosis or gum hypertrophy to suggest cyclosporine adverse effects. In summary, this patient has bilateral lung transplant and is clinically well currently. The likely underlying causes a. Young patient – Cystic fibrosis, AAT, primary pulmonary hypertension, Eisenmenger (heart-lung transplant) b. Old patient – COPD, IPF and bronchiectasis (always bilateral) Presentation 2 Sir, this patient has a left lung transplant as evidenced by a left lateral thoracotomy scar with a tag indicating that he is a lung transplant patient. There is reduced chest excursion bilaterally with normal precussion note. Auscultatory findings include fine late inspiratory crepitations of the left hemithorax a/w normal vocal resonance, suggesting bronchiolitis obliterans syndrome post transplant. Trachea is central and apex beat is not displaced. There are complications of pulmonary hypertension, cor pulmonale ad polycythaemia. He is also in respiratory distress with a respiratory rate of 24 bpm with use of accessory muscles of respiration. Clinically he is in repsiratory failure and is oxygen dependent on INO2 2L/min; there is no central cyanosis and no signs of CO2 retention such as flapping tremors of the hands or a bounding pulse. In terms of aetiology: Examination of the right hemithorax showed presence of ILD/COPD with features of

a. fine late inspiratory crepitations which are Velcro-like, clubbing b. prolonged expiratory phase with ronch and hyperresonance on percussion of the right

hemithorax and loss of liver dullness, nicotine staining Steroid toxicity and cyclosporin toxicity. In summary, this patient has a left lung transplant for an underlying ILD. There are complications of pulmonary hypertension and cor pulmonale and is clinically in respiratory failure. There also

Page 145: Mrcp short case telling skills

signs to suggest bronchiolitis obliterans syndrome with signs of chronic steroid usage and cyclosporine usage. Questions What are the indications for lung transplant?

Cardiopulmonary o Primary pulmonary hypertension o Eisenmenger’s (heart-lung transplant)

Chronic Lung Conditions o Restrictive pulmonary disease - ILD o Obstructive pulmonary disease – COPD, AAT o Suppurative – Bronchiectasis (must be bilateral tplt), Cystic fibrosis o Sarcoidosis

What are the contraindications for lung transplant?

Disease specific guidelines

Includes o Age o Comorbidities: Absence of concomitant disease that shortens life expectancy

(HIV, Hep B) o Contraindications to surgery/GA (recent MI) o Smoking, alcoholic, poor social support

What are the complications?

Graft dysfunction (reperfusion edema in the first week)

Airway complications – dehiscence, stenosis or bronchomalacia

Rejection o Acute – lymphocytic inflammation, adjust steroid and immune suppression o Chronic – alloimmune inflammatory and non-alloimmune fibroproliferative;

bronchiolitis obliterans syndrome

Infection o CMV – ganciclovir o Aspergillus o Bacterial – pneumonia (Ps aeuroginosa)

How would you manage?

Multidisciplinary, regular follow up, transplant coordinator

3 drug maintenance o Calcineurin inhibitors (Cyclosporin or tacrolimus) o Purine synthesis antagonist ( AZA or MMF) o Steroid

CMV and PCP prophylaxis

Page 146: Mrcp short case telling skills

Marfan’s Syndrome

Examination (seated) Overall Tall, disproportionately long limbs compared to trunk

Upper Limbs Arachnodactyly, thumb sign, wrist sign(overlap > 1cm) collapsing pulse reduced extension of elbows

Face Dolichocephalic(long-headed) Blue sclera, iridodonesis, myopia, ectopia lentis (superolateral) High arched palate Meisher’s elastoma No thyroidectomy scar

Chest Pectus excavatum or carinatum Thoractomy scar (Hx of repair of aortic aneurysm) No gynaecomastia

LL Arachnodactyly

Stand up Kyphoscoliosis Abdomen: inguinal or femoral herniae, hernia scars, striae atrophicae Genu recurvatum Pes planus

Request Cardiovascular examination: MVP, AR Respiratory: scar suggestive of chest tube for pneumothorax, pleurodesis Lower limbs for weakness and numbness (complications of dural ectasia) Measure his arm span to height ratio (>1) Measure pubis-sole to pubis-vertex ratio (>1)

Presentation Sir, this patient has Marfan’s syndrome as evidenced by tall stature with disproportinately long limbs (also known as dolichostenomilia). He has got arachnodactyly with hyperextensible joints with positive thumb sign (Steinberg), wrist sign (Walker), hyperextension of the elbows and genu recurvatum and pes planus. There is presence of dolicocephaly, with iridodenesis, blue sclera and is myopic. He has a high arched palate. I did not detect any Meisher’s elastoma (small papules of the skin of the neck). There is also kyphoscoliosis with pectus excavatum. Of note there are chest wall scars suggestive of previous chest tube insertions. There is no obvious inguinal or femoral hernia, scars or striae atrophicae. There is no collapsing pulse.

Page 147: Mrcp short case telling skills

I did not detect any evidence of malar rash or calve swelling suggestive of a DVT which are features of homocystinuria. There is also no neck scars, mucosal neuromas or hyperpigmentation to suggest MEN type 2B as these patients have a marfanoid habitus. There is also no gynaecomastia or eunuchoid habitus to suggest Klinfelter’s syndrome (say this if patient is a man). I would like to complete my examination by measuring his arm span to height ratio as well as his sole-pubis to pubis-vertex ratio; in addition I would like to perform a cardiovascular examination to look for MVP, AR; a respiratory examination for plurodesis, as well as lower limb examination for weakness or numbness secondary to dural ectasia. Questions What are the differential diagnoses for a patient who has a tall stature? Marfan’s syndrome Homocystinuria Malar flush, mental retardation, inferomedial ectopia lentis Hx of epilepsy, IHD(CABG scar), DVT, osteoporosis Presence of homocystine in the urine via cyanide-nitroprusside test Autosomal recessive inborn error of metabolism of amino acid with deficiency of

cystathionine beta synthetase MEN type 2b Hyperpigmentation, mucosal neuromas(lips, tongue, palate, conjunctiva and cornea),

proximal myopathy MEN 1: Pituitary, parathyroid, pancreatic (PPP) MEN 2a: Parathyroid, adrenals(phaechromocytoma), thyroid (MTC) (PAT) MEN 2b: PAT and hyperpigmentation, mucosal neuromas, marfanoid

Klinefelter’s syndrome Male patient, eunuchoid habitus (arm span> height, sole-pubis>pubis vertex, femenine

fat distribution Gynaecomastia, lack of beard and axillary hair, voice is not masculine, pea-sized testes

(normal >3.5cm), varicose veins Mentally subnormally, infertile Rule out hypo-osmia for Kallman’s syndrome (idiopathic hypogonadotrpic hypogonadism

with hypo-osmia, cleft palate/lip, congenital deafness or blindness which can be treated with gonadotropins and GnRH for fertility)

Raised FSH and estradiol with low testosterone and chromosomal analysis 47XXY(buccal smear for karyotyping)

Infertile as majority are 47XXY (80%) and others can be due to more than 2 X or > 1Y or mosaicism (can be fertile)

Most common cause of male hypogonadism, 1:500 Increased risk of DM, Br cancer and SLE Increases with increasing maternal or paternal age

Page 148: Mrcp short case telling skills

What is Marfan’s syndrome? It is an inherited autosomal dominant connective tissue disorder Affecting the skeletal system, cardiovascular system with ocular abnormalities 1 in 15 000 Male=Female What is the mode of transmission of Marfan’s syndrome? Autosomal dominant Chromosome 15q21 Defects in fibrillin gene How is Marfan’s syndrome diagnosed? Based on the Ghent criteria which takes into account

o Family history o Molecular studies o 6 organ systems

Skeletal Skin Eye CVS Pulmonary Dura (dura ectasia)

What are the ocular features of Marfan’s syndrome? Small spherical lens Cataracts Lens subluxation Glaucoma Hypoplasia of dilator pupillae, therfore difficulty with pupillary dilatation Flat cornea Myopia Retinal detachment Increased axial length of the globe How would you investigate? Molecular studies Annual echocardiography Monitor aortic diameter (normal <40mm, composite graft required if >50mm) MV function

Ophthalmic examination How would you manage? Education and psychological counselling Annual cardio review Beta-blockade (retards rate of aortic root dilatation) Aortic root graft >50mm IE prophylaxis

Eye review What is the prognosis?

Page 149: Mrcp short case telling skills

Death due to cardiovascular complications Aortic dissection CCF secondary to AR

Life span is about mid forties What are the complications of pregnancy in Marfan’s syndrome? Early premature abortion Death from aortic dissection (safe if aortic root<40mm) How would you counsel patient? Affected individuals can transmit the condition to 50% of their offspring. The recurrence risk is 50% if 1 parent is affected. The recurrence risk is small if neither

parent is affected. During counseling, the variability of the disease should be emphasized, as an affected child

may be more or less affected than the parent. How do you assess hypermobility?

Beighton’s 9 point scale

passive dorsiflexion of the little finger beyond 90°

passive apposition of the thumb to the flexor aspects of the forearm

hyperextension of the elbow beyond 10°

hyperextension of the knee beyond 10°

forward flexion of the trunk, with the knees straight, so the palms of the hands rested easily on the floor

If 4 or more => Joint laxity What are the causes for hypermobile joints?

Benign Joint Hypermobility Syndrome (Majority)

Ehlers Danlos Syndrome

Marfan’s syndrome

Osteogenesis Imperfecta What are the causes of blue sclera?

Marfan’s syndrome

Ehlers Danlos Syndrome

Osteogenesis Imperfecta

Pseudoxanthoma elasticum

Chronic steroid intake

Median Nerve Palsy

Examination

Upon suspecting median nerve palsy, rule out ulnar and brachial neuritis

Median nerve palsy

Motor

Wasted thenar eminence

Thumb is externally rotated into the plane of the thumb rather than perpendicular

Page 150: Mrcp short case telling skills

Pen-touch test (for abductor pollicis brevis)

Oschner clasping test (flexor digitorum superficialis)

Flexion of the terminal digit of the thumb (flexor pollicis longus)

Flexion of the terminal digit of the index finger (flexor digitorum profundus)

Sensory

Test for reduced sensation in the lateral 31/2 fingers as well as thenar eminence.

Exclude ulnar and radial nerve palsy

Aetiology

Tinel’s sign

Look for RA hands

Look at the wrist and forearm, elbow, arm and axilla for scars.

Test for function

Rule out Myxedema and acromegaly Presentation Sir, this patient has an isolated unilateral right median nerve palsy with wasting of the right thenar eminence associated with an externally rotated thumb. There is weakness of abduction of the thumb as demonstrated by the pen touch test associated with reduced sensation to pinprick in the right lateral 3 1/2 fingers. Oschner’s clasping test is negative and flexion of the terminal phalanx of the thumb and index fingers are preserved, indicating that the level of the lesion is at the wrist. There is no ulna or radial nerve palsies. In terms of aetiology, there is also no evidence of RA of the hands and patient does not have features of hypothyroidism or acromegaly. Tinel’s sign is negative and there are no scars noted on the right upper limb. Both fine and coarse motor functions are intact. In summary, this patient has a right median nerve palsy at the level of the wrist. Possible aetiologies includes surgical causes such as compression, trauma or surgery or medical causes such as mononeuritis multiples, infection, inflammatory and ischaemic causes. Questions What is the course and branches of the median nerve?

It supplies all the muscles of the forearm except the flexor carpi ulnaris and the ulna half of the flexor digitorum profundus and LOAF (lateral 2 lumbricals, opponens pollicis, abductor pollicis brevis and flexor pollicis brevis)

Formed by lateral(C5-7) and medial(C8,T1) cords of the brachial plexus

Enters the arm closely related to the brachial artery with no branches above the elbow

Enters the forearm lateral to the brachialis tendon and in between the pronator teres.

Gives off the anterior interosseous nerve

Above the wrist, gives off the palmar cutaneous branch

Page 151: Mrcp short case telling skills

Enters the carpal tunnel and supplies LOAF and sensory branch to the lateral 3 ½ fingers.

Branches

Forearm – flexor carpi radialis, flexor digitorum superficialis (flexion of fingers at the PIPJ), pronator teres

AIN – Flexor pollicis longus (flexion of the DIPJ thumb), flexor digitorum profundus of the lateral 2 fingers (flexion of at the DIPJ), pronator quadratus

Palmar cutaneous (to the thenar eminence)

Terminal motor (LOAF) (NB for F for flexion at the MCPJ thumb) What are the various levels of lesions and the clinical correlation?

Wrist-Wasting of thenar, ext rotated thumb, pen touch test positive; sensory loss of the lateral 3 ½ fingers

Cubital fossa Above plus Oschner clasping test positive and failure of flexing the terminal digits of the thumb and index finger

Arm and axilla (same as cubital fossa) (For forearm, depends where the lesion is eg AIN syndrome will affect flexor digitorum profundus and flexor pollicis longus only)

What are the causes?

Trauma

Surgical

Compression

Mononeuritis multiplex

Infection – Leprosy

Inflammatory – CIDP

Ischaemic - Vasculitis Causes of Carpal tunnel syndrome

Idiopathic

Pregnancy, OCPs

Endocrine – Hypothyroidism, Acromegaly

Hands – RA, gout, TB tenosynovitis, OA of carpus

Amyloidosis, CRF, sarcoidosis What are the tests to demonstrate a median nerve palsy?

Tinel’s sign (percussion)

Phalen’s test (flexion at the wrist for 60 s)

Hyperextension of the wrist (for 60 s)

Tourniquet test (sphygmomanometer for more than 2 mins above systolic)

Luthy’s sign – Skinfold does not close tightly around a bottle or cup; secondary to thumb abduction paresis

Durkan’s test – apply direct pressure over the carpal tunnel What are the other areas of nerve compression?

Median nerve (CTS)

Page 152: Mrcp short case telling skills

Ulna nerve (elbow tunnel)

Radial nerve (spiral or humeral groove)

Meralgia paraesthetica (lateral cut nerve of the thigh at the ing lig)

Common peroneal nerve (head of the fibula)

Posterior tibial nerve (Tarsal tunnel syndrome)

Plantar nerves of the 3rd/4th toes (Morton’s metatarsalgia) How would you investigate?

o Blood Ix o Imaging – X-rays o Nerve conduction test demonstrating slow sensory conduction across the

transverse carpal ligament. How would you manage? Education OT and wrist splint Medications – treatment of underlying disease, withdrawing OCPs, IA steroid Surgical decompression What is the prognosis?

o Neuropraxia with no disruption to the sheath or the axon Recovery complete and rapid (weeks)

o Axonotmesis with disruption of the axon but an intact Schwann sheath Recovery complete but slower (1mm/day)

o Neuronotmesis Recovery is incomplete

Median Nerve Palsy

Examination

Upon suspecting median nerve palsy, rule out ulnar and brachial neuritis

Median nerve palsy Motor

Wasted thenar eminence

Thumb is externally rotated into the plane of the thumb rather than perpendicular

Pen-touch test (for abductor pollicis brevis)

Oschner clasping test (flexor digitorum superficialis)

Flexion of the terminal digit of the thumb (flexor pollicis longus)

Flexion of the terminal digit of the index finger (flexor digitorum profundus) Sensory

Test for reduced sensation in the lateral 31/2 fingers as well as thenar eminence. Exclude ulnar and radial nerve palsy Aetiology

Tinel’s sign

Look for RA hands

Look at the wrist and forearm, elbow, arm and axilla for scars.

Test for function

Rule out Myxedema and acromegaly

Page 153: Mrcp short case telling skills

Presentation Sir, this patient has an isolated unilateral right median nerve palsy with wasting of the right thenar eminence associated with an externally rotated thumb. There is weakness of abduction of the thumb as demonstrated by the pen touch test associated with reduced sensation to pinprick in the right lateral 3 1/2 fingers. Oschner’s clasping test is negative and flexion of the terminal phalanx of the thumb and index fingers are preserved, indicating that the level of the lesion is at the wrist. There is no ulna or radial nerve palsies. In terms of aetiology, there is also no evidence of RA of the hands and patient does not have features of hypothyroidism or acromegaly. Tinel’s sign is negative and there are no scars noted on the right upper limb.Both fine and coarse motor functions are intact. In summary, this patient has a right median nerve palsy at the level of the wrist. Possible aetiologies includes surgical causes such as compression, trauma or surgery or medical causes such as mononeuritis multiples, infection, inflammatory and ischaemic causes. Questions What is the course and branches of the median nerve?

It supplies all the muscles of the forearm except the flexor carpi ulnaris and the ulna half of the flexor digitorum profundus and LOAF (lateral 2 lumbricals, opponens pollicis, abductor pollicis brevis and flexor pollicis brevis)

Formed by lateral(C5-7) and medial(C8,T1) cords of the brachial plexus

Enters the arm closely related to the brachial artery with no branches above the elbow

Enters the forearm lateral to the brachialis tendon and in between the pronator teres.

Gives off the anterior interosseous nerve

Above the wrist, gives off the palmar cutaneous branch

Enters the carpal tunnel and supplies LOAF and sensory branch to the lateral 3 ½ fingers.

Branches

Forearm – flexor carpi radialis, flexor digitorum superficialis (flexion of fingers at the PIPJ), pronator teres

AIN – Flexor pollicis longus (flexion of the DIPJ thumb), flexor digitorum profundus of the lateral 2 fingers (flexion of at the DIPJ), pronator quadratus

Palmar cutaneous (to the thenar eminence)

Terminal motor (LOAF) (NB for F for flexion at the MCPJ thumb) What are the various levels of lesions and the clinical correlation?

Wrist-Wasting of thenar, ext rotated thumb, pen touch test positive; sensory loss of the lateral 3 ½ fingers

Cubital fossa

Above plus

Oschner clasping test positive and failure of flexing the terminal digits of the thumb and index finger

Page 154: Mrcp short case telling skills

Arm and axilla (same as cubital fossa)

(For forearm, depends where the lesion is eg AIN syndrome will affect flexor digitorum profundus and flexor pollicis longus only)

What are the causes?

Trauma

Surgical

Compression

Mononeuritis multiplex

Infection – Leprosy

Inflammatory – CIDP

Ischaemic - Vasculitis

Causes of Carpal tunnel syndrome

Idiopathic

Pregnancy, OCPs

Endocrine – Hypothyroidism, Acromegaly

Hands – RA, gout, TB tenosynovitis, OA of carpus

Amyloidosis, CRF, sarcoidosis What are the tests to demonstrate a median nerve palsy?

Tinel’s sign (percussion)

Phalen’s test (flexion at the wrist for 60 s)

Hyperextension of the wrist (for 60 s)

Tourniquet test (sphygmomanometer for more than 2 mins above systolic)

Luthy’s sign – Skinfold does not close tightly around a bottle or cup; secondary to thumb abduction paresis

Durkan’s test – apply direct pressure over the carpal tunnel What are the other areas of nerve compression?

Median nerve (CTS)

Ulna nerve (elbow tunnel)

Radial nerve (spiral or humeral groove)

Meralgia paraesthetica (lateral cut nerve of the thigh at the ing lig)

Common peroneal nerve (head of the fibula)

Posterior tibial nerve (Tarsal tunnel syndrome)

Plantar nerves of the 3rd/4th toes (Morton’s metatarsalgia) How would you investigate?

o Blood Ix o Imaging – X-rays o Nerve conduction test demonstrating slow sensory conduction across the

transverse carpal ligament. how would you manage Education OT and wrist splint Medications – treatment of underlying disease, withdrawing OCPs, IA steroid

Page 155: Mrcp short case telling skills

Surgical decompression What is the prognosis?

o Neuropraxia with no disruption to the sheath or the axon- Recovery complete and rapid (weeks)

o Axonotmesis with disruption of the axon but an intact Schwann sheath Recovery complete but slower (1mm/day)

o Neuronotmesis Recovery is incomplete

Panhypopituitarism (Simmond’s disease)

Clinical o Pale, soft skin, loss of axillary hair, breast atrophy, hypogonadism, gynaecomastia and

galactorrhea (hyperprolactinaemia), pallor and hairlessness (=alabaster skin) o Features of hypothyroidism o Postural hypotension

Etiology o Visual fields for bitemporal hemianopia (Chromophobe adenoma) o Fundoscopy for papilloedema o Radiation marks or signs o Surgical scar marks o Others

Postpartum Necrosis (Sheehan’s syndrome) Pit apoplexy (spont infarct or hemorrhage) Craniopharyngioma TB, sarcoid, metastatic

Ix o Blood test

FBC – NCNC anemia U/E – hypoNa, LF/FSH, GH, PRL, TSH, ACTH Testosterone, estradiol, T4, Cortisol level Stimulation test – Synacthen test, insulin hypoglycemia test (gluc, 2.2 and

check cortisol an GH) o Imaging – MRI o Formal perimetry

Notes o Fail in order of FSH/LH, GH, PRL, TSH and ACTH and lastly ADH o Usually macroadenoma for acromegaly and non functioning tumors o Mx – replacement of steroids and thyroid and sex hormones and GH o Must always replace steroids first then thyroid to prevent adrenal crisis o Steroid replacement may unmask diabetes insipidus

Addison’s disease

Clinical

Page 156: Mrcp short case telling skills

o Weakness, LOA and LOW o Hyperpigmentation

Crease of the palms Mouth and lips Nipples, belt, straps, rings

o Sparse axillary hair, pubic hair, postural hypotension o Associations – vitiligo, polyglandular (hypoparathy, DM, thyroid) o Dy/Dx – Nelson’s syndrome = look for abdominal scar and visual field, Liver, renal

Ix o Confirm dx with synacthen test o Confirm level

ACTH Prolonged ACTH test

Will respond if there is suppression by exogenous steroids or ACTH deficiency

Imaging o If adrenals

AXR (calcification) CT adrenals CXR : TB Adrenal Antibodies

Mx o Replace steroids o Fludrocortisone o Steroid card

Notes o Causes of hyperpigmentation

Addison’s, Nelson’s, ectopic ACTH Liver – PBC, hemochromatosis Uremia Race, suntan Porphyria cutanea tarda

o Causes of Addison’s AI (21 hydroxylase) TB Mets HIV

o Association Graves, Hashimoto, Pernicious anemia AI polyglandular syndromes

Type 1: Addison’s, hypoparathy, chronic mucocutaneous candidiasis

Type 2: Addison’s, hypothy, DM

Page 157: Mrcp short case telling skills

Gynaecomastia

Physiological o Newborn o Adolescence o Ageing

Pathological o Cirrhosis of the liver, renal failure o Hyperprolactinoma, thyrotoxicosis o Klinfelter’s o Malignancy (HCC, Lung CA, Testicular)

Pharmacological o Digoxin, spironolactone, cimetidine, methyldopa, diazepam

Carpopedal spasm---Baliga379 Carcinoid syndrome--Baliga 381

Dupytren’s Contractures

Typically o elderly males o pitting and thickening of the palmar skin progressing to a firm, painless nodule fixed to

the skin and fascia, with subsequent cord involvement o involving bilateral 4th and 5th fingers resulting in fixed flexion of the MCPJ and PIPJ o Garrod’s knuckle pads Check function Feet (plantar fibromatosis), Peyronie’s disease (penile shaft) and retroperotineal fibrosis Establish etiology o Primary

AD Young males

o Secondary/ associations Chronic ethanol ingestion – parotidomegaly, hepatomegaly, CLD Antiepileptics – gum hypertrophy, nystagmus DM especially insulin dependent Hyperlipdaemia (xanthelesma) Smoking (nicotine staining of the fingers) Manual labor and hand trauma

Fibroproliferative disorder Mx o Most do not require o Annual follow up for contracture developing o Intralesional injection of collagenase or interferon gamma o Surgical correction (if >30 contractures of MCPJ or any degree of contracture of PIPJ)

Clubbed Fingers

Page 158: Mrcp short case telling skills

Rule out o Pseudoclubbing of scleroderma and clubbing of thyroid acropachy o Grade

1 – Fluctuation of the nail bed (sponginess) 2 – Scaramoth’s sign (obliteration of the diamond when dorsal surfaces of the

terminal phalanges are opposed together), loss of Lovibond’s angle 3 – Drumstick appearance ie enlarged finger tips 4 – associated with hypertrophic pulmonary osteoarthropathy of wrist and ankle

(subperiosteal reaction and new bone formation) o Causes

Lung

Abscess, bronchiectasis

Pulmonary fibrosis

Ca lung CVM

Cyanotic congenital heart disease

Eisenmenger’s syndrome

Infective endocarditis Abdomen

Cirrhosis

Inflammatory bowel disease

Coeliac’s disease Thyroid Familial Idiopathic

o Unilateral causes AVM of the Lung Axillary artery aneurysm

Painful/Swollen Knee Joint

Examine the joint o Look, feel, move approach, disuse atrophy o Gait o Make sure it’s not Charcot’s joint (see Charcot’s joints) o Examine other joints and proceed to exclude causes

OA RA – palindromic type (acute recurrent arthritis, affecting one joint) Seronegative spondyloarthropathy (4) Crystal arthropathy – gout, pseudogout or chondrocalcinosis Septic arthritis Haemophilia Trauma Others – Lyme’s, Rh heart Disease

Still’s disease/Juvenile Chronic Arthritis

Page 159: Mrcp short case telling skills

Cs (Still’s) – micrognathia, arthropathy of the DIPJ, splenomegaly, LNs, Hx maculopapular rash

Dx of JCA o <16 yrs, >6 weeks of arthritis with no other cause, > 6 months evolve

Still’s disease – arthritis with daily temperature spikes Polyarticular (>5 joints) with early fusion of cervical and mandibular joints Pauciarticular (4 or less) – iritis in girls and sacroilitis in boys

Joint involvement with RF and ANA negative

Systemic symptoms of lethargy, anaemia, growth disturbance, amyloidosis

Rx o Education, counseling andsupport o PT/OT o Analgesia, intra-articular steroids, MTX, hydroxychloroquine

Enteropathic Arthropathy

2 types o Peripheral arthropathies

Non-erosive, migratory and reversible of the large joints (knees, ankles and elbows), occasionally MCPJ and PIPJ of the hands

Parallels disease activity and improves when bowel disease improves o Axial arthropathies

Similar to AS (X-ray are similar) Independent of bowel disease activity

Look for abdominal scar, erythema nodosum

Old rickets

Consider this diagnosis o Paget’s (see Pagets’) o Short stature

Achondroplasia (AD, short stature, prominent forehead, saddle nose, short limbs with no bowing, exaggerated lumbar lordosis, normal trunk, occ spinal cord compression) with normal sexual and mental fuction and life span

Noonan’s syndrome (AD)

Short stature, facial abnormalities(hypertelorism, triangular facies, webbed neck, ptosis, down-slanting eyes), pectus excavatum or carinatum, hyperextensible joints

Cardio – PS, ASD, VSD

Mental retardation

Impaired blood clotting Turner’s syndrome

45XO

In females only

Page 160: Mrcp short case telling skills

Cs – short stature, shield chest, short 4th MC/MT, cubitus valgus, webbed neck, absent breast development with normal pubic hair, cutis laxa(excessive skin), hypoplastic/hyperconcex nails, naevi

CVM – coarctation of aorta, biscupid aortic valve, aortic dissection

Hypothyroid

Ovarian failure with high FSH and LH

Cs of Ricket’s o Bilateral o Symmetrical o Bowing (lateral curvature of the tibia and femur, radius and ulna) o Short stature o No increased warmth o Frontal bossing and parietal flattening o Rickety rosary (thickening of costochondral junction), Harrison’s groove (indentation of

the lower ribs at the diaphragmatic attachment)

Causes of rickets or osteomalacia o Decreased Vit D – sun exposure, malnutrition o Malabsorption – gastrectomy, coeliac’s, pancrease o Abnormal metabolism – CRF, liver dz, RTA, anticonvulsants o Others

Familial hypophosphataemia Vit D resistant rickets

Rickets occurs before closure of epiphyses compared to osteomalacia o Osteomalacia with bone pain, prox weakness with pseudofractures or looser’s zones in

the ribs, pelvis and clavicles/scapula

Raised ALP, low Ca, high PTH and low Vit D

Mitral Regurgitation

Presentation Sir, this gentleman has mitral regurgitation that is moderately severe in nature. There is a pansystolic murmur heard best at the apex which radiates to the axilla. (If it radiates to the carotids – posterior mitral leaflet rupture) This is a grade 3/6 murmur and is not associated with a systolic thrill. The first heart sound is soft and there is presence of a third heart sound(S3). I did not detect any mid-dastolic murmur. The apex is thrusting and displaced, located at the 6th IC at the anterior axillary line. This is complicated by pulmonary hypertension as evidenced by a palpable and loud pulmonary component of the second heart sound associated with a left parasternal heave. There are no clinical signs of heart failure. On the peripheral examination, patient is in atrial fibrillation with an irregularly irregular pulse which is rate controlled at 80 beats per min. There is no bruising to suggest overanticoagulation. There are also no stigmata of infective endocarditis.

Page 161: Mrcp short case telling skills

To complete the examination, I would like to take the patient’s blood pressure, as well as temperature chart for any fever. (Mention abdominal examination, urine dipstick and fundoscopy if clinically suggestive of IE) In summary, this gentleman has mitral regurgitation that is moderately severe in nature, with complication atrial fibrillation and pulmonary hypertension. There are no complications of heart failure or infective endocarditis. My differential diagnoses include IHD, MVP and Rh heart disease. (If thoracotomy scar, think of mitral valvotomy for MS) Questions How do you grade the severity of mitral regurgitation clinically?

Mild – No Pulm hypt

Moderate – Pulmonary hypertension

Severe – LVF, S3 What are the causes of mitral regurgitation? Common causes are MVP, IHD, Rh heart disease and endocarditis Left ventricular dilatation, cardiomyopathy, Marfan’s, Rheumatoid, AS Acute causes: MI, IE, Trauma, Surgery, spontaneous rupture Anterior leaflet: radiates to axilla and back Posterior leaflet: radiates to carotids

Mitral valvotomy if a thoracotomy scar seen If elderly and mild to moderate, typically due to annular calcification What are the differential diagnoses for a pansystolic murmur? MR TR VSD What congenital conditions can be associated with MR? Corrected TGA Partial AV canal Ostium primum atrial defect (cleft mitral valve) What causes a third heart sound? Rapid filling of the left ventricle from the large volume of blood from the left atrium

occurring in early diastole Why is the pulse jerky? Pulse is sharp and abbreviated due to lack of sustained forward stroke volume with a

reduced systolic ejection time because of regurgitant leak into the left atrium How do you differentiate an MDM from severe MR vs MS? MS has an opening snap Severe MR associated with S3 MS murmur is longer MS has loud S1 How do you differentiate between MR and TR murmur?

Page 162: Mrcp short case telling skills

Mitral Regurgitation Triscupid Regurgitation

PSM heard best at Apex PSM heard best at the LLSE

Radiates towards the axilla Radiates towards the right of sternum

Louder on expiration Louder on inspiration

Displaced apex beat Apex beat not displaced

Jerky pulse character Normal pulse character

Normal JVP unless complication

Giant V wave with pulsatile liver

How do you differentiate between an MR murmur and that of a VSD?

MR VSD

Loudest at the apex Loudest at the LLSE

High pitched Harsh, low pitched

Soft S1 Normal S1

What are the types of dynamic manoeuvres that you are aware of and what are their uses?

Respiration Murmurs on the right side louder on inspiration due to increased venous return and

blood flow to the right side of heart Converse is true Valsalva manoeuvre (decrease preload) 3 phases Phase 1 – beginning of maneuver Rise in intrathoracic pressure and a transient increase in LV output

Phase 2 – Straining phase Systemic return falls Reduced filling of the right and left heart chambers SV and BP drops while HR increases Most murmurs become softer and shorter except MVP – Systolic click and murmur begins earlier (LV size is smaller), ie longer

and louder HOCM – murmur is louder as LV volume is reduced

Phase 3 Release of maneuvre Right heart murmurs becomes louder followed by left heart murmurs

Squatting (increases venous return and systemic arterial resistance) Most murmurs are louder MVP – click occurs later and murmur is shorter because LV size increased HOCM – LV size increased which reduced the obstruction to outflow and systolic murmur

is softer Standing

Page 163: Mrcp short case telling skills

Most murmurs are softer except MVP – louder and longer and HOCM - louder

Isometric exercises (increases afterload) AS – Softer murmur as there is reduction of pressure gradient across the valve MVP – click occurs later and murmur is shorter because LV size increased HOCM – LV size increased which reduced the obstruction to outflow and systolic murmur

is softer MR/AR/VSD louder

Amyl Nitrite inhalation Initial relative hypotension MR/AR/VSD decrease AS increases because of increased stroke volume

Later tachycardia phase MS and right murmurs increase Can use to differentiate Austin Flint from MS

What are the signs of severity for MR? Presence of S3 Short MDM Apex thrusting and displaced Pulmonary hypertension CCF What is the pathophysiology of MR? MR leads to LV overload Compensatory LV dilatation Eventually, decompensate resulting in heart failure and increased risk of sudden death Also, regurgitation into the LA leads to enlargement of LA with AF and elevated pulmonary

pressures Should all murmurs be investigated?

All should be Ix except 1. mid-systolic, grade 2 or < murmurs with no associated findings or symptoms 2. continuous murmurs of venous hum or mammary soufflé of pregnancy

How would you investigate this patient? ECG LA enlargement – P mitrale (II – P >0.12s, Limb; Bifid P waves in limb leads with inter-

peak > 0.04s, terminal P negativity in V1) LVH – Sokolow & Lyon Criteria (S in V1 and R in V5 or 6 >35mm) AF Pulmonary hypertension

CXR

Page 164: Mrcp short case telling skills

CCF – pulmonary congestion, enlarged heart Left atrial enlargement Pulmonary artery enlargement

Echocardiogram Dx of MR Severity – EF <60% and LV end-systolic diameter >45mm Cause Complications eg IE

Cardiac catheterisation Not indicated in most patients but useful if there is discrepancy between

echocardiographic and clinical findings Useful to stenosis, regurgitation and intracardiac shunting

How would you manage this patient? Education Medical therapy Antibiotic prophylaxis Treatment of underlying cause eg IHD, dilated CMP (Rx of CCF and afterload reduction) Treatment of complications eg AF, IE, CCF

Surgical Indications Symptomatic or EF<60% or LV end-systolic dimension >45mm

Types of surgery Mitral valve repair if technical feasible is best Mitral valve replacement if technically not feasible provided EF >30%

Controversial Varied causes for MR If due to IHD or dilated CMP, then Sx is controversial If due to MVP, timing of surgery Indicated if

symptomatic, AF, pulmonary hypt, EF<60% or ESD>45 If asymptomatic,

risk stratify according to regurgitant orifice(doppler) <20mm2 20-39mm2 >40mm2 (this

affects Px and closer follow up necessary) How do you diagnose infective endocarditis? Duke’s criteria 2 Major, 1 Major 3 Minor or 5 minor Major

Page 165: Mrcp short case telling skills

Persistently positive blood c/s with typical organism Persistently positive blood c/s 2 or more positive c/s > 12h apart 3 or more positive c/s each 1 hr apart if 4 or more taken, >70% positive

Typical organism Strep viridans, Strep bovis, enterococci, Staph aureus HACEK: Haemophilus, Actinobacillus, Cardiobacterium, Eikenella, Kingella

Endocardial involvement Positive echocardiogram: vegetations, abscesses, valve perforation, dehiscence New valvular regurgitation

Minor Predisposing heart condition Fever Vascular phenomena: arterial emboli, septic pulmonary emboli, mycotic aneurysm,

ICH, Janeway lesion Immunologic phenomena: GN, Osler’s nodes, Roth spots, Rh factor Positive blood c/s not satisfying major criteria Positive echocardiogram not satisfying major criteria

What are the types of endocarditis? Native valve endocarditis Strep, enterococci, Staph Rh, Cong HD, MVP with murmur and degenerative valvular disease

Prosthetic Early(<60 days): Staph aureus or S. epidermis Late(>60 days): Similar to native endocarditis

Fungal :IVDA and ICU IVDA: TV involvement, AV also; Staph aureus, MRSA, fungi, Strep, GNB How would you investigate?

Bloods

Blood C/S (as above)

FBC (NCNC anaemia, raised TW with left shift), ESR, CRP

2D echo

CXR, ECG How do you treat infective endocarditis? General measures Eg oxygen, treat fever

Antibiotics IV CP 12-18MU/d 4H for 4 weeks Can also add IV gentamicin 1mg/kg 8H for first 2 weeks

Page 166: Mrcp short case telling skills

If allergic, use vancomycin 30mg/kg/d in 2 divided doses for 4 weeks HACEK organism: IV ceftriaxone MSSA: IV cefazolin or nafcillin or cloxacillin

Surgery Heart failure Failure of medical therapy Presence of fever and inflammatory syndrome after 1 week of appropriate and

adequate antibiotics Presence of mobile vegetation >10mm with 1 major embolism 1 week A/B Presence of mobile vegetation >15mm with 1 week of A/B

Valvular complication eg valvular abscess, valvular obstruction, rupture into the pericardium, septal formation, fistula

Fungal endocarditis Prosthetic valves esp if unstable or early(<60 days) or caused by S aureus

When and how should you prophylax against IE? (3 steps: Risk stratify, Type of Procedure and Type of antibiotics) Risk stratify

Highest risk: prosthetic valves, both bioprosthetic and mechanical; previous IE; congenital cyanotic heart disease; or surgically produced systemic/pulmonary shunts.

Moderate risk: (1) all other congenital cardiac conditions, except isolated secundum atrial septal defects and surgical repairs of an atrial septal defect or patent ductus arteriosus or ventricular septal defect more than 6 months ago; (2) acquired valvular dysfunction (eg, rheumatic heart disease, calcific aortic stenosis); or (3) hypertrophic cardiomyopathy and mitral valve prolapse with valvular regurgitation and/or thickened leaflets. Thickening of the anterior leaflets of the mitral valve correlates with significant mitral insufficiency, especially in men older than 45 years.

Low risk: mitral valve prolapse without significant regurgitation or thickened leaflets on echocardiography, implanted cardiac PMs, implanted defibrillators, implanted coronary stents, or "innocent" murmurs. An important caveat is that in elderly individuals, an innocent murmur may not be hemodynamically significant but may signify the presence of a calcified leaflet that is susceptible to infection during a transient bacteremia.

Procedures that require antibiotic prophylaxis in high-to-moderate risk patients are as follows:

Invasive manipulation of the respiratory tract (eg, tonsillectomies, rigid bronchoscopy)

Gastrointestinal surgery, biliary tract surgery, sclerotherapy of esophageal varices, dilatation of esophageal strictures, and endoscopic retrograde cholangiopancreatography in the presence of biliary obstruction

Prostate surgery, cystoscopy, and urethral dilatation

Generally, hysterectomies, vaginal delivery, cesarean delivery, urethral catheterizations, dilation and curettage, therapeutic abortions, sterilization procedures, insertion or

Page 167: Mrcp short case telling skills

removal of intrauterine devices, cardiac catheterizations, angioplasties, or endoscopies with or without biopsies do not require prophylaxis.

Adult antimicrobial IE preventive regimens for dental, oral, respiratory tract, or esophageal procedures recommended by the American Heart Association to prevent streptococcal IE from oral-dental sources are as follows:

Administer amoxicillin at 2 g orally 1 hour before the procedure or ampicillin at 2 g IM or IV 30 minutes before the procedure.

If the individual is allergic to penicillin, clindamycin at 600 mg, cephalexin at 2 g, or azithromycin at 500 mg orally 1 hour before the procedure are alternatives.

If the individual is allergic to penicillin and is unable to take oral medication, clindamycin at 600 mg IV or cefazolin at 1 g IM or IV should be given 30 minutes before the procedure.

Adult IE prophylactic regimens for individuals undergoing lower gastrointestinal tract surgery or instrumentation of genitourinary tract procedures are for preventing enterococcal endocarditis. They are as follows:

High Risk regimens recommended by the American Heart Association are ampicillin at 2 g IM or IV plus gentamicin at 1.5 mg/kg (not to exceed 120 mg) within 30 minutes of the procedure, followed by ampicillin at 1 g IM, IV, or orally 6 hours later.

High-risk individuals who are allergic to penicillins should receive vancomycin at 1 g IV over 1-2 hours plus gentamicin at 1.5 mg/kg IV or IM (not to exceed 120 mg) within 30 minutes of starting the procedure.

For moderate-risk patients, amoxicillin at 2 g orally 1 hour before the procedure or ampicillin at 2 g IM or IV within 30 minutes of starting the procedure is recommended.

The alternative for patients who are allergic to penicillin who are at moderate risk is vancomycin at 1 g IV over 1-2 hours, completed 30 minutes before the procedure.

What is the prognosis? Depends on underlying cause If IHD or dilated CMP, Px dependent on the underlying disease If due to MVP Asypmtomatic regurgitation is a serious disease with a 5-yr death rate of 22-33% form

cardiac adverse events

Page 168: Mrcp short case telling skills

Mitral Stenosis

Presentation Sir, this patient has mitral stenosis which is severe in nature with complications of infective endocarditis and congestive cardiac failure. There is presence of a mid diastolic murmur heard best at the apex and is accentuated in the left lateral position. It is a grade 3/6 murmur and is not associated with any diastolic thrill. It is severe as it is associated with an early opening snap and a long mid diastolic murmur. The first heart sound is also loud. The apex beat is tapping in nature and is not displaced, located just medial to the mid-clavicular line in the 5th intercostal space. There is associated pulmonary hypertension with a palpable pulmonary component of the second heart sound with left parasternal heave. There is a loud pulmonary component of the second heart with a presence of a functional TR as evidence of PSM at the LLSE that is louder with inspiration associated with a giant V wave which is elevated at 5cm. There is no associated Graham Steel murmur (PR). This is associated with congestive cardiac failure with bilateral basal crepitations and bilateral pedal edema. Examination of the peripheries reveals evidence of stigmata of infective endocarditis. Patient is clubbed with Janeway lesions on the palms and Osler’s nodes noted on the pulp of the fingers. There are also splinter haemorrhages with presence of conjunctival pallor. There is presence of a peripherally inserted central catheter suggesting use of long term antibiotics. The patient is on IV cloxacillin suggesting that the infective organism is MSSA. There is complication of atrial fibrillation. The HR is irregularly irregular with a rate of 84 bpm. There is a characteristic pulsus parvus pulse. There are no bruises to suggest overanticoagulation. I did not notice any mitral facies. The patient’s voice is also not hoarse which may suggest Ortner’s syndrome. (mention the lateral thoracotomy scar with possible mitral valvotomy as intervention) I would like to complete the examination by looking at the patient temperature chart as well as taking the patient’s BP.

Page 169: Mrcp short case telling skills

In summary, this patient has MS that is severe in nature with complications of atrial fibrillation, pulmonary hypertension congestive cardiac failure and infective endocarditis secondary to MSSA. The possible causes for MS include rheumatic heart disease or congenital parachute valves. Questions How do you grade the severity of MS clinically?

Mild – no PHT

Moderate – PHT

Severe - CCF What are your differentials for a mid-diastolic murmur? MS Atrial myxoma Ball-valve thrombosis Flow across the TV in ASD MR with increased flow through the mitral valve during diastole Austin Flint murmur (in severe AR) What are the causes of mitral stenosis? Common Rheumatic heart disease Congenital parachute valve

Rare Calcification of mitral annulus and leaflets CTDs: SLE, RA Carcinoid (malignant)

What causes a tapping apex beat? An accentuated first heart sound What causes an opening snap? Opening of a stenosed mitral valve and indicates that leaflets are pliable Why is the first heart sound loud? The mitral valve is held open during diastole by the transmitral gradient. The valve is

suddenly slammed shut during ventricular contraction What causes presystolic accentuation of the murmur? Occurs in sinus rhythm only during atrial systole which increases flow from the LA to the LV

through the stenotic valve

Page 170: Mrcp short case telling skills

How do patients present? Asymptomatic Women, may have h/o RH heart disease Symptoms ppt especially during pregnancy or development of AF Usually left-sided heart failure: exertional dyspnea, PND, orthopnea Less frequently with right sided heart failure, hemoptysis and hoarseness of voice, although

this symptoms are more specific How do you diagnose rheumatic fever? Duckett-Jones criteria 2 major or 1 major + 2 minor Major Carditis, Sydenham’s chorea, SC nodules, erythema marginatum, arthritis

Minor Past H/o RHD, fever, arthralgia, prolonged PR, ESR, CRP + a h/o streptococcal infection (ASOT raised, recent scarlet fever, +GpA

Strep throat c/s or kits for GpA Strep with high specificity but low sensitivity.

How do you manage Rheumatic fever?

Prevention o Primary prevention: Rx with IM Benzathine Pen G 1 dose or 10 days of Pen V o Secondary prevention: all patient with history of Rh fever should receive

prophylaxis; IM Pen G once/month or PO Pen V daily bd What are the signs of severity for MS? Early opening snap Long MDM Pulmonary hypertension CCF Pulsus parvus What is Ortner’s syndrome? Hoarseness of voice from compression of the recurrent laryngeal nerve from an enlarged left

atrium What is Lutembacher’s syndrome? Association of MS with ASD How would you investigate? ECG P mitrale P pulmonale, RVH, RAD, AF

CXR

Page 171: Mrcp short case telling skills

Calcified mitral valve Enlarged LA (double silhoutte sign, straightening of the left heart border, filling of the

pulmonary bay by enlarged LA, horizontalization of the left bronchus) Prominent pulmonary trunk Pulmonary congestion (Upper lobe diversion, Kerly B lines)

Echocardiogram Dx Assess severity (Valve area calculation and transmitral gradient) Look for complications (eg IE) Assessment suitabilty for balloon valvotomy

What is the normal cross-sectional area of the valve? 4-6 cm2 What is a significantly stenosed mitral valve? <1 cm2 and >10 mmHg gradient across the valve How would you manage? Education Medical treatment Antibiotic prophylaxis Treat complications AF – anticoagulation and rate control CCF – symptomatic treatment vs improvement of mortality

Surgery Indications Symptomatic (limit activity) with significant stenosis (<1 cm2 and >10mmHg) Pulmonary hypertension Hemoptysis Recurrent thromboembolic events despite adequate anticoagulation

Type Valvotomy Closed Closed mitral valvotomy Balloon valvuloplasty (procedure of choice) but need to satisfy Good mobility of valves Minimal calcification No or mild MR Minimal subvalvular disease

Open valvotomy Mitral replacement

In which trimester does pregnancy results in symptomatic MS? Second trimester due to increase in blood volume

Page 172: Mrcp short case telling skills

Mitral Valve Prolapse (Floppy MV, Barlow’s syndrome, Click-murmur syndrome) Presentation Sir, this patient has got a MR that is severe and secondary to a mitral valve prolapse. I say this because there is a presence of a mid-systolic click associated with a late systolic crescendo-decrescendo murmur heard best at the apex. This murmur radiates towards the axilla and is a grade…..(present as for MR) I would like to complete my examination by asking the patient to perform the valsalva manoeuvre as well as to stand to accentuate the murmur; take BP and temperature chart as well as a neurological examination for signs of stroke. Questions What causes a mid-systolic click?

o Inability of the papillary muscles or the chordae tendinea to tether the mitral valves in the late stages of systole

o The prolapsing of the valve leaflet into the LA and sudden tensing of the mitral valve apparatus causes the mid-systolic click

Page 173: Mrcp short case telling skills

What are the differential diagnoses for a systolic murmur?

o AS o PS o MVP o HOCM

What are the causes and associations of a MVP?

o Myxomatous degeneration of the mitral valve tissue o Associated with

o Heart conditions ASD (Secundum type) Cardiomyopathy Myocarditis

o Systemic conditions Marfan’s syndrome Ehlers danlos syndrome Osteogenesis Imperfecta Polycystic Kidney disease SLE

What manoeuvres can accentuate the findings of an MVP and why?

o Valsalva manoeuvre and standing o Decrease preload o Reduction in the cardiac volume o Further imparing the papillary muscles or chordae tendinea from maintaining tension on

the leaflets and preventing the leaflets from prolapsing into the LA o Hence the systolic click occurs earlier with a longer duration of the systolic murmur

How do patients with MVP present?

o Asymptomattic o Symptomatic

o Palpitations o Anxiety o Atypical chest pain o Light-headedness

o Complications of MR o CCF – fatigue and dyspnea o IE o Arrythmias o Embolic phenemenon

o Sudden death How would you Ix?

Page 174: Mrcp short case telling skills

o Echocardiogram o Confirm Dx o Complications of MR

How would you manage?

o Education and reassurance o Medical

o Antibiotic prophylaxis Only if associated with MR Otherwise not necessary if just MVP

o Symptomatic Palpitations Rx with beta blockers (benign ventricular ectopy)

o Rx underlying cause or associations o Rx complications such as MR with CCF, IE or AF or TIA

o Surgical o As for MR

Mixed Aortic Valve Disease

Sir, this patient has got mixed aortic valve disease and the predominant lesion is Aortic Stenosis 1. small volume pulse 2. heaving and undisplaced apex beat 3. Loud and harsh systolic murmur 4. associated with systolic thrill Aortic regurgitation 1. Collapsing pulse 2. Displaced and thrusting apex beat 3. Soft systolic murmur 4. No systolic thrill My findings are:

o Presence of an ESM heard best at he aortic area that radiates towards the carotids. It is a grade 4/6/ murmur and it is associated with a systolic thrill. It is severe as it is associated with an early ejection click with a long systolic murmur with late peaking. There is no S4 detected and the second heart sound is soft; I could not detect a paradoxial splitting of the second heart sound.

o There is also an EDM heard bset at the LLSE and is loudest in expiration with the patient sitting forwards. It is a grade 3/6 murmur and is not associated with any diastolic thrill. (skip the severity markers for AR)

o Apex o CCF o IE, SR and small volume, haemolytic anaemia

Page 175: Mrcp short case telling skills

o Request BP especially for narrow pulse pressure, Temperature chart and enquire symptoms of angina, syncope and dyspnea.

o In summary

o Presence of an EDM heard best at the LLSE and is loudest in expiration with the patient sitting forwards. It is a grade 3/6 murmur as it is not asssociatd with ay diastolic thrill. The second heart sound is soft and there is no third hear sound. There is also no mdm at the paex to suugest an Austin Flint murmur.

o There is also an ESM heard best at the aortic area that radiates towards the carotids. It is a grade 2/6/ murmur and is not associated with any systolic thrill. (skip the severity markers for AS)

o Apex o CCF o IE, SR, collapsing, brachial dance and Corrigan’s o No quinke, muller’s de Musset’s, Duroziez and Traube’s o No Marfans, AS or RA o Requests for BP especially for a wide pulse pressure, temperature chart. o In summary

Questions What are the causes of a mixed aortic valvular lesion?

o Rheumatic heart disease o Biscupid aortic valve

Mixed Mitral Valve disease

Sir, this patient has mixed mitral valve disease and the predominant lesion is 1. Mitral stenosis

Early opening snap with long diastolic murmur

Loud S1

No S3

Tapping apex beat that is not displaced

Pulsus parvus pulse 2. Mitral Regurgitation

Displaced apex beat that is thrusting in nature

Soft S1

S3

Jerky pulse

Mdm is a short diastolic murmur with no opening snap My findings are:

o Presence of a MDM heard best at the apex and accentuated at the left lateral position. It is a grade 4/6 murmur as it is associated with a diastolic thrill. It is severe as it is associated with an early opening snap and a long mdm. There is a loud first heart sound.

Page 176: Mrcp short case telling skills

There is also a MR murmur as evidenced by a PSM heard best at the apex. It is a grade 3/6 murmur and is not associated with any systolic thrill. There is also no third heart sound.

o Apex beat is not displaced o Pulmonary hypertension, CCF o IE, AF, over-anticoagulation o Mitral facies and Ortner’s o No lateral thoracotomy scars to suggest previous operation for MS o Requests o In summary, this patient has mixed mitral valves disease with the predominant lesion

being MS. This is complicated by pulmonary hypertension and AF. He is not in heart failure and signs of IE. The most likely cause in this patient is rheumatic heart disease.

o Presence of a PSM heard best at the apex; 4/6 murmur and is associated with a systolic

thrill. There is a soft first hearts sound and a presence of a third heart sound. There is also a MS as evidenced by MDM heard best at the apex at the left lateral position. It is associated with a late opening snap and short mdm.

o Apex o CCF, Pulmonary hypt o IE, AF o Lateral thoracotomy scar o Marfan’s and SLE o Requests o In summary

Questions What is the significance of a third heart sound in mixed mitral valve disease?

o Presence of S3 implies no significant MS What are the causes?

o Rheumatic heart disease o MS with valvotomy done that has been complicated by MR

What are the frequencies of valvular involvement in rheumatic heart disease?

o MV – 80% o AV – 50% o Mixed MV and AV – 20% o TV – 10% o PV -1%

Myasthenia Gravis

Examination Statement Examine face, CNs eyes

Page 177: Mrcp short case telling skills

Approach to Ptosis (See Ptosis) and weakness Sequence Eyes Ptosis with fatigability Variable strabismus and diplopia that occurs after some time Check for hyperthyroid and thyroid eye disease Check for anaemia Check for malar rash of SLE

Face VII – show your teeth : snarls Assessment of speech : Yeeee or count 1 to 20, nasal voice (bulbar palsy) Masseter weakness but pterygoids normal\ Check neck for goitre and scars

ULs Normal deep tendon reflexes (Eaton Lambert and Miller Fisher are reduced) Normal sensation Fatigability with weakness RA and SLE features

Thymectomy scar and plasmapheresis line Associations Endo: Thyroid, DM, Pernicious anaemia CTD: RA, SLE, Polymyositis

Requests Drug Hx D-penicillamine usage for RA or Wilson’s Antibiotics and CVS drugs that can worsen MG

Temperature chart for fever – precipitate weakness

Presentation Sir, this patient has myasthenia gravis as evidenced by presence of muscle weakness with fatigability. The patient’s deep tendon reflexes and sensation are normal. On examination of the face, there is presence of bilateral facial muscle weakness producing a mask-like facies with ptosis. I noticed furrowing of her forehead musculature in an attempt to compensate for the ptosis. There is presence of variable strabismus and diplopia after sustained gaze. I also noticed the presence of a nasal voice as well as a reduction in volume of his speech when asked to count from 1 to 20. The patient does not require a nasogastric tube. I did notice mid line sternotomy scar which suggest previous thymectomy.

Page 178: Mrcp short case telling skills

There is no goitre or features of hyperthyroidism. Patient also does not have symmetrical deforming polyarthropathy to suggest RA and has no cutaneous features of SLE. I also did not notice any diabetic dermopathy. I would like to complete my examination by Checking his negative inspiratory force Temperature chart Drug history In summary, this patient has myasthenia gravis with mild weakness of which a thymectomy has been done previously. Questions What is Myasthenia Gravis? Autoimmune condition with antibodies targeting the post-synaptic Ach receptors of the

neuromuscular junction Resulting in progressive muscle weakness with use of the muscle and recovery of strength

after a period of rest Weakness experienced once number of receptors is 30% or less How common is the thymus involved? 75% of cases of which 15% are thymomas and 85% are thymic hyperplasia What are the common presentations? Age 2 peaks 20 to 30 years old with female predominance >50 years old with male predominance

Ptosis, diplopia Dysarthria, difficulty swallowing (isolated bulbar muscles involvement occurs in 20%) Generalised weakness or reduced exercise tolerance Respiratory failure in 1% Tends to occur extraocular muscles first, then to facial to bulbar and to limbs and truncal What can exacerbate MG or precipitate crisis? Non compliance to medications Infection Emotions Drugs Antibiotics: aminoglycosides, tetracyclines, macrolides and fluoroquinolones CVS : Beta blockers, Calcium channel blockers (verapamil) Others : Chloroquine, quinidine, procainamide, Li, Mg, Prednisolone, quinine(in gin tonic

drinks), penicillamine

Page 179: Mrcp short case telling skills

What is cholinergic crisis? Can cause confusion between myasthenic crisis from cholinergic crisis Results from excess of cholinesterase inhibitors such as neostigmine and physostigmine Causes flaccid paralysis and SLUDGE (Miosis, salivation, lacrimation, urinary incontinence,

diarrhea, gastrointestinal hypermotility and emesis) How would you investigate? Blood Ix AchR Ab Positive in 80% with generalised MG Positive in only 50% with ocular involvement only also present in 90% of patients with penicillamine induced MG

Antistriated musce Ab Anti Muscle specific kinase Ab (Anti MuSK Ab – positive in patients with AchR Ab –ve) FBC to rule out infection

Imaging CXR – thymus (anterior mediastinal mass), aspiration pneumonia CT for thymus

Tensilon test Dx and distinguishing from cholinergic crisis Edrophonium (T1/2 10 mins) Look for objective improvement in ptosis (require observer) Cardiac monitoring for bradycardia and asystole (Rx with atropine) 1 mg test dose and up to 10 mg In cholinergic crisis, will get increased salivation etc Note that in ALS, improvement in muscle weakness also occurs

Ice Pack test Ice applied with glove to eyelids for 2 mins Improvement in ptosis is dx (positive in 80%)

Electrodiagnostic studies Repetitive nerve stimulation test – shows a decrease in the compound muscle action

potential by 10% in the 4th or 5th response to a train of nerve stimuli Single fibre nerve electromyography – evidence of neuromuscular blockade with

increased jitter How do you grade the severity of the weakness? Myasthenia Gravis Foundation of America

o Grade 1 – affects the ocular muscles only o Grade 2 – mild weakness affecting muscles other than ocular muscles

2A – Affects the limb and axial muscles 2B - Affects the respiratory and bulbar muscles

o Grade 3 – moderate weakness (3A AND 3B) o Grade 4 – Severe weakness (4A and 4B)

Page 180: Mrcp short case telling skills

o Grade 5 – Intubation required Osserman’s grading

o I: Ocular o II A: Mild generalised with slow progression o II B: Moderate generalised o III: Acute fulminant MG o IV: late severe MG (takes 2 yrs to progress from I to II)

How would you manage? Emergencies in crisis (ABC) Treat exacerbating factors Stop medications that can exacerbate Treat fever with antipyretics Treat infections

Oral pyridostigmine, neostigmine Steroids, azathioprine, cyclosporine Plasmapheresis IVIG Thymectomy What are the complications? Myasthenic crisis Severe exacerbation of MG 10% require intubation

Treatment complications Cholinergic crisis Cx of medications

What is Eaton-Lambert syndrome? Myasthenic disorder associated with malignancy such as small cell ca of lung Affects the proximal (especially the pelvic girdle and thigh) and truncal musculature; bulbar

muscles is rarely involved Improves with exercise Presence of Abs to calcium channels

Page 181: Mrcp short case telling skills

Negligence – “Dealing with the Angry Patient” Scenarios

Delay in Investigative Management

Postponing An Investigation

Adverse Incident (Wrong Medications) Introduction Task – Acknowledge/Apologise, Listen, Explain, Plan

Acknowledge & Apologise & Assure (The 3As) o Acknowledge that the delay or the mistake is a serious one o I am very sorry to have caused you so much worry and distress o I want to let you know that as we speak, your mother is now stable

Listen and Look her in the eye o Expect anger, crying, wanting to complain or to sue the doctor

Explain o Never openly criticize your colleague or the department o Factual – eg “breakdown in communication which should not have happen”

The reason/s for the mishap “The antibiotic was given in good faith in the best interest of her mother” “It was the best treatment for her condition and was given as I did not know

then she was allergic” “ I sincerely did not know that your mother was allergic to the medication”

o What you have done to correct the situation The corrective measures to stabilise patient I have labelled all her notes personally with an “Allergy” sticker, tagged her

with a allergy tag and keyed the information into our computer records personally

Plan (for the patient, consultant and incident report) o Rectify the problem right now

eg calling the radiologist now Going down personally to speak to the radiologist Get patient’s phone no, stay in clinic and I’ll get it sorted out now Other measures to speed up process from now onwards to “make up for lost

time”, “earlier report date” “Earlier referral to oncologist/surgeon”

Page 182: Mrcp short case telling skills

“I will personally call the doctor in the ward to handover your mother’s condition and allergy personally”

o Inform the consultant o Clinical Incident Report to the Risk Management Team

“This is a mistake that should not have happened” “measures to ensure that this does not happen again” “I’ll inform you of the outcome in 3 days’ time”

o If patient, satisfied, then proceed as above o If patient wants to complain, then obtain contact information for him to do so

Neurofibromatosis (von Recklinghausen’s disease)

Examination

Spot diagnosis

Look at the arms for café-au-lait spots, axilla for freckles

Look at the face o Eyes – Lisch nodules (brown pigmentation of the iris) o Ears – deafness

Lower limbs o Bowed legs, pseudoarthrosis

Request o Chest – café-au-lait spots, axillary freckling, kyphoscoliosis o Fundi – optic gliomas, retinal harmatomas o Abdomen – auscultate for renal bruit o Pulses for coarctation o BP – hypertension in renal artery stenosis, coarctation, phaeochromocytoma o Family Hx

Presentation Sir, this patient has neurofibromatosis type 1 as evidenced by presence of multiple neurofibromas which are subcutaneous nodules some of which are pedunculated with a generalised distribution associated with cafe-au-lat spots which are brown macules present on the upper limbs and chest as well as axillary freckling. On examination of his eyes, there are Lisch nodules detected and no deafness on screening (acoustic neuroma). Examination of the lower limb does not reveal any bowed legs or pseudoarthrosis. I would like to complete the examination by:

o Chest – café-au-lait spots, axillary freckling, kyphoscoliosis, lung fibrosis o Abdominal examination o Pulses o Fundi – optic gliomas, retinl harmatomas o Cranial nerve examination – V, VI, VII, VIII, Cerebellar

Page 183: Mrcp short case telling skills

o BP – hypertension in renal artery stenosis, coarctation, phaeochromocytoma o Family Hx

Questions What are the type of neurofibromatosis? Type 1 and 2 What are the diagnostic criteria? Type 1 - 2 or more of: Café-au-lait spots (6 or more, each >15mm in diameter) Neurofibroma (2 or more) or plexiform Freckles in the axillae or inguinal (Crowe’s sign) Bone lesions – Sphenoid dysplasia Lisch nodules FHx – 1 or more first degree relative Type 2 – Either 1. Bilateral eight nerve palsy on CT/MRI OR 2. First degree relative with type 2 and either a. unilateral eighth nerve mass or b. 2 or more: neurofibroma, glioma, schawnomma, meningioma or

juvenile posterior subcapsular lenticular opacity. What are Lisch Nodules? These are melanocytic harmatomas of the iris that appear as well-defined, dome-shaped elevations projecting from the surface of the iris which are yellow or brown in color. Incidence increases with age and by 20 years, 100% have them. Why are the possible causes of hypertension? Renal artery stenosis Pheachromocytoma Coarctation of the aorta

Page 184: Mrcp short case telling skills

What are the histology of the skin tumors? Schwanommas Neurofibromas What is the mode of inheritance and on which chromosome? Autosomal dominant Type 1 – Chromosome 17 (Neurofibromin which is a tumor supp gene) Type 2 – Chromosome 22 (tumor supp gene) If a parent is affected, child has 50% of being affected If family has history of type 2, others should be screen with hearing tests and

brainstem auditory evoked response.

How would you investigate? Diagnosis is clinical. How would you manage? Education and counselling Most do not require treatment Managing complications hypertension, excision of painful neurofibroma,

radiation of optic glioma. What are the associations and complications of the disease?

1. Childhood leukaemia 2. CNS complications

a. Mental retardation b. Epilepsy c. Obstructive hydrocephalus sec to stenosis of aqueduct of Sylvius d. Optic gliomas, acoustic neuromas, meningiomas e. Cord compression form spinal nerve root involvement

3. Sarcomatous changes 4. Bony complications

a. Intraosseous bone cysts b. Bowed legs secondary to thinning of the cortices of long bones c. Pseudoarthrosis of the tibia d. Rib notching e. Sphenoidal dysplasia

Page 185: Mrcp short case telling skills

5. Lung cysts 6. Hypertension

a. Coarctation b. Renal artery stenosis c. Phaeochromcytoma (5%)

What can be a possible differential diagnosis? Dercum’s disease. What are the other neurocutaneous conditions that you are aware of?

Tuberous sclerosis (spot diagnosis) (= Bourneville’s or Pringle’s disease) o Hx of epilepsy or seizures, FHx – Aut dominant o Face – adenoma sebaceum (angiofibromas distributed in a butterfly pattern over

the cheeks, chin and forehead (dy/dx acne, Cushingoid?) o Chest and back

Shagreen patches (leathery thickenings localised patches over the lumbosacral region)

Ash-leaf hypopigmentation Café-au-lait macules

o Hands – subungal fibromata o Systemic

CVS – CCF and arrythmias, cardiac rhabdomyomas Resp – fibrosis Abdomen – polycystic kidneys, renal angiomyolipomas CNS – retinal harmatomas

o Mx Education Rx seizures

Sturge-weber syndrome o Spot Dx o Hx – seizures, hemiparesis, hemisensory, mentally retarded o Signs

Port wine stains in V1 and V2 distribution Hypertrophy of area involved Hemangiomas of the iris Fundus for choroidal haemangiomas BP for hypertension secondary to phaeochromocytoma

o Ix – SXR tramline calcification parietal-occipital lobe o Mx

Seizures control Skin – photothermolysis Eye – screen for glaucoma and Mx choroidal angiomas

Von-Hippel-Lindau disease

Page 186: Mrcp short case telling skills

Optic atrophy Examination

Fundus o Papilloedema o Optic cup (Glaucoma) o DM changes o Retinitis pigmentosa o Attenuated arterioles and veins as in CRAO

Eye o RAPD (MS) o Argyll-Robertson pupil o INO (MS) o Nystagmus (MS and FA)

Head o Tender temporal arteritis o Pb lines in the gums o Paget’s facies

Hands o Cerebellar o AF

Presentation Sir, this patient has unilateral/bilateral optic atrophy. On examination of his fundus, I did not detect any presence of papilloedema, deep optic cup, DM changes or RP. The vessels are also not attenuated. I would like to complete the examination by testing patient’s visual acuity and visual fields as well as for features of MS with RAPD, INO, nystagmus/cerebellar signs, palpate the temporal artery and examine the gums for Pb lines and pulse for AF Questions What are your differential diagnoses?

Unilateral o Demyelinating disease o Compression (Tumor, aneurysm, Paget’s) o Glaucoma o Ischaemic

Thromboembolic Vasculitis – temporal arteritis, tertiary syphilis

Bilateral o Toxic

Nicotine, alcohol Drugs (ethambutol, chloroquine, methanol, Pb, Arsenic)

Page 187: Mrcp short case telling skills

o Metabolic B12 deficiency, B1 and B6 Diabetes mellitus

o Hereditary FA Leber’s (mitochondrial dz with pt mutations) DIDMOAD (DI, DM, Optic atrophy, Deafness) – rare recessive

o Others sec to papilloedema sec to retinitis pigmentosa

How do patients present?

o Reduction in visual acuity What would you find on visual filed testing?

o Central scotoma What is Leber’s optic atrophy or Leber’s hereditary optic neuropathy (LHON)?

o Hereditary condition affecting males o Progressive visual loss o Onset form second decade onwards o Mitochondrial disease with point mutations

What investigations would you do?

o Electroretinography and pattern evoked visual response o Blood test

Blood glucose ESR VDRL B12 levels

o Imaging SXR – enlarged sella turcica – pit tumor CT scan – SOL MRI – demyelinating disease

o (History for toxic and hereditary causes)

Osteoarthritis of the Hands

Examination

Herbeden’s nodes and Bouchard’s nodes, squaring of the thumb

Presence of active arthritis

No muscle wasting

Tinel’s sign

Function

Page 188: Mrcp short case telling skills

o ROM o Coarse fn o Fine fn

Establish cause o Primary o Secondary to Acromegaly, Hemochromatosis

Request o Examine other joints

Knee – varus/valgus deformity, crepitus, wasting of the quadriceps Hips Cervical spondylosis, Lumbar spondylosis Gait (Trendenlenberg’s sign – downward tilting of the pelvis on the affected side)

Presentation Sir, this middle-age lady has OA of the hands as evidenced by presence of Herbeden’s nodes which are bony swelling affecting the DIPJ. I did not detect any Bouchard’s nodes but there is presence of squaring of both hands as a result of subluxation of the first MC. There is no significant muscle wasting with preservation of function. ROM was good and patient is able to perform coarse fn such as turning a door knob and fine motor fn such as transferring coins. Tinel’s sign is negative. I would like to complete the examination by examining other joints for OA in particular

Knees

Hips

Gait for Trendelenberg’s sign

Cx and Lx spondylosis There are no features of acromegaly or hemochromatosis. I would like to offer the dx of Nodal OA or primary generalized OA with OA of the hands occurring in this middle aged lady. Questions What are Herbeden’s nodes?

Bony swellings at the DIPJ in OA What are Bouchard’s nodes?

Bony swelling at the PIPJ in OA Why is there squaring of the hand?

This is due to subluxation of the first MC

Page 189: Mrcp short case telling skills

What are the types of OA?

Primary generalized OA aka nodal OA

Middle-aged women, Autosomal dominant

OA of the DIPJ with Herbeden’s with marked deformity and preservation of fn

Also affects the carpometacarpal joint of the thumb, knees and hips

Secondary

Trauma

Inflammatory arthropathies – RA, Septic arthritis, gout

Endocrine – Acromegaly, hyperparathyroidism

Metabolic – chondrocalcinosis, hemochromatosis

Neuropathic joints – DM, Tabes, syringomyelia How would you Ix?

Radiographical

Subchondral bone cysts and sclerosis

osteophytes

narrowed joint space

Varus/varus deformity

If a synovial aspirate is done to r/o other causes

<100wbc/ml How would you manage?

Education and counseling o Appropriate footware o Weight management

PT/OT

Pharmocotherapy o Glucosamine, chondroitin sulphate, glycosaminoglycans o Drugs

Analgesia – paracetamol, NSAIDs, Opiods Tetracycline (inhibit enzyme that breaks down cartilage) Diacerin (Anti IL-1) Hydroxychloroquine

o Intra-articular steroids o Visco-supplementation (hyaluronic intra-articular injection)

Surgery - Knee and hip replacement

Other Eye Conditions

Visual Acuity

Examine each eye with finger counting o If unable to do so, proceed with finger movement and then light perception o If able to do so, proceed with Snellen chart

Page 190: Mrcp short case telling skills

Determine unilateral or bilateral, acute or chronic

Causes o Bilateral Acute – front (methyl alc poisoning) vs back( occipital lobe infarction

trauma) o Bilateral chronic – glaucoma, cataracts, DM, bilateral nerve damage or compression) o Unilateral acute

CRVO, CRAO, arteritis, non arteritic isch optic neuritis Retinal detachment Vitreous hemorrhage

Cataracts

Causes o Systemic

Senile cataracts DM

In prroly controlled younf type 1 DM, can get snowflakes cataracts Hypoparathyroidism Drugs

Steroids (>10mg/day of prednisolone > 1year)

Chloroquine

Chlorpromazine o Local

Trauma to the eye Glaucoma Radiation

o Hereditary Dystrophia myotonica (stellate) Wilson’s diseae (sunflower cataracts) Refsum’s disease

Nystagmus

Rule out nystagmus at extremes of gaze which is physiological

Obvious type of nystagmus o Pendular – congenital, macular disease o Rotatory only – central causes o Upbeat nystagmus (fast phase upwards)

Upper brainstem – MS, stroke, Wernicke’s ( triad of confusion, ophthalmoplegia and nystagmus, ataxia a/w Korsakoff’s Psy)

o Downbeat nystagmus

Page 191: Mrcp short case telling skills

Cervicomedullary junction – AC malformation, syringobulbia,MS o Ocular bobbing – pontine lesions

Jerky nystagmus o Occurs at primary gaze (means central)

Cerebellar Vestibular (MS or stroke)

o Occurs on horizontal gaze Multidirectional gaze evoked nystagmus

Central – cerebellar or vestibular Right or left horizontal gaze evoked nystagmus

Central or

Peripheral o Vestibular neuronitis, Meniere’s

o Ataxic nystagmus ie INO NB: To differentiate between central and peripheral, central is sustained and peripheral can be fatigued and often associated with severe vertigo

Pupillary defects

Large pupil o Differential diagnoses

RAPD III nerve palsy Holmes Adie pupil

Unilateral

Slow reaction to bright light and incomplete constriction to convergence

Young women

Reduced or absent reflexes

Degeneration of ciliary ganglion Mydriatic drugs Sympathetic overdrive (drugs)

Small pupil o Argyll Robertson pupil

Characteristic

Small (2mm), irregular pupils

Absent light reflex

Intact accommodation reflex

Does not dilate with mydiatrics Sign of tertiary syphilis Begins unilaterally and involves both pupils with time (months to years) Pathophysiology unknown Differential diagnoses for light-near dissociation

Syphilis

Page 192: Mrcp short case telling skills

DM

Pituitary tumors

Midbrain lesions

Adie’s tonic pupil

Dystrophia myotonica

Aberrant regeneration of CN III

Familial amyloidosis o Horner’s syndrome o Long Standing Adie’s tonic pupil (initially large pupil) o DM o Encephailitis o Sarcoidosis o Lyme’s disease o Parinaud’s (triad of psuedo AG pupil, vertical gaze palsy and nystagmus on

convergence and causes include MS, vascular and pinealoma)

Approach to Examination of the Face

(I) Assessment of Higher Cortical Function (II) Approach to the examination of the Eyes (III) Cranial Nerves

Isolated CN abnormalities

Multiple CN abnormalities Conforming

Cavernous sinus syndrome

Superior Orbital syndrome

Cerebellar Pontine Angle

Syringobulbia

Lateral Medullary syndrome

Medial Medullary syndrome

Bulbar Palsy

Pseudobulbar Palsy

Jugular Foramen syndrome Non-conforming

Brainstem Stroke

Multiple sclerosis

Base of skull metastasis/leptomeningeal metastasis/NPC

Basal meningitis

Paget’s disease

Myasthenia Gravis

Miller Fisher Syndrome

Page 193: Mrcp short case telling skills

Guillain Barre Syndrome

Mononeuritis multiple

Migraines (Paralytic) (IV) Speech

Dysarthria

Cerebellar Speech

Bulbar Palsy

Pseudobulbar Palsy

Slurred Speech (VII )

Parkinson’s Syndrome

Dysphasia

Expressive

Receptive

Nominal

Conductive

Global

Dysphonia

Recurrent Laryngeal Nerve Abnormality, VC abnormalities

Paget’s disease-(Spot diagnosis – large head with hearing aid and it’s not Acromegaly!)

Examination Head Enlargement of the skull, especially in the frontal and parietal Measure circumference (>55cm = abnormal) Prominent scalp veins Palpate skull for warmth Auscultate the skull for bruit Face RAPD, VA and visual fields Hearing aids, tests for deafness (conduction vs SNHL) Other CNs

Neck Platybasia (basilar invagination) – short neck, low hairline JVP

Back for kyphosis, tenderness, warmth and systolic bruits Upper limbs Obvious bowing of the long bones Cerebellar signs from platybasia

Lower limbs Lateral bowing of the femur Anterior bowing of the tibia Warmth OA knees Obvious paraplegia

Page 194: Mrcp short case telling skills

Bilateral pedal edema Request for Fundoscopy to look for optic atrophy and angiod streaks Neurological examination of the LLs and ULs for cord compression or nerve root

compression signs Urinalysis----Hematuria from urolithiasis

Presentation Sir, this patient has Paget’s disease as evidenced Bony features enlarged skull, >55cm, short neck and low hairline, back, UL and LL bowing Warmth, tenderness, systolic bruits OA knees

Neurological VIII nerve (hearing aid), CNs Cerebellar Obvious paraplegia

CVS – no raised JVP or bilateral pedal edema Complete my examination Fundoscopy Urinalysis Neurological examination History of increase in hat size

In summary, patient has Paget’s disease with complication of left-sided deafness requiring a hearing aid.

Questions What are the differential diagnoses for bowing of the tibia? Paget’s disease (Asymmetrical) Rickets disease (bilateral symmetrical) Congenital syphilis Yaws Periosteitis with apparent bowing What is Paget’s disease? Metabolic disease characterised by excessive and abnormal remodelling of bone Primary defect lies in increased osteoclastic activity with increased bone resorption and

increased osteoblastic activity There is excessive bone resorption with compensatory disorganised deposition of new bone Males 2X more common and increases with age Association with measles and paramyxovirus, cause is unknown Stages Lytic phase then Mixed intermediate phase (lytic and blastic) Sclerotic phase

Page 195: Mrcp short case telling skills

What are the complications? Bony and immobilisation Pathological fractures Sarcomatous change in 1% OA Protrusio acetabuli

Neurological Obstructive hydrocephalus CNs Hearing loss Conductive more commonly for otosclerosis of the ossicles Sensori-neural hearing loss from auditory nerve compression

Optic atrophy Spinal cord compression(basilar invagination) or nerve root compression

High-output cardiac failure Metabolic Gout – hyperuricaemia from rapid bone resorption during prolonged immobilisation Urolithiasis from hypercalciuria Hypercalcaemia from immobilisation

How would you investigate? Urinary hydroxyproline – increased Reflects increased osteoclastic activity and bone resorption Fasting sample required False positive if diet contains hydroxyproline, skin disease

Blood Ix Serum Ca and PO4 normal but high in prolonged immobilisation or malignancy Serum ALP high (increased osteoblastic activity)

Imaging Skull - “cotton wool appearance”, osteoporosis circumscripta Pelvis – “brim sign” thickening of the iliopectineal line Vertebrae – “picture frame” sign with sclerotic margins Long bones – increased trabeculation and localised bone enlargement

Bone scan – increased uptake reflects activity and useful for monitoring disease How would you manage? Education and counselling Most are asymptomatic and do not require treatment PT, OT and ST Symptomatic Painkillers

Treat disease Indications

Page 196: Mrcp short case telling skills

Bone pain, osteolytic lesions in weight bearing bones, delayed or non-union of fractures, neurological complications (except hearing loss), cardiac complications

Bisphosphonates and salmon calcitonin Treat complications What are angiod streaks? Linear disruptions of Bruch’s membrane with proliferative connective tissue emerging

through the defects.

Papilloedema

Examination

On noticing papilloedema

Attempt to identify the different stages of papilloedema present

Increase in venous calibre and tortusity

Optic cup pinker with disappearance of vessels over the disc

Disc is suffused and slightly elevated with blurring of margins; optic cup is filled and presence of haemorrhages around the disc

Look at the retina

Features of hypertension (flame-shaped haemorrhages, cotton wool spots and hard exudates)

Features of CRVO (heamorrhages)

Severe anaemia (haemorrhages)

Check for

Pallor (severe anaemia)

Obvious proptosis

Grave’s ophthalmopathy

Cavernous sinus thrombosis

Spectacles (for hypermetropia)

Requests to examine the other eye if told to examine one eye only (bilateral papilloedema vs Foster-Kennedy syndrome)

Requests

VA

Visual fields

Color testing

Pupillary reflex (may not be possible if dilated)

Eye movements

Pain on eye movements

VI nerve palsy

Palpate the temporal region if elderly for tenderness (temporal arteritis)

Blood pressure Presentation

Page 197: Mrcp short case telling skills

Sir, this patient has papilloedema affecting his right eye as evidenced by a suffused and slightly elevated optic disc associated with blurring of the disc margins with filling in of the optic cup and dilated tortuos veins. There was no evidenced of hypertensive retinopathy such as silverwiring of the blood vessels, arterio-venous nipping, flamed-shaped haemorrhages or exudates. I could not detect any haemorrhages on the retina to suggest severe aneamia or CRVO. I noticed that there was no conjunctival pallor and no obvious proptosis of the eye. I would like to complete my examination by examining the other eye for features of papilloedema or optic atrophy; checking his blood pressure; testing his VA and VF and asking him about color vision loss; eye movements for VI nerve palsy and pain on eye movement as well as RAPD. Questions What are the differential diagnoses of optic nerve swelling?

Papilloedema

Papillitis

Ischaemic optic neuropathy

Pseudopapilloedema

Hypermetropia (margins is blurred)

Drusen (yellowish-white deposits at the optic disc)

Myelinated nerve fibres

Bergmeister’s papilla (whitish elevation of the center of disc; common, seen in all ages, races and equal sex distribution)

How do you differentiate beteween papilloedema and pseudopilloedema?

Fundal fluroscein angiography What are the differences between papilloedema and papillitis?

Page 198: Mrcp short case telling skills

Papilloedema Papillitis

VA Preserved Reduced

VF Enlargement of blind spot; loss of peripheral vision

Central scotoma

Color testing Normal Loss of red

Pupillary reflex Not affected RAPD

Eye movements

No pain on movements

Pain on movement

Others Bilateral Unilateral

Absent of venous pulsation

Venous pulsations present

*Retrobulbar neuritis presents exactly like papillitis without the optic nerve head swelling appearance What are the causes of papilloedema? (6)

Space Occupying lesion

Malignancy – SOL tumor

Abscess

Hematoma

Hydrocephalus

Obstructive (ventricle, aqueduct, outlet to 4th ventricle)

Communicative

Increased CSF formation (rare – Choriod plexus papilloma)

Reduced CSF absorption

Meningitis

SAH

Benign intracranial hypertension

Idiopathic

Addison’s disease

Drugs – Nitrofurantoin, tetracycline, vit A, steroids, OCPs

Hypertension

CRVO

Others(3s)

Metabolic

Hypoparathyroidsm

CO2 retention

Graves congestive ophthalomopathy

Hamotological

Severe anaemia

Polycythaemia rubra vera/leukaemia/Multiple myeloma

Sagittal/caverbous sinus thrombosis

Sarcoid

Page 199: Mrcp short case telling skills

GBS (impaired CSF absorption due to elevated protein content)

Paget’s disease, Hurler’s syndrome What are the stages of papilloedema?

Stage 1 – increase in venous size and tortousity with loss of venous pulsation

Stage 2 – Optic cup is pinker and less distinct with disappearance of vessels over the disc

Stage 3 – Blurring of the disc on the nasal side

Stage 4 – Disc is suffused and elevated with blurred margins, optic cup filled in and haemorrhages around the disc

What is ischaemic optic neuropathy?

Ischaemia to optic nerve head from arteriolar sclerosis or temporal arteritis

Elderly with sudden visual loss

Swollen optic disc What is Foster-Kennedy Syndrome?

Characterised by papilloedema in one eye and optic atrophy in the other eye

Caused by a tumor of the frontal lobe, classically subfrontal olfactory groove meningioma

Results in optic atrophy due to compression of that optic nerve by the tumor and papilloedema in the contralateral eye from raised ICP

Look for features of NPH and loss of smell What is Benign intracranial hypertension?

Dandy’s diagnostic criteria

Patient is alert

Clinical features of raised ICP

No localising signs except VI CN palsy

LP opening pressure >20cm H2O and CSF composition normal

Normal ventricle size and normal CT head

Management

Discontinue drugs, weight loss

Diuretics, carbonic anhydrase inhibitors

Serial LPs or lumbopertoneal shunt

Optic nerve fenestration or subtemporal decompression

Parkinsonism--Examination

Introduce

Mask like facies, monotonous speech, dyskinesias

Upper limbs

Resting tremors which disappears with use

Bradykinesia (thumb to finger, rotate wrist and “twinkle stars”

Page 200: Mrcp short case telling skills

Leadpipe rigidity and cogwheeling

Acute dystonia or alien limb syndrome

Pronator drift and cerebellar signs

Palmomental reflex, grasp reflex

Face

Eye movements, vertical Doll’s if vertical gaze impaired

Close eyes for blepharospasm

Feel for seborrhea

Look for KF rings

Count 1-20

Unbutton shirt, write, cap a pen, comb his hair

Gait – typical parkinsonian gait; also rule out gait apraxia

Request

Speech if not done

Swallowing

Handwriting

Postural BP

AMT Presentation Sir, this elderly gentleman has Parkinsonism with mask like, expressionless facies. He has asymmetrical resting tremor of the right hand with characteristic pill rolling movements of the thumb that disappears with use of the hand. There is also presence of bradykinesia with leadpipe rigidity at the elbows and cogwheeling at the wrist. Movement of the contralateral upper limb accentuates these features. There is presence of seborrhea and Myerson’s sign or the glabella tap sign is positive. He has difficulty initiating his gait and has a stooped posture associated with shuffling gait with festination and lack of normal arm swing. He also turns in numbers. His gait is not apraxic and he is not on any urinary catheter to suggest NPH. Functionally he is able to walk unaided and can perform keyturning movements and unbutton his short unaided. There is no evidence of dyskinesias which can result as a result of L-dopa therapy. He dose not have features suggesting presence of Parkinson-plus syndrome. There is no evidence of Progressive Supranuclear Palsy such as impairment of the vertical gaze, blepharospasm or frontal lobe signs such as palmomental reflex and the grasp reflex. There are also no cerebellar signs to suggest multisystem atrophy. There is also no evidence of corticobasal ganglia degeneration such as dystonic arm or alien limb syndrome.

Page 201: Mrcp short case telling skills

In summary, this patient has Parkinsonism most likely due to Parkinson’s disease and relative preservation of his function; there is no evidence of dyskinesia currently to suggest side effects of L-dopa therapy. Questions What is Parkinson’s disease?

It is a progressive neurodegenerative disorder associated with degeneration of the dopaminergic nigrostriatal neurons.

Dx clinically with 2 out of 3 signs comprising of resting tremors (3-5Hz), bradykinesia and rigidity. The 4th sign of postural instability occurs later in the course of the disease. What are the causes of Parkinsonism?

1. Parkinson’s disease 2. Parkinson-plus syndromes 3. Drugs (Neuroleptics, antiemetics, MPTP- 1 methyl 4 phenyl 1,2,3,6 tetrahydropyridine) 4. Anoxic brain damage ( Post cardiac arrest, Manganese, CO) 5. Post encephalitis ( encephalitis lethargica or von Economo’s disease) 6. Tumor such as giant frontal meningioma

What are the pathologic findings in Parkinson’s disease?

Loss of pigmented dopaminergic neurons in the substantia nigra

Presence of Lewy Bodies (eosinophilic cytoplasmic inclusions) What are the Parkinson-plus syndromes?

Progressive supranuclear palsy (most common) (frontal lobe) (3)

Vertical gaze palsy

Downgaze affected first, then upgaze, then horizontal

Can be overcome by vertical Doll’s

Other features such as blepharospam and slow pursuit or saccadic eye movements

Postural instability and axial rigidity with falls early in the course of the disease

Frontal lobe signs

Multiple sytem atrophy (Cerebellar)

MSA-P = Parkinsonism features

MSA-C = Cerebellar features

Features (3)

Cerebellar signs

Autonomic features – orthostatic hypotension, urinary dysfn and erectile dysfn

Corticospinal signs – hyperreflexia and extensor plantar response

Corticobasalganglionic degeneration (frontoparietal lobe)

2 features

Limb apraxia or alien limb syndrome

Dystonia

Page 202: Mrcp short case telling skills

Parkinsonism-dementia-ALS complex

Diffuse Lewy Body disease (Parkinsonism, dementia and neuropsychiatry) What is the significance of diagnosing Parkinson Plus syndrome?

Poorer prognosis

Poor response to L-dopa therapy What are the features that suggest that patient may have Parkinson plus syndromes?

Early onset of dementia

Presence of hallucination or psychosis

Early onset of postural instability

Truncal symptoms more prominent than appendicular symptoms

Marked symmetry of signs early in the stage of the disease

Lack of response to levo-dopa therapy in the early stage of the disease

Presence of symptoms and signs suggestive of Parkinson-plus syndromes. What are the stages of Parkinson’s disease? Staged via the Hohen and Yahr staging system comprising of 5 stages:

Stage 1 – symptoms and signs unilateral and mild

Stage 2 – Bilateral and minimal disability

Stage 3 – Generalised dysfunction with sig bradykinesia and gait impairment

Stage 4 – Rigid and bradykinesia, severe symptoms with limited walking

Stage 5 – Completely invalid and requires nursing care How would you investigate?

Brain scan to rule out

NPH

Multi-infarct syndromes

Frontal meningiomas

Parkinson-plus syndrome

MSA – atrophy of the cerebellum, brainstem

PSP – Atrophy and hyperintensity of the midbrain and red nucleus

CBGD – Frontoparietal cortical atrophy

If patient is young, ie<50, rule Wilson’s disease

Slit-lamp examination

Serum ceruloplasmin and 24Hr Urinary Copper

How would you manage?

Multidisciplinary approach

Education and counselling, PT/OT/ST

Page 203: Mrcp short case telling skills

Medications

Dopamine agonist, eg pramipexole or ropinirole

Early in the course of the disease or younger patients

Delay onset of motor fluctuations and dyskinesias

Nausea, orthostatic hypotension, hallucinations or somnolence, edema

L-dopa therapy (usually combined with a peripheral decarboxylase inhibitor)

Should be used if there is disability

Never used in patients with melanoma

Peak dose dyskinesia and end of dose rigidity

Nausea, orthostatic hypotension, hallucinations

Tremors – anticholinergic (dry eyes and mouth, urinary retention, arrhythmia), e.g. arcane or benztropine.

Rigidity – beside Dopa and D agonist, can use MAO-B inhibitors or amantadine (cognitive side effects)

COMT inhibitors (diarrhea and hepatotoxicity)

Surgical

Lesion surgery – thalomotomy (tremors) and pallidotomy (for all features)

Deep brain stimulation (for all features especially tremors) How would you manage dyskinesia?

Peak dose dyskinesia

Reduce the dose and increased frequency

Add on COMT inhibitors i.e. entacapone which increases half life of L-dopa therapy

Initiating with dopamine agonist and adding on L-dopa therapy resulted in less motor fluctuations

End of dose

Increase dose, frequency

Switch to CR formulations

Add DA, MAO-B inhibitors, COMT inhibitors What is the prognosis?

Chronic

Progressive

Some will have mild while other will have severe symptoms

Some will be troubled mainly with tremors, other by rigidity and bradykinesia What is the abbreviated mental test?

Use as a screening for possible dementia in the elderly

A score of less than 6 warrants further assessment

Includes

Age

DOB

Page 204: Mrcp short case telling skills

Remember this address – 42 West Street

Time (nearest hour)

Year

Recognition of 2 persons

Place

Prime Minister of UK

First Year of WW1 (1914-1918)

Count backwards 20 to 1

Peripheral Neuropathy

(Think: Sensory, motor, or mixed. Are nerves palpable nerves?) Presentation 1

Sir, this patient has predominantly sensory peripheral neuropathy as evidenced by o Loss of sensation to pinprick and light touch and o Impairment of vibration and joint position sense o In a stocking distribution

The motor system is intact; I did not notice any o Wasting or fasciculations of the lower limb muscles o Tone and reflexes are normal with downgoing plantars o Power is normal

OR

Sir, this patient has mixed motor and sensory peripheral neuropathy as evidenced by o Loss of sensation to pinprick and light touch and o Impairment of vibration and joint position sense o In a stocking distribution

Associated with o Wasting and fasciculations of the lower limb muscles o Reduced tone and reflexes with downgoing plantars o Diminished power of 4 in the lower limb muscles especially affecting plantarflexion,

dorsiflexion and flexion and extension of the knees bilaterally AND

There is presence of o Loss of hair on the lower half of the legs bilaterally o No charcot joints

The most likely underlying aetiology is diabetes mellitus as I noticed o Presence of diabetic dermopathy o I screened for other possible causes:

No thickened nerves or hypopigmentation patch (leprosy) Parotidomegaly, dupytren (chronic ethanol ingestion) Not sallow (uremia) Not pale (B12 deficiency) Not cachexic and no clubbing of toes (paraneoplastic) No symmetrical deforming polyarthropathy (RA) No clinical features of acromegaly, hypothyroidism

Page 205: Mrcp short case telling skills

I would like to complete the examination o Gait (if not done) to look for high steppage gait (sensory ataxia) o Upper limbs for distal sensory impairment although I noticed that there is no obvious

wasting of the hands o Urine dipstick for glycosuria (DM) o Ask history

Drug history – INH, nitrofurantoin, phenytoin, chloroquine, penicillamine, vincristine, cyclosporine A

Chronic ethanol ingestion Presentation 2

Sir this patient has predominantly motor neuropathy as evidenced by o Wasting and fasciculations of the lower limb muscles o Reduced tone and reflexes o With diminished power of 4 affecting knee flexion and extension as well as plantar

and dorsiflexion

Sensation is intact with normal pinprick sensation, vibration sense and propioception.

The most likely aetiology in this patient is o diabetes mellitus as I noticed presence of diabetic dermopathy in the lower limbs o Other possible aetiologies for a predominantly motor peripheral neuropathy

Drugs – cyclosporine A, Gold, penicillamine Pb, Hg Metabolic – DM and AIP Infectious/Inflammatory – HIV, GBS, Amyloid, sarcoid PAN HSMN type 1

Questions What are the causes of peripheral neuropathy (mixed, sensory and motor)?

DAMIT BICH o Drugs

INH, nitrofurantoin, chloroquine Penicillamine, gold, cyclosporin A, phenytoin vincristine, cisplatinum

o Alcohol, Arsenic(Mees, raindrop pigmentation), Pb(wrists and Pb lines in gums), Hg o Metabolic – DM, Uremia, AIP o Infectious - Leprosy, HIV, botulism, diphtheria o Inflammatory – GBS (look for facial diplegia), CIDP o Tumor – paraproteinemia, paraneoplastic (Ca Lung), Hodgkin’s o B12, B6 and B1 o Infiltrative – Amyloid (look for thickened nerves and autonomic), sarcoid o Immunological – PAN, SLE, RA o Congenital – HMSN, Refsum’s disease, porphyria

Page 206: Mrcp short case telling skills

o Cryptogenic o Hormonal – Acromegaly, hypothyroidism, hyperthyroidism o POEMS (Polyneuropathy, Organomegaly, Endocrinoapthy, Monoclonal gammopathy,

Skin changes – a/w osteosclerotic myeloma) (NB: DM can be sensory, motor or mixed) What are the causes of a painful peripheral neuropathy? (DAB, CAP)

DM, Alcohol, B12 deficiency

Carcinoma, porphyria, Arsenic What are the causes of thickened peripheral nerves?

Median nerve (wrist), ulna nerve (elbow), common peroneal nerve (head of fibula), Greater auricular nerve (neck)

CHAOS o CIDP o HMSN o Acromegaly, Amyloid o Others

LS DNR – Leprosy, sarcoid, DM, Dejerine Sotta disease (hypertrophic peripheral neuropathy), NF, Refsum’s disease (retinitis pigmentosa, optic atrophy, cerebellar and deafness, cardiomyopathy ad ichthyosis)

What are the causes of mononeuritis multiplex (separate involvement of more than one peripheral or cranial nerve by the same disease)?

-Endocrine-DM, Hypertension, Acromegaly -AI-RA, SLE, PAN, Sjogren, Churg-Strauss, Wegener’s -Infection-Leprosy, Lyme, HIV -Infiltrative-Amyloid, sarcoid -Carcinomatosis

What are the types of neuropathy in DM?

Symmetrical sensory neuropathy (glove and stocking)

Predominantly motor, asymmetrical (diabetic amyotrophy)

Mixed motor and sensory peripheral neuropathy

Mononeuropathy

Mononeuritis multiplex

Autonomic neuropathy What are the neurological complications of alcohol?

Wernicke’s (confusion, ophthalmoplegia, cerebellar, neuropathy)

Page 207: Mrcp short case telling skills

Korsakoff’s psychosis (recent memory loss and confabulation)

Cerebellar degeneration

Central pontine myelinosis

Epilepsy

Myopathy and rhabdomyolysis

Peripheral neuropathy

Pleural Effusion

Presentation Sir, this patient has a right sided moderate pleural effusion affecting the lower two thirds of the right hemithorax. There is reduced chest excursion of the right hemithorax associated with stony dullness with reduced/absent vesicular breath sounds and vocal resonance affecting the lower two thirds of the right hemithorax. Apex beat was not displaced and trachea was central in position. There are no scars on the chest wall to suggest a previous chest tube or a thoracotomy. Patient respiratory rate is 14 bpm with no signs of respiratory failure or distress. With regards to aetiology: (State the positives first and rearrange accordingly) 1. Patient does not have any raised JVP, S3or pedal oedema to suggest CCF. There are also no stigmata of chronic liver disease or generalised oedema or renal biopsy scar to suggest nephrotic syndrome. Patient does not have features of hypothyroidism. 2. There is no nicotine staining of the fingers, no clubbing, no palpable cervical lymph nodes and he is not cachexic. I did not detect any signs of SVCO, Horner’s syndrome and patient does not have a hoarse voice to suggest a malignant effusion. I could not detect any bronchial breathing above the effusion. The patient is not toxic looking and did not detect any Mantoux testing with respect to TB pleural effusion. There is also no deforming polyarthropathy of RA or cutaneous signs of SLE. There is no calf swelling or tenderness noted to suggest DVT. With regards to treatment, I did not notice (if there are features of malignancy) any radiation marks on the right hemithorax and there are also no features of side effects of chemotherapy such as alopecia or oral ulcers. I would like to complete the examination by looking at the patient’s temperature chart as well as examining his sputum and examine the patient’s breasts (if female). In summary, this patient has got a moderate size right sided pleural effusion. He is not in respiratory distress or failure. In view: 1. Patient is cachexic, the likely underlying aetiology includes tuberculous pleural effusion or a

malignant pleural effusion.

Page 208: Mrcp short case telling skills

2. Fever, (young patient, short history) the most likely aetiology for this patient includes a parapneumonic effusion. Other diagnosis includes tuberculous effusion, malignant effusion or autoimmune cause.

3. Patient has Cx lymphadenopathy, I would like to offer the diagnosis of tuberculous effusion. Another possibility is a malignant effusion.

4. Patient has complications of SVCO/Horner’s syndrome/Clubbing with HPOA/Nicotine staining of the nails/tender ribs/chest wall, he has a malignant effusion.

5. Patient has vasculitic lesions of the hands, joint deformities/tenderness, malar rash. The aetiology of the effusion is most likely due to collagen vascular disease/SLE/RA.

6. Young female, aetiologies include CTD, hypothyroidism and TB Questions What are your differential diagnoses for dullness on percussion of the right lower zone?

1. Pleural thickening :Old TB, old empyema, mesothelioma, asbestosis, PHx of hemothorax 2. Basal consolidation 3. Lower lobe collapse 4. Raised hemidiaphragm:

a. phrenic nerve palsy from Ca or phrenic nerve crush for old TB treatment with supraclavicular fossa scar

b. hepatomegaly 5. Mitotic mass

What are the causes of a pleural effusion?

o Transudative o CCF, constrictive pericarditis o Nephrotic syndrome, hypoalbuminemia, peritoneal dialysis o Chronic liver disease (hepatic hydrothorax) o Myxoedema o Atelectasis

o Exudative o Malignancy

Primary - bronchial or pleural Secondaries – Breast, pancreas, kidneys, ovaries, lymphomas

o Infective – parapneumonic, TB o CTD – SLE, RA o PE (can also be transudative but less common) o Pancreatitis (left sided) o Drug induced – nitrofurantoin, bromocriptine o Meig’s syndrome (ovarian fibroma with ascites and pl effusion) o Yellow nail syndrome (triad of yellow nails/onycholysis, pl effusion/bronchiectasis

and lymphedema) What are the causes of hemothorax?

o Trauma o Rupture of pleural adhesion containing blood vessel, carcinoma

Page 209: Mrcp short case telling skills

What are the causes of a chylothorax?

o Trauma or surgery to the thoracic duct o Carcinoma or lymphoma affecting the thoracic duct

What are the causes of an empyema?

o Pneumonia o Abscess o Bronchiectasis o TB

How do you confirm the diagnosis of a pleural effusion?

o Perform a lateral decubitus film to look for layering o USS

How do you differentiate between an exudative and transudative effusion?

o Light’s criteria o Sensitive but not specific for exudates o One out of 3 criteria

Pl fluid protein: serum protein > 0.5 Pl fluid LDH:serum LDH>0.6 Pl fluid LDH > 2/3 upper limit of serum LDH

o If suspect transudate still, can do the serum to pleural albumin gradient (difference). If > 1.2 g/dL, transudate. (less sensitive for exudates)

How would you investigate?

o CXR o Sputum for gram stain, c/s and AFB smear, cytology o Diagnostic pleural tap and pleural biopsy

o Indications for a diagnostic thoracocentesis >10mm thick on a lateral decubitus film or USS

o Appearance Bloody appearance

<1% insignificant

1-20% = malignancy, PE or trauma

>50% cf to peripheral Hct = hemothorax Turbid (parapnemonic, chylothorax) Putrid odour (anaerobic)

o Haemotological Ix Total cell > 1500 cell/ml >50% neutrophils (parapneumonic) Lymphocyte predominant (cancer, TB, lymphoma, CTDs) Mononuclear cells – chronic Eosinophils

Page 210: Mrcp short case telling skills

Blood or air in the pleural space

Drugs – nitrofurantoin, bromocriptine, dantrolene

Churg-Strauss

Paragonimiasis

Asbestosis o Biochemical

Light’s criteria and serum pleural albumin gradient pH Glucose (<0.5 cf to peripheries = TB, malignancy, RA) Amylase level (pancreatitis)

o Microbiological Smear and C/S AFB smears PCR, ADA(adenosine deaminase) or gamma interferon

o Cytology Fluid Pleural biopsy

o Others o Blood Ix, mantoux o Bronchoscope, CT scan

When must you order a CXR post Dx tap?

o When you suspect complications of pneumothorax o Air is aspirated o Patient develops cough, chest pain or dyspnea o Loss of tactile fremitus over the superior part of the aspirated hemithorax

How would you manage?

o Treat the underlying cause o Treat symptoms

o Fever, pain o SOB – therapeutic thoracentesis (up to 1500mls)

o Chest tube insertion (tube thoracostomy) o Complicated parapneumonic effusion

Gross pus or empyema pH <7.2 Glucose <3.0 G/S positive for organism LDH >1000

o Hemothorax o Pleurodesis for malignant effusions

What are the surface markings for a respiratory examination?

Page 211: Mrcp short case telling skills

Anteriorly o 4th rib and above = upper lobe on right o between 4th to 6th rib = middle lobe on the right o 6th rib and below on the left = lower lobe

Horizontal fissure extends on the right extends from 4th rib to midaxillary line where it bisects the oblique fissure at the 5th rib

Oblique fissure extends from the 6th rib anteriorly to the back at T2

Lower lobe extends posteriorly from T2 to T11

Upper lobe posteriorly extends from T2 and above

Prominent Cx spine = C7

Inferior angle of the scapula = T7

Pregnancy

Remember this is communication station – informed consent and patient makes decision!

Inform her condition and show concern

Explain procedure, risk to mother and to fetus

DVT and PE

o H/O of clotting abnormalities or recurrent miscarriages (Antiphospholipid) o Rule out PE in a DVT case o Rx – LMWH is safe; warfarin is not

LMWH

S/C daily till 24 hr before delivery, does not cross placenta

Induction of labour (DVT and VTE is higher post caesarean cf to vaginal)

Restart LMWH and warfarin overlap for 5 days or so

Continue on warfarin for 4 weeks

May cause epidural hematomas, osteoporosis and thrombocytopenia

Dosing regimen and treatment is not determined yet Warfarin (10% risk of embyopathy)

Oral but in 1st trimester, Cx of fetal epiphyseal haemorrhage and nasal hypoplasia

2nd and 3rd causes optic atrophy and mental retardation

Also causes placental or fetal haemorrhage

Safe during breastfeeding

Skin necrosis for mother o Always refer to haematologist and obstetrician and inform her GP o Always offer pain relief (Paracetamol) and stockings o Hypercoagulable state highest in the 3rd trimester and a few weeks postpartum o 25% of women with DVT during pregnancy carry the factor V leiden allele, which

results in APC resistance; this increases risk of pre-eclampsia o Fetus which is factor V leiden also has risk of extensive placental infarction o Investigation of PE

Concerns of teratogenicity and oncogenecity

Page 212: Mrcp short case telling skills

Can lead shielding of the fetus, minimising fluoroscopy and reduction of dose of radiosiotpes

Compression USS of DVT is sensitive and specific and safe

If isolated calf DVT, rarely cause PE

Distal DVT can progress to prox DVT in 20-30% within the 2 weeks of presentation of DVT

So require serial DVT screen; USS on day 1 and day 7 if positive; if negative on d1, repeat in 2-3 days

If suspect iliac DVT, can consider MRV, venogram with shielding For PE

Do V/Q scan – if high prob = dx

If low probability = no PE

If non diagnostic, go for USS DVT, if positive = PE

If negative, then can do pulmonary angiogram

Pulmonary angiogram is possible – informed consent

Ventilation-perfusion scan can be carried out (Tc labelled microaggregated albumin is low dose radioactive iodine and Krypton gas half life is short in terms of seconds)

Spiral CT is not recommended because of high radiation and not validated; also a normal result dose not exclude PE

o If a woman has prior VTE, can consider omitting anticoagulation; if want to prophylax with anticoagulation, is also a reasonable option, especially if underlying haematological consition (Blood 2002)

Valvular Heart Disease and Preganancy

Pregnancy discouraged for cyanotic heart disease, Eisenmenger and severe pulmonary hypertension

Symptomatic and severe valvular lesions addressed before conception and pregnancy

Simple measures such as bed rest and avoidance of supine position should not be overlooked

For low to moderate risk patients, no need antibiotic prophylaxis for vaginal or caesarean delivery unless infection suspected

For high risk such as PHx of IE, prosthetic valves and complex cyanotic heart disease, antibiotics can be used such as Ampicillin/gentamicin or vanco/genta

Prosthetic valves in pregnancy management is slightly different from DVT/PE o Assess risk and mother’s preference Maternal vs fetal safety) o This is due high risk of valvular thrombosis with LMWH/UFH o If high risk (Starr Edwards, Bjork Shiley, MVR or multiple valves)

Materal safety – warfarin for 36 weeks and UFH for labour Fetal safety – UFH/LMWH for 12 weeks then warfarin till 36 weeks then UFH

for labour o If low risk (St Judes, AVR, chronic AF)

Page 213: Mrcp short case telling skills

Maternal safety – UFH/LMWH for 12 weeks then warfarin till 36th week and then UFH for labour

Fetal safety – LMWH till delivery Contraception and TOP

Combined OCPs high risk for pt at risk for VTE

Can use progesterone pill only but less effective

Can also offer IUD with progesterone

Barrier methods

Can offer sterilisation if high risk pregnancy and decision not to have children (Preg rate post sterilisation is 0.5%)

TOP – suction curettage under LA is the preferred method; up to 24 weeks; shouldn’t use medical abortion

Drugs and Pregnancy

RHEZ is safe

Most CVS safe except, ACE inhibitors (renal failure, renal tubular defects, IUGR) Amiodarone(hypothyroid), warfarin, spironolactone (external genitalia)

Fits and pregnancy o Phenytoin (IUGR, heart defects and orofacial anomalies), valproate and CMZ

(spina bifida and neural tube defects; Valproate risk > CMZ; reduce by taking folate supplements)

o Overall risk is 4-8% vs 2-3% of the general population o Risk is great to fetus if mother has poorly controlled fits o Try to use 1 type of medication and lowest possible dose; review meds with

neurologist especially if no hx of fits for a relatively long period of time o No increased risk of fits, but has increased risk of PV bleed and morning sickness

Asthma exacerbation o Should still give steroid course as risk of hypoxia> risk of steroids

General principles o Consider mother and illness o Consider fetus o Consider risk of not treating to mother and fetus o Options available eg omitting some and changing or reducing dose o Always consult your consultant and specialist o Patient to decide after an informed decision

Prosthetic Heart Valves

Sir, this patient has got mechanical mitral/aortic valve which has been done for an underlying mitral/aortic stenosis/regurgitation.

Page 214: Mrcp short case telling skills

I say this because there presence of a mid-line sternotomy scar associated with audible metallic clicks to the unaided ear. There is presence of a mitral valve replacement with a metallic first heart sound and a normal second heart sound. There is no pan-systolic murmur to suggest a valve leakage. (There is presence of an aortic valve replacement as evidenced by a normal first heart sound followed by a metallic click and a metallic second heart sound. There is no early diastolic murmur or a collapsing pulse to suggest a valve leakage.) (There are both mitral and aortic valve replacement as evidenced by dual metallic heart sounds. There is no pan-systolic murmur to suggest a mitral valvular leakage or an early diastolic murmur which indicates an aortic valve leakage.) Ther metallic sounds are crisps (no valvular thrombosis) and there is no conjunctival pallor or jaundice to suggest hemolytic anaemia. The apex beat is displaced at the 6th IC at the ant axillary line. (Displaced and MVR = MR; undisplaced and MVR = MS; Displaced and AVR = AR). There is no evidence of pulmonary hypt(MVR). Patient is in CCF as evidenced by presence of bibasal crepitations, raised JVP of 3 cm and bipedal edema. Patient is not in AF(MVR) and pulse is not collapsing in nature (mention this if AVR for leakage). There is no peripheral stigmata of IE such as clubbing, Janeway’s lesion, Osler’s nodes or splinter haemorrhages. This is associated with bruises which suggest overanticoagulation. There is no evidence of any Marfan’s, RA, AS or Syphilis (mention this if AVR for AR or MVR for MR) I would like to complete my examination by taking the BP of the patient and looking at his temperature chart and neurological examination for strokes. In summary, this patient has got MVR/AVR or both which is most likely done for MR/MS/AR/AS (which is due to underlying Marfan’s syndrome). There is no clinical evidence of valvular leakage, thrombosis or haemolytic anaemia. There is also no pulm hypt but pt is in heart failure and in AF. There are no signs of IE or overanticoagulation. Questions What are the indications of a mitral/aortic valve replacement?

o See respective MS/MR/AS/AR What are the types of prosthetic valves? Mechanical valves Ball and cage valve (Starr-Edwards) Single tilting disc (Bjork-Shiley) Double tilting disc (St Jude)

Bioprosthetic – Homograft or heterograft

Page 215: Mrcp short case telling skills

What are their differences? Duration Mechanical valves last 20-30yrs Bioprosthetic may fail within 10-15 years

Thrombogenecity Mechanical require lifelong anticoagulation (Starr-Edwards>single disc>double disc) Bioprosthetic does not require lifelong anticoagulation

Therefore in the young and those who already require long term anticoagulation, mechanical

valves preferred And in the elderly(lifespan <10-15 years) or those that cannot tolerate anticoagulation,

bioprosthetic valve preferred What are the complications? Complications of prosthesis Valve leakage (mild- hemolytic anaemia, severe – CHF) Valve thrombosis Valve strut failure (rare, acute presentation with high mortality, Bjork-Shiley) Hemolytic anaemia (from valvular leakage due to partial dehiscence; Rx with Fe, folate,

transfusions, B blockers or if fit for op, repair of valve replacement) Complications of valvular heart disease Infective endocarditis Congestive cardiac failure Thromboembolism (rule out IE and thrombosis)

Complications of management Overanticoagulation Bleeding

What are the causes of anaemia in such patients? Bleeding from anticoagulant Hemolytic anaemia Infective endocarditis How do you tell clinically that the valve has malfunction? New murmur Change in characteristic of a preexisting murmur Change in intensity or characteristic of an audible sound How would you investigate a patient suspected of having valve dysfunction? Cinefluoroscopy – rapid, fast ad inexpensive for structural integrity TTE – often difficulty study due to reverberations from the metal TEE – useful for assessing MV prosthesis but limited in AV prosthesis

Page 216: Mrcp short case telling skills

Can MRI be done for a patient with mechanical heart valves? Yes it is safe except those with pre 6000 Starr-Edwards prosthesis (1960-64) Valve thrombosis Up to 5% per patient-year Factors – inadequate anticoagulation and mitral location Manisfest as pulmonary congestion, poor peripheral perfusion or systemic embolisation, acute

deterioration Change in audible sounds or murmur

Ix shows reduced movement of the disc or poppet, reduced orifice area, increased regurgitation or transvulvular pressure

Mx <5mm – IV heparin >5mm – Fibrinolysis (if high operative mortality) or valve replacement

Proximal Myopathy Examination

On detecting proximal weakness, proceed to test sensation to rule out neuropathic weakness; skip the cerebellar; waddling gait

If unilateral proximal weakness, think of diabetic amyotrophy (a/w pain and sensory impairment)

Check the ULs Acromegaly, Cushing’s Dupuytren contracture Dermatomyositis/Polymyositis Proximal weakness

Check the Face Eyes for MG Cushing’s, Acromegaly, Thyroid Parotids

Presentation Sir this patient has proximal weakness of the upper and lower limbs that is due to proximal myopathy. There is presence of weakness with a power of 4 on the upper and lower limb girdle muscles. I was able to overcome his abduction of the arms and he has difficulty standing from a sitting/squatting position. There is presence of a waddling gait. There is no evidence of any sensory involvement. There is also no fatigability. There were no features of Dermatomyositis/polymyositis Acromegaly/Cushing’s/Thyroid Chronic ethanol ingestion – Dupuytren and parotidomegaly Sarcoid – lupus pernio

Page 217: Mrcp short case telling skills

Cancer – cachexia, clubbed I would like to take a drug history Cholesterol lowering drugs Corticosteroids Cyclosporine A Chloroquine The possible etiologies include: Congenital Duchenne’s Sex linked, young male child, onset 3-4yrs Pseudohypertrophy of the calves Proximal weakness – Gower’s sign, usually cannot ambulate by 15yrs No facial involvement Low IQ Dilated cardiomyopathy

Becker’s Similar but less severe to Duchenne Can ambulate beyond 15 years Usually onset 5-15 but maybe 3rd/4th decades Majority survive to 4th/5th decades Dx – Western blot of muscle biopsy – abnormal/reduced dystrophin

Limb Girdle AR, 10-30 yrs old, progressive with severe disability in 20yrs Shoulder and pelvic girdle affected Deltoids are spared initially – pseudohypertrophy Biceps and brachioradialis are involved late Hip flexors and glutei are weak Early wasting of medial quads and tibialis anterior with lateral quads and calves

being pseudo hypertrophied Face is never involved and normal IQ and lifespan Normal muscle enzymes

Fascioscapular and oculopharyngeal – see ULs Acquired P – Polymyositis/Dermatomyositis, polymyalgia rheumatica A – Alcohol C – Cancer H – HIV E – Endocrine (Acromegaly, Cushing’s, Thyroid), ESRF M – Mitochondrial myopathy (Myopathy, External ophthalmoplegia, red ragged fibres

and lactic acidemia), McArdle’s syndrome (weakness after exercise) P – Periodic Paralysis O – osteomalacia D – Drugs S - Sarcoid

Page 218: Mrcp short case telling skills

Psoriasis – Locomotor (10% of Psoriasis with Joint involvement) Presentation

Sir, this patient has psoriatic arthropathy affecting the hands of the 1. Arthritis mutilans type (bilateral deforming arthropathy, telescoping of the digits) 2. RA type (symmetrical joint involvement) 3. OA type (asymmetrical terminal joint involvement) 4. mono/oligoarticular type 5. AS type (Sacroilitis, but the syndesmophytes arise from the lateral and anterior surface

and not at the margins unlike AS) With

1. Bilateral deforming polyarthropathy, and joint deformities, tender (activity) 2. sausage shaped fingers, tenosynovitis 3. wasting dorsal guttering and wasting of the thenar and hypothenar eminence 4. nails – pitting, onycholysis, subungal hyperkeratosis, discoloration of the nails (80%

involvement with arthropathy) 5. Skin patches – well circumscribed plagues on the extensor surfaces of the elbows and

scalp, with salmon pink hue and silvery scales 6. surgical scars

Joint function 1. Impaired or preserved 2. able to grip and do pincer movement 3. coarse function – turn a doorknob 4. fine function – cap a pen, transfer coins, unbutton clothes 5. able to abduct and internally rotate her shoulder joints which are important for her ADLS

Treatment complications – Steroids for arthritis Mention no evidence of Gout (as this is associated with Psoriasis) Complete my examination by

examining for other joint involvement

Skin – especially scalp, knees, natal cleft, intragluteal folds, submammary folds, Koebner’s phenomenon

Enquire on aggravating factors Questions What are the types of skin lesions?

Plague

Guttate (numerous small papular, hx of streptococcal infection

Pustular (localized or generalized, superficial pustules may stud the plagues)

Erythrodermic (generalized erythema and scaling which may be life threatening)

Inverse psoriasis (plagues evolving in the intertriginous area without typical silvery scales due to moisture and maceration)

Where are the typical sites of distribution?

Page 219: Mrcp short case telling skills

Extensor surfaces of knees, elbows, scalp, navel, natal cleft, submammary and intragluteal folds

How do you assess severity?

Psoriasis Area and Severity Index – area, thickness, redness and scaling Total score 72 - <10, 10-50, >50 for mild, moderate and severe respectively What are the types of joint involvement in psoriasis?

OA

RA

AS

Oligo/mono

Arthritis mutilans Radiological features of psoriatic arthritis?

Periostitis – “fluffy”

Destruction of small joints

“Pencil in cup appearance”

Non –marginal syndesmophytes in AS type What are the unique characteristics of psoriatic lesions?

Salmon pink hue with silvery scales

Koebner’s - New psoriatic skin lesions at site of cutaneous trauma

Moist red surface on removing of scales (Bulkeley’s membrane)

Auspitz’s sign – capillary bleeding when silver scales are picked from the plague What is Koebner’s phenomenon and which other conditions is it seen?

New skin lesions at the site of cutaneous trauma

Occurs in 30% of patients with psoriasis, usually occurring 10-20 days post- Trauma, ranges from 3 to 2 years

Also occurs in eczema, lichen planus, vitiligo and lichen sclerosus et atrophicus What is the pathology?

Hyperproliferation of the epidermis with inflammation of the dermis and epidermis. What are the differential diagnoses for onycholysis?

Psoriasis

Fungal infection

Thyrotoxicosis (Plummer’s nails)

Lichen Planus What are the aggravating factors?

Emotional stress

Alcohol

Drugs – beta blockers, ACE inhibitors, Indocid, Lithium & antimalarials

Streptococcal infection (classically associated with guttate psoriasis)

Injury to the skin – mechanical injury and sunburn What are the principles of management?

Page 220: Mrcp short case telling skills

Education

Avoidance of aggravating factors

Topical – WSP, salicyclic, aqueous cream

Topical – Topical steroids, coal tar, Dithranol, Calcipotriol (Vit D3 which acts to increase keratinocytes differentiation as a result of increased extracellular calcium therefore decreased cellular proliferation and scaling), topical retinoids

Systemic – UVB, MTX, Retinoids, systemic steroids, cyclo, tacrolimus and MMF

Novel – immunodulators (infliximab, etarnacept) What is the prognosis?

Deforming and erosive in 40%

10% are disabled by the arthropathy What other joint pathology can patients have especially if disease is active?

Gout – because of hyperproliferation Others

30% have family history

Psoriasiform lesions on the fingers, toes, nose and ears – exclude SCC of the Oropharynx, tracheobronchial tree and esophagus – Bazex syndrome.

Purpura

(Approach: Establish purpura, rule out anaemia and neutropenia, establish cause) Examination

Introduce, thank pt, ask for pain and request to undress, note any nasal speech (Wegener’s)

General inspection

Age

Cushingoid

Renal failure, CLD

Extent – ULs, LLs trunk Examine individual lesions in the ULs or LLs

Palpable = vasculitis

Central hemorrhagic necrosis of HSP

Petechiae, ecchymosis

Cockscrew hair, perifollicular haemorrhages

Thin skin Upper limbs

Hands

RA/SLE/Scleroderma

IE signs – Osler’s nodes, splinters, clubbing

Nails involvement

CLD stigmata Elbows

Page 221: Mrcp short case telling skills

RA nodules, thickened nerves (leprosy)

Face

Jaundice

Conjunctival pallor (haematological disease)

Malar rash

Mouth – Ulcers, rashes, bleeding gums (scurvy for elderly patient) Chest

CLD stigmata Lower limbs

Arthritis of knees and ankles

Examine the feet Requests

LNs

Abdominal examination – hepatosplenomegaly

Peripheral neuropathy

Temperature chart

Urine dipstik – hematuria in vascultis with renal involvement

Drug history Presentation Sir, this patient has

Purpura/palpable purpura as evidenced by non blanchable, well-demarcated reddish/purplish patches

Presence/absence of petechiae, ecchymosis

Distribution and extent

Anaemia and mouth ulcers (neutropenia)

Cause (purpura)

Age (Mention perifollicular haemorrhages and cockscrew hair if elderly)

Cushingoid

renal failure

Liver failure

Chest scars – anticoagulation

Obvious haemarthrosis (haemophilia)

Ehlers Danlos

Cause (palpable purpura)

AI conditions

Infections

Malignancy

Drugs

Treatment (Cushingoid – can be cause of purpura or treatment for vasculitic rash)

Complete examination for spleen, liver and LNs Questions

Page 222: Mrcp short case telling skills

What are your differential diagnoses for purpura?

Thrombocytopenia o ITP o BM infiltration – haematological malignancies o BM aplasia o CLD

Capillary fragility o Senile purpura o Chronic steroid ingestion o Vasculitis eg HSP o Renal failure

Coagulation factors (ecchymoses) o Haemophilia o Christmas disease o Anticoagulation o CLD

What are the causes of a palpable purpura/vasculitis rash?

Autoimmune o SLE/RA/SSc o Churg-Strauss/PAN/Wegener’s o PBC/UC/Cryoglobulinaemia

Mitotic o Solid o Haematological – Lymphoproliferative, paraproteinaemia

Infective o Viral – HIV, Hep B, Hep C, EBV, influenzae o Bacterial – IE, TB, leprosy, Streptococcal

Drug o Aspirin o Antibiotics (penicillins, sulphonamide) o Allopurinol o Anticoagulant o Phenytoin, Gold

Idiopathic What are the common causes of purpura?

Senile purpura

Secondary to steroids and anticoagulants

Thrombocytopenia from leukaemia or marrow aplasia How would you investigate this patient?

After taking a detailed drug history

Blood Ix – FBC, biochemistry, liver function test, coagulation profile and protein electrophoresis (rule out paraproteinaemia), ANAs, dsDNA, ANCAs

Skin Bx – small vessel vasculitis

Page 223: Mrcp short case telling skills

Others – Bone marrow biopsy, trephine biopsy of the iliac crest

Radial Nerve Palsy

Examination

Suspect this on an apparently normal looking pair of ULs

Proceed to examine the ULs as per normal, concentrating on median and ulnar nerve as well as brachial plexopathy

Once radial nerve palsy detected, proceed to look for level o Demonstrate weakness of extension at the MCPJ o Preservation of IPJ extension (lumbricals and interossei muscles) o Weakness of wrist extension o (don’t forget to extend wrist before testing grip strength) o (don’t forget to test finger abduction and adduction with hands flat on a surface) o Test brachioradialis o Test triceps muscles, triceps jerk o Thumb abduction and Oschner’s clasping test for median screen o Finger abduction and Froment’s sign for ulnar screen o Look for reduced sensation in the first dorsal interosseous or anatomical snuffbox o Inspect the forearm, elbow, humerus and shoulder o Check the gums for lead poisoning – blue-black line on the gingival margin o Test Function

Presentation Sir, this patient has got an isolated right radial nerve palsy at the level of the upper third of the humerus or above. I say this because of weakness of extension of the fingers at the MCPJ and at the wrist associated with weakness of the brachioradialis muscle, triceps muscles with weakness of extension at the elbow. In addition, there is also numbness of the first dorsal interosseous space. There is no evidence of concomitant ulnar or median nerve palsies. I did not detect any scars or deformities over the humerus or the axilla. (Mentioned other areas if the level is lower) There is also no clinical evidence of lead poisoning such as a blue-black line on the gingival margin. Possible causes include compression of the right radial nerve such as crutch palsy at the axilla or Saturday night palsy at the humerus. I also note that there is presence of a splint for his wrist and finger drop. He is able to perform coarse and fine motor function. Questions

What is the course of the radial nerve and its branches? o C5, 6,7,8, T1 and emerges from the posterior cord of the brachial plexus o Leaves the axilla and enters the arm between the long head and medial heads of

the triceps and supplies the triceps o Spiral groove on the back of the humerus between the lateral and medial heads

of the triceps

Page 224: Mrcp short case telling skills

o Lower third of the humerus, it pierces the intermuscular septum to enter the anterior compartment of the arm where it supplies the brachioradialis

o It gives off a branch supplying the extensor carpi radialis longus o At the elbow, ie lateral epicondyle of the humerus, it gives off the posterior

interosseous nerve which supplies all the extensors of the forearms including the abductor pollicis longus and supinator except the extensor carpi radialis longus

o The radial nerve continues as the superficial radial nerve which provides sensory innervation of the posterior aspects of the radial 3 ½ digits.

What are the various levels of lesions and what are the correlating clinical features? o Axilla eg crutch palsy – All gone including triceps and triceps reflex o Humerus

Upper third – all is lost Middle third

triceps and triceps reflex preserved and brachioradialis and below is lost

Saturday night palsy Lower third – triceps and brachioradialis is preserved

o Elbow Like lower third Only the PIN involved

Extensors of the fingers at the MCPJ affected only

Wrist drop is not a feature as the extensor carpi radialis longus is intact and this alone can effect wrist extension

o Forearm PIN involvement Superfical radial nerve palsy; aka Watenberg syndrome which is an

entrapment syndrome where there is pain and numbness over its distribution of the first web space dorsally only because of overlap

What are the causes? o Trauma form accident or surgery o Compression or entrapment o Part of a mononeuritis multiplex o Lead poisoning o (for PIN, finger drop can be secondary to synovitis from RA)

How would you investigate? o Detailed history for the cause o X-ray – for fracture, healing callus or tumor o EMG and NCT to locate the level of injury and to monitor recovery progress

How would you manage? o Education and counselling o OT and PT with a wrist splint and cock-up splint for finger drops

Page 225: Mrcp short case telling skills

o Surgical

What’s the prognosis? o Neuropraxia with no disruption to the sheath or the axon

Recovery complete and rapid (weeks) o Axonotmesis with disruption of the axon but an intact Schwann sheath

Recovery complete but slower (1mm/day) o Neuronotmesis

Recovery is incomplete

Resuscitation in a Terminally Ill Patient

Introduction (Introduce, clarify, comfortable, aim, what relative understands) Issues

Task – Grave Px and Resuscitation

(Medical/legal/ethical, listen/advise/summarise, Social, ICE) (9 issues)

Grave Px o Form what she understands, elaborate o Offer condolences o “your father is very ill” o Explain underlying condition o What Ix showed, what has been done, what his doctors has discussed with him o Ask if father had discussed his condition with her o Remove guilt from daughter

Father did not want to burden you because you lived very far away She could not have prevented this

o Get her to agree that the main aim is to keep patient “as comfortable as possible”, “free of pain”

o Inform her that patient islikely to die this admission and possibly in the next few hours

Resuscitation o Ask her if she understands what this is o If she does not

If father were to stop breathing or hear to stop beating Giving oxygen through a plastic device in the throat Giving medication to “artificially” make the heart beat Giving chest compressions

o Has her father express his wishes o Ask her about her thoughts o This may bring his heart back for a while

But painful process for patient and does not help him It will be unsuccessful, prolonging his suffering Comfortable with dignity and self esteem Medical team has made the decision about DNR

Page 226: Mrcp short case telling skills

o Offer a plan Admit him to a side room for privacy Comfort

Oxygen, fluids

Subcutaneous morphine Ask religion, offer hospital chaplain to say prayer

o If she insists Respect her wishes Offer a plan as above Will speak to consultant and arrange another appt If religion is the issue, engage spiritual guide of her faith

Closing Retinitis Pigmentosa

Examination

Upon noting the pigmented bony spicules at the eye peripheries

Macular edema or bull’s-eye maculopathy

Attenuated arterioles

Way pallor of the optic disc

Cataracts

External ophthalmoplegia

Ptosis

Deafness

Hands for polydactyly

Sallow appearance for renal impairment

Short Stature Presentation (think in terms of 3s) Sir, this patient has retinitis pigmentosa as there are bony spiculated pigmentation on the peripheries of the retina bilaterally. This is not associated with any macular edema or bull’s eye maculopathy, attenuated arterioles or a waxy pale optic disc. I also noted that the patient has cataracts, which maybe associated with retinitis pigmentosa. There is no ptosis that I noted and the patient is not sallow in appearance and does not have short stature. I would like to complete the examination by examining his visual field and acuity, enquire about night blindness and assess eye movements to look for external ophthalmoplegia. I would also liked to screen for deafness as well as examine his hands for polydactyly and his limbs for spinocerebellar degeneration. Questions What is retinitis pigmentosa? It is a dystrophy of the photoreceptors and pigment epithelium with an incidence of 1 in 4000. Occurs bilaterally and begins in childhood or young adults with a progressive course.

Page 227: Mrcp short case telling skills

What is the mode of inheritance? 9% X linked, 16% is AR and 22% is AD; the rest or approx 50% is sporadic. What are the clinical features of RP? Commoner rod-cone type Ring scotoma, Peripheral visual field loss, tunnel vision Night blindness Cone-rod type Visual acuity loss Loss of color discrimination Day vision problems What are the causes of RP? Primary where no known cause Secondary to inflammatory retinitis Associated with other syndromes

Usher’s syndrome (RP with hearing loss)

Alport’s syndrome (RP with hearing loss and nephritis)

Refsum’s disease (RP with deafness, hypertrophic peripheral neuropathy and cerbellar ataxia- it is a phytanic acid storage disease)

Abetalipoproteinaemia (RP, with fat malabsorption and spinocerebellar degeneration)

Friederich’s ataxia

Kearnes Sayre Syndrome (RP with ext ophthalmoplegia, ptosis and heart block)

Laurence-Moon-Biedl Syndrome (RP with short stature, polydactyly, renal dysfn) What are the associated ocular abnormalities? Posterior subcapsular cataracts Myopia Keratoconus Open angle glaucoma How would you Ix? Careful FHx and rule out phenothiazine toxicity Formal visual fields testing, color testing (Ishihara charts) and electroretinogram (ERG) Ix if cause is suspected, eg, ECG for heart block in Kearnes Sayre syndrome, lipids and

protein elctrophoresis for abetalipoproteinaemia, serum phytanic acid for Refsum’s disease

How would you manage? Education and counselling especially genetic counselling Impaired vision training and aids for ADLs and job retraining Medication such as high dose of Vit A which slow the progress of RP by 2% a year and

acetazolamide for complications of cystoid macular edema.

Rheumatoid Arthritis

Presentation Sir, this patient has Rheumatoid arthritis affecting the hands as evidenced by

Page 228: Mrcp short case telling skills

Presence of symmetrical deforming polyarthropathy PIPJ/MCPJ Swan neck, Boutonniere’s, Z-thumb, ulna deviation Subluxation (MCPJ, dorsal subluxation of the ulna at the carpal joint) Active arthritis/quiescent Intrinsic muscle wasting CTS Dropped fingers from tendon rupture Synovial thickening Vasculitic lesions, nail-fold infarcts Palmar erythema No nail changes and skin lesions of Psoriasis SLE skin changes Elbows for Rh nodules Function Preserved vs impaired Coarse and fine functions Treatment Steroid – atrophied skin, bruisability Surgical intervention – CTS decompression, tendon release Requests Other joint involvement (MTPJ, knees) Extra articular features of RA Questions What are the extra-articular features of RA?

Eye o Conjunctiva – Keratoconjunctivitis sicca, pallor o Sclera – episcleritis, scleritis, scleromalacia perforans o Lens – Cataracts from chronic steroid usage o Retina – vasculitis, drug induced (Gold, Hydroxychloroquine) o Extra-ocular muscles – mononeuritis multiplex, myasthenia sec to penicillamine

Respiratory o Upper airways – Cricoarytenoid o Pleura – pleurisy, effusions o Airway – BOOP o Parenchyma – Pulmonary fibrosis, pneumonitis, PHT ( RA or MTX) o Caplan’s, Nodules

Neurological o Peripheral neuropathy o Mononeuritis multiplex o Nerve entrapment o Cx atlanto-axial subluxation +/- Cx myelopathy

Page 229: Mrcp short case telling skills

o Muscle atrophy, proximal myopathy sec to steroids, penicillamine induced myasthenia

Abdomen o Splenomegaly in Felty’s syndrome

What are the causes of anaemia in RA?

Fe deficiency – GI bleed from NSAIDS

Megaloblastic anaemia – Pernicious anaemia

Anaemia of Chronic disease

Hypersplenism from Felty’s Syndrome

Aplasia – Gold, Penicillamine What are the poor prognostic indicators?

Insidious onset and high activity at onset

Rh nodules or early erosions within 1 year

Extra-articular features

Persistent activity after 1 year – Active arthritis, ESR

High levels of Rh factor and Anti CCP Ab (anti cyclic citrullinated peptide Ab) What are the criteria for Dx RA (American college of Rheumatology)? Any 4 of the following:

Morning Stiffness for >1 hr duration for > 6 weeks

Arthritis of 3 or more joints for > 6 weeks

Arthritis of wrists, MTCP, PIPJ

Symmetric

Rh nodules

Rh factor

Radiographic changes typical changes including erosions or unequivocal decalcification How would you investigate this patient?

Blood Ix – Rh factor, anti-CCP, ESR, CRP

X-rays of the joints – erosions and periarticular osteopenia How would you manage this patient?

Education and counselling

Non-pharmacological o OT, PT

Pharmacological which will depend on the severity o Analgesia – NASIDS o DMARDS

Methotrexate (Check FBC and LFT) Sulphasalazine Hydroxychloroquine Low dose prednisolone Newer agents

Leflunomide

Tacrolimus

Cyclosporine A

Page 230: Mrcp short case telling skills

Rapamune (sirolimus) o Immunomodulators (biologics)

Anti TNF – Etarnacept (FDA approved), infliximab, humira

Beware of TB and atypical pneumonia resulting from their use Anti CD20 – Rituximab

Surgical What is Z thumb deformity?

Deformity that occurs in RA hands

With hyperextension of the first IPJ and fixed flexion and subluxation of the first MCPJ

Resulting squaring appearance of the hands What is Boutonnière’s deformity?

Hyperflexion of the PIPJ and hyperextension of the DIPJ

Due to rupture of the central slip of the extensor tendon over the PIPJ with imbalance of the flexion and extension forces of the finger

What is swan neck deformity of the fingers?

Hyperextension of the PIPJ and hyperflexion of the DIPJ

Due to synovitis of the flexor tendons leading to flexion at the MCPJ with constant effort to extend the finger; leading to stretching of the collateral ligaments and the volar plate of the PIPJ; intrinsic muscle balance leads to swan neck deformity

What are the differential diagnoses for deforming polyarthropathy of the hands?

Rheumatoid arthritis

Psoriatic arthritis of the RA type

Jaccoud’s arthropathy which is ulna deviation with subluxation of the 2nd to 5th fingers at the MCPJ which is voluntarily correctable; initially described in patient’s with Rh fever but now used synonymously with SLE deforming arthropathy

Scleroderma

Seated (hands on pillow if available) This is a middle-aged lady has got scleroderma as evidenced by Hands – evidences of sclerodactyly and smooth/shiny/tight/taut shiny skin of the hands which extends proximal to the MCPJ (double pinch test). There is also digital tip pitting, finger pulp atrophy and acroosteolysis. There is also presence of Raynaud’s phenomenon, beaking of nails (pseudoclubbing), atrophic nails, nail fold telangiectasia (especially 4th digit via magnifying glass) and vasculitic rashes at the finger tips. There is also calcinosis and subcutaneous calcification located fingers, elbows and extensor aspect of the forearms. This is associated with wasting of the intrinsic muscles of the hands and vitiligo/hyperpigmentation (salt and pepper appearance).

Page 231: Mrcp short case telling skills

In terms of function, she is able to perform pincer movements and hand grip is good with a power of 5; however there is limitation of finger extension with flexion contractures and she finds it difficult to unbutton/button of clothes as well as to perform turning door knob manoeuvres. There is no proximal myopathy (myositis) There is also involvement of her face as evidenced by: Face – Bird – like facies, smooth/shiny/tight/taut skin of her face with difficulty closing her eyes, blotchy telangiectasia, pinched nose, microstomia, perioral tethering with pseudorhagades. I also noticed that the patient is cachexic looking; note pallor Legs – On examination of her legs, I also noticed presence of scleroderma as well as vasculitis, telangiectasia and ulcerations, vitiligo. Treatment complications - Steroids I would like to complete the examination by taking the patient’s blood pressure, urine dipstick, cardiovascular examination, respiratory examination and abdominal examination and ask her about dysphagia (examine stools for steatorrhea), Raynaud’s phenomenon as well as dry eyes and mouth. Questions What are the criteria for diagnosis of scleroderma? Major Proximal scleroderma (affecting MCPJ/MTPJ) Minor Sclerodactyly Digital tip pitting/pulp atrophy Bibasal pulmonary fibrosis Require 1 major or 2 minor What are the types of Scleroderma? CREST (a/w anticentromere antibodies) Limited cutaneous (extremities) Diffuse cutaneous (involving of skin of trunk) Scleroderma sine scleroderma (systemic complications without skin) What are the phases of skin changes? Edematous phase Dermal phase (induration) Atrophic phase with contractures What are the complications? Lung Pulmonary fibrosis Reflux pneumonitis Pleural effusion Alveolar cell carcinoma

Page 232: Mrcp short case telling skills

Cardiovascular Primary Pulmonary Hypertension Cor Pulmonale from Pulm fibrosis Pericarditis, Pericardial effusion Myocardial fibrosis Abdomen Esophageal dysmotility Malabsorption with steatorrhea from dilated second part of the duodenum resulting in bacterial overgrowth Kidneys – Renal failure (Malignant hypertension–responsive to ACE -) Primary Biliary Cirrhosis (Woman – rare) What are the possible etiologies for anaemia in a patient with scleroderma? Anaemia of chronic disease Fe deficiency anaemia from esophagitis B12 and folate deficient anaemia from malabsorption Microangiopathic hemolytic anaemia (MAHA) Aplasia from medications such as MTX What is Thibierge-Weissenbach syndrome? Acrosclerosis associated with deposition of calcium in the subcut tissue What is mixed connective disease? 2 or more - SLE, polymyositis, dermatomyositis, SSc Antiribonuclear protein antibody (Speckled pattern) Why must you avoid high dose corticosteroids? This may precipitate renal crisis. How would you investigate? ANA – Speckled pattern Anti-topoisomerase DNA 1 (Scl-70) for systemic sclerosis Anti-centromere antibody (CREST) Outline your management. Education and support PT/OT for hand function Treat symptoms and complications Raynaud’s – avoid cold, calcium channel blockers, prostacyclin Esophagitis – PPIs ACE inhibitors for hypertension Bosentan (endothelial receptor antagonist – for pulmonary hypertension) Treatment of disease Immunosuppressants – Steroids, MTX, Aza, cyclophosphamide Antifibrotic – penicillamine, interferon What is the Px? Male (poor) Renal involvement (poor) 70% for skin involvement only – 10 yr survival 20% 10 yr survival if kidneys and lung involved

Page 233: Mrcp short case telling skills

Spastic Paraparesis

(Think: Cerebellar, sensory level, dorsal column, mixed, ULs and others) Presentation Sir, this patient has spastic paraparesis as evidenced by

Increased tone and clonus

Hyper- reflexia with upgoing-plantars

Weakness of the lower limbs of power 4 with wasting

Complications of o DVT o Pressure sores o Urinary catheter, diapers

Functionally o Wheelchair by the bedside o Walking aids, orthotic shoes o Gait (Rhomberg’s sign positive with dorsal column loss, broad based gait)

This is associated with 1. Cerebellar signs

o Dyssynergia on heel-shin test o Dysmetria and intention tremor on toe to finger test o Dysdiadochokinesia on foot tapping test o There is no sensory level and no dorsal column loss o The possible differential diagnoses includes

Spinocerebellar degeneration such as FA

Young

Pes cavus, loss of ankle reflexes, dorsal column loss Multiple sclerosis Craniospinal junction abnormalities

Congenital - AC malformation

Acquired - meningioma at the CS junction, syringomyelia Lewitic meningomyelitis

o Hence I would like to complete the examination by Features of MS – RAPD, INO and optic atrophy Features of CS junction – CNs, neck, dissociated sensory loss Argyll Robertson pupils for lewitic disease

2. Sensory level at T4 o Loss of sensation to pinprick up to the level of the nipples bilaterally o There are no cerebellar sign and dorsal column sensory system is intact o Hence the lesion is likely due to spinal cord lesion at T4 level and possible etiologies

are Cord compression

Extramedullary (Root pains, LMN localised, spasticity early, no sacral sparing, abnormal CSF)

Page 234: Mrcp short case telling skills

o Vertebral – spondylosis, trauma, PID, tumor, infection o Extradural – abscess, metastases, lymphoma o Intradural – meningioma, NF

Intramedullary (root pains rare, LMN signs extend over several segments, late spasticity, may sacral spare, normal CSF)

o Syringomyelia o Tumor – glioma, ependymoma o Hematomyelia

Cord infarction

Anterior spinal artery thrombosis

Vasculitis (PAN, syphilis)

Thoracic/AAA and dissection

Causes of cord compression Myelitis

Infective o Mumps, measles, EBV, HIV o Mycoplasma, syphilis, TB

Neoplastic - Carcinomatosis meningitis

Nutritional – B12

Demyelinating – MS o I would like to complete the examination

PR for saddle anaesthesia and a lax anal tone, although I did notice that he is not on ant urinary catheter or diapers

Back for any scars or bony tenderness ULs (if sensory level is at or above the ULs)

3. Dorsal column loss o With loss of proprioception involving the first MTPJ and ankle joints o As well as loss of vibration sense up to the level of the knees o The sensation to pinprick is intact and there are/no cerebellar signs o The possible etiologies include

Spinocerebellar degeneration Multiple sclerosis Subacute combined degeneration of the cord Taboparesis

o Hence I would like to complete the examination by Features of MS Features of SACD – pallor, splenomegaly Argyll Robertson pupils

4. Upper limbs o Intact cerebellar and sensation to pinprick, proprioception and vibration (sensation

may be lost) o I would like to examine the upper limbs for

LMN signs, wasted hands

Page 235: Mrcp short case telling skills

Syringomyelia, cervical myelopathy, MND Inverted supinator jerks

C5-6 lesion UMN signs

Bilateral strokes, high cervical myelopathy

5. Absent ankle jerks +/- knee jerks o Fasciculations – MND o Cerebellar – SCD (FA) o Dorsal column loss – SACD, Taboparesis o Conus medullaris lesion o Combined pathologies

Cord compression and pre-existing peripheral neuropathy

DM and Cx myelopathy

Alcohol and Cord compression Cx and lumbar myelopathies

6. Others – Cerebral palsy, parasagittal falx meningioma Questions What is spinocerebellar ataxia?

A inherited disorders with multiple subtypes >20

Cs by cerebellar and spinal degeneration, slowly progressive with atrophy of cerebellum

Ataxia o Acquired (see cerebellar) o Hereditary

AD

Some involve trinucleotide repeats with high penetrance and anticipation

AR (FA) X-linked

What is Friedreich Ataxia?

Hereditary ataxia, autosomal recessive, Ch 9, trinucleotide repeat

Cs by o Symptomatic - during childhood and teenagers, inability to walk by 20s; onset <25 o Physical

pes cavus, distal wasting, spastic LL (pyramidal weakness), cerebellar signs, dorsal column loss (marked loss of cells in the dorsal root ganglion) and absent knee and ankle reflexes (degeneration of peripheral nerves)

Scoliosis

Request for o Spine - scoliosis o CVS – HOCM o Fundoscopy – optic atrophy

Page 236: Mrcp short case telling skills

o Urinalysis for glycosuria o IQ – intellectual deterioration

Differential diagnosis for spasticity, cerebellar and dorsal column loss is Multiple sclerosis; note that multiple sclerosis usually has increased reflexes

Forme fruste of this condition is pes cavus or hammer toes without other signs

Dx – Harding’s criteria o Clinical as above o Ix – small or absent sensory nerve action potentials, increased motor conduction

velocity

Other well know conditions for spinocerebellar degeneration o Refsum’s disease (elevated serum phytanic acid from defective lipid alpha oxidase –

optic atrophy, retinitis pigmentosa, cardiomyopathy, ichthyosis and ataxia) o Olivopontocerebellar degeneration o Machado-Joseph disease (CAG repeats)

What is Multiple sclerosis?

See History Taking Notes What is subacute combined degeneration of the cord?

Progressive disorder due to Vit B12 deficiency with degeneration of the corticospinal tracts (pyramids/lateral columns) and dorsal column of the spinal cord

It also affects o Nervous system – CNs, peripheral neuropathy, transverse myelitis, dementia,

psychiatric o Pernicious anaemia

Typically patient feels paraesthesia

B12 deficiency o Low intake, Vegan o Impaired absorption

Stomach

Gastrectomy

Pernicious anemia o Anti –parietal cell antibody o Anti IF antibody

Small intestines

Ileal disease (Crohn’s disease), coeliac’s disease

Bacterial overgrowth

Fish tapeworm

Colchicine

Ix o FBC – macrocytic anaemia o B12 and folate o Anti IF Ab and Anti Parietal cell Ab

Page 237: Mrcp short case telling skills

o Schilling test IM B12 to saturate all B12 protein binding sites PO Co-B12 If >10% excretion, normal If <10%, abnormal

Repeat with oral IF o If normal, means IF deficiency o If abnormal, Rx with antibiotics

If normal = bacterial overgrowth If abnormal = terminal ileal disease

o Screen with OGD if pernicious anemia, with higher risk of Gastric Ca o MRI of spinal cord – degeneration of the lateral and dorsal columns

Rx with parenteral B12 or PO B12 high doses

Px – may recover if treated early; otherwise damage is usually permanent What is taboparesis?

Cs o Lightning pains – electric shocks in the limbs, throat, stomach or rectum o Physical

Spasticity with dorsal column loss (high stoppage gait), absent ankle jerks Charcot’s joints, trophic ulcers LL before ULs, rarely ULs involved first (= cervical tabes) Incontinence and constipation Argyll-Robertson pupils

Due to lewitic disease, neurosyphilis of which: o Acute syphilitic meningitis o Acute transverse myelitis o Meningovascular disease (stroke in young patient, CN abnormalities) o Tabes dorsalis

3 stages

Pre-ataxia

Ataxia

Paresis (= taboparesis) o Taboparesis o Generalised paralysis of the insane (GPI)/Dementia paralytica

Chronic progressive frontoparietal meningoencephalitis with atrophy Dementia which classically progresses to grandeur and delusions Trombone tremor (hands, lips and tongue)

o Gummata in the CNS

Caused by spirochetal infection, Treponema pallidum

Stages o Primary – painless chancre o Secondary – maculopapular rash, acute syphilitic meningitis

Page 238: Mrcp short case telling skills

o Tertiary Neurosyphilis – obliterative small vessel endarteritis, affecting the vasa

vasorum Cardio syphilis Gummatous syphilis

o Quaternary Fulminant anergic necrotising encephalitis in HIV patients

Ix with VDRL/RPR, TPHA/FTA o VDRL

Non-specific False positive (EBV, malaria, SLE, RA, pregnancy, non syphilitic treponemal

infection) Titre use to monitor treatment and reinfection

o TPHA/FTA Specific Once positive, will remain positive even after treatment

Rx o Symptomatic treatment

Lightning pains – analgesia, TCAs, carbamazepine Ataxia – PT/OT Bladder – avoid anticholinergics, self catheterisation

o Penicillin Beware the Jarisch Herxheimer reaction

From toxins released from killed spirochaetes

Starts 3-4 hrs and peaks at 6-8 hrs

Fever, HR, RR, myalgia, lethargy

Rx with steroids 1 day before and with salicylates What is motor neuron disease?

Cs o Onset >40 years old usually, progressive o Cs by presence of UMN and LMN of a single spinal segment, with motor dysfn

involving at least 2 limbs or 1 limb and bulbar o No sensory deficit, no cerebellar, no extrapyramidal, no ocular involvement

Types o Bulbar/pseudobulbar palsy (25%) o Amyotrophic lateral sclerosis (50%)

UMNs and LMNs UMN from degeneration of the lateral corticospinal tracts and Betz cells LMNs from degeneration of CN nuclei or anterior horn cell

o Progressive muscular atrophy (25%) Severe muscular dystrophy with distal wasting Preserved deep tendon reflexes

Page 239: Mrcp short case telling skills

Due to lesions in the anterior horn cells o Primary lateral sclerosis (rare)

UMN progressing to LMN

Dy/Dx o Cervical cord compression o Syphilitic amyotrophy o Anterior horn cell disease

Polio Spinal muscular atrophy

No definitive Ix test

Mx o Counselling and support o Symptomatic

Dysphagia – NG, PEG PT, OT Antidepressants

o Riluzole – glutamate antagonist o Rx complications

Pneumonia, UTI Pressure sores DVT

Poor Px – progressive with median survival of 3-5 years What are the clinical features of cervical myelopathy? Symptoms exceeds signs and spasticity exceeds weakness Cs Motor look for wasting and weakness of C5 and C6 level as this is commonest area for

spondylosis involvement passive abduction of the little finger (myelopathy hand sign) spasticity of LL

Inverted supinator jerk Sensory – often no sensory loss but may have pseudoathetosis with loss of

proprioception and vibration What causes fasciculation?

Sign of LMN disease with denervation

Spontaneous firing of large motor units formed by branching fibres of surviving axons striving to innervate the muscles that have lost their nerve supply

What is Lhermitte’s phenomenon? Tingling or electric sensation that passes down the spine with flexion of the neck

Page 240: Mrcp short case telling skills

Causes includes Multiple sclerosis Cervical myelopathy (spondylosis and tumor) – also has similar sensation on extension of

the neck i.e. reversed Lhermitte’s sign SACD

What are the various manoeuvres to elicit Babinski’s sign? Chaddock’s manoeuvre (dorsolateral aspect of the sole of feet) Oppenheim’s sign (pressure applied on the inner border of the tibia) Gordon’s reflex (pinching the Achilles’ tendon)

Splenomegaly

Presentation Sir, this patient has a moderately enlarged spleen without evidence of liver cirrhosis. There is associated with tenderness/pallor/lymphadenopathy. The spleen is moderately enlarged at 4 cm from the left costal margin. There is a palpable notch with a regular edge and smooth surface, firm consistency and is non tender. I did not detect a splenic rub. This is not associated with hepatomegaly or ascites. The kidneys are also not ballotable. Peripheral examination showed that there is no evidence of any stigmata of CLD and patient is not jaundiced with no bruises or petechiae. Aetiology: 1. Patient is not cachexic looking with no conjunctival pallor or enlarged Cx LNs. There is also no evidence of polycythemia such as plethoric facies or conjunctival suffusion or bone marrow biopsy scar. 2. Patient is not toxic looking and no rashes or enlarged tonsils are noted. 3. There are no splinter haemorrhages or stigmata of IE. 4. There are no features of SLE or RA or chronic haemolytic anaemia. I would like to complete the examination by looking at the patient’s temperature chart and take a history of night sweats, LOW and travel history My differential diagnoses for this young patient with moderately enlarged spleen with anaemia are Massive Splenomegaly (>8 cm)

CML

Myelofibrosis

PRV

Chronic malaria

Kala-azar (visceral leshmaniasis)

Page 241: Mrcp short case telling skills

Others(Gaucher’s, rapidly progressive lymphoma) Moderately Enlarged (4 to 8 cm/ 2-4 FB)

Myeloproliferative

Lymphoproliferative

Haematological – AI, ITP, Thalassemia and HS

Chronic malaria

Cirrhosis Mildly Enlarged(4cm</1-2FB)

Myeloproliferative, Lymphoproliferative

Infections

Viral – CMV,EBV

SBE, splenic abscesses, leptospirosis, Meliodosis, TB, Typhoid, Brucellosis(farmer)

Acute malaria

Infiltrative – Amyloidosis, Sarcoidosis

Endocrine – Acromegaly, thyrotoxicosis

Collagen vascular – SLE, Felty’s

Chronic haemolytic – Thalassemia, AI, HS, ITP Tender

Infective causes

Acute myeloproliferative and lymphoproliferative Pallor

Myeloproliferative

Lymphoproliferative

Malaria

Hemolytic anaemia(Thalassemia and AIHA)

AI – Felty’s, SLE

Cirrhosis of the liver with portal hypertension Lymph Nodes

Lymphoproliferative(CLL/lymphoma)

Infective(IMS, Meliodosis, CMV, TB, HIV) Questions What are the causes? See above What are the features of an enlarged spleen in contrast to an enlarged left kidney?

Palpation o Unable to get above it o Notch border o Not bimanually palpable or ballotable

Page 242: Mrcp short case telling skills

Percussion note over the mass is dull from the left 9th rib in the mid-axillary line extending inferior-medially in the axis of the 10th rib

Inspection shows that the mass moves inferior-medially with inspiration rather than inferiorly

Auscultation may reveal a splenic rub Is a normal spleen palpable or percussible?

A palpable spleen implies that is at least twice enlarged

A normal spleen can be percussed along the 9th, 10th and 11th rib but is not percussible beyond the anterior axillary line

How would you Ix? How would you manage? Notes on conditions (See History Section)

Syringomyelia

Examination

Proceed as per normal for the upper limbs

Once dx is made, request o Examine the neck

Scars of previous Sx Scoliosis

o The cranial nerves Horner’s syndrome Ataxia and nystagmus Bulbar palsy (syringobulbia) Loss of temperature and pain sensation from the outer part of the face

progressing towards the center o The lower limbs

Spastic paraparesis Presentation Sir, this patient has got syringomyelia as evidenced by

LMN pattern of weakness of both ULs o Wasting and weakness of the small muscles of the hands and forearms o Reduced tone and reflexes

There is dissociated sensory loss with o Loss of sensation to pinprick in the ULs and upper chest o With intact sensation to vibration and proprioception

I also noticed presence of o Scars and old burn marks on his fingers

Page 243: Mrcp short case telling skills

o But I did not detect any Charcot’s joints of the ULs o La main succulente – ugly, cold, puffy, cyanosed hands with stumpy fingers and

podgy soft palms

Examination of the face o There was no evidence of bulbar palsy

Palatal movements were normal, and CN XI and XII were intact o There was also no Horner’s syndrome o No ataxia or nystagmus o However there is loss of sensation to pinprick of the face in an “onion skin pattern”

Examination of the neck o No surgical scars noted o No kyphoscoliosis

Examination of the lower limbs o Spastic paraparesis

In summary, this patient has syringomyelia with presence of wasting of the upper limbs, dissociated sensory loss and spastic paraparesis of the lower limbs. This has resulted with complications of repeated trauma of his hands.

Questions What is syringomyelia?

Cavity formation with presence of a large fluid filled cavity in the grey matter of the cervical spinal cord which is in communication with the central canal and contains CSF.

Triad of LMN weakness of the ULs, dissociated sensory loss in the ULs and UMN weakness in the LLs

How do patients present?

Rare disorder, 4th to 5th decades, male=females

Painless trauma or burns in the upper limbs, poorly localised pain in the ULs What is the pathophysiology?

At the level of the syrinx o LMN – anterior horn cell affected o Dissociated sensory loss – affects the decussating fibres of the spinothalamic tract

Below the level of the syrinx o Affecting the pyramidal corticospinal tract with spastic paraparesis of the LLs and

preservation of sphincters

Extension into the upper cervical cord and medulla o Horner’s syndrome o Bulbar palsy (CN IX-XII) o Ataxia and nystagmus ( affects the medial longitudinal bundle if lesion from C5

upwards)

Page 244: Mrcp short case telling skills

o Onion skin pattern loss of pain in the face (spinal nucleus of V CN which extends from the pons to the upper cervical cord)

What are the differential diagnoses for dissociated sensory loss?

Anterior spinal artery occlusion (affects the spinothalamic tract)

DM neuropathy, leprosy, hereditary amyloidotic polyneuropathy What are your differential diagnoses for syringomyelia?

Craniovertebral anomalies

Spinal cord injuries

Intramedullary tumours of the spinal cord

Arachnoiditis around the foramen magnum obstructing CSF flow

Hematomyelia What are the associated abnormalities?

Arnold-Chiari malformation

Bony defects around the foramen magnum

Hydrocephalus

Spina bifida

Spinal cord tumours How would you Ix?

MRI scan of the spinal cord How would you manage?

Drainage of the syrinx to the subarachnoid space

Syringoperitoneal drainage

In AC malformation, cervical laminectomy and removal of the lower central portion of the occipital bone

Intramedullary tumour excision What is syringobulbia?

Syrinx in the medulla of the brainstem

Usually extension of the syringomyelia but can be isolated

Results in o Horner’s o Ataxia and nystagmus o Bulbar palsy o CN V, VII, IX and X especially o Onion skin pattern of loss of pain sensation of the face

To Ventilate or Not

Page 245: Mrcp short case telling skills

Opening (Introduce/clarify/comfortable/purpose/what he understands) Issues

Task – Informing how ill father is, premorbid and ventilation

(medical/legal/ethical, listen/summarise/advise, social, ICE and 9 issues)

Task 1 – Informing how ill father is o Update on latest findings at the A&E eg pneumonia, clinical state and blood test and

CXR results o Coupled with his underlying condition o Currently, how doctors are managing him o Patient is ill such that he will pass away soon unless he is artificially supported by a

machine to breathe o “In order to advice you on the options, I need to discuss with you about his

condition”

Task 2 – premorbid o Normal state, ADLs o Ex tolerance o Depressed? o Condition eg COPD

Severity, LTOT, Neb, requirement on LTOT or prn Admission, ICU, intubation, cx, difficulty weaning off

o Other condition

Task 3 – ventilation o Options – Place him on a face mask vs NIPPV vs ventilation o The first 2 are likely to fail and his risk of death is high o Ventilation

Aim and benefits – remedial reason, first episode, acceptable quality of life Discouraging factors – advance COPD with poor quality of life Procedure – “a plastic tube passed into windpipe and connected to a

machine” and “tubes in the arms and neck for fluids and antibiotics” Risk – infection, pneumothorax “lung bursting”, difficulty weaning off,

psychosis and death o Check if patient has stated any views, living will o Check family and relatives’ view

Closing (Summarise/support/plan/next appointment/pager no)

Transplanted Kidney

Presentation Sir, this patient has a transplanted kidney in the right iliac fossa associated with bilateral enlarged kidneys with a functioning AVF with features of cyclosporine and chronic steroid use.

Page 246: Mrcp short case telling skills

There is presence of a rounded palpable mass in the right iliac fossa with an overlying scar. It is non-tender. In addition there are bilateral masses in the flanks which are bimanually palpable and ballotable with a nodular surface. I am able to get above these masses and they are not tender. They move inferiorly with respiration and percussion note is resonant over them. There is no associated ascites and no renal bruit. The liver and spleen are both not enlarged. The patient does have features of renal impairment with a sallow appearance and is thin looking. He does not have any bruises or pruritic scratch marks and no leukonychia or Terry’s mails were detected. There is also no conjuctival pallor to suggest anaemia and there are also no features of polycythemia such as a plethoric facies or conjunctival effusion. He is also not in fluid overload with no pedal edema and is able to lie flat and is not oxygen dependent. There is no Kussmaul’s breathing with no uremic fetor or flapping tremor of the hands. There is presence of an arterio-venous fistula in the right upper limb. It is functioning with a good thrill. There are no recent needle puncture marks and no aneurysm was noted. There is presence of diabetic dermopathy noted on the lower limbs. There is no evidence of transplant related hepatitis B or C with no jaundice or stigmata of chronic liver disease. Patient has hypertrichosis and gum hypertrophy which are complications of cyclosporine usage. Moreover, he has a Cushingnoid habitus with steroid purpura and thin skin, suggesting chronic steroid usage. I would like to complete my examination:

Temperature chart for fever

BP for hypertension

Fundoscopy for hypertensive changes

Urine analysis for hematuria, proteinura or pyuria

CVM – MVP or AR

Neurological – III nerve palsy or PHx of stroke In summary, this middle age gentleman has a transplanted kidney for underlying Adult Polycystic kidney disease with previous dialysis. The graft is functioning well as he is not uremic and is well with features of cyclosporin and steroid use. Questions What are the differential diagnoses for a right iliac fossa mass?

Page 247: Mrcp short case telling skills

Transplanted kidneys

Carcinoma of the caecum (hard mass, LNs)

Abscess – appendicular, ileocecal

Crohn’s disease (mouth ulcers, PR for fistulas)

Ovarian tumors (in females)

Others o Amoebiasis, TB lymphadenitis, actinomycosis o Carcinoid o Ectopic kidney

What are your differential diagnoses for a left iliac fossa mass?

Transplanted kidney

Colonic carcinoma (hard mass, hepatomegaly LNs)

Diverticular abscess

Fecal mass

Ovarian tumors

Others – lymphadenitis What are the common kidney diseases leading to transplant?

DM

Hypertension

GN How does renal transplant compare with dialysis?

Higher patient survival rates

Better quality of life with lower hospitalisation rates What are the causes of transplant loss?

Patient death

Allograft failure o Immunological

Acute rejection

Single most important event determining graft survival

Can result in rapid loss of graft or progression to chronic rejection or chronic allograft nephropathy

Treated with pulse steroid or anti-lymphocyte antibody therapy Chronic rejection

o Non-immunological Renovascular thrombosis Ischaemia reperfusion injury Nephrotoxicity from calcineurin inhibitors CMV, polyoma virus DM, hypertension, hyperlipdaemia

Page 248: Mrcp short case telling skills

o Others Recurrence of primary disease (GN and DM) Chronic allograft nephropathy

What is delayed graft function?

Defined as requirement of dialysis in the first week post transplant o Immunological – acute rejection o Non-immunological – ischaemia reperfusion injury, donor hypertension

What are the strategies one can use to reduce graft loss?

Immunological o Live donor better than cadaveric o HLA matched at A, B and DR loci o Absence of pre-sensitisation

Previous transplant

Pregnancies

Transfusions

Idiopathic o Immunosuppresive therapy to reduce acute rejection

Traditionally use of steroid and cyclosporin

Others

Calcineurin inhibitors eg Cyclosporin and tacrolimus

Mycophenolate mofetil

Sirolimus

Non-immunological o Pre-transplant

Donor factors – old age, CVA, hypertension

Recepient factors – older, male, obese, diabetic, hypertension o Technical factors

increase cold ischemia time – LD transplant, renoprotective preservative solutions

hyperfiltration from inadequate nephron dose – match size and better if male to female; use of ACE inhibitors

o Post-transplant

Calcineurin inhibitors induced nephrotoxicity

Monitor levels

Use others such as sirolimus or MMF

CMV infections and polyoma virus

Prophylaxis with ganciclovir for CMV

No Rx for polyoma virus

Treat BP (<130/80) and hyperlipidaemia and DM What are the complications of cyclosporine?

Hirsutism/hypertrichosis

Hypertrophy of the gums

Page 249: Mrcp short case telling skills

Hypertension

Hyperkalaemia, hyperuricaemia, hypercholesterolaemia, hypomagnesemia

Hepatotoxicity

Hemolytic uremic syndrome

Hiccuping (gastroparesis)

Hole-in-bones (osteoporosis)

Nephrotoxicity

Neoplasia (lymphoproliferative)

Neurological (tremors, headaches, seizures and strokes) What are the complications of chronic steroid use?

Skin – thin skin, telangiectasia, steroid purpura

Cushingoid habitus

Osteoporosis, AVN femoral head

Peptic ulcer disease

Hypertension

Diabetes mellitus

Cataracts

Steroid psychosis How do yo manage?

Education and counselling, regular follow up, compliant

Treat underlying cause

Require preparation prior to transplant

Post transplant management to reduce graft loss (See above)

Ulnar Nerve Palsy

Examination

Rule out median, radial and brachial neuritis

Inspecting

Wasting of the muscles of the hands, hypothenar eminence and partial clawing of the 4th and 5th fingers, sparing of the thenar eminence, ulnar paradox

Proceed to tests for finger abduction and Froment’s sign (weakness of the adduction of the thumb)

Test finger flexion of the 5th finger for flexor digitorum profundus involvement; test for wrist flexion at the ulna side and look for the tendon of the flexor carpi ulnaris

Rule out median nerve (thenar eminence and ext rot thumb, pen touch test and Oschner clasping test) and radial nerve

Sensory testing in the medial 1 ½ fingers; test T1 sensory loss

Examine the wrist and elbows (feel for thickened nerve, wide carrying angle))

Function

Thickened nerve (cf with Pb for radial and Acromeg etc for median)

Page 250: Mrcp short case telling skills

Presentation sir, this patient has got a isolated left ulnar palsy as evidenced by a left ulnar claw hand with wasting of the small muscles of the hands with dorsal guttering as well as wasting of the hypothenar eminence. There is sparing of the thenar eminence. There is weakness of finger abduction and Froment’s sign is positive. There is preservation of the flexion of the DIPJ of the 4th and 5th fingers; when the hand is flexed to the ulna side against resistance, the tendon of the flexor carpi ulnaris is palpable. This is associated with reduced sensation to pinprick in the medial 1/1/2 fingers. There are no associated median or radial nerve palsies and T1 involvement. In terms of aetiology, there is a scar at the wrist associated with a marked ulnar claw hand, demonstrating the ulna paradox. I did not find any signs to suggest leprosy such as thickened nerves, hypopigmentation patches or finger resorption. Both coarse and fine motor function of the hand is preserved. In summary, this patient has a left ulna claw hand due to a traumatic injury to the left wrist. Questions What is the anatomical course of the ulnar nerve?

It provides motor to all muscles of the hands except the LOAF; flexor carpi ulnaris and flexor digitorum profundus to the 4th and 5th fingers.

Sensory to the ulna 1 ½ fingers

Begins from the medial cord of the brachial plexus (C8 and T1)

No branches in the arm

Enters the forearm via the cubital tunnel (medial epicondyle and the olecranon process) and motor supply to the flexor carpi ulnaris and ulna half of the flexor digitorum profundus

It gives off a sensory branch just above the wrist and enters Guyon’s canal and supplies the sensory medial 1½ fingers and hypothenar as well as motor to all intrinsic muscles of the hands except LOAF.

What is the level of lesions and its clinical correlation?

Wrist – Hypothenar eminence wasting, Froment’s positive, weakness of finger abduction, pronounced claw and loss of sensation

Elbow – less pronounced claw and loss of terminal flexion of the DIPJ and loss of flexor carpi ulnaris tendon on ulna flexion of the wrist

How do you differentiate ulnar nerve palsy vs a T1 lesion? Motor – wasting of the thenar eminence in addition for T1 Sensory – loss in T1 dermatomal distribution What is the ulna claw hand? It refers to the hyperextension of the 4th and 5th MCPJ associated with flexion of the IPJs of the 4th and 5th fingers as a result of ulnar nerve palsy. It is due to the unopposed long extensors of the 4th and 5th fingers in contrast to the IF and MF which are counteracted by the lumbricals which are served by the median nerve. What is the ulnar paradox? It means that the ulnar claw deformity is more pronounced for lesions distally e.g. at the wrist as compared to a more proximal lesion e.g. at the elbow.

Page 251: Mrcp short case telling skills

This is because a more proximal lesion at the elbow also causes weakness of the ulnar half of the flexor digitorum profundus, resulting in less flexion of the IPJs of the 4th and 5th fingers. What is Froment’s sign? Patient is asked to grasp a piece of paper between the thumbs and the lateral aspect of the index finger. The affected thumb will flex as the adductor pollicis muscles are weak. (Patient is trying to compensate by using the flexor pollicis longus supplied by median nerve) What are the causes of an ulnar nerve palsy?

Compression or entrapment (Cubital tunnel at the elbow and Guyon’s canal at the wrist)

Trauma (Fractures or dislocation – cubitus valgus leads to tardive ulnar nerve palsy)

Surgical

Mononeuritis multiplex

Infection – leprosy

Ischaemia – Vasculitis

Inflammatory - CIDP How would you investigate?

Blood Ix to rule out DM if no obvious cause

X-rays of the elbow and wrist (both must be done to rule out double crush syndrome) (KIV C-spine and CXR)

EMG(axonal degeneration for chronic) and NCT(motor and sensory conduction velocities useful for recent entrapment as well as chronic) to locate level and monitor

How would you manage?

Education and avoidance of resting on elbow

OT, PT

Medical – NSAIDs and Vit B6

Surgical decompression with anterior transposition of the nerve NB: LOAF – lateral 2 lumbricals, opponens pollicis, abductor pollicis brevis and flexor pollicis brevis

Unilateral Enlarged Kidney

1. Sir, this patient has an enlarged left kidney and is on hemodialysis. There is presence of an enlarged left kidney as evidence by a left flank mass that is bimanually palpable and ballotable with a nodular surface. I am able to get above the mass and there was no palpable notch. Percussion note is resonant above the mass and it moves inferiorly with respiration. It is non tender and not associated renal bruit. 2. Sir, this patient has an enlarged right kidney and is on hemodialysis. There is presence of a right flank mass that is bimanually palpable and ballotable with a nodular surface. I am able to get above this mass and the mass does not cross the midline. Percussion note is resonant over the mass and it moves inferiorly with respiration. It is non-tender and there is no associated renal bruit.

Page 252: Mrcp short case telling skills

There is no ascites. The liver is not enlarged with a span of 12 cm in the right midclavicular line. The spleen is also not enlarged. I did not notice any scars. The patient has evidenced of chronic renal failure of which he is receiving hemodialysis. There is presence of a sallow appearance. I did not notice any pruritic scratch marks or bruises on the ULs or LLs and he is not cachexic looking. There are no signs of leukonychia or Terry’s nails. There is no conjunctival pallor to suggest anaemia and he does not have features of polycythemia. Patient is not in fluid overload, has no uremic fetor or flapping tremor of the hands and no Kussmaul’s breathing. Patient is undergoing hemodialysis via a left arterio-venous fistula. There is a strong thrill felt over the fistula with recent needle injection marks. There are no complications of aneurysm of the fistula. There are no abdominal scars to suggest previous TK insertion or renal transplant. I would like to complete my examination by

Temperature chart for fever

Blood pressure for hypertension

Fundoscopy for hypertensive changes

(don’t mention urine analysis if ESRF on RRT!)

Cardiovascular examination – for MVP and AR

Neurological examination for III nerve palsy of stroke

FHx of stroke or aneurysm In summary, this middle age gentleman has an enlarged left sided kidney with complications of ESRF of which he is undergoing hemodialysis. The most likely aetiology is asymmetrically enlarged Adult Polycystic Kidney disease. Questions What are the causes of a unilaterally enlarged kidney?

Causes of bilateral asymmetrical enlargement o APCKD, Acromegaly, DM, bilateral HN o Tuberous sclerosis, VHL, Amyloidosis

Unilateral disease o RCC o Acute renal vein thrombosis o Pyonephrosis o Hypertrophy of a single functioning kidney

See “Bilateral enlarged Kidneys” for other questions.

Approach to Unilateral Ptosis

Rule out pseudoptosis o Life up any droopy eyelids

Muscle

Page 253: Mrcp short case telling skills

o Dystrophia myotonica (see Dystrophia Myotonica)

Neuromuscular o Myasthenia gravis (See Myasthenia Gravis)

Nerve o III nerve palsy (see III nerve palsy) o Horner’s syndrome

Unilateral Horner’s syndrome Examination

Examine the other cranial nerves o Cavernous sinus syndrome o Superior orbital syndrome o Lateral medullary syndrome (see CN syndrome) o Syringobulbia (V, VII, IX –XII) o Multiple sclerosis (INO, Cerebellar, RAPD)

Neck o Scars – trauma, surgery o Neoplasia o Carotid aneurysm o Cervical rib

Upper limbs (examine in this sequence) o Pronator drift then cerebellar signs (Lateral medullary syndrome) o Wasting of ipsilateral small muscles of hands (T1) o Clubbing o sensory loss T1 o Dissociated sensory loss (Syringomyelia) o Contralateral loss to pain and temperature (Lateral medullary syndrome) o Axilla – trauma to brachial plexus

Chest o Pancoast tumor

Inspection, dullness, auscultation

Trachea deviation

Ask for Loss of sweating and level Presentation

Sir, this patient has a right sided isolated Horner’s syndrome as evidenced by o Partial ptosis of the right eyelid o Miosis of the right pupil with an intact light reflex o Enophthalmos o Elevation of the lower eyelid o (Anhydrosis if you were allowed to ask the patient)

There was no associated CN abnormalities in particular o Cavernous sinus syndrome o Superior orbital syndrome

Page 254: Mrcp short case telling skills

o Lateral medullary syndrome (see CN syndrome) o Syringobulbia (V, VII, IX –XII) o Multiple sclerosis (INO, Cerebellar, RAPD)

Examination of the neck

Upper limbs o Present axilla findings (stat the other findings with CN syndromes)

Chest for Pancoast lesion

DM dermopathy, xanthelasma

Request to ask patient for loss of sweating.

Summary Questions What are the features of Horner’s syndrome?

Partial ptosis – paralysis of the upper tarsal muscle (Muller’s muscle)

Miosis – paralysis of pupil dilator

Enophthalmos – paralysis of muscle of Muller

Slight elevation of the lower eyelid – paralysis of the lower tarsal muscle

Loss of sweating How do you delineate the site of lesion clinically?

Loss of sweating o Central lesion – loss in the head, upper trunk and arm (1St order) o Neck

Proximal to the superior cervical ganglion – loss in face (2nd order) Distal to superior crvical ganglion – no loss (3rd Order)

Adrenaline 1:1000 in both eyes (denervation hypersensitivity) o Above the superior cervical ganglion (peripheral) = dilates the affected eye o Below/Proximal to superior Cx ganglion or normal eye = no effect

Cocaine 4% o Dilates normal eyes o No effect on the affected side if above/distal to superior cervical ganglion

What are the causes of Horner’s syndrome?

Hypothalamus or brainstem o Stroke o Pontine glioma o Coning of temporal lobe

Cervical cord (C8-T2 : intermediolateral column) o Syringomyelia o Multiple sclerosis o Tumor

Superior Mediastinum (2nd order nerves exits the spinal cord and synapses at the superior cervical ganglion)

o Pancoast lesion (SCC of lung) o Trauma to brachial plexus

Page 255: Mrcp short case telling skills

Neck (carotid sympathetic plexus and superior cervical ganglion) o Neoplasia o Trauma o Surgery (cervical sympathectomy) o Carotid aneurysm o Carotid Dissection (triad of pain, ipsilateral Horner’s and cerebral or retinal

ischaemia)

Idiopathic

Congenital – heterochromia of the iris (grey-blue on the affected side)

Migraine – causes intermittent Horner’s syndrome

Upper Limbs Overview

Motor o Unilateral vs bilateral o Proximal vs distal vs entire UL o Myopathy, Neuromuscular and Neuropathy

Sensory (See Lower Limbs) o Peripheral neuropathy o Mononeuropathy/mononeuritis multiplex o Rediculopathy

Movement disorders Motor

Distally

Wasted hands o Myopathies o Nerve (Think of levels)

Mononeuropathy

Ulnar

Median

Radial

Combination of above three Peripheral neuropathy Brachial plexus Roots Anterior Horn Spinal cord

Claw hands o Partial claw – ulna claw hand (r/o dupytren’s contracture) o Total claw

Neurological – combined ulna and median, leprosy, brachial, polio, syringomyelia

Non-neurological – RA, ischaemic contracture, Scleroderma

Page 256: Mrcp short case telling skills

Proximally

Proximal myopathy, Dystrophia myotonica

Myasthenia gravis

Syringomyelia, Radiculopathy, upper brachial plexus

Entire Upper Limb

Bilateral o Cervical myelopathy o Syringomyelia o MND o Spinal Muscular Atrophy o Polyradiculopathy o Bilateral Brachial Plexus (trauma or bilateral cx Rib)

Unilateral o Polio o Brachial neuritis o Polyradiculopathy o Hansen’s disease o Hemiparesis o Infantile hemiplegia o Brown-Sequard

Movement disorders

Parkinsonism

Chorea and movement disorders o Think: 3,3,3

Peripheral : Parkinsonsim, Rh heart, SLE Face: Wilson’s, Hyperthyroidism, Polycythaemia Request: CVS, Drug, AMT for Huntington

Hemiballismus

Cerebellar o Think

Unilateral: 4

Stroke and CNs and risk factors

SOL and CNs

Parkinsonism

MS Bilateral (2/2/2/2/2/2)

ULs o Alcohol o Parkinsonism o (NF)

Eyes o Wilson’s, MS

Page 257: Mrcp short case telling skills

Mouth o Hypothyroidism o Phenytoin o (Alcohol)

LLs o FA o (Ataxia telangiectasia)

General o Wasting (paraneoplastic) o Bilateral strokes

Requests o Drugs o Infection

VII Nerve Palsy

Examination

Upon noticing facial asymmetry, proceed to tests VII nerve functions

Look up (frontalis) and attempt to push the folds down, close your eyes (orbicularis oculi) and attempt to force them open, and frown (corrugator superficialis)

Look for exposure keratitis, tarsorraphy

Nasolabial fold, show your teeth and and blow against closed lips

Look for drooling of saliva

Determine UMN or LMN, unilateral or bilateral

UMN unilateral

Examine UL and look for hemiparesis on the same side of the facial weakness

Check for xanthelesma, DM signs and BP

LMN unilateral

Examine other CN

VI nerve and contralateral weakness in brainstem lesions

CPA lesion (V, VI, VII and VIII with cerebellar)

Other CN nerves involvement non-conforming type

Basal meningitis lesions

Mononeuritis multiplex, MG

Therefore proceed to examine the neck ook at the Palate for vesicles Examine the parotids and for surgical scars Mastoid tenderness Examine the neck for cervical LNs Upper limbs

Contralateral hemiparesis Ipsilateral cerebellar

Ask to examine

Otoscopy for vesicles in EC and otitis media

Page 258: Mrcp short case telling skills

For hyperacusis (sensitive to high-pitched or loud sounds)

For loss of taste in the anterior two-thirds of the tongue

Urine dipstick for glucose and BP

Upon noticing facial diplegia, proceed with

Rule out MG (Bilateral ptosis)

Rule out Dystrophia myotonica or fascio-scapular-humeral dystrophy

Bilateral LMN VII

Test for frontalis, corrugator and orbicularis oculi

Ask patient to show teeth and blow against closed lips

Look for V, VI, VIII

Examine parotids (Sarcoidosis, amyloidosis)

Examine tongue (scrotal tongue for MR syndrome)

Examine the upper limbs for GBS, MND, leprosy, Lyme’s (radiculopathy) and bilateral cerebellar signs if suggestive of bilateral CPA tumors

Think of rare: Melkersson-Rosenthal syndrome, Mobius syndrome Presentation Sir, this patient has got a right sided lower motor neurone facial nerve palsy as evidenced by:

Paralysis of both the upper and lower facial muscles on the right

Loss of wrinkling of the right side of the forehead, inability to fully close his right eye shut with Bell’s phenemenon

Associated with loss of the right nasolabial fold and drooping of the right angle of the mouth There are no complications of exposure keratitis and there is no drooling of saliva. I also did not notice any evidence of a right sided tarsorraphy There is no associated VI nerve palsy to suggest a brainstem lesion. There are also no features of Cerebellopontine angle lesion with no involvement of the V, VI, VIII or cerebellar signs on the right. I did not find any evidence of a parotid swelling or a surgical scar and there are no vesicles on the palate. There was no right sided facial oedema or plication of the tongue to suggest the rare syndrome of MR syndrome. There is no mastoid tenderness and no enlarged cervical LNs. I did not detect any contralateral hemiparesis or cerebellar signs. I would like to complete my examination by

Otoscopy for vesicles in EC and otitis media

Ask about hyperacusis (sensitive to high-pitched or loud sounds)

Ask for loss of taste in the anterior two-thirds of the tongue

Urine dipstick for glucose and BP (mononeuritis multiplex) The most likely cause for this patient would be a right sided Bell’s palsy. Questions What is the course of the facial nerve?

VII nerve nucleus lies in the pons in close proximity with VI nerve nuclei

VII leave the pons with VIII via the cerebellopontine angle

Page 259: Mrcp short case telling skills

It enters the facial canal and enlarges to become the geniculate ganglion

A branch is given off to the stapedius muscle and the greater superficial petrosal branch goes to the lacrimal glands

The chorda tympani which supplies taste sensation to the anterior two thirds of the tongue joins the VII nerve in the facial canal

VII nerve exits the skull via the stylomastoid foramen, through the parotids with the following branches

Temporalis

Zygomatic

Buccal

Mandibular

Cervical What are the causes of a unilateral LMN VII nerve palsy?

Brainstem (Infarct/haemorrhage, MS, abscess and tumour, syringobulbia)

Base of skull lesions (infective, tumour, infiltrative)

CPA lesions (acoustic neuroma, meningioma, neurofibroma)

Petrous temporal bone (Bell’s palsy, Ramsay Hunt, OM)

Parotid (tumour, sarcoidosis, surgery)

Mononeuritis multiplex NB: Most common is Bell’s palsy What are the causes of bilateral LMN VII nerve palsies?

After ruling out MG and myopathies

Bilateral CPA tumor as in NF type 2

Bilateral Bell’s palsy

Bilateral Parotid enlargement (Sarcoidosis – Uvoeoparotid fever or Heerfordt’s fever)

GBS, MND and leprosy, Lyme disease

Rare: Rosenthal Melkersson syndrome (triad of VII palsy with facial edema and plication of the tongue, Mobius syndrome (congenital facial diplegia, oculoparalysis from III and VI and infantile nuclear hypoplasia)

What is Bell’s phenemenon?

t refers to the upward movement of the right eyeball with incomplete closure of the right eyelid in an attempt to close the right eye.

Why are the muscles of the upper face spared in a upper motor neurone lesion?

The upper facial muscles are preserved in an UMN lesion as there are bilateral cortical representations of these muscles.

What is Bell’s palsy?

An idiopathic facial paralysis, believed to be due to viral-mediated cranial neuritis from HSV

Typically presents with abrupt onset of weakness with worsening the following day, associated with facial or retroauricular pain, hyperacusis and excessive tearing

What is Ramsay Hunt syndrome?

Herpes zoster infection of the geniculate ganglion

Presents with vertigo, hearing loss, facial weakness, pain in the ear with vesicles seen on the external auditory meatus and palate

Page 260: Mrcp short case telling skills

What is facial synkinesis?

Attempt to move one group of facial muscles results in movement of another group

Occurs as a result of anomalous regeneration of the facial nerve

Egs if nerve fibres which innervate the facial muscles later innervate the lacrmial glands, then patient shed tears on mastication (crocodile tears)

How would you investigate?

Targeted Ix according to history and physical examination

Blood Ix eg Lyme’s disease

Imaging How would you manage (Bell’s palsy)?

Educate

Lubricating eye drops, eye patch, physiotherapy

PO Pred 1mg/kg/d for 7-10 days and PO acyclovir 400mg 5X/d for 7 days (within first 72hrs)

Regular follow up to look for resolution and exclude new developing signs suggestive of other conditions

Surgical (tarsorraphy) for chronic non-resolving cases How would you educate or counsel patient with Bell’s palsy?

Common condition

Course

Improvement onset: 10 days to 2 months

Plateau: 6 weeks to 9 months

Residual signs

Synkinesis

Frequency: ~50%; May be reduced by corticosteroid treatment

May be treated with botulinum

Probably due to anomalous regeneration of nerve

Crocodile tears: 6%

Face weakness: 30%

Contracture: 20%

Blepharospasm: May occur years after paralysis Prognosis better

Incomplete paralysis

Early improvement

Slow progression

Younger age

Normal salivary flow

Normal taste

Electrodiagnostic tests normal

Nerve excitability

Electrogustometry

Visual Field Defects

Page 261: Mrcp short case telling skills

Examination

Stem statements o Examine visual fields o Examine eyes o Patient complain of knocking into objects

General o Acromegaly o Hemiparesis o Dysphasia

Visual fields o Introduce o Sit about an arm’s length o “Can you see my whole face” o Test for gross VA – counting fingers (wear spectacles!) o Test for gross visual fields using finger movements as well as for visual inattention o Patient to cover his right eye with right hand and instructed to look straight into my

left eye o Test using white hat pin from all quadrants o If single eye defect

Proceed with fundoscopy

BRAO, haemorrhages, chorioretinitis

Optic atrophy, glaucoma, RP Possibilities

Constricted field o Chronic papilloedema o Chronic glaucoma o Retinitis pigmentosa o Chorioretinitis o Hysteria (visual field does not widen as object is brought

further away from the patient cf to organic cause)

Scotoma (red hat pin) o Retinal haemorrhage or infarct (paracentral or peripheral

scotomas) Does not cross the horizontal midline

o Optic nerve (pale in atrophy, normal in retrobulbar neuritis and pink and swollen in papillitis) resulting in central scotomas Compression – tumor, aneurysm, Paget’s Glaucoma Neuritis

MS

Ischaemic (C/BRAO, syphilis, temporal arteritis

and idiopathic)

Page 262: Mrcp short case telling skills

Toxic (methanol, tobacco, Pb, arsenic)

B12 defeiciency

Hereditary – Friederich’s ataxia, LHON

Secondary to retinitis pigmentosa

Altitudinal defects o Retina infarcts o Ischaemic optic neuropathy

Totally blind in one eye o Retina o Optic nerve

o If bilateral peripheral field loss Bilateral retinal lesion Bilateral optic nerve lesion

o If bitemporal defect Upper> lower = inferior chaismal

Pituitary tumor

Suparsellar meningioma Lower > upper

Craniopharyngioma Other causes

Aneursym

Metastasis

glioma o If homonymous hemianopia (infarcts, haemorrhages or tumor)

Left or right homonymous hemianopia = right or left lesion respectively Incongruous

Optic tract Congruous

Upper quandrantonopia o Temporal lobe

Lower quandrantonopia o Parietal lobe

Macula sparing (test with a red hat pin) o Occipital cortex

No macula sparing o Optic radiation

Note any DM dermopathy, xanthelasma and AF, hemiparesis Presentation Sir this patient has

A left/right o Eye blindness

Unable to perceive light

Page 263: Mrcp short case telling skills

o Scotoma o Constricted visual field defect o Upper/lower, temporal/nasal field

Bitemporal hemianopia o Mention any acromegalic features o Request to screen for hypopituitarism o Causes includes … (see above) o Investigate

Lateral SXR enlarged sella turcica, calcification for cranipharingioma CT or MRI head Formal field perimetry Serum prolactin

Left/right, upper/lower homonymous quandrantonopia

Left or right homonymous hemianopia, incongruous or congruous, macula sparing o Mention obvious signs

Hemiparesis Dysphasia (for right homonymous hemianopia) Visual inattention

o Request for neurological examination for CVA and tumor o Look for

CVA risk factors – DM dermopathy, xanthelasma, AF Tumor

Cachexia, clubbing for metastatic disease o Ix

CT head Formal field testing, perimetry

VSD

(Clue: Young patient; look for associated conditions of Down and Fallot’s Tetrology) Presentation Sir, this patient has got a Ventricular Septal Defect that is hemodynamically significant as evidenced by: Presence of a pan-systolic murmur heard best at the left lower sternal edge with radiation towards the right side of the sternum. This murmur can be heard at the apex but there is no radiation to the axilla. It is louder on expiration. It is a grade of 5/6 murmur and is associated with a systolic thrill. The first heart sound is not soft. I did not detect any third heart sound.

Page 264: Mrcp short case telling skills

I did not detect any early diastolic murmur at the left lower sternal edge to suggest an associated AR. This is also no associated mid-diastolic apical rumble at the apex to suggest a flow murmur at mitral valve which can be a/w VSD. Apex beat is displaced and is located at the anterior axillary line at the 6th IC. It is thrusting in nature. No evidence of Eisenmenger’s syndrome such as central cyanosis, clubbing. There is evidence of pulmonary hypertension such as palpable or loud P2, no parasternal heave. There are no signs of CCF such as bilateral pedal edema, no basal crepitations or raised JVP; she is comfortable at rest with a RR of 14 bpm and does not require any supplemental oxygen. There is no peripheral stigmata of IE. The pulse is regular at 70 bpm and character of the pulse is normal. There are no features to suggest Down syndrome or (Turner syndrome - if female). I would like to complete my examination by looking at the patient’s temperature chart and taking his blood pressure. In summary, this young man has got a VSD that is severe with a displaced apex beat and is complicated by pulmonary hypertension. Clinically, there is no heart failure or Eisenmenger’s syndrome or IE. The most likely cause is congenital VSD. Dy/Dx – MR, TR, VSD – For VSD, murmur radiates to the right of the sternum, young patient and a palpable thrill Questions What are your differential diagnoses for a PSM?

o MR – PSM at apex radiates towards the axilla, soft S1 o TR – PSM heard at the triscupid area, louder on inspiration; usually secondary to

pulmonary hypertension or seen in IVDAs; Giant V wave, pulsatile liver o VSD – PSM heard at the LLSE which is louder on expiration

What are the causes of a VSD?

o Congenital o Acquired

o MI How common is VSD?

o The most common congenital heart condition o 2 per 1000 o Usually in the membranous portion (can also be found in the muscular) o Small defects close spontaneously in early childhood in about 50%

Page 265: Mrcp short case telling skills

What are the types of VSD? o Supracristal (above the crista supraventricularis) o Infracristal

o Upper membranous o Lower muscular (<5%)

o Different morphology o Maladie de Roger o Swiss cheese o Large o Gerbode defects (opens into the RA)

What are the conditions in which VSD is part of?

o Fallot’s tetralogy o Truncus arteriosus o AV canal defects o DORV (double outlet RV)

What are the complications of VSD?

o AR o Pulmonary Hypt o Eisenmenger’s complex o CCF o IE

Does the loudness of the murmur correlate with severity?

o No; in fact, a small VSD results in a louud murmur and the converse is true. What is Maladie de Roger?

o A term used to describe a small VSD that is hemodynamically insignificant with normal heart size, ECG and CXR; it is a loud murmur on ausculatation

How do you differentiate an isolated VSD with one that is associated with Fallot’s tetralogy?

o Pulmonary thrill, PS murmur o Clubbed and central cyanosis (but could be VSD with Eisenmenger’s)

How do you differentiate VSD from HOCM?

o ESM rather than PSM o Apex is not displaced, double apical impulse o Jerky pulse

How would you Ix?

o ECG o Normal in small defects o LVH, RVH, p mitrale

Page 266: Mrcp short case telling skills

o Pulmonary hypertension – P pulmonale, RAD o CXR

o Normal in small defects o Cardiomegaly, LA and LVH o Pulmonary plethora initially o Pulmonary hypertension later with prominent pulmonary trunks, rapid tapering

of the peripheral pulmonary arteries and oligaemic lung fields o CCF

o Echocardiogram o Diagnostic o Determine severity and direction of shunt via color doppler

How would you manage?

o Counsel o Medical

o Antibiotic prophylaxis o Rx complications of CCF

o Surgical o Small, asymptomatic and normal pulmonary pressure do not need surgery o Indications

Evidence of pulmonary hypertension or CCF Right ventricular pressure >50 mmHg Right to left flow ratio or pulmonary to systemic resistance ratio >1.5 Recurrent IE Cx by AR Acquired cause eg rupture of septum form MI

o Contraindication Development of Eisenmenger

o Types Surgery Percutaneous transcatheter

Wasted Hands

Unilateral vs Bilateral (think of levels!) Unilateral

Think of (no myopathy, got brachial plexus)

Peripheral nerve (median, ulnar or combined)

Mononeuropathy vs peripheral neuropathy (asymmetric involvement)

Brachial plexus (trauma, tumor, radiation, Cx rib)

C8-T1 root lesions (Cx spondylosis)

Anterior Horn Cell (Poliomyelitis)

Cervical cord

Proceed as:

Page 267: Mrcp short case telling skills

Long case – as per protocol, check also neck and chest

Short case

On inspection, unilateral wasted hands noted

Neurological hand screen

Examine for ulnar and median nerve palsies.

Check for sensory for nerve vs root (peripheral nerve vs brachial plexus) and no loss (ie anterior horn cell)

Note sensory for ulnar, median and radial

Note sensory of peripheral neuropathy

Note dermatomal sensory

Feel for thickened nerves, look for hypoaesthetic macules, fasciculations

Look for scars in the axilla and neck (neck pain, tenderness), Cx rib

Check function

Requests

Palpate for cervical rib and features of Pancoast’s tumor (dullness to percussion, Horner’s syndrome, hoarseness voice)

Check for winging of scapula (for brachial plexus involvement)

If brachial plexus

Upper vs lower (wasting of muscles of hands) vs complete

Surgical(Cx rib, Pancoast) vs medical cause(brachial neuritis)

Test for proximal involvement

Serratus anterior (winging of scapula on pushing against wall) ie C5,6,7

Supraspinatus (abduction of UL from hands by your side position) C5

Infraspinatus (elbow flexed and push backwards) C5

Rhomboids (hand on hip and push backwards) C4,5,6

Reflexes (inverted supinator jerk) Bilateral

Think of

Rule out the obvious (hand screen)

RA, gouty hands

Dystrophia myotonica

Levels (got myopathy, maybe brachial plexus if bilateral Cx ribs)

Distal myopathy (reflexes normal; rare), dystrophia myotonica

Peripheral nerve lesions

Combined CTS (see median nerve palsy)

Combined ulnar and median nerve

Leprosy (resorption, hypoaesthetic macule and thickened nerve)

HMSN (look at the feet for pes cavus deformities, thickened nerves)

Peripheral motor neuropathy

(Not likely to be brachial plexus unless bilateral Cx ribs)

Nerve roots

Page 268: Mrcp short case telling skills

Cervical spondylosis (inverted supinator jerk, increased jerks for high cervical cord lesions)

Anterior Horn cell (no sensory loss)

MND (fasciculations)

Poliomyelitis

SMA

Spinal cord lesions

Intramedullary (Syringomyelia – dissociated sensory loss)

Extramedullary

Request

LL – spastic paraparesis ( if suspect Cx cord, MND)

Lower cranial nerve (bulbar palsy – if suspect MND or syringomyelia)

Proceed as

Long case

Proceed as per normal

Examine or request to examine the neck (pain tenderness and pain on neck movements), chest, CNs and LLs accordingly

Short case

Neurological hand screen

Median and ulnar nerve testing, and wrist drop( because this is also weak in C8 root lesions)

Sensory – peripheral nerve vs neuropathy vs root

Check the elbows for thickened nerves

Look for fasciculations (peripheral nerve, neuropathy, MND), hypoaesthetic macules

Inspect the neck

Quick glance at the face (NG tube – bulbar palsy, LLs – HMSN)

Check function

Request for reflexes, percussion myotonia if deemed appropriate (if suspect Cx cord lesion or dystrophia myotonica)

Questions What are the levels and causes?

Disuse atrophy (RA hands)

Myopathy (distal myopathies or dystrophia myotonica – usually forearms more affected)

Peripheral neuropathy - motor (see causes in Neurology segment)

Mononeuropathy

Surgical, trauma or compression

Mononeuritis multiplex, infection, inflammatory and ischaemic

Brachial Plexus

Surgical, trauma compression (Pancoast’s, Cx rib)

Brachial neuritis

Nerve root (Disc prolapse)

Page 269: Mrcp short case telling skills

Anterior Horn cell

MND, poliomyelitis, SMA

Spinal cord

Intramedullary

Extramedullary How would you Ix? Blood Ix according to causes as above Imaging – X-rays, CT or MRI of spine NCT/EMG What are the causes of a claw hand?

Partial claw

Ulnar nerve palsy (See Ulnar nerve)

True Claw

Non-neurological

RA

Severe Volkmann’s ischaemic contracture

Neurological (5)

Combined median and ulnar nerve

Leprosy (reflexes present. Pain loss, thickened nerves)

Lower brachial plexus ( C7-T1, selective loss of reflexes, pain loss)

Poliomyelitis (reflexes selective, pain intact)

Syringomyelia (reflexes absent, pain loss)

Withdrawing Treatment

Opening (Always clarify identity and decision maker) Task

Task – Breaking bad news, withdrawing treatment

BBN o Find out what relative understand o Summarise events so far o Sorry that he has not responded to treatment and remains ill despite all efforts to

treat him o Outlook poor and patient is going to die

Withdrawing treatment o Explain that everyday, the team discusses treatment regimes as well as withdrawing

ad withholding treatments o ITU team feels that continuing treatment is not going to help patient

Page 270: Mrcp short case telling skills

o Difficult decision for the ITU team to make, withdrawing is a sad undertaking for the medical team

o Aim to do what is best for patient o Does relative understand “withdrawing treatment” o Find out patient’s wishes, request, AMD, Living will o Find out relatives wishes o Procedure of withdrawing and outcome o Say that should not feel guilty as decision to withdraw is a medical one o Reassure palliative measures

Closing

Do not need to make a decision right away

Summarise, support, advice, next appt, pager Discussion

Ethics

A – Autonomy : Drs need not obey a request for a procedure /treatment but must follow patient’s wishes to withdraw or withhold treatment

B – Beneficence : can be from treatment or not treating

C – Confidentiality

D – Do no harm

E – Equity : It is unethical to deny acess to treatment in order to make it available to another individual

Legal o Withdrawing artificial nutrition and hydration for persistent vegetative state o Court’s decision

Page 271: Mrcp short case telling skills

Notes on MND

1) MND/ ALS is a multisystem disease with extramotor involvement. Muscle atrophy and spasticity in limb and bulbar muscles result in weakness and loss of ambulation, oropharyngeal dysfunction, weight loss, and ultimately respiratory failure. Cognitive

Page 272: Mrcp short case telling skills

impairment of the frontal lobe type and even fronto temporal dementia are not uncommon. Oculomotor and sphincter functions are relatively spared. Mean time from onset of symptoms to confirmation is 13-18 months. Death due to respiratory failure 2-4 years after onset.

2) DX: Positive criteria: LMN and UMN signs, progression of symptoms and signs Exclude: sensory signs, sphincter disturbances, visual disturbances, autonomic features, basal ganglia dysfunction, Alzhiemer-type dementia, ALS “ mimic” syndromes Supportive: fasciculation, neurogenic changes in EMG, normal motor and sensory nerve conduction, absence of conduction block 4 reasons for early Dx: psychogically, as the progressive loss of motor symptoms causes anxiety and discomfort, ethically, so they can better plan the remaining part of life, economically to avoid unnecessary tests, neurologically, to initiate neuroprotective medication

3) Heredity: familial ALS age of onset 47-52 years (58-63 in sporadic cases). Pre symptomatic genetic testing only in first degree adult blood-relatives of patients with known SOD1 gene mutation. Clinical DNA analysis for SOD1 gene mutation only in known familial history of ALS or in SALS with characteristic phenotype of the D90A mutation

4) DDX: progressive spinal muscular atrophy, cervical radiculomyelopathy, thoraco lumbar sacral disc disease, multifocal motor neuropathy, CIDP, adult onset SMA, MG, spino bulbar muscular atrophy (kennedy’s disease), multiple sclerosis, cerebrovascular disease, and MSA. Therefore, advisable to have brain and spine imaging, CXR, routine labs eg ESR, CRP, FBC, LFT, TFT, B12, folate, myeloma panel, CK, RP, glu, and electrophysiological studies. Difficulties arise when only upper or lower motor neuron signs

5) Most useful clinical measure are vital capacity and the revised ALS functional rating scale. 6) Survival: 50% about 30 months, some beyond 10 years (esp if younger, and mostly UMN

signs at presentation. Bulbar onset of ALS/ definite ALS at presentation were a/w survival of > 10 years. Poor prognosis if higher El Escorial category at presentation. Long survival also seen in primary lateral sclerosis or in predominantly LMN syndromes eg flail arm/ leg syndrome. PLS is defined as isolated UMN signs that occur 3 years after the onset of symptoms and is a slowly progressing syndrome. UMN-dominant ALS has disability similar to ALS but slower progression.

7) Breaking bad news

Location quiet, comfortable, and private

Structure face to face, At least 45-60 mins, enough time to ensure no rushing/ interruptions, eye contact and sit close to patient, Switch off mobile phone, Do not disturb signs

Participants know the patient before the meeting including

Page 273: Mrcp short case telling skills

family, emotion and social situation, case history and all relevant test results. Have all the facts at hand. Have patient’s support network present (relatives) Have NC present

What is said What does he already know about the condition. It is a progressive disease of the motor nerves. Use the correct ALS-term, not “wear and tear of motor nerves” Explain the anatomy of the disease (make simple drawing) The name of the disease should be mentioned and explained, to avoid misunderstanding. Positive aspects eg no pain, no disturbance in sensation, mentation, continence etc should be emphasised. Intact mentation offers possibility to develop coping mechanisms which can lead to serene acceptance of the disease state. This is coupled with a wish for active involvement in the process of administering adequate symptomatic treatment Consider the cultural and psychosocial context of patient and family How much does he want to know about ALS and

Page 274: Mrcp short case telling skills

tailor information accordingly. Does he wish to receive information or prefer that it is told to a family member Give diagnosis and discuss its implications. Give warning comment that bad news is coming. The whole truth may need to come by instalments. If the patient indicates that they want to know the course of the disease, be honest about the probable progression and prognosis but give a broad time frame, and recognise the limitations of any predictions There is no cure, symptoms tend to steadily worsen and the prognosis is highly variable. Some survive 5-10 or more years Acknowledge and explore patient’s reaction and allow for emotional expression Summarise the dicsucssion verbally, in writing, and / or audiotape Printed materials about disease and support and advocacy organisations Allow plenty of time for questions Stepwise, checking repeatedly if understands, and reacting appropriately to the

Page 275: Mrcp short case telling skills

verbal and non verbal cues of the patient Avoid withholding diagnosis, providing insufficient information, delivering information callously, taking away or not providing hope. Care at each visit to find out where the patient stands and move on from there

Reassurance Partial relief by having a label. Humour is a sign of living and should not be forgotten acknowledge this is devastating news but discuss reasons for hope eg research, drug trials, and the variability of the disease Explain that the complications of ALS are treatable Reassure that every attempt will be made to maintain the function nand that the treatment decisions will be respected Reassure that the patient and family will continue to be cared for and will not be abandoned. Inform about patient support group (contact details and leaflets),ALS-care team Inform about neuroprotective treatment (eg riluzole) and ongoing research Discuss opportunities to participate in research

Page 276: Mrcp short case telling skills

treatment protocols if available Acknowledge willingness to get a second opinion of the patient’s wishes Warn at the outset from dubious and costly healing methods, even if they are administered by doctors. As a rule, the more expensive the drug and the higher the promised clinical effect, the greater is the caution that should be exercised. Informative websites If asked re: prognosis, ranges like “several days” or “days to weeks” are preferable. Except for the state of mobility, no other factors for a poor prognosis have been identified. (relatives should be encouraged to see the body after the death)

How to say emotional: warm, caring, empatic, respectful. Honest, sympathetic but not sentimental. Give news at person’s pace; allow her to dictate what she is told.

Language simple and careful word choice, yet direct. No euphemism/ medical jargon

8) Coming to terms with diagnosis and disability- family member present, can ask questions,

action points and care plan, how to tell children 9) -> coping with substantially impaired function

Page 277: Mrcp short case telling skills

10) -> end of life stage- respiratory insufficiency, inadequate nutrition, and hydration, increasing discomfort, severe psychological distress, accommodate that death is the eventual outcome

11) -> after death- financial hardship, burn out, need for ongoing information and involvement in ALS –related topics, grief and bereavement support eg letter of condolence, the availability to talk to someone, provision of appropriate referrals to physicians, psychologists, psychiatrists.

12) Symptom control

Drugs Other treatments

Cramps/ fasciculations From degeneration of the intramuscular motor axons CMZ 200mg bd, phenytoin 100mg tds-qds, alternatives: Mg 5mmol tds-qds, diazepam, verapamil 120mg qds, gabapentin

PT, exercise, massage, hydrotherapy

Spasticity Degeneration of the UMNs Need to titrate to subjective clinical effect, as a moderate degree of spasticity is better for mobility than flaccid paresis Intrathecal Baclofen, tizanidine 6-24 mg daily, dantrolene 25-100mg daily, Botox type A for trismus and stridor, gabapentin 900-2400mg daily, memantine 10-60mg daily, diazepam 10-30mg daily, BZD,

PT, hydrotherapy in heated pools 3x/ week 32-34 degrees warm water, cryotherapy official muscles, heat, cold, ultrasound, electrical stimulation

Excessive watery saliva Atropine drops (sublingually 0.25-0.75mg TDS), hysocine hydrobromide, hyoscine butylbromide, hyoscine scopolamine (PO or dermal patch 1.5mg every 3 days-2 patches if severe drooling), glycopyrronium aka glycopyrolate, benztropine, amitryptylline (not more

Home suction device, dark grape juice, sugar-free citrus lozenges, nebulisation, steam inhalation, injections of botox into partid glands, irradiation of the salivary glands 7-8 Gy

Page 278: Mrcp short case telling skills

than 25-50mg BD-TDS) Stimulation of cholinergic receptors produce thin serous secretions, while stimulation of beta receptors produces thick protein and mucus rich secretions

Persistent saliva and bronchial secretions

Carbocisteine, propranolol, metoprolol

Home suction device and room humidifier, assisting expiratory movments using a manual assisted cough, assisted cough insufflator-exsufflator esp in acute respiratory infection, rehydration (jelly or ice) pineapple or papaya juice, reduced intake of dairy products, alcohol and caffeine, butter, cricopharyngeal myotomy if frequent episodes with cricopharyngeal spasm and severe bronchial secretions

Excessive or violent yawning

Baclofen

Laryngospasm Lorazepam Reassurance

Pain-stiff joints, muscle cramps, pressure on the skin/ joints from immobility, unphysiological stress on the bones and joints which have lost their protective muscular sheath due to atrophy

Simple analgesics, NSAIDS, opiods-tramadol, then morphine or ketobemidon. Secondary advantages of alleviating dyspnea and anxiety. Constipation is a problem. Nursing, changes in position frequently If toxic concentrations develop in the terminal phase, an opiod rotation with hydration may alleviate symptoms such as myoclonus, delirium, and nausea. Headache due to raised

Comfort (seating, sleeping, day and night care)

Page 279: Mrcp short case telling skills

ICP: steroids, kiv NSAIDs +/- opioids, titrated in dose against the effect. Once terminally stuporous, the interval can be prolonged; at least ¼ of original dose is needed to prevent withdrawal symptoms. If previously agreed with patient, the steroid dose may be tapered when terminal stupor is reached. A prophylactic AED +/- analgesic should then be considered. Neuralgia- CMZ, valproate, opioids, nerve block Dysaesthesia- amitryptilline, clomipramine, levomepromazine, opioids Nerve compression- dexamethasone, opiids, nerve block Bone mets- steroids, NSAIDs, bisphosphonates, radiations, opiods

Emotional lability dextrometophan and quinidine. TCA eg amitriptyline (10-150 mg), SSRI eg fluvoxamine, citalopram, litium (400-800mg), levodopa (600-1000mg), Treat only if troublesome

Inform it is due to an organic lesion in the brain and is not a mood disorder

Communication difficulties Speaking techniques, low- tech augmentative and alternative communication tools, voice amplifiers, light writers, scanning systems operated by switches, brain- computer interfaces

Constipation Lack of exercise. Review medication, since

Hydration, increased fibre intake

Page 280: Mrcp short case telling skills

muscle relaxants, sedatives and anticholinergics reduce bowel mobility. Lactulose, senna

Depression Amitriptyline best and cheapest, citalopram, SSRI sertraline, fluoxetine, paroxetine

Psychological support, counselling

Insomnia Anxiety, depression, nightmares Inability to change position during sleep due to weakness Fasciculations and muscle cramps Dysphagia with aspiration of saliva Respiratory insufficiency with hypoxia and dyspnea Amitriptyline, zolpidem, diphenhydramine

Comfort, analgesia

Anxiety Sublingual Lorazepam 0.5 -2 mg, PO lorazepam 0.5 mg BD-TDS, IV 5-10mg midazolam, oral diazepam, bupropion,

Psychological support, counselling

Fatigue DDX depression, insomnia, abnormal muscle activation, immobility, or respiratory dysfunction, or riluzole

At least 50% may profit from pharmacotherapy Better family counselling Treat underlying symptoms Identify psychosocial factors If onset rapid: psychostimulants eg Dextroamphetamine 5-30mg/ day PO, methylphenidate 5-30mg/day PO, pemoline 37.5-150mg/ day PO If onset of action may take weeks: TCA eg amitryptilline, imipramine doxepine, desipramine, nortriptyline

Modafinil, consider withholding riluzole

Page 281: Mrcp short case telling skills

all 25-125mg/d PO SSRI eg fluoxetine 20mg/ day, sertraline 50-200mg/ day

Secretions in the lower airways can be difficult to reach

Mucolytics like guaifenesin/ N- acetylcysteine 200-400mg TDS, neb saline and beta blocker eg metoprolol or propranolol, theophylline, frusemide, anticholinergic bronchodilator

Venous thrombosis Due to leg paralysis

Not for prophylactic anticoagulation

PT, limb elevation, compression stockings

Terminal restlessness and confusion due to hypercapnia

Neuroleptics eg chlorpromazine 12.5mg 4-12 hrly PO/ IV/ PR

Cognitive or behavioural impairment

Screening or cognitive and behavioural impairment using neuropsychological assessment tools

Weakness, immobility, vomiting, candidiasis, incontinence

Stamina undergoes fluctuations- reasurre that this is normal and does not herald an increase in pace of the disease’s progression. Active and passive PT is important, especially for the prevention of muscle contractures and joint stiffness. Should never exercise to exhaustion. IF legs are severely affected and there is increased irsk of falling, exercising in a water pool can be helpful. Maintain independence in everyday life activities. Cane->Ankle foot orthoses-> wheelchairs-> special eating tools, higher toilet seats, bath tub lift, etc.

Page 282: Mrcp short case telling skills

discuss the forthcoming need for these devices ahead of time, so that patient and family have time to adjust mentally. Home evaluation by OT to pinpoint needs home modification Achase inhibitors may lead to a short term improvement in muscle strength eg plane trip with pyridostigmine 40mg tds as it lasts only few days to a few weeks and not seen in all patients

Psychological problems -fear of becoming a burden

Supportive psychotherapy (best administered as family therapy) Extensively discuss with patients and relatives Antidepressants

Restlessness Treat causes (treatable ones are pain, distended bladder/ rectum, cerebral anoxia, dyspnea, inability to move, paradoxical reaction to BZD, drug or nicotine withdrawal restlessness. May be indistinguishable from akathisia, resulting from initiation of neuroleptics or opiates. If RLS suspected, try L-dopa Is it Myoclonus, delirium or pure motor restlessness, without mental disturbances Calm presence of family BZD eg lorazepam 0.5-2mg every 1-4 hr (PO, sublingual, IV, IM); midazolam 2.5-10mg; 30-

Page 283: Mrcp short case telling skills

60 mg in 24 hr (IV, IM, SC); diazepam 5-10mg every 4-12h (PO, rectal, IV) If necessary, sedating neuroleptic eg levomepromazin 10-20mg every 4-8 hr (PO, SC, IV, IM)

Death rattle Explain to family. Caused by secretions in the trachea and larynx. Patient possibly feels SOB Stop overhydration Antichol only stop production of new secretions, therefore Gentle aspiration and lateral placement 10-20mg N-butylscopolamin SC (no central effect) Alternatively scopolamine 0.5-1mg SC or IM every 2-4 hours; if atropine (centrally excitatory) is used, add anxiolytic eg lorazepam If necessary add midaz 5 mg SC If noisy tachypnea of 30-50/ min morphine can be parenterally titrated to 10-15/ min

Delirium- may worsen during the evening and night, or with sedatives

Terminal delirium is associated with hallucination, and paranoid ideas and may occur in an agitated, hyper/ hypoactive form. Treat causes (overhydration, drugs, hypercalcemia, CNS involvement by underlying pathology, terminal multiorgan failure) Review medications

Page 284: Mrcp short case telling skills

Calm presence of family Propofol has been advocated for the therapy of terminal delirium because of its rapid onset, short half-life and lack of accumulation Potent neuroleptics eg haloperidol 5-20mg PO, SC, IV or levopromazin 25-200mg PO, SC, IV. Caution: extrapyramidal movement disorder, and BP drop. Increase antiepileptic as neuroleptics can decrease seizure control. Combinations with other drugs, eg TCA, may even ppt a delirium. Add BZD if necessary: (may worsen delirium) lorazepam 0.5-2mg (PO, sublingual, IM, IV), midazolam 5-20mg IV, SC, or diazepam 10-20mg rectal

LOC Raised ICP Epileptic seizure Hypoxia Infection Drug side effect (anticholinergics, BZD, opiates, H2 blockers, phenothiazine, steroids) Lab(Ca, Na, glu, ketone, uremia, hepatic enceph, endocrine disturbance) Hemato (anemia, coagulopathy) Intox (alcohol, drugs) Side effect of radiation

Myoclonus Hypoxic: piracetam Metabolic eg hypoglycemia/ drug

Page 285: Mrcp short case telling skills

induced eg anticholinergics, morphine or terminal multiorgan failure: BZD eg midazolam 5-10mg SC/ hr, diazepam 5-10 mg rectally, clonazepam, lorazepam PO/ sublingual

Drowsiness Exclude reversible causes Reorganise drug regimen Explain to relatives that patient may still hear them, therefore they should be encouraged to stay with the patients and talk to them. Tachycardia, tachypnea and sweating should be monitored as they may signal potentially treatable distressing symptoms

Epileptic seizure Non convulsive status epilepticus? If change in mentation suddenly. Rectal diazepam 10mg/ midazolam 10mg parenterally, followed by diazepam 10mg rectally every hour until the seizures stop, then 20mg rectally at night/ midazolam 10mg SC/ IV every hour until seizures stop, then 30mg/ 24 hr SC infusion. If steroids are tapered in the terminal phase, antiepileptic (and analgesic) should be increased prophylactically

Nausea and vomiting Raised ICP Other causes should be identified and treated Daily 4mg dexamethasone is enough if no impaired

Page 286: Mrcp short case telling skills

consciousness. Max 3x 8mg/ day Promethazine 25-50mg rectally/ SC Q8H Haloperidol 2-10mg IM, SC Q8-12H

Terminal dry mouth As long as can express thirst, no need to automatically substitute fluids. Excess of replacement can lead to over hydration leading to SOB, death rattle, brain edema. Good and regular oral hygiene (Q2H, show to relatives) 1-2ml of water by pipette or syringe Q30-60 min Smooth lips with Vaseline or dexpanthenol Moist air If candidiasis, treat with nystatin Pieces of pineapple, ice cream in small portions, crushed ice cubes in fabric etc If unconscious fluid replace to avoid dehydration, leading to delirium, rise in toxic metabolites, given SC, about 700ml/ 24 hours

Sedation Opposes the idea of palliation which intends to enable the patients to live until the last moment in a mentally alert state, so that they may enjoy their remaining time, say boodbye to relatives or set up their wills. Refractory suffering: barbiturates instead of opiates or BZDs.

Page 287: Mrcp short case telling skills

Titration of these centrally acting drugs must be stopped when sufficiently deep sedation is obtained without indirect signs of suffering (tachypnea, tachycardia, restlessness) “you matter until the last moment of your life. We will do all we can, not only to help you to die peacefully, but also to live until you die”

13) Riluzole 50mg BD prolongs 3 months after 18 months treatment- life prolonging effect (6

-21 months) is greater the earlier it is initiated. stop if LFT > 5x upper limit of normal, or neutropenia. Also linked to hypersensitivity pneumonitis. Not clear when treatment should be terminated. Realistic expectations for treatment effects and potential side effects. Consider in ALS < 5 years, with FVC> 60%, and without tracehostomy. Considered in PMA/ PLS patients who have first degree relative with ALS, and in symptomatic SOD1 gene mutation

14) Assisted ventilation

Symptoms of respiratory insufficiency

Signs of respiratory insufficiency

SOB on exertion Stress that patient will not “choke to death” -> prevented by suction, PT. There are enough medications to prevent this in every patient. Also before the lack of oxygen is felt, the paitent will calmly “go to sleep” due to rising pCO2. Treat reversible causes eg bronchospasm,

Tachypnea

Page 288: Mrcp short case telling skills

heart insufficiency, pneumonia, over hydration Non medical: explanation, calm presence of family, cool draught, PT, fan, space, trunk elevation Aim of this is suppression of respiratory awareness. RR of 15-20/ min is aim. Intermittent, short bouts-sublingual lorazepam 0.5-2.5mg Longer attacks >30 mins- inhaled morphine or IV midazolam 5-10mg Chronic – regular morphine 2.5mg PO 4-6x daily. For severe SOB, give sc morphine or IV infusion 0.5mg/ hr and titrate Oxygen only if symptomatic hypoxia and close to death as it can exacerbate CO2 retention and mouth dryness

Page 289: Mrcp short case telling skills

Opioids alone or in combination with BZDs if anxiety is present.

Orthopnea Use of accessory muscles

Difficulty clearing secretions

Paradoxical abdominal movement

Restless and unrefreshing sleep

Decreased chest movment

Excessive daytime sleepiness

Weak cough

Morning headache Dry mouth

Poor concentration and memory

sweating

Depression tachycardia

Lethargy dizziness

Fatigue syncope

Nocturia Confusion, hallucinations, rarely papilloedema

Poor appetite Weight loss

Diffuse pain in head, neck and extremities, cyanosis and edema, recurrent or chronic URTI, reduced appetite, recurrent gastritis, phonation difficulties, depression, anxiety, tremor, morning headache, difficulty falling asleep, disturbed sleep,

Page 290: Mrcp short case telling skills

nightmares, daytime fatigue and sleepiness, concentration problems

FVC: standing/ sitting and lying, symptoms, signs-> discuss options eg temporary NIV-> agreeable to consider ->sniff nasal pressure (for inspiratory muscle strength), pulse oxymetry, morning ABG -> if FVC < 80% or SNP <40cm H2O/ significant nocturnal desaturation on overnight oxymetry (>90% for >5% of the time asleep) or morning pCO2> 6.5kPa/ dyspnea, orthopnea, disturbed sleep, morning headache, poor concentration, loss of appetite or Epworth sleepiness scale> 9, supine VC <25% of sitting or standing VC -> for assisted ventilation NIV offered (BIPAP) (survival benefit greater than riluzole, median benefit up to 250 days, slows the decline of FVC, early initiation increases compliance) (initially for intermittent nocturnal support. As respiratory muscle strength declines, daytime NIV usually becomes necessary and patients may become dependent on nonstop ventilation.) (tolerance is reduced with bulbar onset but this does not prevent successful ventilation. Increased risk of aspiration in patients with bulbar onset and problems because of difficulties in clearing secretions or obstructions, such as those related to abnormal function of the vocal cords should be considered. Flaccid paresis of facial muscles may have difficulty using NIV) (FVC can be inaccurate in bulbar weakness, pseudobulbar palsy with apraxia of facial movements. Choice of ventilation depends on hypoventilation symptoms, Upper airway obstruction symptoms, bronchial secretions and factors eg availability, cost, patient preference and care) (TV for progression or cannot cooperate with NIV because of loss of bulbar tone and difficulty clearing secretions. But unable to communicate, costly, emotional and social impact -> intolerant-> invasive ventilation-> end of life care. Informed about terminal phase, tracheotomy ventilation, hospice referral and palliative care. Parenteral diamorphine, BZD, and antiemetic when ventilation withdrawn. -> tolerant but later declines-> / invasive ventilation/end of life care If impractical-> end of life care If refuses NIV, intermittent oxygen administration only during the day, when patient is awake, because of the danger of respiratory depression in chronically hypercapnic patients receiving oxygen during sleep Any infection or airway obstruction should be treated with antibiotics and physiotherapy, and they should have annual influenza immunisation If for tracheotomy ventilation, patients and carers must agree on the circumstances under which ventilator support will be withdrawn. When no reasonable hope of

Page 291: Mrcp short case telling skills

recovery, and death was anticipated without ventilator support, relieve suffering, not to hasten death, although death may be hastened as a consequence. Consequences of decision must be discussed with the patient, the family, and the home physician eg use of opioids in the terminal phase. Reassure that whenever she decides to stop NIV, all necessary care and appropriate medication will be available to prevent death by choking. When initiating ventilatory withdrawal, the primary goal is to maintain comfort. First discontinue supplemental oxygen, and end positive expiratory pressure, then convert to T piece and spontaneous breathing. At each step in the withdrawal, pretreat with sedative-hypnotic equivalent to 15mg/ hr of IV diazepam +/- opioid equivalent to 15mg/ hr IV morphine, raising the dose to relieve SOB and anxiety, even to point of unconsciousness if necessary and requested. Respiratory insufficiency in ALS is caused mainly by respiratory muscle or bulbar weakness and can be aggravated by aspiration and bronchopneumonia Early discussion of end of life to avoid unplanned TV VC < 1 L or < 25-30% predicted indicates significant risk of impending respiratory failure or death. Bulbar impairment increases risk of aspiration, and acute respiratory infection can ppt sudden respiratory failure

Advantages of TV disadvantages

Prevents aspiration, more secure ventilator, ability to provide higher ventilator pressure

More secretions generating, impairing swallowing, increasing aspiration, increasing risk of infection, tracheoesophageal fistula, tracheal stenosis or tracheomalacia, costs, 24hr nursing care, social, emotional burden. Possible subsequent clinical course which may end in a “locked-in” syndrome in the ICU, where any possibility of communication with the outside world is rendered impossible by a complete tetraplegia including extraocular msucles

15) Nutritional support

Contributory factors to malnutrition and weight loss: dysphagia, upper limb weakness slows eating and makes them dependent on others for cutting their food and feeding them, fatigue, weakness, worsening muscle atrophy, which prolongs meal times, and they do not finish their meals. Anxiety and depression can lead to anorexia. Immobility, reduced fluid and fibre and use of opioids can lead to constipation-> Loss of appetite Assessment of nutritional status: dietary history, BMI, anthropometry, weight at each clinic visit Swallow safety assessment by ST: frequency of choking, texture of food, drooling, meal duration, fatigue during meals kiv VFS Food modification: dietary counselling, blending/ adding thickeners to liquids, taught lip-seal and tongue exercises, chin-tuck manoeuvre (flexing neck forward), smaller and more frequent meals, high protein and caloric supplements, education of patient and carers in

Page 292: Mrcp short case telling skills

feeding and swallowing techniques such as supraglottic swallowing and postural changes. Can drink carbonated fluids or ice-cold fluids if difficulty swallowing tap water, esp if spastic dysphagia. Sufficient oral fluid intake is important to improve articulation, maintain oral hygiene and reduce risk of constipation. Recipe books available from Muscular dystrophy association (MDA) in US Poor dietary intake and dehydration/ unsafe to swallow/ losing > 10% baseline weight/ patient’s request-> assessment -> unlikely to survive > 3 months/ unable to manage feeds-> oral intake as tolerated/ palliative intravenous hydration or NG feed -> respiratory assessment: FVC, SNP, overnight oxymetry, morning ABG -> FVC >50%, SNP > 40 cm H20-> PEG/ radiologically inserted gatrostomy -> FVC <50%, SNP<40 cmH20-> NIV and NG feed x 2-4/52-> RIG

16) PEG/ RIG to give nutrition, hydration, stabilise weight, give drugs Complications: vomiting, diarrhoea, constipation, hemorrage, peristomal infection, pneumoperitoneum, peritonitis, gastrocolic fistula formation, parietal necrosis, weight gain and buried bumper syndrome. Death can occur secondary to aspiration and respiratory arrest after PEG placement or peritonitis after RIG NGT-> limit physical activity, nasopharyngeal discomfort, pain, ulceration, increase oropharyngeal secretions and lead to aspiration PEG-> needs mild sedation, which is hazardous is FVC< 50%, (possible insurgence of basal atelactases through pressure of the air-inflated stomach against the weakened diaphragm) can be done with NIV if respiratory imparirment. Timing based on symptoms, nutritionlal status and respiratory function. More mechanical obstruction or tube migration vs RIG. Complications include transient laryngeal spasm, localised infection, gastric hemorrhage, failure to place PEG due to technical difficulties, death due to respiratory arrest. Risk factors for post-PEG pneumonia include pre- PEG history of aspiration pneumonia, presence of reflux esophagitis during endoscopy, and concurrent infection. Cisapride is a prokinetic agnet that can be used to enhance gastric emptying. In aphonic patients, they can be treated with conservative laryngectomy or laryngeal diversion. Live 1-4 months longer on PEG than without. RIG-> no need sedation, can be done upright, but more pain vs PEG Note this does not eliminate oral feeding

17) Communication: progressive dysarthria or if cognitive impairment of frontal type- reduced verbal output, reduced spelling ability, word finding difficulty, and auditory comprehension of more complex input. Assessed by ST. Need to optimise personal partner-to-partner communication as well. When dysarthria progresess the use of an

Page 293: Mrcp short case telling skills

augmentative and alternative communication (AAC) system is needed. Prosthetic treatment (palatal lift and/ or palatal augmentation prosthesis) can be useful in reduction of hypernasality and improvement of articulation. For ventilated patients eye-pointing or eye-gaze augmentive high –tech communication devices are useful. Brain –computer-interfaces, EEG and EP (SCP) methods, thought translation devices can be used as new communication channels.

18) Palliative care early from time of diagnosis- maximise QOL of patients and family by relieving symptoms, providing emotional, psychological and spiritual support as needed, removing obstacles to a peaceful death and supporting the family in bereavement. Discuss preferences and decisions about interventions (eg assisted ventilation and advanced directives for life support or resuscitation.) Ensure whole- person approach to care that is local and supplements primary care with expertise in symptom control and end of life care. Include bereavement support. Adjustments to management plan every 2-3 months. Early referral to hospice or home care team to facilitate the work. Arrangements for care near the end of life must be triggered by severity of symptoms

19) Advance care plan- prolongation of life or comfort care-died in location of preference, prepared for death, treated in a respectful and sensitive manner by healthcare professionals. Nor ready to accept the limitations in life sustaining services that are required by some hospice facilities. In ALS patients whose primary goal is extension of life, they may present patients in whom the imminence of life threatening complications cannot be easily anticipated, do not accept their diagnosis or prognosis, or who simple have a strong desire to live. Patients want to initiate discussion about advance directives earlier than their physicians, but it should be the physician who initiates the discussion. To enhance the chance of a peaceful death. Typical symptoms in the terminal phase of neurological disease should be anticipated by the neurologist and explained to patients and relatives. Stress that there are enough therapeutic possibilities to prevent the patient from suffering. It is the small details which make the difference in the management of terminal patients. In addition to medical information, information as to where relatives may turn for help can be given. Prefered route for drug administration is oral. Unnecessary IV lines reduce mobility. Alternative routes are rectal/ sublingual. If unconscious, can reduce some of the medication, but stopping opoids or BZDs abruptly may cause withdrawal symptoms.

20) Advanced directive-legal situation, naming of health care proxy, re evaluate at least 6 monthly

21) Multidisciplinary care improves care, and extends survival 22) Clinic: every 2-3 months esp first half year following diagnosis, and in later stages of the

disease. Very slowly progressing disease can be seen 1x/ 2x / year. Check for s/s of respiratory insufficiency. FVC. Weigh each visit. Dysphagia assessment. Discuss preference for life-sustaining treatment every 6 months

23) If long term limitations with intermittent exacerbation and sudden dying-Ongoing disease management, advance care planning, mobilising services to the home. Requires continuity and comprehensiveness of care. Services must match the course of the illness.

24) If prolonged dwindling- supportive services at home, like Meals on wheels and home health aides, then institutional long term care facilities are central to good care

Page 294: Mrcp short case telling skills

25) Care needs generated by symptoms or disabilities are urgent priorities 26) Adjust care to family and patient resources, to varying needs, and preferences 27) Care system that works: correct medic al treatment, reliable symptoms relief, no gaps,

no surprises in the course of care, customised care, consideration for family situation, help to make the best of every day. Teach essentials of disease management, how to recognise symptoms and prevent worsening of illness. Ensure availability of key medications. Advance directives for sudden death. Early intervention for signs of exacerbation, In home adaptations and equipment to ensure comfort. Tailor care plant to patient and family.

28) Ill and dying patients: value adequate symptom control, independence, autonomy, strong relationships and completion. They seek to prepare for death, contribute to other, and avoid prolongation of dying. Thus palliative care includes symptoms assessment and management, advance care planning, identification of goals, assessment and management of psychological and spiritual needs, assessment of patients’ support system and communication of estimated survival. The degree to which patients require extensive care giving may conflict with the patients’ overall goal to contribute to, not burden, their family. May be admitted to hospice institutions for the last days of life, if care at home is no longer possible

29) Information must be timed appropriately for decision making, and well in advance of major management crossroads. Decision making is a dynamic process that may change as disease progresses

30) Nurture therapeutic relationship from diagnosis to palliation 31) QOL can be measured with generic measure such as SF-36 together with a disease-

specific measure Attention of one symptom improves control of all symptoms. Clarify who is bothered by the symptom (caregiver, patient, family), determine what was helpful in the past, determine whether treatment was correctly given as prescribed. Residual symptoms should be carefully reviewed. An attitude of quiet confidence and cautious optimism is reassuring. Need to take time to listen, admit the problem, explain

what is happening, convey to the patient your continuing invo Acute visual failure

Transient: monocular: carotid TIA, ocular causes (glaucoma) binocular: raised ICP, migraine, epilepsy, VB TIA persistent: monocular: AION, optic neuritis (ON), tumour, aneurysm (complicated), infection (sphenoid-ethmoid) binocular: occipital infarct, toxic neuropathy, pituitary apoplexy, migraine, seizure visual loss-> patient reports one- sided-> one eye/ one hemifield in front of chiasma-monocular at chiasma- bitemporal hemianopia/ binasal behind chiasma- homonymous hemianopia

Page 295: Mrcp short case telling skills

patients don’t complain because

1) overlapping nasal fields 2) associated inattention 3) filling in phenomenon from intact cortex 4) most visual functions in central fields

eye pain in occipital infarct (tentorial nerve derived from recurrent branch of ophthalmic division of CN5) monocular (optic nerve/ ocular cause)/ binocular (visual pathway behind chiasma) -alternate eye cover -altitudinal visual loss-monocular -positive scotoma-retinal (monocular) -associated symptoms like vertigo, giddiness Optic nerve: Symptoms: Dimming or darkening of vision Colour desaturation (looks faded) Signs: RAPD Optic nerve head pallor/ swelling VF defect Not optic nerve: positive scotoma, ghost images, halves, metamorphopsia (lines curved or bent) ocular causes: CRAO CRVO Retinal detachment Central serous retinopathy Acute zonal occult outer retinopathy (AZOOR) AION: Ciliary artery occlusion Altitudinal field defect Arteritic (GCA)/ non arteritic

Arteritic (GCA) Non arteritic

½ will develop visual loss in the other eye (1/3 within 1/7, 1/3 within 1/52, 1/3

Usually on waking (?viagra night before) Lower altitudinal VF defect

Page 296: Mrcp short case telling skills

within 1/12) Age> 50 ESR> 50 Temporal tenderness Dx: fluorescein angiography, Doppler sonography, temporal artery biopsy GCA: AION (90%), CRAO (3-5%), AION and CRAO, AION and diplopia GCA- new headache, preceding amaurosis fugax/ diplopia, jaw claudication

Hyperemic disc swelling Disc at risk- follow eye (cup disc ratio < 0.1 (cup small, normal ratio 0.4) Spontaneous improvement in 40% Sequential involvement of other eye (16% over next 5 years PION (posterior ION)-difficult to diagnose, compressive lesion Optic nerve enhancement on MRI -97%-ON, 6%-AION -non specific -signifies disruption of BBB -inflammation/ mediated mediated/ infectious -infiltrative-sarcoid -neoplastic -ischemic

Optic neuritis: Immune mediated

1) primary CNS (Demyelinating)-MS, NMO, ADEM, CIS (idiopathic) 2) secondary CNS (systemic disease- SLE, sarcoid, behcet’s, sjogren’s; parainfectious;

paraneoplastic) infective: syphilis, TB, lyme’s, CMV Symptoms:

1) visual loss- acute, nadir within 10/7, improvement within 30/7 (4-6/52) 2) pain- 80%, ocular/ periorbital pain, on ocular movement 3) Uthoff’s phenomenon- 30-50% with exercise/ heat- thermal lability of a partially

demyelinated nerve (even 0.5 degrees increase) 4) Pulfrich’s phenomenon- stereo-illusion (unequal conduction time in both eyes), disorder

of depth, DDX ataxia (swing pen/ pendulum with linear course- patient perceives it as elliptical (poor man’s VEP)

Signs: VA impairment, RAPD, disc swelling 1/3 (papillitis), normal disc 2/3 (retrobulbar neuritis), VF defects (diffuse 48%, altitudinal 20%), Classically taught-> central or centrocecal 8%, colour vision desaturation Singapore: papillitis> retrobulbar neuritis

Page 297: Mrcp short case telling skills

-less association with MS

Syndrome of optic neuritis and transverse myelitis (NMO syndrome) CNS autoimmune disease: MS, Devic’s, ADEM Systemic AI disease: CTD- SLE, Sjogren’s, Behcet’s, sarcoid; vasculitis; APLS; paraneoplastic

MS: macdonald’s criteria 2005 At least 2 attacks and at least 2 lesions At least 2 attacks and 1 clinical lesion and 1 MRI lesion (in space) 1 attack and at least 2 lesions and 1 MRI lesion (in space) CIS and MRI lesion (in space) and MRI lesion (in time) 1 attack: last at least 24 hours, confirmed by examination, likely to be inflammatory/ demyelinating New attack- interval at least 1/12 MS lesions: juxtacortical (u fibres) must touch the cortex, dawson fingers MS 2008: 14-1157-1174 (DDx of suspected MS Devic’s disease (NMO): Core features: ON and myelitis Supportive features: Brain MRI not diagnostic of MS cord lesion at least 3 vertebral segments NMO Ig G positive 73% sensitive, 90% specific Attacks are more severe Longer cord segments MRI: normal/ non specific white matter changes, unique lesion 10% (periependymal regions (around 3rd/ 4th ventricles) and hypothalamus) NMO spectrum: NMO Limited forms (forme fruste)-cord alone (recurrent myelitis), ON without cord involvement Asian optospinal MS (controversial) ON +/- longitudinal extensive myelitis with associated systemic AI diseases ON/ myelitis or other systemic syndromes with brain lesions typical of NMO NMO coexists with ANA positive (40%), sjogren’s Ab positive (15%) Treatment of NMO (different from MS- IFN)

a) acute attack- IV methylprednisolone, Plasma exchange b) relapse- Aza/ Pred/ Mitoxantrone OR rituximab

ADEM: acute/ subacute onset multifocal areas of CNS

Page 298: Mrcp short case telling skills

C/F: polysymptomatic encephalopathic-altered consciousness/ behavioural changes

1) monophasic -usually white matter -may involve grey matter -larger lesions 2) recurrent ADEM

- at least 3/12 after first episode - without involvement of new clinical areas - at least 1/12 after completing steroid therapy

3) multiphasic- new areas - at least 3/12 after first event - at least 1/12 after completing steroid therapy - (proposed but not accepted, unable to differentiate from MS)

SLE: Transverse myelitis: infarct (arteritis) SDH (coagulopathy) Subpial leucomyelopathy (dysimmune) ON: acute optic neuritis, AION, chronic progressive visual loss (unexplained) Sjogren’s: Primary Secondary Usually transverse myelitis (multifocal stimulating MS) Usually no optic neuritis Behcet’s Sarcoid: Myelopathy 3rd most common presentation CN2 (1/3 bilateral)/ CN7 Often subacute Steroid dependent Uveitis and granuloma are other eye manifestations Visual loss in paraneoplastic syndromes: Cancer associated retinopathy (CAR)-recoverin Ab Myeloma associated retinopathy (MAR) ON (CRMP-5)

Page 299: Mrcp short case telling skills

Myelopathy in paraneoplastic syndromes CRMP-5 amphiphysin Ab

32) lvement. Foster sense of personhood and independence Transient neurological syndromes

1) TIA 2) Seizures 3) Migraine 4) Demyelinating disease

Approach to white matter lesions on MRI

1) infectious-syphilis 2) immune mediated -CNS vasculitis -Hashimoto’s vasulitis -MS (psychiatric symptoms very unlikely) (white matter) (white and grey matter- NMO) -ADEM 3) paraneoplastic 4) vasculitis 5) metabolic/ endocrine, metachromatic leucodystrophy, adrenocorticoleucodystrophy 6) toxic/ drug 7) mitochondrial Always refer eye- many diseases affect eye and brain Diffuse white matter changes on MRI with a background of migraine -mitochondrial disease -CADASIL -PFO (with migraine) -Hashimoto’s -Leucodystrophy (cognitive changes)

Cerebellar stroke Cerebellum: median vermis and 2 lateral hemispheres Clinical examination Supine: Eyes

1) nystagmus- vestibular, gaze holding

Page 300: Mrcp short case telling skills

2) jerky pursuit 3) OKN-reduced 4) VOR-impaired visual cancellation (orange stick in mouth, turn head side to side)-

cerebellum helps to inhibit VOR-if cerebellum affected, there is impaired visual cancellation

5) Saccades-dysmetria (hypometria-fingers far apart; or hypermetria-overshooting of gaze-fingers close together), posst saccadic drift

6) Binocular control-esotropia (horizontal), skew deviation (vertical) UL

1) finger nose (finger-ear/ finger-glabellar) 2) dysdiadochokinesia 3) dyschronometria (slowness on affected side) 4) hypotonia (asymmetry of wrist flopping )

LL 1) heel shin- 3 parts 2) finger toe

sitting up pendular knee jerk standing up

1) both feet together eyes open, look straight 2) both feet together eyes closed (10 sec) 3) stand with 1 foot forward, eyes open (5 sec) 4) stand on 1 leg (check both) (5 sec) 5) hop around on 1 leg 6) pull test- 2 steps of compensation are allowed

walking 1) wide based gait 2) imbalance while turning 3) tandem walk

talking 1) british constitution/ baby hippotamus

prehension sign- opening up of the affected hand before reaching the object in order to grab it grip-ungrip your finger with both hands- affected hand is slower ( same like dyschronometria) radar maker step sign- stand 1 foot in front of another, then push from behind -if normal foot behind- compensate by bringing that foot forward to prevent a fall -if abnormal foot is behind-affected foot can’t move forward to support when pushed Arterial supply:

1) superior cerebellar artery-from top of the basilar artery 2) anterior inferior cerebellar artery-from bottom of the basilar artery 3) posterior inferior cerebellar artery-from vertebral artery

Page 301: Mrcp short case telling skills

frequency of affected arteries in stroke: PICA> SCA> AICA 3 anatomical facts: 1) brainstem signs are common 2) internal auditory artery (mostly from AICA) present with deafness and vertigo in cerebellar stroke if affected 3) numerous anatomical variations – can’t be dogmatic SCA: usually posterior cerebral artery and thalamosubthalamic artery is also affected. SCA rarely affected alone

Structures affected Clinical features

Rostral ½ of cerebellum Limb ataxia

Rostral vermis Dysarthria Gait ataxia

Superior cerebellar peduncle

Lateralopulsion

Dorsolateral midbrain Ipsilateral horner’s Ipsilateral limb ataxia Contralateral pain and temp (spinothalamic)

Dorsolateral pons CN4

AICA:

Structures affected Clinical features

Middle cerebellar peduncle Ataxia Nystagmus Lateralopulsion

Pons 5,6,7,8,PPRF, INO, sympathetics, spinothalamic

Labyrinth (internal auditory artery)

8 (vertigo, tinnitus, deafness)

PICA:

Structures affected Clinical features

Medial branch Isolated vertigo

Vermis and adjacent cerebelllar hemisphere

Lateralopulsion

Small branches to medulla Lateral medullary syndrome (most common cause is disease of the vertebral

Page 302: Mrcp short case telling skills

artery (and not PICA))

Lateral PICA unknown

three functional cerebellar circuits

1) vestibular loop (archicerebellum- old) from vestibulocerebellar (flocculonodular-uvular) to cerebellovestibular (inferior cerebellar peduncle) (stand upright from crawling) affects equilibrium (truncal ataxia), and oculomotor (nystagmus) eg medulloblastoma in a child- other cerebellar signs are absent 2) spinal loop (paleocerebellum-anterior cerebellum) dorsal and ventral (anterior) spino-cerebellar tracts (superior cerebellar peduncle-> anterior cerebellum)-cerebello (fastigeal) vestibulospinal (walk, run) affects tone (hypotonia, pendular knee jerk), posture and gait (gait ataxia) eg alcoholic cerebellar degeneration (only severe gait ataxia but no other cerebellar signs in eyes, ULs) 3) cortical loop (neocerebellum) corticopontocerebellar (middle cerebellar peduncle) dentatorubrathalamocortical (superior cerebellar peduncle) affects coordination of limb movement (intention tremor, dysmetria), articulation (dysarthria) inferior cerebellar peduncle- input from vestibular nuclei (medulla) middle cerebellar peduncle -input from cerebral hemispheres superior cerebellar peduncle- input from spinal cord, output to cerebral hemispheres

functionally and phyllogenetically:

1) floculonodular lobe 2) anterior lobe 3) posterior lobe

cerebellar infarct presenting as isolated vertigo

1) direction changing nystagmus 2) ataxic/ jerky pursuit 3) lateralopulsion (severe posture instability) 4) head thrust normal (17% of cerebellar strokes present as pseudo vestibular neuronitis)

Management: Normal CT head does not exclude cerebellar infarct MRI brain to look for cerebellar lesions Complications:

1) brainstem compression 2) hydrocephalus

Page 303: Mrcp short case telling skills

3) upward herniation of cerebellar tissue 16-20% will deteriorate, onset anytime in the first week, with peak 3rd day after infarction

Surgical intervention: Drowsiness: IV mannital/ surgery (indications: hematoma> 3 cm, hydrocephalus, brainstem compression) No drowsiness: monitor Vertigo: Only with posture change: BPPV Spontaneous: with tinnitus and deafness: meniere’s disease without tinnitus and deafness: ?vestibular neuronitis with brainstem/ cerebellar signs: verebrobasilar ischemia Gait Gait apraxia: simulate brushing of teeth light cigarette wave goodbye say “shh…” gait analysis: 6 steps: start with watching patient get up from bed/ chair

1) vertical axis (posture): stooped- PD waddling- muscular dystrophy 2) UL arm swing- PD effeminate gait- Chorea 3) feet a) initiation- prompt and ignition failure: late PD (part of freezing) gait apraxia (NPH) b) steps slow, shallow and small (3s)- shuffling gait c) high stepping- foot drop of LMN type d) broad based- ataxic 4) walking in a straight line (tandem gait) 5) turning- turning by numbers, turn en bloc, loss of balance on turning 6) festination-walk/ run fast, then stop suddenly

Chorea Classically fidgety, clumsy and have facial movments Signs include: darting tongue, milkmaid’s grip, pronator sign, dish spoon hand, feminine gait

Page 304: Mrcp short case telling skills

DDX: Fidgetiness, tics (but these two can be voluntarily suppressed) CIDP: Typical: Proximal and distal Motor and sensory UL and LL Hypo/ areflexia of all 4 limbs Atypical: Symmetrical: predominantly DADs, distal, pure motor, pure sensory Asymmetrical: multifocal acquired demyelinating sensory and motor polyneuropathy (MADSAM) Chronic progression: at least 2/12 CIDP umbrella: not included are anti MAG and MMNCB (these don’t respond to steroids) rule out: myeloma screen, AI screen, hematological malignancy before saying no objective sensory involvement- rule out 2 point discrimination latest reports- no increased incidence of CIDP in DM patients Do MRA to rule out vascular cause for all cases of CN12 palsy LMN: Genioglossus movts tongue in opposite directions, and the resultant effect is the tongue in the centre Genioglossus-> tongue protrusion/ deviation Tongue -intrinsic muscle-horizontal and vertical (tongue inside mouth) -extrinsic muscle-genioglossus Iatrogenic -NPC radiation (CN12 most commonly affected) -CEA Intrinsic muscle (of tongue) wasting-> curving of median raphe Cause:

1) Neoplasm ½ of CN12 palsy are due to tumours >1/2 are malignant NPC/ metastases Mets affecting CN12 usually located at medulla, leptomeninges, base of skull, neck

Page 305: Mrcp short case telling skills

12% of meningeal mets cause CN12 palsy Can be sole manifestation of meningeal met

2) Arnold- chiari malformation CN12 palsy seen in 15-18% of AC malformation Due to herniation of cerebellar tonsils compressing the CN12

3) Surgery CN12 can be a transient complication in 5% of CEA

4) Vascular 1) Medial medullary syndrome- ipsilateral tongue paralysis, contralateral hemiplegia,

contralateral position sense 2) Internal carotid artery dissection-mechanisms a) Compression by subadventitial mural hematoma b) Compression by false aneurysm c) Nutrient artery ischemia

Associated with horner’s syndrome Can also cause CN 9,10,11,12 palsies CN12 is most common CN affected 12% of ICA dissection present with CN palsies

3) Rarely hypoglossal vertebral entrapment syndrome- by ecstatic right vertebral artery 5) Radiation

Most common CN affected by radiation due to long, exposed course in the neck 6) Infection 7) Trauma

Japia’s syndrome: tongue/ larynx paralysis, sparing of palate, classically in bull fighter’s/ post intubation, lesion is below emergence of pharyngeal nerve Godtfredson syndrome: CN12 and CN6 on the same side (clivus lesion) UMN: Location: 1) Cortical

Operculum syndrome Lesion in bilateral anterior perisylvian lesions Loss of voluntary control by facial muscles, tongue, pharyngeal, mastication +/- ocular muscles Reflex control is preserved Preservation of automatic function of these muscles

2) Corona radiata Transient tongue stiffness, slurring of speech, contralateral tongue weakness

3) Genu of the internal capsule Isolated weakness of genioglossus (+/- facial weakness)

Page 306: Mrcp short case telling skills

4) Medulla Medial medullary syndrome (tongue and facial paralysis) Ipsilateral tongue paralysis + UMN contralateral facial palsy (lateral medullary -ipsilateral UMN face) CN12 goes to ipsilateral and contralateral nucleus, with CN7 coursing down in between the 2 nuclei. Therefore medial medullary syndrome can cause 1) Ipsilateral UMN tongue and contralateral UMN face 2) Contralateral UMN tongue and contralateral UMN face 3) Ipsilateral LMN tongue and ipsilateral LMN face 4) Contralateral LMN tongue and ipsilateral LMN face CN12 palsy (UMN and LMN) -total tongue paralysis -seen in right ICA dissection -right CN12 LMN if focal, - if stroke- hemiplegia and left UMN 12

CN6 palsy Abduction failure- lateral rectus eg myositis or CN6 Esotropia eg on corneal reflection

1) Is it isolated? -check corneal reflex (cavernous sinus/ superior orbital fissure) -check plantar reflex (brainstem) -check horner’s (cavernous sinus) -check conduction deafness (NPC) Causes: Cavernous sinus (CN6 and horner’s -> cavernous sinus lesion; CN6 and V1- cavernous sinus/ superior orbital fissure but CN6 and V2/3- cavernous sinus lesion) cavernous sinus divided by trigeminal nerve into 3 segments (anterior V1, middle V2, posterior V3) 1) vascular: carotico cavernous fistula (triad of conjunctival injection (dilated arterialised vessels), proptosis, bruit (episodic tinnitus) –ask the patient! Unilateral CC fistula can cause bilateral CN6 palsy. Dipolpia from mm/ nerve ischemia. high flow- traumatic, acute onset, tear in carotid artery; low flow- spontaneous, insidious, small dural vessels), aneurysm (cavernous carotid artery aneurysm- 13% mirror image aneurysm on other side (asymptomatic), 75% present with opthalmoplegia (CN3, 4, and 6 (most common as CN6 is closest to ICA inside the cavernous sinus)) 2) extracranial ICA dissection- classically causes a horner’s- CN6 palsy- due to extension into cavernous sinus 3) inflammatory: immune mediated-tolosa hunt (non specific granulomatous inflammation, painful CN3/ 6 diagnosis of exclusion, good prognosis with steoroid); or infective- aspergilloma

Page 307: Mrcp short case telling skills

4) neoplastic: meningioma (meningioma en plaque), pituitary tumour (lateral extension of tumour; pituitary apoplexy- hemorrhage/ infarct; shrinkage of pituitary tumour by bromocriptine; can also causes CN6 palsy due to adhesions causing traction of CN6; extension superior- visual loss (chiasma), lateral- cavernous sinus lesion) skull base 1) cerebral venous thrombosis 2) meningioma 3) clival chordoma 4) gradenigo syndrome (painful 6th) (CN6 and CN5 (facial pain) +/- CN7/ 8 (or conductive deafness from otitis media) (petrositis- spreading infection from otitis media (underlying mastoid) subarachnoid space/ meninges 1) ischemic (usually CN6, also optic neuropathy- most common cause of CN6 in eye clinic) (criteria: isolated CN6, >40 yo, ischemic risk factors eg DM (OGTT), recovery within 6/12 (86%)) if no recovery in 6/12- repeat MRI for pontine glioma, clivus chordoma, cavernous sinus, thin cuts (brainstem, clivus, cavernous sinus) (to rule out false localizing sign/ anterior inferior cerebellar artery (AICA) causing CN6 palsy due to compression of CN6 along its course) (all patients with idiopathic CN6 should have an LP) 2) intracranial hypertension 3) meningitis 4) siderosis brainstem 1) vascular 2) inflammation eg MS, ADEM (can present with isolated CN6 (1.6%), rarely sole sign of exacerbation, CN6 is the most common nerve presentation) 3) metabolic eg wernicke’s (triad of nystagmus (in decreasing order of frequency), (upbeat/ gaze evoked), bilateral CN6, horizontal/ vertical gaze palsy; involvement of tissues surrounding 3rd/ 4th ventricle), Central pontine myelinolysis (bilateral CN6, gaze (PPRF), bilateral miosis (sympathetic chain for horner’s), bilateral V sensory (corneal)) 4) neoplastic (<2% gliomas, 50% diffusely infiltrative, CN involvement is common, children more than adults) Brainstem glioma causes CN palsy in 87% CN7 37% CN6 33% CN8 33% CN9,10,11 33% CN5 21% CN3 4% CN4, 12 2% Pyramidal signs 58%

Page 308: Mrcp short case telling skills

Sensory loss 25% Babinski can be used for localization but absence of pyramidal signs does not rule out brainstem glioma Nuclear CN6 palsy does not cause lateral rectus palsy but causes gaze palsy like PPRF lesion CN6 vs PPRF lesion: CN6 if oculocephalic reflex is absent (does not need PPRF) CN6 if CN7 also involved (both nuclei in vicinity) CN6 and conductive deafness= NPC until proven otherwise (NPC starts in fossa of Russenmuller and blocks the eustachian tube opening) Chronic isolated CN6 (at least 6/12) -check plantar response and MRI brain to rule out pontine glioma, chordoma, meningioma en plaque diplopia - monocular/ bonocular - horizontal/ vertical vertical diplopia without ptosis – CN4 palsy causes: adult: ischemic - OGTT, ESR, VDRL/ neoplasm- cervical nodes, hearing, ENT, MRI children: brainstem glioma- MRI, post infectious (non specific viral)- observe any age: raised ICP- fundoscopy Psuedo CN6 LR/ MR: 1)myositis/ Thyroid associated orbitopathy-forced duction test/ FT4/ CT orbits/ MRI 2)MG-tensilon/ EMG 3) blow out fracture- history of trauma 4) Duane syndrome (absence of CN6 nucleus)-narrowing of palpebral fissure on adduction 5)non paralytic squint, decompensated squint- FAT (family album tomography) scan 6)bilateral convergence spasm (Spasm of the near reflex, as first described by Cogan in 1955, is a triad of intermittent convergent strabismus, accommodative spasm, and pupillary miosis)-miosis 7) thalamic esotropia (usually medial thalamus) 8) anticholinergic esotropia- benztropine eye drop induced- for near vision, needs miosis, accommodation (paralysed by benztropine), convergence (excessive to compensate for paralysis of accommodation) Lengthening of medial rectus and causing pseudo paresis

Page 309: Mrcp short case telling skills

Disorders of higher visual function Visual cortical areas (V1,V4,V5) V1- primary visual cortex

1) Cortical blindness (with denial)- anton’s syndrome (confabulation/ confusion/ denial of blindness, bilateral retrogeniculate lesion, visual loss equal in both eyes, any degree of blindness, light perception preserved usually, visual defect- bilateral hemianopia and macular sparing (constricted visual field), difficult to differentiate from non organic causes because pupil reflex, EOM and fundoscopy normal)

DDX of constricted visual field (pure tunnel vision) -glaucoma, retinitis pigmentosa, central retinal artery occlusion, non organic (functional tunnel vision) Causes of cortical blindness -stroke (top of the basilar), migraine, head injury (tentorial herniation-> PCA occlusion), encephalitis, anoxia, CJD variant, demyelinating disease

2) Blind sight -ability to detect in the blind field stimulus presence, direction of motion, target displacement, object discrimination due to surviving connections from lateral geniculate body) -association areas that bypass striate cortex -residual functioning islands of striate cortex

3) Riddoch phenomenon -preservation of motor perception in otherwise complete scotoma -striato-kinetic dissociation - theories: preserved islands of function, extrastriate areas (V5) involved

4) ‘Filling-in’ phenomenon -the reason for not appreciating visual loss in HH -appreciate only temporal visual loss

V4-colour centre (usually bilateral) Impairment causes cerebral dyschromatopsia Dsis- sudden in onset, associated visual field defects-> bilateral superior homonymous quadrantanopia V5-motion centre-perception of motion Impairment causes akinetopsia -now reported in Alzhiemer’s disease (post cortical variant) Visual processing pathways (dorsal pathway: “where”, speed; ventral pathway: “what”)

Page 310: Mrcp short case telling skills

Dorsal occipitofugal pathway “where pathway” V1-> V2,V3,V5-> post parietal cortex Causes:

1) hemispatial neglect -line bisection (with very short lines <20 mm, the bisection bias reverses (cross-over effect) -line cancellation -clock face -flower 2) visual allesthesia -transposition of visual fields -environmental rotation -lesions in lateral medulla or occipito parietal junction theory: disturbance of integration of visual information with vestibular and proprioceptive input 3) Balint’s syndrome Triad of 1) simultanagnosia –can’t attend to >1 item of a complex background, can’t simultaneously

integrate the visual clues eg “cookie theft picture” 2) optic ataxia-defective hand movement under visual guidance (visual spasm of fixation)

DDX: cerebellar ataxia- no intention tremor/ dysdiadochokinesia, can reach targets on own body accurately 3) occulomotor apraxia - if fixes on central target, unable to move eyes to peripheral target -not a true apraxia -visually guided saccade problem -occulomotor apraxia will have dysfunction of frontal lobe (frontal eye field) and posterior parieto-occipital regions) -inability to voluntarily direct gaze to visual target -all voluntary eye movements are affected if true Causes of Balint’s syndrome 1) bilateral parieto-occipital lesions in CVA 2) alzheimer’s disease 3) CJD 4) Diffuse lewy body disease 5) CO poisoning

Page 311: Mrcp short case telling skills

4)Palinopsia -persistence or recurrence of a previously seen visual image (palin-again in latin) -cerebral palinopsia –both images equally clear, image persists on closing eye- one of the causes of monocular diplopia

a) temporal (image in time) b) spatial (image in space)

-causes: right parieto-occipital infarcts/ lesions drugs (clomiphene, risperidone, topiramate) metabolic (non ketotic hyperglycemia) ventral occipitofugal pathway-what pathway V1-> V2 and V4

1) language (angular gyrus) (visual-verbal disconnection) - alexia-cannot read written words (with agraphia-can’t read or write, speech

centre is involved; without agraphia-speech centre is intact, hemialexia-in hemifield; hemianopic alexia-impairment in text reading > single word, due to VF defect, right homonymouis hemianopia encroaching into central or parafoveal vision, lesion in medial occipital cortex (primary visual cortex); pure alexia-visual agnosia for words, impairment in single word recognition, lesion in posterior fusiform/ inferior temporal gyrus, +/- hemianopia

- colour anomia-can’t name colour, intact ability to match colours, intact semantic recall eg apple is green in colour

- object anomia-can’t name object by sight, intact ability to name it by touch or sound, can tell function of objects

2) object recognition-inferior temporal cortex (visual-visual disconnection) - visual agnosia-can’t identify object by sight - subtype is prosopagnosia-inability to recognise familiar faces, identify from non –

visual clues eg voice - facial recognition requires intact middle fusiform gyrus, lingual gyrus and has right

sided dominance - lesion in lingual and fusiform gyrus of occipital lobe (usually bilateral)

3) memory, emotion-limbic system (visual-limbic disconnection) - visual hyperemotionality-blunting or absence of emotional response to visual

stimuli (intact to music etc) - visual amneisa

Alexia without agraphia: seen in occipital lobe infarcts of dominant hemisphere which involve the splenium (although occipital lobe infarcts rarely involve the splenium) Splenium of the corpus -reversible lesion -excellent prognosis DDX:

Page 312: Mrcp short case telling skills

1) infective encephalitis: viral (influenza, VZV, adenoV, rotaV), bacterial (E. coli, Salmonella) 2) ADEM/ MS-bilateral symmetrical, subcortical white matter 3) Stroke –Vertebrobasilar circulation 4) PRES 5) Machiafava-Bigami disease 6) Lymphoma 7) Pontine myelinolysis 8) Anti-epileptic drugs (phenytoin toxicity, vigabatrin) 9) Transcallosal seizure

Pathology: intramyelinic edema, inflammation Dizziness 4 questions

1) Vertiginous (problem with vestibular system; associated with nausea and vomiting) vs non vertiginous giddiness (syncopal giddiness; sensation of near faint or syncope, transient hypoperfusion to the brain, associated blurring of vision, no nausea/ vomiting)

2) If vertiginous Peripheral (inner ear/ CN8)- tinnitus, hearing loss, ear pain, ear discharge, ear

fullness nausea and vomiting severe, Central (cerebellum/ brainstem)- diplopia, facial numbness, facial asymmetry,

swallowing, limb weakness/ numbness, visual field, speech problems, rule out central signs, nystagmus, head thrust sign

Gaze evoked (dependent) nystagmus-> changes direction, not present at rest/ primary gaze -> central nystagmus- can have upbeating and downbeating components on gaze

Unilateral nystagmus- central or peripheral Central nystagmus

- pure vertical (upbeating or downbeating) - pure rotatory at primary position - gaze evoked nystagmus

horizontal nystagmus at rest -usually peripheral head thrust (fast VOR) -positive in peripheral nystagmus -also positive when CN8 vestibular nucleus involed -uses saccades to remove pursuit compensation as pursuit can compensate even though VOR impaired -abnormal left head thrust- left vestibular system, left ear problem

Page 313: Mrcp short case telling skills

- slow component- eyes towards right - fast component- towards left to keep gaze central

central integrators (gaze holding system) -“stay there”neurons -faulty in cerebellar and brainstem disorders vestibular system- gives space and velocity information independent of vision and somato sensory inputs fukuda’s marching test -less sensitive than rhomberg’s -not localizing

3) duration of vertigo BPPV

- seconds/ mins - Dix hallpike positive, cannot rule out BPPV if Dix hallpike negative

Acute vestibular neuronitis/ labyrinthitis -preceding URTI, giddiness for 1-2 days, vomiting

migraine -hours

meniere’s -triad of recurrent episodes of vertigo (lasts hours) -nausea and vomiting -fullness of ears (hydrops of ear like glaucoma in eye)= increased pressure

TIA/ stroke -lasts minutes or hours

posterior circulation stroke -ongoing 4) recurrent or non recurrent

characteristic of first attack is very important. Sometimes, what follows is just residual vestibulopathy Recurrent: BPPV, meniere’s, posterior circulation TIA (not frequent, either stop or proceed to stroke), migraine, labyrinthine fistula Non recurrent: acute vestibular neuronitis, posterior circulation stroke, first presentation of recurrent vertigo

Page 314: Mrcp short case telling skills

Scenarios

1) vertiginous, no CNS/ PNS features, hours, first episode - posterior circulation/ vestibular neuronitis 2) vertiginous, no CNS/ PNS features, short spells, recurrent - TIA/ BPPV 3) vertiginous, CNS features, hours, recurrent -posterior circulation 4) vertiginous, PNS features, hours, recurrent -migraine/ meniere’s

non vertiginous giddiness

1) anxiety (hyper ventilation) (psychosomatisation- anxiety neurosis, hyperventilation syndromes, phobias.)

2) diffuse central hypo oxygenation/ hypoperfusion -blood- hypovolemia, anemia, hypoNa -low BP- drugs, autonomic failure, hypocortisolism -blood pump- cardiac arrythmia, cardiac outflow obstruction -blood vessel- posterior circulation TIA/ stroke (can present with LOC due to involvement of reticular activating system in the brainstem-usually no diffuse brain hypoperfusion due to collateral -nerves to blood vessel- vasovagal syncope (normal physiological response, giddiness before fainting, special circumstances, warning, characteristic surroundings, short period of fainting, history of frequent faints before

Psycho physical vertigo- needs vestibular rehab and sedatives eg BZDs Vestibular sedatives

- interferes with the body’s adapting mechanisms (cerebellum can’t adapt) - only give when patient is acutely symptomatic for very short periods - recovery is variable- hypervigilance of visceral symptoms eg IBD

beware of bilateral vestibular problem- not much of signs eg gentamicin toxicity -no nystagmus -rhomberg’s positive (in absense of peripheral neuropathy) -bilateral positive head thrust -snellens chart-if drop by 2 lines on shaking

Page 315: Mrcp short case telling skills

pure vertigo- likely vestibular- no auditory symptoms/ brainstem symptoms vestibular neuronitis -vertigo is symptom, sign is nystagmus -VOR (head thrust) -vestibulo spinal reflex- balance

- standing, - high step march- deviate to one side - forward/ backward 2 steps- make a star

nystagmus on primary gaze (2nd degree nystagmus) Entrapment neuropathies (cortical hand) Peripheral neuropathy Polyneuropathy: diffuse, symmetrical, simultaneous; Causes: hereditary, immune mediated, metabolic/ toxic, nutritional Mononeuritis multiplex: multiple nerves, asymmetrical, random; Causes: immune mediated, infective, infiltrative Mononeuropathy: entrapment syndromes 2 features: focal , mechanical factors in play Predisposing factors: -long length of nerve cell (unlike any other cell) -Passage of nerve cell through mechanical hazards like septum, bony prominences, tunnels etc Factors increasing risk of entrapment neuropathies:

1) Decrease in tunnel size (fracture, arthritis) 2) Increase in tunnel components (myxedema, acromegaly) 3) Underlying polyneuropathy (DM, renal , hereditary)

Common cuases:

1) Radial a) Spiral groove b) Supinator syndrome

Wrist drop on waking up DDX of wrist drop: a) UMN wrist drop (cortical hand) Hyperreflexia, finger abduction weakness, watenburg’s sign No voluntary wrist extension but if wrist is extended passively, able to sustain position (can maintain but cannot initiate)(UMN controls voluntary function Associated movements are spared (if asked to attempt to make a fist quickly, wrist will automatically go into extension)

Page 316: Mrcp short case telling skills

To check for finger abduction weakness in presence of wrist drop- flatten hand against a surface, then check (or else finger extension weakness can cause falsely weak finger abduction) Lesion –cortical (hand area –humunculus), subcortical, look for associated face weakness b) C7 root,

Radial nerve Segmental C7 weakness

Brachioradialis and supinator weak (C5,6)

Spared

Normal triceps jerk, triceps spared

Elbow extension weak, absent triceps jerk

Snuff box sensation C7 sensation

c) Saturday night palsy (spiral groove)

Ulnaris and radialis affected, snuff box numbness Normal elbow extension, and triceps jerk Brachioradialis and supinator affected (C5,C6 (supplied by radial nerve)

d) supinator syndrome (PIN alone as it passes through the supintar muscle and pierces the fibrous arcade of Froshe sparing of a) Extensor carpi radialis (ulnaris affected)(radial deviation on wrist extension,

sparing of triceps and brachioradialis) b) Snuff box sensation spared

Hx: new bed, alcohol, sedative Cortical hand: UMN signs: Hyperreflexia, finger abduction weakness, watenburg’s sign No voluntary wrist extension but if wrist is extended passively, able to sustain position (can maintain but cannot initiate)(UMN controls voluntary function Associated movements are spared (if asked to attempt to make a fist quickly, wrist will automatically go into extension) To check for finger abduction weakness in presence of wrist drop- flatten hand against a surface, then check (or else finger extension weakness can cause falsely weak finger abduction) Lesion –cortical (hand knob infarct –humunculus), subcortical, look for associated face weakness

2) Ulnar Wasting of intrinsic muscles of hand a) at the elbow- weakness of long flexors of 4th and 5th digits (flexor digitorum

profundus)

Page 317: Mrcp short case telling skills

b) at the wrist (guyon’s canal)- sparing of the above presents with interossei muscle weakness or interossei muscle weakness + sensory interossei muscle weakness + abductor digiti minimi weakness interossei muscle weakness + abductor digiti minimi weakness + sensory pure sensory cause: leprosy

Ulnar at elbow Ulnar at wrist C8/T1 lesion

Weak flexors 4th and 5th digit

Flexion normal

Sensory medial 1.5

No sensory Sensory C8/T1

Normal thumb abduction

Normal thumb abudction

Weak thumb abduction

3) median

a) CTS DDX cervical spondylosis Nocturnal, on waking, on repeated use of hand, flick sign, sensory usually involves all 4 digits with proximal pain (At times as high as neck) Wasting of thenar eminence Phalen’s sign- 1 min Tinel’s sign Splitting of sensory in ring finger (digital splitting) (also tested using 2 point discrimination) Rarely swelling proximal to the tunnel Occupation: housewife, mechanics Other causes: median artery thrombosis causing median nerve compression EMG findings: CTS- commonest cause of hand numbness in women

b) Pronator syndrome (anterior interosseous nerve) Median nerve at elbow Carpal tunnel signs and weakness of 1) IPJ of thumb 2) Distal IPJ of index and middle finger 3) Flexor carpi radialis 4) Pronator quadrates

Triangle sign- can’t make an O with thumb and index finger (distal flexors are weak)

4) lateral femoral cutaneous nerve patch of numbness at lateral aspect of upper thigh, knee jerk normal, no weakness of knee extension DDX L3/4 root lesion

Lateral femoral cutaneous nerve L3/4 lesion

Page 318: Mrcp short case telling skills

Sensory classical L3/4 segment

Knee jerk normal Absent knee jerk

Knee extension normal Knee extension weak

5) Lateral popliteal nerve

Foot drop: UMN LMN

Lateral poplieal nerve L5 segmental region Sciatic nerve

Classical sensory Sensory L5 Peroneal component – weak knee flexion, foot inversion, ankle jerk absent (greater vulnerability of lateral division of sciatic nerve (peroneal component) than medial division (posterior tibial component) to physical injury

Normal hip internal rotation Weak internal rotation of hip (gluteus medias L5)

Normal hip internal rotation

Normal foot inversion Weak foot inversion (tibialis posterior L5)

Weak foot inversion

Ankle jerk present Ankle jerk present Absent ankle jerk

Normal knee flexion Normal knee flexion Weak knee flexion

Superficial peroneal NAP

Sural NAP Paraspinal EMG

Lateral popliteal Abn Normal Normal

Sciatic Abn Abn Normal

L5 plexus Abn L5 Normal (s1) Normal

L5 root Normal (preganglionic)

Normal (S1) Abn

Uncommon causes: tarsal tunnel syndrome (numbness of feet DDX intermittent Claudication), ulnar sensory neuropathy (mouse use), suprascapular nerve entrapment

Brown sequard syndrome secondary to sjogren’s

Page 319: Mrcp short case telling skills

Sjogren’s: neurologic manifestations can come before systemic symptoms (dry eyes, mouth etc) and before patient becomes antibody positive (anti Ro/La) Posterior column- vibration, proprioception (big toe-> 4th toe), directional scratch Finger flexor (hoffman’s sign)- asymmetrical-pathological, symmetrical- can be normal/ pathological can be graded according to which individual finger can elicit the sign (distance of muscle spindles) eg when patient is very hyperreflexic, even the little finger flick can elicit hoffmann’s. Foot drop UMN: Drag their feet Hit the stairs, kick objects with their shoes, hit the floor Causes: Ankle dorsiflexion cannot be compensated for by hip flexion (both weak) Spasticity Strumpell’s phenomenon (knee flexion is accompanied by foot inversion) LMN: High stepping gait (hip flexion spared, use hip flexion to lift up their feet high above the ground) DDX

1) lateral popliteal lesion -ankle dorsiflexion weak, ankle eversion weak, AJ spared, foot inversion spared 2) L5 segmental lesion -internal rotation of the hip (L5) weak by gluteus medius, foot inversion by tibialis posterior (L5 weak) 3)sciatic nerve lesion -lateral division (peroneal nerve- knee flexion weak (hamstrings), foot drop, AJ absent) -medial division (posterior tibial nerve- foot inversion weak) -clinical:

Internal rotation of hip

Foot inversion

Knee flexion

Ankle jerk

Lat pop N N N N

L5 Weak Weak N N

Sciatic nerve

N Weak Weak weak

-EMG:

Superficial Sural nerve Paraspinal

Page 320: Mrcp short case telling skills

peroneal nerve action potential

EMG

Common personal nerve

AbN N N

L5 plexus AbN (L5) (post ganglionic)

N (S1) N

L5root N (pregnaglionic)

N AbN

Sciatic N AbN AbN N

On turning head to the right, rightVIII CN VOR -> contralateral 6th CN (lateral gaze centre)-> ipsilateral lateral rectus, via MLF to contralateral (right) CN3-> medial rectus

right Frontal eye field: Therefore, Right gaze palsy-> right CN6 at pons (eyes away from lesion), or left frontal eye field at cortical level (eyes towards lesion) Vertical gaze- III and IV CN nuclei VIII CN via MLF to III and IV-> rotatory nystagmus 1 ½ eye- MLF and PPRF/ CN 6 on the same side saccade -visually guided -voluntarily guided pursuit -vision (cerebellum) OKN eye movements- ipsilateral parieto occipital junction Convergence- accurate vision and depth perception Nystagmus worse on closing eyes because the proprioceptive input is taken away Eye muscles lack proprioceptive spindles Central nystagmus- no change with eyes closed Peripheral nystagmus- worse with eyes close Head shaking enhances nystagmus

Page 321: Mrcp short case telling skills

Fundoscopy test: cover the other eye with hand, and can see the nystagmus of the optic disc in the other eye via fundoscopy Vertigo: psycho-physical vertigo, migrainous vertigo, possible/ probable vestibular neuropathy If no deficit- no need to treat Seizure and cognitive impairment- limbic encephalitis, paraneoplastic Cognitive impairment with seizure- Alzheimer’s and FTD

GBS Autoimmune mediated polyneuropathy Groups:

1) GBS: AIDP (classic GBS), AMAN (acute motor axonal neuropathy), AMSAN, pharyngeal cervico brachial

2) MFS-acute ataxia, opthalmoplegia; bickerstaff encephalitis (GQ1b-66%) 3) Acute sensory neuropathy, acute autonomic neuropathy

Classic GBS: Acute, predominantly motor CN7 involved in 70% , Oropharyngeal weakness involved in > 50% Respiratory system involved in 30% Autonomic system involved 60% Monophasic with progressive phase <4 weeks Cytoalbuminemic dissociation Motor involvment: ½ LL> UL (ascending), 1/3 UL= UL, rest of them UL> LL (descending) or CN/ UL-> LL proximal weakness earlier, more profound, more than distal weakness dissociated weakness eg leg flexors affected, but arm extensors affected (UMN type-> may mimic cord syndrome) dissociation between knee flexors and extensors is a useful diagnostic clue weakness at presentation- predominantly proximal 58%, distal 26%, equal 16% limb weakness is always asymmetrical -rarely virtual monoparesis -rarely use-related paresis (more characteristic of polio, lead neuropathy) neck flexors, shoulder abductor weakness correlates well with diaphragmatic weakness in UMN weakness - weakest are shoulder abduction and finger abduction -hoffman’s- hyper reflexia of finger flexors -speed of movement affected-> slowness of movment -synkinesis (foot tapping-> other foot flexors)

Page 322: Mrcp short case telling skills

(posterior column-> vibration, proprioception, directional scratch; lhermitte’s sign is for posterior column involvment) CN involvemnt CN7 palsy -most frequent, > 50% -early if limb weakness severe -may worsen without worsening of limb weakness -unlike bell’s it is mostly bilateral, no loss of taste/ hyperacusis/ retroauricular pain -before dysphagia oropharyngeal weakness lid abnormality-> incomplete ptosis without opthalmoplegia respiratory failure -first 7 days -VC < 1L- observe in ICU- 1/3 will require intubation -alarming signs of VC <12-15ml / kg, NIF <25cm, hypoxemia-> send to ICU Autonomic dysfunction -BP fluctuations -cardiac arrythmias -urinary retention -GI ileus prominent pain 60-70%; 3 types

1) symmetrical deep muscle pain involving the hips, upper legs, and back (large muscles) 2) asymmetrical sciatica (radicular pain) 3) foot/ hand pain (distal)- associated with intense parasthesia and burning

can have hyper reflexia with GBS due to corticospinal tract dysfunction (can have white matter brain lesions which are seen in AMAN) 2 useful bedside tests

1) idiomuscular reflex-decreased/ absent in axonal disease, increased in demyelination/ conduction block

2) bilateral ocular pressure – pressure on eyelid >25 seconds-> bradycardia (<40)-> autonomic dysfunction

CSF -protein elevated by 2/52, 66% in 1/52, 82% by 2nd week -cells normal in >90% of cases -if CSF cell count >10, does not exclude GBS , but consider HIV/ Lyme disease electrophysiology:

Page 323: Mrcp short case telling skills

NCS findings Early-> decreased H reflex, SNAPs in UL abnormal, sural normal in > 12, decreased F waves -demyelination or conduction block at different levels of nerve/ root -> focal then diffuse -earliest –most proximal level (central roots), most distal (nerve twigs)-> low CMAP- can be confused with axonal neuropathy -disproportionately at sites of entrapment antibodies: classic GBS-IgM/G GM1 AMAN-GM1/GD1a AMSAN-GM1 Pharyngeal Cervico brachial-GT1a +/- GQ1b and GD1a MFS-GQ1b (98%), GT1a Acute opthalmoplegic/ ataxic GBS - GQ1b (98%), GT1a Bickerstaff –GQ1b (66%), GT1a Ataxic neuropathy- GD1b Isolated cranial neuropathy-GT1a, GQ1b Isolate bulbar palsy-GT1a>GQ1b Acute sensory neuropathy-GD2, GD1b, GT1b, GQ1b CANOMAD-GD1b (IgM), GD3, GT1b, GQ1b Biomarkers: CSF tau protein elevation- poor prognosis, indication of axonal damage, useful because it can be detected in 10/7 (vs EMG 2/52) Treatment: Immunotherpay (60%) Plasma exchange is equal to IVIG in benefit -IVIG better if preceding C. Jejuni infection, axonal, predominantly motor, anti GM1 Ab positive treatment related fluctuation -6-16%, -patient responds, then deteriorates within 2/12 of disease -give 2nd dose of IVIG -deterioration starts early (within 9/52) (vs CIDP) -fewer deteriorations (1/2) (vs CIDP) 3% of GBS turn out to be acute presentation of CIDP

Page 324: Mrcp short case telling skills

recurrent AIDP vs CIDP- more likely AIDP if 1) long asymptomatic periods with return of tendon reflexes (in CIDP, reflexes don’t return) 2) frequent antecedent illness 3) CSF normal very early (CIDP-> raised protein) 4) Facial weakness 5) Acute onset (10-15% of CIDP-> acute onset)

Re treatment- If no response-> reconfirm correct diagnosis, then assess value of re treatment Plasma exchange if IVIG fails-> RCT-> slight trend towards a better outcome with combined treatment, so plasma exchange if IVIG fails or 2nd course of IVIG, then plasma exchange if fails. Practice parameter on immunotherapy for GBS furnished by the Quality Standards Subcommittee of the American Academy of Neurology (Hughes et al, 2003) has the following recommendations:

1. PE is recommended for non-ambulant adult patients with GBS who seek treatment within 4 weeks of the onset of neuropathic symptoms. PE should also be considered for ambulant patients examined within 2 weeks of the onset of neuropathic symptoms;

2. IVIG is recommended for non-ambulant adult patients with GBS within 2 or possibly 4 weeks of the onset of neuropathic symptoms. The effects of PE and IVIG are equivalent;

3. Corticosteroids are not recommended for the management of GBS; 4. Sequential treatment with PE followed by IVIG, or immunoabsorption followed by IVIG is

not recommended for patients with GBS; and 5. PE and IVIG are treatment options for children with severe GBS

Poor prognosis 1) immediate – rapid disease progression, bulbar dysfunction, severe neck

weakness, dysautonomia 2) demyelinating GBS-> higher incidence of respiratory failure, needs ventilation

(because affects phrenic nerve more than axonal GBS), mortality 4-15%, severe residual disability 20%

3) in the long term (residual disability) – old age, ventilatory support, fast progression (short latency to nadir), C. jejuni/ CMV infection, axonal severe neuropathy (NCS)

recurrence in GBS 1-6% -2/12 separated by full recovery or 4/12 separated by partial recovery -especially if young, milder disease, MFS -recur in the same pattern for eg MFS-> MFS, with similar severity and trigger -need to rule out CIDP and secondary GBS (lymphoma, collagen vascular disease) Differential diagnosis Muscle- hypokalemic periodic paralysis (absense of idiomuscular reflex) NMJ- MG Peripheral nerve- CIDP, AIP (porphyria), secondary GBS (collagen vascular disease, lymphoma, sarcoid, vasculitis, infective eg HIV, Lyme)

Page 325: Mrcp short case telling skills

Root- carcinomatous meningitis- marked asymmetrical limb weakness, arreflexia, other CN involvement; headache and impaired sensorium Cord- transverse myelopathy Differentiate from bilateral bell’s palsy -in bilateral bell’s palsy, pain behind mastoid, hyperacusis, loss of taste anterior 2/3 of tongue- absent in GBS GBS also usually causes asymmetrical facial weakness Anti GQ1b IgG Ab syndrome comprises

1) fisher syndrome 2) BBE (bickerstaff brainstem encephalitis) 3) GBS 4) Ataxic GBS 5) Acute opthalmoparesis without ataxia 6) Isolated internal opthalmoplegia 7) Acute oropharyngeal palsy 8) Pharyngeal cervical brachial palsy

Important headache profiles (adequate knowledge) AND Patient’s headache protile (accurate construction)-> diagnosis, inx, treatment

A) Migraine 1) migraine without aura (common)

unilateral, throbbing, moderate to severe, up to 72 hours, aggravated by head shaking (in contrast to TTH), associated with nausea and vomiting, photophobia and phonophobia

2) migraine with aura (classical) aura precedes and subsides as headache begins associated focal neurological disturbance but gradual build up over at least 4 mins, 2 auras in succession, headache within 60 mins (contrast with TIA) occipital aura- zig zag fortification spectra parietal-temporal- alice in wonderland phenomenon aura without headache-migrainous equivalent/ migrainous accompaniment DDX: 1) cerebral amyloid angiopathy -MRI GRE -present like TIA 2) ICA stenosis-resolved with CEA Basilar artery migraine-like TIA of basilar artery-diagnosis of exclusion

3) retinal migraine

Page 326: Mrcp short case telling skills

reversible monocular visual loss headache before or within 60 mins penlight test (to rule out angle closure glaucoma) opthalmoplegic migraine-also diagnosis of exclusion rule out-cavernous sinus syndrome, tolosa hunt, miller-fischer

4) complicated migraine Chronic migraine-at least 15 days/ month Status migrainous-lasts at least 72 hours with treatment Persistent aura without infarction-more then 2 weeks of aura Neurological disease with migraine 1) cadasil 2) mitochondrial disease 3) sjogren’s syndrome 4) episodic ataxia 5) celiac disease

DDX of migraine: angioma- can have aura But not side-locked, no gradual buildup, presents with fits TX: migraine- abortive, prophylactic Aura (difficult to treat): abortive (inhalation of 100% Oxygen) and prophylactic (valproate, lamotrigine)

B) tension type headache usually no nausea/ vomiting photophobia/phonophobia (only 1) allowed not aggravated by head shaking or routine physical activity seen in frowners, jaw clenchers, stiff neckers tension-vascular headache not clear cut -in between the above -no nausea/ vomiting/ photo/phonophobia -responds to propranolol or tranquilisers

C) cluster headache -excruciating pain -prednisolone -autonomic features including horner’s syndrome Trigeminal –autonomic cephalgia 1) cluster headache

Page 327: Mrcp short case telling skills

2) paroxysmal hemicranias-increased frequency then cluster headache; short duration; dramatic response to indomethacin

3) SUNCT: short lasting unilateral neuralgiform conjunctival injection tearing, RX: CMZ, gabapentin

Hemicranias continua - daily headache -good response to indomethacin Secondary hemicranias continua -parasellar lesion -cerebellopotine lesions

D) trigeminal neuralgia strictly CN5 territory does not cross midline does not involve the posterior part of head treat with carbamazepine –dramatic MRI or not: about 10% of trigeminal neuralgia is caused by tumour (only 2% in those without neurological deficits)-no difference in clinical features or response to med in tumour vs no tumours- MRI all patients!

E) Ice-pick headache Head pain occurring as a single stab or a series of stabs sudden sharp pain Exclusively or predominantly felt in the orbit, temple and parietal area.

Fraction of a second and recur with irregular frequency ranging from one to many per day Single or repeated volleys RX indomethacin DDX: preherpetic neuralgia

F) Subarachnoid hemorrhage with headache- Thunderclap “unlike anything before” Unilateral/ bilateral Neck->arm-> legs (as blood tracks down) Vomiting, photophobia, LOC, fever, cold sweats Signs: neck rigidity, fundus- subhyaloid hemorrhage, papilloedema SAH detection: CT 98% within 12 hours, declines rapidly 50% within 1/52, 0 in 3/52 MRI GRE: 95% in first 4/7, 100% 4-14 days LP (xanthochromia)- within 2/52 DDX of thunderclap headache: SAH ICA dissection

Page 328: Mrcp short case telling skills

Pituitary apoplexy Spontaneous intracranial hypotension Cerebral venous thrombosis Primary thunderclap headache (due to cerebral vasospasm Benign sex headache 3rd ventricle colloid cyst

G) Brain tumour headache- Does not fit into primary profiles Unlike old headache Side locked Symptoms of raised ICP Interfere with sleep No improvement/ relief with analgesia

H) Primary cough headache Exclude AC malformation/ ICA aneurysm/ intracranial tumour

I) Headache with sex Orgasmic-explosive, nausea and vomiting, RX: propranolol, ergot (need to exclude SAH) Pre orgasmic- dull, RX: tranquilisers

J) Hypnic headache nocturnal, awakes from sleep RX: lithium, topiramate

K) Turtle headache: nummular headache-over parietal area Dx: 10 questions 1) Total duration- longer more likely benign 2) Frequency

- daily- Tension -periodic- migraine -remission for weeks- migraine, cluster

3) Duration of each attack 4) Onset time and mode 5) Site of lesion-

Unilateral alternating- migraine Unilateral, fixed- migraine, cluster, trigeminal Including eye-migraine

6) Character 7) Severity-intere with work, recreation, sleep 8) Associated symptoms:

Page 329: Mrcp short case telling skills

Nausea and vomiting- migraine Raised ICP Visual hallucation/ scotoma Photo/phonophobia Autonomic dusfunction Fever Drowsiness Scalp tenderness (extracranial) Prominent pulsating vessels

9) Aggravating factors 10) Relieving factors Inx: Indication for MRI: change in character, red flag, most severe headache of life CT/ MRI/ MRA/ MRV If Hb <10, blood can appear isodense on CT CT negative headache: SAH Meningitis Benign intracranial hypertension Spontanoues intracranial hypotension Cerebral venous thrombosis

INO 1)Ocular movements-CN 3,4,6 Each eye- abduction, adduction, elevation, depression, intorsion, extorsion 2) gaze (version) control-rotational movements of both eyes in the same direction Supranuclear: Cortical- frontal eye fields, PEF, supplementary eye fields, dorsolateral prefrontal cortex Thalamus-BG-cerebellum- Brainstem premotor nuclei-riMLF, PC, INC (vertical and torsional eye movements), PPRF (horizontal eye movements) Infranuclear: 3,4,6 3) itropia: misalignment of one eye relative to the other (during binocular viewing) Esotropia-inward, exotropia- outward hypertropia-upward (hypotropia downward but always use hypertropia by convention) MLF connects CN6 with CN3 nucleus (internuclear problem) INO-failure of adduction, normal abduction +/- nystagmus of abducting eye

Page 330: Mrcp short case telling skills

MLF lesions: same side as adduction failure Mild/ subtle INO -range of adduction is normal but speed of adduction is reduced -adduction lag may be enhanced by OKN tape -ratio of saccade metrics between abduction and adducting eyes can be used to detect subclinical INO Oculographic studies-show nystagmus (bilaterally) in adducting eye also Nystagmus in abducting eye: Pathophysiology:

1) adaptive response to overcome weakness of yoke muscle (contralateral MR)-Hering’s law 2) dissociated gaze evoked nystagmus (subclinical nystagmus in adducting eye)

if INO: midbrain vs pontine reaction to near vision: accommodation, convergence, miosis convergence in INO: premotor neurons for convergence are in the midbrain INO + impaired convergence: midbrain lesion INO, sparing convergence: pons lesion Sparing of convergence DDX: partial CN3 palsy Other features of INO:

1) exotropia abnormalities of vertical eye position/ movements in MLF lesion MLF-interneurons between 6 and 3, and also conveys axons for horizontal, vertical, torsional conjudate eye movements -Abducens interneurons for horizontal conjugate eye movement- INO -vestibular input for vertical conjugate eye movements: 1) otolith-ocular reflex 2) vestibulo-ocular reflex OKN 3) smooth pursuit 4) gaze holding via MLF to 3,4, INC Impairment in above circuits cause INO + 1) Ocular Tilt Reaction(OTR)

3components: 1) head tilt- away from higher eye (eg owl) 2) ocular counterrolling (torsional eye movmenet eg cats, human)towards lesion (if

below pons), away from lesion ( if at/ above pons) -can be recognised during fundoscopy

3) skew deviation (eg fish)

Page 331: Mrcp short case telling skills

higher eye-contralateral to lesion (medulla), ipsilateral to lesion (midpons or midbrain) (high-high, low-low rule) pathophysiology: imbalance of inputs from otoliths and vertical SSCs to occulomotor nuclei bilateral INO-no skew, diminish imbalance between eyes

OTR in humans-primitive vestigial remnant, only released under pathological conditions (like MLF lesions) Physiological: compensatory head tilt predominates, other components not prominent Pathological: all 3 components become very obvious

2) Dissociated vertical nystagmus Bilateral INO:

3) Vertical smooth pursuit impairment 4) Vertical gaze holding impairment 5) Vertical VOR imparirment MLF lesions cause horizontal eye problem and vertical eye movement impairment All Semicircular canals also have projections via MLF but anterior SCC, only, has projections via alternative pathways eg ventral tegmental tract Clinical implications of difference between anterior and posterior SCC projections: In MLF lesions 1) Dissociated vertical nystagmus 2) Relative sparing of upward or downward gaze

Otolith projections in MLF- detect linear acceleration

Site of lesion causes

Brainstem (pons> midbrain/ medulla)

MS, vascular, demyelination, neoplasms

Cerebellar –rare

Peripheral vestibulopathy

Neural integrators for gaze holding: horizontal- nucleus prepositus hypoglossi (NPH) Vertical-interstitial nucleus of cajal (INC) Both: medial vestibular nucleus, cellgroup of paramedian tract, cerebellar (flocculus) DDX for INO (Adduction failure)

Site Causes Investigation

Muscle Restrictive-dysthyroid Inflammatory-myositis

Forced duction CT Pain, CT

NMJ MG Tensilon (saccade velocity normal)

N Partial 3rd Pupils, other

Page 332: Mrcp short case telling skills

muscles

MLF INO Brainstem infarct (commonly unilateral) MS (commonly bilateral)

MS: 2nd most common cause of INO Most common cause of INO in the young (<45 yo), usually bilateral Inflammation in MS: Postcapillary venules provide scaffolding for adhesion and trafficking into the CNS These venules have greatest concentration in periventricular zones MLF runs below floor of 4th ventricle and aqueduct Prognosis of INO MS: recover over few weeks-months Stroke: recovery is less likely INO Plus syndromes: coincident involvement of adjacent structures to MLF

1) WEMINO (wall-eyed monocular INO) 2) WEBINO (wall-eyed bilateral INO)

Bilateral MLF and convergence pathways are involved (medial rectus subnuclei) Lesion: midbrain

3) 11/2 syndrome (INO and gaze palsy): lesion affecting MLF and PPRF or abducens nucleus complex. Right sided lesion- only left eye can move and look to left a) With contralateral exotropia (to INO) (because PPRF predominates) b) With ipsilateral esotropia (due to fascicular CN6 palsy)

Rarer INO plus syndromes: 4) Monocular paralysis: associated adjacenet abducens nerve fasciculus involvement 5) Contralateral CN4 palsy (With INO): MLF in caudal midbrain with extension into trochleal

nerve nucleus on the same side Lacunar stroke syndromes Lacune- hole Perforating artery occlusion- classically internal capsule -lenticulostriate artery (from MCA) -lipohyalinosis 10% of LACS have a lesion other than a lacnunar infarct 5% may be hemorrhagic

Page 333: Mrcp short case telling skills

Can cause cranial neuropathy/ INO/ other brainstem deficits Can’t distinguish between occlusion of perforating arteries in anterior or posterior circulation Classical syndromes: 1) Pure motor stroke Involves the entire 1/12- face, arm, leg Classically IC/ Pons (if FAL all involved) If restricted deficits (face and arm/ face/ arm/ leg or leg alone)- lesion in CR/ junctional zone The more restricted a deficit, the more the lesion is likely to arise from cortex-frontal subcortical white matter, frontal / parietal cortex 2) Pure sensory stroke Entire ½ of body also Lesion in thalamus Partial-?Lesion Less accepted: 1)Ataxic hemiparesis (no nystagmus because cortical loop) Homolateral ataxia and caudal paresis- dysmetria of arm and leg, and weakness of only leg (esp toes/ ankles) CT/ MRI: infarct 72%, hemorrhage 5% Location: IC 39%, pons 19%, thalamus 13%, CR 13%, LN 8%, cerebellum (SCA) 4%, frontal cortex (4%) May also be in precentral gyrus Usually IC, Pons, CR Lacunar infarct can be related to large artery disease Although lacunar, lesion can be bigger than a lacune Do complete imaging (MRI/ MRA) 2)Sensory motor Most frequent after pure motor stroke Lesion: IC, CR, thalamus, genu of IC 3)Clumsy hand dysarthria syndrome (DCHS) Lesion in IC 40%, pons 17%, CR 8.6% Dysarthria- corticobulbar tracts, ?cerebellar (neocircuit)

Ataxic hemiparesis DCHS

Dysarthria - Mod to severe

Hemiparesis Moderate Mild if any

Clumsiness/ ataxia Ipsilateral limb +/- gait ataxia

Similar

Sensory Frequently ipsilateral

Absent

Others Affects neocerebellar circuit (cortical

+/- tongue deviation and palatal

Page 334: Mrcp short case telling skills

loop)- coordination of limb + corticospinal tract: homolateral ataxia and weakness of only leg or homolateral ataxia and weakness of arm and leg

weakness

Lymphocytic hypophysitis Classification of hypophysitis:

Primary: Lymphocytic hypophysitis Granulomatours hypophysitis Xanthomatous hypophysitis

Secondary: Systemic disease: takayasu’s disease, crohn’s disease, langerhans cell histiocytosis, sarcoidosis, inflammatory pseudotumour Infective etiology: bacterial, viral, fungal

Pathological classification of the main forms of hypophysitis

Primary forms: lymphocytic (Autoimmune) granulomatous

secondary forms: local lesions: germinomas, Rathke’s cleft cysts, craniopharyngiomas, pituitary adenomas Systemic disases: granulomatous disease: sarcoidosis, wegener’s granulomatousis, langerhans cell histiocytosis, syphilis, tuberculosis

General characteristics of lymphocytic hypophysitis Female: male 8:1 Mean age at start of disease:34.5 (F), 44.7 (M) Race: Caucasian/ Japanese 3:1 Frequent appearance from 6 months before to 6 months after delivery Familial or personal history of autoimmunity HLA aplotype: HLA DR4 (44%); HLA DR5 (23%) Symptoms and signs of lymphocytic hypophysitis

Page 335: Mrcp short case telling skills

Related to pituitary enlargement: Headache (60%) Visual field impairment (40%) More rarely diplopia

Related to pituitary secretion alterations: Unusually normal pituitary function Hyperprolactinemia (30%) More rarely hypoprolactinemia Subclinical hypopituitarism (25%) ACTH deficiency (the earliest and most frequent isolated pituitary deficiency) Rarely acute hyposurrealism with high mortality Hypogonadotropic hypogonadism (usually diagnosed only in males) Rarely isolated GH deficiency

Differences between lymphocytic hypophysitis and pituitary adenoma usng different imaging techniques

lymphocytic hypophysitis

pituitary adenoma

Sellar x ray Sellar floor uniformly flat

Unilateral depression of sellar floor

Pituitary NMR Pituitary enlargement with symmetrical sovrasellar expansion Compression and displacement of pituitary mass Stalk thickened but not deviated

Unilateral endosellar mass (microadenoma) or inhomogenously expanding pituitary mass with asymmetrical sovrasellar extension (macroadenoma) Contralateral deviation of stalk

After gadolinium Intense and homogenous enhancmenet of pituitary mass Appearance of “dural tail” Loss of “bright

Slight, delayed and inhomogenous enhancement Usually lack of “dural tail” Persistence of

Page 336: Mrcp short case telling skills

spot” of neurohypophysis if diabetes insipidus is associated

“bright spot”

Lymphocytic hypophysitis

1) Endocrine autoimmune disease: Hashimoto’s thyroiditis Diabetes mellitus Parathyroiditis Graves’ disease Addison’s disease APS type 1 APS type 3a

2) Non endocrine organ specific autoimmune diseases: Vitiligo Myasthenia Alopecia Pernicious anemia Primary biliary cirrhosis Chronic atrophic gastritis

3) Other non endocrine diseases: SLE Iidiopathic retroperitoneal fibrosis Discoid lupus erythematosus Germinoma Dacryoadenitis Treatment: (only symptomatic) a) Spontaneous remission b) Mass reduction:

1) surgical transphenoidal treatment Histological diagnosis, decompression pituitary biopsy-diagnostic, therapeutic

2) lymphocytic drugs steroids, MTX, Aza, cyclosporine A, high dose pulsed methylpred steroids first line if no compressive symptoms-> sequential MRI

3) radiotherapy c) if panhypopituitarism-> replacement hormone therapy

Page 337: Mrcp short case telling skills

MND Progressive disease -should tell patient unless he doesn’t want to know because

1) save costly pilgrimage from doctor to doctor in search of answers 2) entitled to warranty time

Neurology Menagerie

1) Humming bird sign/ Penguin sign-in PSP Sagittal MRI-midbrain atrophy, tegmentum atrophy, sparing of pons Axial MRI- Mickey Mouse sign due to sparing of cerebral peduncles Midbrain: Tectum Tegmentum Substantia nigra Cerebral peduncles PSP: Postural instability (Early falls) Parkinsonism Supranuclear gaze palsy -slowing before palsy -saccades before pursuit -vertical before horizontal -down before up -square wave jerks -uninhibited blink to light Midbrain contains premotor nuclei for ocular movements MSA P –marked atrophy of pons

2) Face of giant panda sign- in Wilson’s Hyperintensity in midbrain tegmentum Sparing of the red nucleus (eyes) Sparing of the substantia nigra (ears) Hypointensity of superior colliculus (mouth) “miniature panda” sign Hypointensity due to iron/ copper deposition

Page 338: Mrcp short case telling skills

Hyperintensity due to edema/ demyelination

3) Eye of the tiger sign- in Hallervorden-Spatz syndrome(EPS-dystonia/ chorea, dementia, ocular symptoms) (now known as PKAN-panthothenate kinase) Hyperintensity wthin a hypointense globus pallidus on t2 MRI Hyperintensity due to edema and necrosis/ gliosis Hypointensity due to Fe deposition (early disease) Acute insult of GP- cyanide poisoning, CO poisoning Chronic insult of GP- Hallervorden-Spatz, wilson’s disease NBIA (neurodegeneration with brain iron accumulation) 1)PKAN (100% have eye of the tiger, not very specific- also in PSP and CBGD) 2) neuroferritinopathy 3)Infantile neuroaxonal dystrophy (INAD) 4) aceruloplasminemia (no eye of the tiger)

4) Tiger skin appearance (stripes), leopard skin appearance (spots) in metachromatic leucodystrophy Sparing of myelin Focal areas of lipid deposition Dysplastic gangliocytoma-patch of tiger skin in cerebellum

5) Elephant sign (bilateral) in alzheimer’s disease due to hippocampal atrophy

6) Butterfly sign- around corpus callosum (heavily myelinated fibres) in malignant glioma, tumefactive MS

7) Hot cross buns a) 60% MSA, 80% MSA C b) 25% SCA2, 1% SCA3 c) Vasculitis d) Variant CJD

Hyperintensity- degenerate transverse pontocerebellar fibres Hypointensity- preservation of corticospinal tracts and pontine tegmentum Inflammatory- infective/ immune mediated Markers- abnormal CSF, lesion enhancement on MRI Brunn’s nystagmus Low frequency, high amplitude on one side High frequency, low amplitude on the other side Classically in lesions affecting cerebellum, brainstem like acoustic neuroma, AICA syndrome eg left acoustic neuroma: affects left vestibular nerve-high frequency,

Page 339: Mrcp short case telling skills

small amplitude nystagmus to the right, later, left cerebellum involved- low frequency, high amplitude nystagmus to the left

Palatal myoclonus/ tremor/ nystagmus Involuntary palatal movements, bilateral -rhythmic to and fro -jerky component -typically 2Hz frequency (1.5-3Hz) -doesn’t disappear during sleep -associated with pendular, vertical nystagmus (look straight ahead) -can involve face, neck muscles, tongue muscles synchronously Classically after pontine hemorrhage/ infarct -In contrast to usual brain atrophy after stroke, there is hypertrophy of the inferior olivary nucleus -neuronal hypertrophy with vacuolization and astrocytosis -usually takes >1/12 to start, maximum size up to 612 after stroke -Other muscles involved are : pharyngeal muscles, laryngeal muscles, neck muscles -pathophysiology: disruption of the guillain mollaret triangle (dentate nuc-> red nuc-> inferior olivary nucleus -> sup. Cerebellar peduncle) -may be unilateral TX: anecdotal success -clonazepam, carbamazepine, baclofen, anticholinergics -Sx: tensor veli palatine -Botox (for tinnitus) 2 types:

Idiopathic/ essential: Acquired (symptomatic)

Tensor veli palatine -innervated by V3 (opens the Eustachian tube) -originates near Eustachian tube -clicking tinnitus -no hypertrophy of contralateral inferior olivary nucleus -disappears during sleep (eye and other structures not involved)

Levator veli palatine -innervated by CN7 -other facial muscles involved -no clicking tinnitus -hypertrophy of contralateral inferior olivary nucleus (hyperintense on T2) -persists during sleep

Causes: stroke, encephalitis, MS, tumour

Parkinsonism Rest tremor, rigidity, hypokinesia (2/3)

Page 340: Mrcp short case telling skills

4th: postural instability Hypokinesia: parkinsonism, apraxia, freezing, catatonia, psychomotor depression, rigidity Hyperkinesias: tremor, dystonia, chorea, myoclonus, tics IS it PD or atypical Red flags for/ against Need to know

1) Typical features of PD 2) Other diseases causing parkinsonism

Parkinsonian disorders:

Neurodegenerative

Non neurodegenerative (secondary)

Primary (idiopathic) parkinsonism

Atypical parkinsonian disorders

Drug: antipsychotic, antiemetics, methyldopa Vascular: multi-infarct, Binswanger’s disease Structural: NPH, brain tumour, aneurysm, infectious mass (other signs- hemiparesis, hyperreflexia, aphasia, sensory loss, seizures) Toxin: MPTP, CO, Mn, Hb, cyanide metabolic: Wilson’s, PTH, T4, liver failure, central pontine myelinolysis Infection: postencephalitic, HIV, CJD, infectious masses, neuroacanthocytosis Trauma: pugilistic encephalopathy

Sporadic (look for red flags to suggest atypical PD)

Familial (up to 15%)

Sporadic: PSP CBD

Familial: FTDP-IT (fronto-temp

Page 341: Mrcp short case telling skills

MSA (DLB- ?variant of PD- early onset of dementia, delusions and hallucinations, agitation) AD

dementia with parkinsonism) SCAs, Machado-Joseph disease (SCA-3), FTD-PROGR FXTAS (fragile X tremor ataxia syndrome) Wilson’s disease, Juvenile Huntington’s disease, DRPLA (dentate-rubro-pallidoluysian atrophy)

Red flags:

symptoms that come too early for PD 1) Early dementia (<1 year) (DLB, NPH) 2) Early hallucinations (<3 years) (DLB) 3) Early prominent postural instability (<3 years)(PSP, NPH) 4) early gait disorder (NPH) 5) early freezing (<3 years) (MSA) 6) early severe autonomic dysfunction (MSA) 7) early saccadic slowing (PSP) 8) early dysphagia (MSA)

absence of features atypical for PD: 1) absence of resting tremor, (MSA, PSP, NPH) 2) absence of asymmetry (MSA, PSP, NPH) 3) absence of levodopa response (1g/ day x 1/12-> if no response) (MSA, PSP, CBD)

symptoms highly suggestive of atypical parkinson’s disorders 1) respiratory stridor (MSA) 2) myoclonus (MSA, CBD)

Page 342: Mrcp short case telling skills

3) cerebellar/ pyramidal signs (MSA, PSP, SCA, Fx TAs) 4) limb apraxia (CBD) 5) rapid progression (MSA, PSP)

Rest tremors At onset of PD 65% During course of PD 75-100% Resoved (lost) 9% Characteristics: 3-5 Hz, unilateral/ asymmetrical, distal prominence (pronator-supination, pill rolling), also involves lips, chin, jaw, legs, rarely neck, head or voice (DDX ET) Induction/ aggravation of tremor- produce stress (count 100 backwards), ask patient to walk Pure tremor (benign tremulous PD) –monosymptomatic PD -isolated tremor, preceds other symptoms by years -responds to levodopa Postural tremor: in addition to rest tremor in many patients more prominent, more disabling than rest tremor first manifestation of the disease extreme pronation/ supination bring out the postural tremor better PD related postural tremor (re emergent tremor) DDX ET -same frequency as rest tremor -latency in appearance of postural tremor (rest tremor breaking then) -responsive to levodopa therapy Co existent ET responds to propranolol

PD tremor ET

Freq 4-6 Hz 5-10 Hz

Movment Sup-pronation Flexion-extension

Rest Increased Decreased

Action Decreased Increased (can interfere with rapidly alternating movments- cogwheel can be found in ET on distraction)

Walking Increased Decreased

Alcohol - Decreased

Page 343: Mrcp short case telling skills

Distribution other than hands

Face, jaws, lips, chin

Head, voice

No other signs of parkinsonism

Rigidity: Increased tone Flexion/ extension of wrist Throughout the range of movement Lead pipe and cogwheeling-underlying superimposed tremor Froment’s sign: exercise opposite limb to bring out the rigidity- non specific, also present in benign ET Rigidity may be associated with pain eg painful shoulder Hypokinesia: Decrease in speed and amplitude Bradykinesia refers to decrease in speed alone Fusion of discrete movement Affects-voluntary movements, associated movements of facial expression, arm swing “kinesia paradoxica” able to run if “fire” Anatomical localisation for hypokinesia- putamen and globus pallidus Postural instability -check all steps for gait, pull test appears late, important in staging, refractory to levodopa (non dopaminergic pathways are involved) H and Y stag 1-unilateral 1.5-+axial involvement 2- bilateral 2.5-recovery on pull test 3-postural instability, physically independent 4-severe disability, able to walk or stand unassisted 5-wheelchair bound or bedridden unless aided Average latency (in months) to first fall

Disease Time (in months)

PD 108

MSA 42

PSP 16.8

5th symptom: mirror movment -high prevalence in unaffected or less affected limbs

Page 344: Mrcp short case telling skills

Postural deformities in PD -appear late -younger onset of disease -flexion deformities (neck, trunk) -striatal limb deformities-striatal hand, foot -dropped head -camptocrnua -PISA syndrome Loss of neurons in pedunculopontine nucleus DBS targets- GPi and STN Gait: lack of arm swing slow, shallow and small steps (3s) turning en masse flexion deformities freezing- start, turn, narrow line focal dystonias preceding PD

1) foot dystonia- precedes development of PD by several years 2) extension of great toe (striatal foot) 3) luicifer’s cramp 4) fonzanelli’s thumb up sign

non motor symptoms in PD: sleep disorders:

a) REM sleep behaviour disorder 1/3 of patients Pre parkinsonism state (preceds disease) Violent dream content

b) Insomnia > 50% related to hypocretin secretion

c) Hyposmia Frequent Early marker of PD Neuronal loss in olfactory bulb

d) Pain-oral and genital

Ophthalmic signs in PD 1) Dry eyes (decreased blink rate) 2) Formed visual hallucinations (26.7%)

-sensation of presence -auditory hallucinations (rare)

Page 345: Mrcp short case telling skills

3) slowness in remembered saccades- cognitive impairment, correlates with long duration of PD

Visual hallucinations in all patients - mean 11 years -early visual hallucinations- if cognitive impairment, autonomic involvement, axial rigidity -DLB if early Dementia -20-40% -6x increased risk -profile- dys-executive syndrome, 2 x increased risk of MCI, MCI may represent a key feature of PD Neuroimaging -PET -differentiate PD from atypical parkinsonism -early, preclinical PD -Sonography of brain parenchyma-hyperintensity in PD, also in benign ET -3T MRI scanner- diffusion tensor pulse sequence, FA (Fractional anisotrophy)- 100% sensitive and specific Involvement of dopaminergic pathway, autonomic pathway, cognitive pathway Synkinesis seen in early PD Deep brain stimulation -GPi (globus pallidus internus), STN (subthalamic nucleus- junction between midbrain and pons, controls postural balance- stimulate to improve postural stability), PPN (peduncular pontine nucleus)-below -indications : motor fluctuations not responding to drugs, disabling dyskinesias -good candidate-no cogwheeling, good response to levodopa, no comorbids which increase the risk of surgery -improves positive symptoms: rest tremor, rigidity -negative symptoms do not improve: hypokinesia, postural instability -complications: infection, bleeding, cognitive problems, speech problems, emotional problems, -of STN and pedunculopontine nucleus- effective for gait and posture Fetal cells transplant failed due to

1) Ethical reasons 2) Cost 3) No consistent good response in clinical trials 4) Neurons that survived were found to have developed lewy bodies

Page 346: Mrcp short case telling skills

Stem cells only affect the dopaminergic pathway- other pathways are not affected PSP:

1) Postural instability-pedunculopontine atrophy (midbrain atrophy) 2) Parkinson’s- predominant axial rigidit, lack of asymmetry, poor response to levodopa 3) Supranuclear gaze palsy

Classic: steele-richardson-olzschewski syndrome Variants: PSP P etc Upright posture/ frequent falls Pseudobulbar emotionality Staring look, furrowed brow/ stare Square wave jerks (saccadic intrusions)- very pathognomonic- in primary gaze, use fundoscope and fix on optic disc (easier to see) Slow saccades (speed affected before limitation of movement) Slowing affects saccades more than pursuit Vertical gaze affected before horizontal Down before up Oculocephalic response:Vertical doll’s and horizontal doll’s normal Eyelid apraxia Parkinsonism/ hypo/ bradykinesia Cognitive impairment Motor disinhibition: Applause sign (frontal), Luria hand sign (frontal), go/ no-go tasks Executive dementia Early midbrain atrophy: MRI: humming bird sign/ penguin sign Vascular parkinsonism: Acute/ subacute onset, stepwise progression Lower body parkinsonism with shuffling gait and freezing Presence of vascular risk factors At least 2 infarcts in BG +- widespread white matter disease on MRI consistent with Binswanger’s pathology Hydrocephalus induced parkinsonism -can be communicating or obstructive -NPH is idiopathic- parkinsonism/ gait dzr + urinary/ fecal incontinence + dementia MSA:

a) Autonomic ( urinary dysfunction (frequency, urgency)/ impotence, erectile dysfunction, postural hypotension (stand 3 mins, systolic 30 mmHg, diastolic 10 mmHg)

Page 347: Mrcp short case telling skills

b) Parkinsonism poorly responsive to levodopa c) Cerebellar dysfunction

Pyramidal signs Pathologically: 1)supratentorial: MSA P, MSA C, predominant autonomic (shy dragger involvement) 2) infratentorial: PD, PSP, CBD, SCA, atypical friedrich’s, pure autonomic failure Spinal Features:

1) Tremor- postural, kinetic, or jerky 2) Postural instability not as early as PSP 3) Autonomic failure-cold dusky extremities 4) Speech 5) Dysphagia 6) Emotional incontinence 7) REM sleep disturbances 8) Stridor 9) Dystonia 10) Stimulus sensitive myoclonus 11) Poor response to levodopa 12) Rapid progression (wheelchair sign on 1 year)

Hot cross bun sign: ponto-cerebellar atrophy: loss of pontine neurons and myelinated transverse cerebellar fibres with preservation of the corticospinal tracts

MSA PD PSP DLB

Orthostatic hypotension

87% 78% 45% 15%

Urinary incontinence

87% 82% 75% 64%

CBD: Cortical: frontal atrophy- alien hand, frontal gait d/o apraxias, dementia parietal atrophy-cortical sensory loss (2 point discrimination), apraxia, agnosia, parietal eye fields basal ganglia- parkinsonism, dystonia, myoclonus, tremor, chorea, blepharospasm saccades horizontal and vertical vs PSP (vertical mostly) Unilateral, coarse tremor/ rigidity Limb apraxia/ limb dystonia/ alien limb DLB: Progressive cognitive decline Probable (2) of

Page 348: Mrcp short case telling skills

1) parkinsonism 2) fluctuation of cognition 3) hallucinations Possible (1) Severe neuroleptic sensitivity DLB vs PDD –“infamous “ 1 year rule Dementia <1 year- DLB, > 1 year –PDD Of all sporadic causes, relative frequency of PD is most common MSA/ PSP- less common SCAs more likely to have parkinsonism 2x3=6 SCA3- type 1-dystonia, type 2- ataxia, type 3-peripheral neruoapthy / wasting, type 4-parkinsonism (dopa responsive) Drug induced -symmetrical -less likely to have tremor -dramatic response to anticholinergics -static until treated -associated orofacial dyskinesias -within 1-3/12 of starting drugs -recovery few weeks to a few months -drugs: neuroleptics, Ca antagonists (cinnarizine), hidden neuroleptics (chlorpromazine, promethazine) Neurological diseases with acanthocytes: Apolipoproteinemia Choreoacanthosis (parkinsonism, No myopathy, AR) Mcleod syndrome (parkinsonism, myopathy, X linked) Gait apraxia – simulate brushing of teeth, light a cigarette, wave goodbye, ask to say “shh” Gait analysis 6 steps Start with watching patient get up from bed/ chair

1) Vertical axis (posture) -stooped-PD -waddling-muscular dystrophy

2) UL –arm swing- PD -effeminate gait-chorea

3) Feet a) initiation –prompt/ ignition failure- late PD (freezing), gait apraxia (NPH) b) steps –slow, shallow, small (3s)-shuffling gait

Page 349: Mrcp short case telling skills

c) high stepping-foot drop of LMN Type d) broad based-ataxic

4) walking in a straight line (tandem) 5) turning- turning by numbers, turn enblock, loss of balance on turning 6) festination-walk/ run fast, then stop suddenly

Balance:

1) both feet together- look straight 2) both feet together- eyes closed (10 secs) 3) eyes open, 1 foot in front (5 secs) 4) 1 foot up (5 secs) 5) Hop around on 1 foot 6) Pull test-2 steps of compensation are allowed

Ptosis HORNER’s Sympathetic ptosis (Horner’s) - mild ptosis -Upside down ptosis ->inferior Muller’s muscle also weak 25% of population have physiological anisocoria 15% of population have asymmetrical palpebral fissure -these disappear on elevation 3-5% of population have both physiological anisocoria, and asymmetrical palpebral fissure neck: Painful Horner’s->dysphagia, nystagmus brainstem: PICA- nystagmus, skew deviation of eyes, lateralopulsion of eyes/ body Can also cause -ipsilateral 7th UMN (abberant fibres), -psuedo 5th sensory neuropathy -contralateral facial numbness -ipsilateral body-> limb paraesthesia if ipsilateral hemiparesis –suggests extending cerebellar infarct -repeat scan urgently PICA- misnomer (vertebral artery disease (67%), PICA (10%)) Young patient – vertebral artery dissection-> anticoagulation AICA: Ocular tilt reaction -> head hilt, ocular counterolling, skew deviation( vertical divergence, supranuclear origin, hypertropia) Horizontal deviation: Exotropia, esotropia Wall eye bilateral INO: exotropia with bilateral INO

Page 350: Mrcp short case telling skills

Lung apex: pancoast Cord:Transverse myelitis-> acute/ subacute-> vascular –arterial – anterior spinal artery thrombosis, posterior spinal artery thrombosis; venous; dural AVF post infectious autoimmune, Behcets (eye signs), Sjogrens’ (eye signs) Sarcoid (eye signs), paraneoplastic (antiampiphysin, CRMP 5), Lymphoma (eye signs),

MYASTHENIA GRAVIS

Ptosis-> look at pupils-> see if it disappears with elevation (Horner’s) Eye signs of myasthenia: Upward gaze:

1) fatiguability on sustained upward gaze (use glare from torchlight to enhance) 2) enhanced ptosis (look up, elevate the other eyelid, the unassisted one comes down) 3) see-saw ptosis

lateral gaze 4) look laterally-> fatigue faster (look to contralateral side) 5) up and down lid sign->on looking laterally, patient compensates 6) Mary Walker phenomena-> weaknesss in another part of body (eg ptosis) following

effort of arms/ legs, eg grip hand, eyelid comes down 7) Ice pack test: cold enhances neurotransmission. Put ice pack over eyelid for 2 mins-> get

better 8) Sleep test: eyelid ptosis gets better after a nap

Lid twitch signs 9) cogan’s lid twitch- from below to primary gaze level (cogan’s) 10) close eyestight (gently) and then open suddenly- twitching 11) look sideways- lid hop sign (twitching > once)

lid retraction signs: compensate for ptosis by retracting lids of the other eye 12) unilateral lid retraction with contralateral ptosis 13) unilateral lid retraction without contralateral ptosis/ bilateral lid retraction without

ptosis

Page 351: Mrcp short case telling skills

eye lid signs: 14) eye lash sign- can’t bury eyelashes fully 15) eye peek sign- complete closure initially, then the eye lashes peek out (can’t sustain

closure) 16) forcibly close eyes for 30 secs, then open suddenly- ptosis disappears (work the

antagonist muscle) 17) impairment of ocular muscle movements

lambert’s sign- grip finger- facilitation and enhancement-> grip becomes stronger practical points:

1) cold enhances neurotransmission, so warm limb to >/= 35 degrees before testing 2) stop pyridostigmine for >/= 12 hrs prior to RNS 3) if exercise to bring on fatiguability, wait for 1-2 mins before checking EMG/ RNS (post

exercise facilitation) Ramsay hunt syndrome Type 1: Zoster oticus Type 2: Zoster oticus and LMN facial Type 3: Zoster oticus and LMN facial and tinnitus/ deafness Type 4: Zoster oticus and LMN facial and tinnitus/ deafness and vestibular dysfunction Other nerves involved: 8,9,5,10,6, rarely 2 Acute vestibular neuronitis: can have vertical diplopia -ocular tilt reaction -usually patients don’t complain Bell’s palsy:

1) HSV1 Bell’s -pain behind the ear - hyperacusis- no acyclovir

2) Herpes zoster without a rash - pain inside the ear -severe pain, deafness instead of hyperacusis -redness of ear lobe (red ear sign) -acyclovir, prednisolone up to 14/7 60% neuropraxic- good recovery 40% denervation> neuropraxia, incomplete recovery

Page 352: Mrcp short case telling skills

Neuropraxia, recovery: First: facial asymmetry at rest-> frontalis-> eye closure-> mouth closure (up to down) Sequential recovery-> when upper part of face recovers, can be mistaken from UMN 7th palsy, explain to patient- mouth will recover last Speech Dysarthria-articulation Dysphonia-phonation Dysphasia-language, all types have abnormality of verbal output Dysphasia in speech is accompanied by dysphasia in writing Neuroanatomy of language Cerebral dominance of right handed people-> left hemispheric dominance Main cortical areas for language: (auditory centre) Visual input for written words, auditory input for spoken words-> wernicke’s area-> (indirect pathway) Geschwind’s area-> broca’s area-> motor cortex (direct pathway) arcuate fasciculus (phonologically based- sound based)-> broca’s area for vocalization semantically based (meaning of words)= other pathways (non arcuate fasciculus) bedside assessment: fluency (spontaneous speech, talk a lot, non sensible content, usually posterior hemispheric lesion, no hemiplegia, confused with psychiatric patients) -fluent/ non fluent (phrase length, words/ minute, articulatory ability, agrammatic, melody of language affected -few words, difficulty in expression, usually the anterior hemispheric lesions, associated hemiplegia due to motor cortex involvment, easily diagnosed) -word finding difficulty -paraphasia -neologism

Page 353: Mrcp short case telling skills

comprehension (1 step commands-> 2 step-> 3 step commands-> syntactically difficult questions-> semantically reversible eg lion eaten by tiger who survives OR semantically constrained eg book read by boy VS syntactically difficult eg susan gave the ruby ring which belonged to her mother to her daughter mary- what type of ring? Who has her ring now? Who originally had the ring? repetition (polysyllabic words) make hay while the sun shines, don’t cry over spilt milk- only spared in transcortical dysphasias naming spelling- sensitive for wernicke’s aphasia reading – read aloud, reading-> comprehension, paragraph reading writing- copying, dictation, (I went to market to buy a fat pig; today is a beautiful day), spontaneous production of sensation (why in hospital) fluency of spontaneous speech, comprehension and repetition can help to differentiate different types of dysphasia

dysphasia Site of lesion Fluency Comprehension repetition

broca 44,45 - + -

wernicke 22,39,40 + - -

global 22,39,40,44,45 + - -

Conduction dysphasia

40,41,42 + + -

Transcortical motor

Near broca - + +

Transcortical sensory

Near wernicke + - +

Transcortical mixed sensory and motor

- - +

Broca’s -44,45+ (large area) - aphemia -non fluent dysphasia -agrammatical -comprehension intact for simple material

Page 354: Mrcp short case telling skills

-associated hemiplegia -repetition hesitant -naming reduced -difficulty in reading aloud -writing intact -recovery good -?apraxia of speech -associated buccolingual apraxia wernicke’s -22,39,40 -spontaneous speech- fluent, paraphasic -auditory comprehension impaired, even for simple questions -repetition impaired -spelling, reading, writing impaired -no hemiplegia -+/- homonymous hemianopia -naming impaired conduction aphasia -repetition impaired out of proportion to other language function disturbance -AF (arcuate fasciculus involved) -subcortical lesion (AF) -but in real life lesion is usually temporal or parietal transcortical -sparing of repetition (perisylvian language circuit) -can be perseveration – echolalia -motor (similar to Broca dysphasia + can repeat, usually watershed infarcts between ACA and MCA (anterior watershed), or MCA and PCA (posterior watershed)) -sensory (like wernicke’s + can repeat, usually near temporoparietal junction, echolalic) -mixed (global aphasia + can repeat, echolalic)

Subcortical areas involved in dysphasia

1) thalamic aphasia -typically fluent, paraphasic errors -medial and anterior thalami -like sensory transcortical dysphasia (impaired comprehension, intact repetition) 2) basal ganglia (non fluent without hemiparesis- need to rule out simple partial seizure)

Page 355: Mrcp short case telling skills

-like trasncortical motor dysphasia -decreased fluency -comprehension only mildly affected -repetition spared

If global dysphasia without hemiparesis

-2 separate lesions with sparing of motor strip eg embolic infarcts, mets, hemorrhage -if single lesion- pre rolandic or temporoparietal. Need to rule out embolic cause -recovery and prognosis variable (in the past, thought to have a very good prognosis) crossed aphasia (rare-0-4%)-diagnostic criteria -strong right handedness -absense of family history of left handedness -right hemispheric lesion (>50% of left handed still have left hemisphere dominance) 2 types

1) “mirror image” of aphasia from left hemispheric lesion 2) anomalous (non fluent regardless of location)

visuospatial cortex still in right hemisphere in bilingual people, after dysphasia -mother tongue recovers first (not necessarily) -most familiar language (written, read and spoken) at the time of insult recovers first vs spoken only language language and non dominant hemispheric lesions -discourse impaired (ability to narrate) -ability to appreciate a story or point of joke may be impaired -prosody (melody of language) may be impaired ND hear- probable source of residual automatic utterances Epilepsy-> simple partial seizures can present with aphasia Epileptic foci

1) motor cortex of either hemisphere (anterior apercular area)- phonatory seizure/ dysarthric seizure- comprehension/ writing normal

2) speech areas (dominant hemisphere) –writing/ comprehension normal- broca’s Splenium of the corpus callosum Alexia without agraphia -seen in occipital lobe infarcts of dominant hemisphere which involve the splenium (although occipital lobe infarcts rarely involve the splenium -reversible lesions

Page 356: Mrcp short case telling skills

-excellent prognosis DDX:

1) Infective encephalitis-viral (influenza, VZV, adenovirus, rotavirus), bacterial (E. Coli, salmonella)

2) ADEM/ MS- bilateral, symmetrical, subcortical white matter 3) Stroke –Vertebrobasilar circulation 4) PRES 5) Machiafava-bigami disease 6) Lymphoma 7) Pontine myelinolysis 8) AEDs (phenytoin toxicity, vigabatrin) 9) Transcallosal seizure

Pathology: intramyelinic edema, inflammation Spontaneous intracranial hypotension -resembles post LP headache-initally orthostatic -begins and increases slowly -thunderclap/ exertional/ cough/ paradoxical postural headache -other symptoms -> neck ache, nausea/ vomiting, diarrhoea, diplopia, tinnitus, dementia, chorea -mechanism- caudal brain sagging -aetiology- unclear, coughing, vigorous physical activity, minor physical trauma (chiropractor) -associated with cerebral venous sinus thrombosis (2%) -suspect if change in character of headache (lose positional character), impaired sensorium, seizures -treat CVST first and look for usual risk factors of CVST -typical MRI features: 1) diffuse pachymeningeal gadolinium enhancement without leptomeningeal enhancement-compensatory venous dilatation 2) thickening of the meninges with linear enhancement (not noluar like sarcoid, TB, mets) 3) pachymeningeal hyperintensity on T2 4) MRI findings predictive of outcome-good outcome of MRI abnormality- good compensation 5) MRI spine may be useful if head MRI is normal, to find CSF leak into the subdural, epidural, paraspinal regions -other abnormalities include pituitary enlargement, and subdural hygroma -CT myelography- site of leak -radionuclide cisternography- presence of leak

Page 357: Mrcp short case telling skills

-CSF: pressure> 40 in >50% of patients (hypovolemia rather than hypotension) mild lymphocytosis raised protein level xanthochromia LP not necessary if MRI and headache is classical -Treatment: bed rest, hydration, corticosteroids, analgesics, caffeine (coca cola, cafergot) blood patch (or ligation or patching of epidural space with muscle and gelfoam), -cervical/ thoracic blood patch (higher yield as directed at site of CSF leak)

Thalamic stroke syndrome

4 types of thalamic nuclei and connection with midbrain Not uncommon 25% of all posterior circulation -Vertebrobasilar infarcts -basilar artery occlusion -occipital infarcts isolated thalamic infarcts-11% of POCI Thalamus is a vague and complicated structure at the base of the brain According to vascular territory, thalamus can be divided into: Anterior: polar arteries- off PCOM artery -30% supplied by thalamo-subthalamic artery (polar artery absent in 30%) Medial: thalamo-subthalamic arteries (paramediam perforating) -also supplies midbrain, Contralateral medial thalamus, anterior thalamus Lateral: thalamogeniculate artery- comes off PCA Posterior: posterior choroidal artery -also supplies midbrain

MEDIAL THALAMUS: NEUROPSYCHIATRIC MANIFESTATIONS, AND DISTURBANCE OF CONSCIOUSNESS

Thalamic-subthalamic artery Medial thalamic nuclei

Page 358: Mrcp short case telling skills

Connections with frontal cortices, limbic system and reticular activating system (“frontal cortical-subcortical circuit”)

Frontal cortex ↗ ↘ Thalamus striatum ↖ ↙ Globus pallidus/ substantia nigra Cognitive impairment and behavioural disturbances (in subcortical lesions which disrupt circuit)

a) Dorsolateral prefrontal cortex Dorsolateral prefrontal cortex ↗ ↘

Thalamus (ventral anterior medial dorsal) caudate (dorsolateral)

↖ ↙ GP (lateral dorsomedial)

DP syndrome: executive dysfunction affected (cognitive) –subcortical dementia

b) Lateral orbitofrontal circuit Lateral orbitofrontal cortex ↗ ↘ Thalamus Caudate (ventromedial) ↑ ↖ ↙ SN← Globus pallidus Lateral orbitofrontal syndrome (behaviour): change in personality, disinhibition (social misconduct), hyperorality

c) Anterior cingulated circuit Anterior cingulate cortex ↗ ↘ Thalamus Nucleus accumbens ↖ ↙ GP-substantia nigra AC syndrome: apathy, abulia, akinetic mutism

Page 359: Mrcp short case telling skills

LATERAL THALAMIC NUCLEI: SENSORY AND MOTOR

-marked hemianesthesia -paroxysmal pain, hyperpathia, -hemiataxia “thalamo-geniculate arteries”

1) Sensory relay nuclei - somatosensory: sensory loss, whole hemi/ partial- facial, cheiro-oral, cheiro-podal- oral, psuedoradicular; all modalities/ sparing of pain and temperature; paroxysmal pain / parasthesia (thalamic pain); hyperpathia (unpleasant sensory stimulus) -vestibular sensory: disequilibrium- astasia (tendency to fall to one side) (vestigial-vestibular-thalamic) -gustatory sensory :contralateral ageusia (loss of taste)

2) Motor relay nuclei - cerebellar: limb ataxia (dentate-rubral-thalamo-cortical) - basal ganglia: dystonia “thalamic hand”- flex at MCP, hyperextend at IP; chorea; myoclonus Involuntary movements in lesion of lateral thalami-myoclonus, dystonia, pseudoathetoid, action and postural tremors

POSTERIOR THALAMUS: VISUAL FIELD DEFECTS

Pulvinar and lateral geniculate body Posterior choroidal artery C/F homonymous quadrantanopia/ hemianopia; homonymous wedge shaped hemianopia, quadruple sectoranopia (isolated VF defects may be the sole manifestation of lacunar thalamic infarct) Geniculate body-neglect, transcortical aphasia, amnesia Anterior choroidal artery-thalamo-capsular: HHH syndrome- hemiataxia, hemianesthesia, hemiplegia Connections with:

Page 360: Mrcp short case telling skills

1) RAS-acute depression of consciousness, hypersomnolence (persist for weeks) 2) Frontal cortex-neuropsychiatric manifestations 3) Limbic system-neuropsychiatric manifestations

DDX wernicke’s encephalopathy- affects medial thalamus Other structures: fibres to convergence centre

1) Thalamic esotropia (pseudo 6th) 2) Wrong way deviation (irritative lesion) conjugate deviation contralateral to side of

lesion 3) Skew deviation

Midbrain syndrome

- occulomotor disturbances- INO, vertical gaze disorder, 3rd nerve palsy (nuclear/ fascicular) -pyramidal dysfunction -cerebellar ataxia -red nucleus tremor (substantia nigra nearby- so combination of cerebellar, postural and rest tremor) -peduncular hallucinosis: strongly vivid, colourful images, lasting mins to hours (throughout waking), aetiology- mainly vascular Site of lesion- rostral brainstem, thalamus (dissociation of components of REM sleep)

ANTERIOR THALAMIC NUCLEI: NEUROPSYCHIATRIC MANIFESTATIONS

By polar arteries Main connections: frontal cortices, limbic system (mammilary bodies), RAS C/F: as of medial thalamic involvement + amnesia -sensory transcortical dysphasia -perseveration in thinking and speech -palipsychism (superimposition of temporally unrelated information) -chronotanaxis (loss of time orientation and loss of time sense -time blindness (if medial dorsal nuclei of thalamus is involved) -emotional centre facial paralysis (voluntarily-no facial paralysis) Single infarct dementia-bilateral medial thalamic infarcts, also anterior thalamic infarcts, disruption of frontal cortical-subcortical circuits UMN wrist drop -associated movements still present eg reflex is extend when asked to make a fist -UMN signs eg waterburg’s sign positive -inability to initiate but able to maintain